Surgery (non-ortho) Flashcards Preview

MRCS Qu's > Surgery (non-ortho) > Flashcards

Flashcards in Surgery (non-ortho) Deck (270)
Loading flashcards...
1
Q

A 29 year old man has had a sore throat for the past 5 days. Over the past 24 hours he has notices increasing and severe throbbing pain in the region of his right tonsil. He is pyrexial and on examination he is noted to have a swelling of this area. What is the most likely cause?

	Tonsillar cancer
	Lymphoma
	Quinsy
	Glandular fever
	Common cold
A

Unilateral swelling and fever is usually indicative of quinsy. Surgical drainage usually produces prompt resolution of symptoms.

Acute tonsillitis
Characterised by pharyngitis, fever, malaise and lymphadenopathy.
Over half of all cases are bacterial with Streptococcus pyogenes the most common organism
The tonsils are typically oedematous and yellow or white pustules may be present
Infectious mononucleosis may mimic the condition.
Treatment with penicillin type antibiotics is indicated for bacterial tonsillitis.
Bacterial tonsillitis may result in local abscess formation (quinsy)

2
Q

A 76 year old man presents with a painful right arm (he is right handed). On examination, he has a cool right forearm and absent radial and brachial pulses. A duplex scan shows thrombus occluding the brachial artery. What is the most appropriate course of action?

	Administration of therapeutic low molecular weight heparin
	Brachial embolectomy without fasciotomy
	Intra arterial thrombolysis
	Systemic thrombolysis
	Brachial embolectomy with fasciotomy
A

Brachial embolectomy without fasciotomy

Options to treat upper limb embolic events include either anticoagulation or surgery. Background arterial lesions are very rarely present in the upper limb so embolectomy is usually successful. Anticoagulation with intravenous unfractionated heparin is a reasonable alternative. However, note that low molecular weight heparin is not used in this setting as its difficult to control perioperatively.

Acute limb ischaemia
Thrombosis of a pre-existing site of atherosclerosis if the commonest cause of acute limb ischaemia
Acute thrombosis of popliteal aneurysms poses the greatest threat to the limb
Sudden occlusion of a large proximal vessel results in the typical appearances of acute limb ischaemia

Clinical appearances
Less than 6 hours = White leg
At 6 -12 hours = Mottled limb with blanching on pressure
More than 12-24 hours = Fixed mottling

Management of acutely ischaemic leg

  • White leg with sensorimotor deficit Surgery and embolectomy
  • Dusky leg, mild anaesthesia Angiography
  • Fixed mottling Primary amputation

Role of thrombolysis
Intra arterial thrombolysis is better than peripheral thrombolysis
Mainly indicated in acute on chronic thrombosis
Avoid if within 2 months of CVA or 2 weeks of surgery
Aspiration of clot may improve success rate if the thrombosis is large

Surgery
Both groins should be prepared
Transverse arteriotomy is easier to close
Poor inflow should be managed with iliac trawl- if this fails to improve then consider a femoro-femoral cross over or axillo-femoral cross over.
A check angiogram should be performed on table and prior to closure
Systemic heparinisation should follow surgery
Fasciotomy should be considered if the time between onset and surgery exceeds 6 hours

3
Q

A 14 year old boy presents with enlarged tonsils that meet in the midline. Oropharyngeal examination confirms this finding and you also notice peticheal haemorrhages affecting the oropharynx. On systemic examination he is noted to have splenomegaly. What is the most likely cause?

	Oral candidiasis
	Infection with Streptococcus pyogenes
	Infection with Rickettsia rickettsii
	Infection with Epstein Barr virus
	Infection with Staphylococcus aureus
A

EBV - A combination of pharyngitis and tonsillitis is often seen in glandular fever. Antibiotics containing penicillin may produce a rash when given in this situation, leading to a mistaken label of allergy. Infection with candidiasis can occur in individuals with systemic illness of which splenomegaly may be a feature. However, its unlikely to only affect the tonsils.

Acute tonsillitis
Characterised by pharyngitis, fever, malaise and lymphadenopathy.
Over half of all cases are bacterial with Streptococcus pyogenes the most common organism
The tonsils are typically oedematous and yellow or white pustules may be present
Infectious mononucleosis may mimic the condition.
Treatment with penicillin type antibiotics is indicated for bacterial tonsillitis.
Bacterial tonsillitis may result in local abscess formation (quinsy)

4
Q

An 8 year old boy presents with abdominal pain,a twelve hour history of vomiting, a fever of 38.3 0C and four day history of diarrhoea. His abdominal pain has been present for the past week. What is the most likely cause?

	Coeliac disease
	Appendix abscess
	Irritable bowel syndrome
	Mesenteric adenitis
	Diverticulitis
A

Appendix Abscess

The high fever and diarrhoea together with vomiting all point to a pelvic abscess. The presence of pelvic pus is highly irritant to the rectum, and many patients in this situation will complain of diarrhoea. Mesenteric adenitis is less likely to run such a protracted course. IBS does not typically produce such marked systemic symptoms. Diverticulitis is almost unheard of in children.

Appendicitis	
Pain radiating to right iliac fossa
Anorexia (very common)
Short history
Diarrhoea and profuse vomiting rare

Crohn’s disease
Often long history
Signs of malnutrition
Change in bowel habit, especially diarrhoea

Mesenteric adenitis
Mainly affects children
Causes include Adenoviruses, Epstein Barr Virus, beta-haemolytic Streptococcus, Staphylococcus spp., Escherichia coli, Streptococcus viridans and Yersinia spp.
Patients have a higher temperature than those with appendicitis
If laparotomy is performed, enlarged mesenteric lymph nodes will be present

Diverticulitis
Both left and right sided disease may present with right iliac fossa pain
Clinical history may be similar, although some change in bowel habit is usual
When suspected, a CT scan may help in refining the diagnosis

Meckel’s diverticulitis
A Meckel’s diverticulum is a congenital abnormality that is present in about 2% of the population
Typically 2 feet proximal to the ileocaecal valve
May be lined by ectopic gastric mucosal tissue and produce bleeding

Perforated peptic ulcer
This usually produces upper quadrant pain but pain may be lower
Perforations typically have a sharp sudden onset of pain in the history
Incarcerated right inguinal or femoral hernia
Usually only right iliac fossa pain if right sided or bowel obstruction.
Bowel perforation secondary to caecal or colon carcinoma
Seldom localised to right iliac fossa, although complete large bowel obstruction with caecal distension may cause pain prior to perforation.

Gynaecological causes
Pelvic inflammatory disease/salpingitis/pelvic abscess/Ectopic pregnancy/Ovarian torsion/Threatened or complete abortion/Mittelschmerz

Urological causes
Ureteric colic/UTI/Testicular torsion
Other causes
TB/Typhoid/Herpes Zoster/AAA/Situs inversus

5
Q

A 74 year old man presents with a painful right leg. The pain developed suddenly the preceeding evening. However, he wondered if it might resolve overnight, instead, it has got worse. On examination, his right leg is cold and white with diminished distal sensation. A CT angiogram shows a thrombus occluding the external iliac artery with no atheromatous disease. What is the most appropriate course of action?

Femoral embolectomy and below knee fasciotomy
Femoral embolectomy alone
Femoral popliteal bypass graft without fasciotomy
Femoral embolectomy with above and below knee fasciotomy
Femoro-femoro cross over graft with above and below knee fasciotomy
A

Femoral embolectomy and below knee fasciotomy

Delayed limb re-perfusion = Risk of compartment syndrome

Delayed surgery for limb ischaemic carries a risk of re-perfusion injury and compartment syndrome. Where surgery is delayed beyond 6 hours, most surgeons would perform a fasciotomy at the same time as the embolectomy. However, whilst the compartments below the knee are vulnerable, its very rare for this to occur in the thigh and so, as a rule, the thigh is not treated in this manner.

Acute limb ischaemia
Thrombosis of a pre-existing site of atherosclerosis if the commonest cause of acute limb ischaemia
Acute thrombosis of popliteal aneurysms poses the greatest threat to the limb
Sudden occlusion of a large proximal vessel results in the typical appearances of acute limb ischaemia

Clinical appearances
Less than 6 hours = White leg
At 6 -12 hours = Mottled limb with blanching on pressure
More than 12-24 hours = Fixed mottling

Management of acutely ischaemic leg

  • White leg with sensorimotor deficit Surgery and embolectomy
  • Dusky leg, mild anaesthesia Angiography
  • Fixed mottling Primary amputation

Role of thrombolysis
Intra arterial thrombolysis is better than peripheral thrombolysis
Mainly indicated in acute on chronic thrombosis
Avoid if within 2 months of CVA or 2 weeks of surgery
Aspiration of clot may improve success rate if the thrombosis is large

Surgery
Both groins should be prepared
Transverse arteriotomy is easier to close
Poor inflow should be managed with iliac trawl- if this fails to improve then consider a femoro-femoral cross over or axillo-femoral cross over.
A check angiogram should be performed on table and prior to closure
Systemic heparinisation should follow surgery
Fasciotomy should be considered if the time between onset and surgery exceeds 6 hours

6
Q

A 22 year old man is investigated for weight loss. A duodenal biopsy taken as part of his investigations shows total villous atrophy and lymphocytic infiltrate. He has a skin lesion that has small itchy papules.

A.	Pyoderma gangrenosum
B.	Erythroderma
C.	Dermatitis herpetiformis
D.	Acanthosis nigricans
E.	Multiple lipomata
F.	Multiple neurofibromata
G.	Multiple telangectasia
H.	None of the above
A

Dermatitis herpetiformis

The patient has coeliac disease and this is associated with dermatitis herpetiformis.

Skin lesions may be referred for surgical assessment, but more commonly will come via a dermatologist for definitive surgical management.

Basal Cell Carcinoma
Most common form of skin cancer.
Commonly occur on sun exposed sites apart from the ear.
Sub types include nodular, morphoeic, superficial and pigmented.
Typically slow growing with low metastatic potential.
Standard surgical excision, topical chemotherapy and radiotherapy are all successful.
As a minimum a diagnostic punch biopsy should be taken if treatment other than standard surgical excision is planned.

Squamous Cell Carcinoma
Again related to sun exposure.
May arise in pre - existing solar keratoses.
May metastasize if left.
Immunosupression (e.g. following transplant), increases risk.
Wide local excision is the treatment of choice and where a diagnostic excision biopsy has demonstrated SCC, repeat surgery to gain adequate margins may be required.

Malignant Melanoma
The main diagnostic features (major criteria):
Change in size
Change in shape
Change in colour
Secondary features (minor criteria)
Diameter >6mm
Inflammation
Oozing or bleeding
Altered sensation

Treatment
Suspicious lesions should undergo excision biopsy. The lesion should be removed in completely as incision biopsy can make subsequent histopathological assessment difficult.
Once the diagnosis is confirmed the pathology report should be reviewed to determine whether further re-excision of margins is required (see below):

Margins of excision-Related to Breslow thickness
Lesions 0-1mm thick 1cm
Lesions 1-2mm thick 1- 2cm (Depending upon site and pathological features)
Lesions 2-4mm thick 2-3 cm (Depending upon site and pathological features)
Lesions >4 mm thick 3cm

Further treatments such as sentinel lymph node mapping, isolated limb perfusion and block dissection of regional lymph node groups should be selectively applied.

Kaposi Sarcoma
Tumour of vascular and lymphatic endothelium.
Purple cutaneous nodules.
Associated with immuno supression.
Classical form affects elderly males and is slow growing.
Immunosupression form is much more aggressive and tends to affect those with HIV related disease.

Non malignant skin disease:

Dermatitis Herpetiformis
Chronic itchy clusters of blisters.
Linked to underlying gluten enteropathy (coeliac disease).

Dermatofibroma
Benign lesion.
Firm elevated nodules.
Usually history of trauma.
Lesion consists of histiocytes, blood vessels and fibrotic changes.
Pyogenic granuloma
Overgrowth of blood vessels.
Red nodules.
Usually follow trauma.
May mimic amelanotic melanoma.

Acanthosis nigricans
Brown to black, poorly defined, velvety hyperpigmentation of the skin.
Usually found in body folds such as the posterior and lateral folds of the neck, the axilla, groin, umbilicus, forehead, and other areas.
The most common cause of acanthosis nigricans is insulin resistance, which leads to increased circulating insulin levels. Insulin spillover into the skin results in its abnormal increase in growth (hyperplasia of the skin).
In the context of a malignant disease, acanthosis nigricans is a paraneoplastic syndrome and is then commonly referred to as acanthosis nigricans maligna. Involvement of mucous membranes is rare and suggests a coexisting malignant condition.

7
Q

A 72 year old man is investigated for weight loss. On examination he is deeply jaundiced and cachectic. He also has a dark velvety lesion coating his tongue.

A.	Pyoderma gangrenosum
B.	Erythroderma
C.	Dermatitis herpetiformis
D.	Acanthosis nigricans
E.	Multiple lipomata
F.	Multiple neurofibromata
G.	Multiple telangectasia
H.	None of the above
A

Acanthosis nigricans

Acanthosis nigricans may be associated with GI malignancies such as gastric and pancreatic cancer.

Skin lesions may be referred for surgical assessment, but more commonly will come via a dermatologist for definitive surgical management.

Basal Cell Carcinoma
Most common form of skin cancer.
Commonly occur on sun exposed sites apart from the ear.
Sub types include nodular, morphoeic, superficial and pigmented.
Typically slow growing with low metastatic potential.
Standard surgical excision, topical chemotherapy and radiotherapy are all successful.
As a minimum a diagnostic punch biopsy should be taken if treatment other than standard surgical excision is planned.

Squamous Cell Carcinoma
Again related to sun exposure.
May arise in pre - existing solar keratoses.
May metastasize if left.
Immunosupression (e.g. following transplant), increases risk.
Wide local excision is the treatment of choice and where a diagnostic excision biopsy has demonstrated SCC, repeat surgery to gain adequate margins may be required.

Malignant Melanoma
The main diagnostic features (major criteria):
Change in size
Change in shape
Change in colour
Secondary features (minor criteria)
Diameter >6mm
Inflammation
Oozing or bleeding
Altered sensation

Treatment
Suspicious lesions should undergo excision biopsy. The lesion should be removed in completely as incision biopsy can make subsequent histopathological assessment difficult.
Once the diagnosis is confirmed the pathology report should be reviewed to determine whether further re-excision of margins is required (see below):

Margins of excision-Related to Breslow thickness
Lesions 0-1mm thick 1cm
Lesions 1-2mm thick 1- 2cm (Depending upon site and pathological features)
Lesions 2-4mm thick 2-3 cm (Depending upon site and pathological features)
Lesions >4 mm thick 3cm

Further treatments such as sentinel lymph node mapping, isolated limb perfusion and block dissection of regional lymph node groups should be selectively applied.

Kaposi Sarcoma
Tumour of vascular and lymphatic endothelium.
Purple cutaneous nodules.
Associated with immuno supression.
Classical form affects elderly males and is slow growing.
Immunosupression form is much more aggressive and tends to affect those with HIV related disease.

Non malignant skin disease:

Dermatitis Herpetiformis
Chronic itchy clusters of blisters.
Linked to underlying gluten enteropathy (coeliac disease).

Dermatofibroma
Benign lesion.
Firm elevated nodules.
Usually history of trauma.
Lesion consists of histiocytes, blood vessels and fibrotic changes.
Pyogenic granuloma
Overgrowth of blood vessels.
Red nodules.
Usually follow trauma.
May mimic amelanotic melanoma.

Acanthosis nigricans
Brown to black, poorly defined, velvety hyperpigmentation of the skin.
Usually found in body folds such as the posterior and lateral folds of the neck, the axilla, groin, umbilicus, forehead, and other areas.
The most common cause of acanthosis nigricans is insulin resistance, which leads to increased circulating insulin levels. Insulin spillover into the skin results in its abnormal increase in growth (hyperplasia of the skin).
In the context of a malignant disease, acanthosis nigricans is a paraneoplastic syndrome and is then commonly referred to as acanthosis nigricans maligna. Involvement of mucous membranes is rare and suggests a coexisting malignant condition.

8
Q

A lesion that may occur in a 32 year old man with long standing Crohns disease.

A.	Pyoderma gangrenosum
B.	Erythroderma
C.	Dermatitis herpetiformis
D.	Acanthosis nigricans
E.	Multiple lipomata
F.	Multiple neurofibromata
G.	Multiple telangectasia
H.	None of the above
A

Pyoderma gangrenosum

Pyoderma gangrenosum may occur in Crohns disease.

Skin lesions may be referred for surgical assessment, but more commonly will come via a dermatologist for definitive surgical management.

Basal Cell Carcinoma
Most common form of skin cancer.
Commonly occur on sun exposed sites apart from the ear.
Sub types include nodular, morphoeic, superficial and pigmented.
Typically slow growing with low metastatic potential.
Standard surgical excision, topical chemotherapy and radiotherapy are all successful.
As a minimum a diagnostic punch biopsy should be taken if treatment other than standard surgical excision is planned.

Squamous Cell Carcinoma
Again related to sun exposure.
May arise in pre - existing solar keratoses.
May metastasize if left.
Immunosupression (e.g. following transplant), increases risk.
Wide local excision is the treatment of choice and where a diagnostic excision biopsy has demonstrated SCC, repeat surgery to gain adequate margins may be required.

Malignant Melanoma
The main diagnostic features (major criteria):
Change in size
Change in shape
Change in colour
Secondary features (minor criteria)
Diameter >6mm
Inflammation
Oozing or bleeding
Altered sensation

Treatment
Suspicious lesions should undergo excision biopsy. The lesion should be removed in completely as incision biopsy can make subsequent histopathological assessment difficult.
Once the diagnosis is confirmed the pathology report should be reviewed to determine whether further re-excision of margins is required (see below):

Margins of excision-Related to Breslow thickness
Lesions 0-1mm thick 1cm
Lesions 1-2mm thick 1- 2cm (Depending upon site and pathological features)
Lesions 2-4mm thick 2-3 cm (Depending upon site and pathological features)
Lesions >4 mm thick 3cm

Further treatments such as sentinel lymph node mapping, isolated limb perfusion and block dissection of regional lymph node groups should be selectively applied.

Kaposi Sarcoma
Tumour of vascular and lymphatic endothelium.
Purple cutaneous nodules.
Associated with immuno supression.
Classical form affects elderly males and is slow growing.
Immunosupression form is much more aggressive and tends to affect those with HIV related disease.

Non malignant skin disease:

Dermatitis Herpetiformis
Chronic itchy clusters of blisters.
Linked to underlying gluten enteropathy (coeliac disease).

Dermatofibroma
Benign lesion.
Firm elevated nodules.
Usually history of trauma.
Lesion consists of histiocytes, blood vessels and fibrotic changes.
Pyogenic granuloma
Overgrowth of blood vessels.
Red nodules.
Usually follow trauma.
May mimic amelanotic melanoma.

Acanthosis nigricans
Brown to black, poorly defined, velvety hyperpigmentation of the skin.
Usually found in body folds such as the posterior and lateral folds of the neck, the axilla, groin, umbilicus, forehead, and other areas.
The most common cause of acanthosis nigricans is insulin resistance, which leads to increased circulating insulin levels. Insulin spillover into the skin results in its abnormal increase in growth (hyperplasia of the skin).
In the context of a malignant disease, acanthosis nigricans is a paraneoplastic syndrome and is then commonly referred to as acanthosis nigricans maligna. Involvement of mucous membranes is rare and suggests a coexisting malignant condition.

9
Q

A 67 year old male is diagnosed as having a 7cm infra renal abdominal aortic aneurysm. What is the likely risk of rupture over the next 5 years?

	<10%
	20%
	25%
	75%
	35%
A

Risks of abdominal aortic aneurysm rupture (over 5 years):
5-5.9cm = 25%
6-6.9cm= 35%
7cm and over = 75%

Aneuryms greater than 5cm in diameter on USS should be formally assessed using CT scanning with arterial phases to delineate anatomy and facilitate surgical planning.

Abdominal aortic aneurysms are a common problem in vascular surgery.
They may occur as either true or false aneurysm. With the former all 3 layers of the arterial wall are involved, in the latter only a single layer of fibrous tissue forms the aneurysm wall.
True abdominal aortic aneurysms have an approximate incidence of 0.06 per 1000 people. They are commonest in elderly men and for this reason the UK is now introducing the aneurysm screening program with the aim of performing an abdominal aortic ultrasound measurement in all men aged 65 years.

Causes
Several different groups of patients suffer from aneurysmal disease.
The commonest group is those who suffer from standard arterial disease, i.e. Those who are hypertensive and have been or are smokers.
Other patients such as those suffering from connective tissue diseases such as Marfan’s may also develop aneurysms. In patients with abdominal aortic aneurysms the extracellular matrix becomes disrupted with a change in the balance of collagen and elastic fibres.

Management
Most abdominal aortic aneurysms are an incidental finding.
Symptoms most often relate to rupture or impending rupture.
20% rupture anteriorly into the peritoneal cavity. Very poor prognosis.
80% rupture posteriorly into the retroperitoneal space
The risk of rupture is related to aneurysm size, only 2% of aneurysms measuring less than 4cm in diameter will rupture over a 5 year period. This contrasts with 75% of aneurysms measuring over 7cm in diameter.
This is well explained by Laplaces’ law which relates size to transmural pressure.
For this reason most vascular surgeons will subject patients with an aneurysm size of 5cm or greater to CT scanning of the chest, abdomen and pelvis with the aim of delineating anatomy and planning treatment. Depending upon co-morbidities, surgery is generally offered once the aneurysm is between 5.5cm and 6cm.

Indications for surgery
Symptomatic aneurysms (80% annual mortality if untreated)
Increasing size above 5.5cm if asymptomatic
Rupture (100% mortality without surgery)

Surgical procedures
Abdominal aortic aneurysm repair

Procedure:

GA
Invasive monitoring (A-line, CVP, catheter)
Incision: Midline or transverse
Bowel and distal duodenum mobilised to access aorta.
Aneurysm neck and base dissected out and prepared for cross clamp
Systemic heparinisation
Cross clamp (proximal first)
Longitudinal aortotomy
Atherectomy
Deal with back bleeding from lumbar vessels and inferior mesenteric artery
Insert graft either tube or bifurcated depending upon anatomy
Suture using Prolene (3/0 for proximal , distal anastomosis suture varies according to site)
Clamps off: End tidal CO2 will rise owing to effects of reperfusion, at this point major risk of myocardial events.
Haemostasis
Closure of aneurysm sac to minimise risk of aorto-enteric fistula
Closure: Loop 1 PDS or Prolene to abdominal wall
Skin- surgeons preference

Post operatively:

ITU (Almost all)
Greatest risk of complications following emergency repair
Complications: Embolic- gut and foot infarcts
Cardiac - owing to premorbid states, re-perfusion injury and effects of cross clamp
Wound problems
Later risks related to graft- infection and aorto-enteric fistula

Special groups

Supra renal AAA
These patients will require a supra renal clamp and this carries a far higher risk of complications and risk of renal failure.

Ruptured AAA
Pre-operatively the management depends upon haemodynamic instability. In patients with symptoms of rupture (typical pain, haemodynamic compromise and risk factors) then ideally prompt laparotomy. In those with vague symptoms and haemodynamic stability the ideal test is CT scan to determine whether rupture has occurred or not. Most common rupture site is retroperitoneal 80%. These patients will tend to develop retroperitoneal haematoma. This can be disrupted if Bp is allowed to rise too high so aim for Bp 100mmHg.
Operative details are similar to elective repair although surgery should be swift, blind rushing often makes the situation worse. Plunging vascular clamps blindly into a pool of blood at the aneurysm neck carries the risk of injury the vena cava that these patients do not withstand. Occasionally a supracoeliac clamp is needed to effect temporary control, although leaving this applied for more than 20 minutes tends to carry a dismal outcome.

EVAR
Increasingly patients are now being offered endovascular aortic aneurysm repair. This is undertaken by surgeons and radiologists working jointly. The morphology of the aneurysm is important and not all are suitable. Here is a typical list of those features favoring a suitable aneurysm:
Long neck
Straight iliac vessels
Healthy groin vessels

Clearly few AAA patients possess the above and compromise has to be made. The use of fenestrated grafts can allow supra renal AAA to be treated.

Procedure:

GA
Radiology or theatre
Bilateral groin incisions
Common femoral artery dissected out
Heparinisation
Arteriotomy and insertion of guide wire
Dilation of arteriotomy
Insertion of EVAR Device
Once in satisfactory position it is released
Arteriotomy closed once check angiogram shows good position and no endoleak

Complications:
Endoleaks depending upon site are either Type I or 2. These may necessitate re-intervention and all EVAR patients require follow up . Details are not needed for MRCS.

10
Q

A 48 hour old neonate develops increasing abdominal distension. He had a normal delivery but has yet to pass any meconium. Following digital rectal examination liquid stool is released.

A.	Meckel diverticulum
B.	Pyloric stenosis
C.	Acute appendicitis
D.	Mesenteric adenitis
E.	Intussusception
F.	Malrotation
G.	Hirschsprung disease
A

Hirschsprung disease

Hirschsprungs may present either with features of bowel obstruction in the neonatal period or more insidiously during childhood. After the PR there may be an improvement in symptoms. Diagnosis is by full thickness rectal biopsy.

Paediatric Gastrointestinal disorders
When occurs may present atypically
Mesenteric adenitis
Central abdominal pain and URTI
Conservative management
Intussusception
Telescoping bowel
Proximal to or at the level of, ileocaecal valve
6-9 months age
Colicky pain, diarrhoea and vomiting, sausage shaped mass, red jelly stool.
Treatment: reduction with air insufflation
Malrotation
High caecum at the midline
Feature in exomphalos, congenital diaphragmatic hernia, intrinsic duodenal atresia
May be complicated by development of volvulus, infant with volvulus may have bile stained vomiting
Diagnosis is made by upper GI contrast study and USS
Treatment is by laparotomy, if volvulus is present (or at high risk of occurring then a ladds procedure is performed
Hirschsprung’s disease
Absence of ganglion cells from myenteric and submucosal plexuses
Occurs in 1/5000 births
Full thickness rectal biopsy for diagnosis
Delayed passage of meconium and abdominal distension
Treatment is with rectal washouts initially, thereafter an anorectal pull through procedure
Oesophageal atresia
Associated with tracheo-oesophageal fistula and polyhydramnios
May present with choking and cyanotic spells following aspiration
VACTERL associations
Meconium ileus
Usually delayed passage of meconium and abdominal distension
Majority have cystic fibrosis
X-Rays may not show a fluid level as the meconium is viscid (depends upon feeding), PR contrast studies may dislodge meconium plugs and be therapeutic
Infants who do not respond to PR contrast and NG N-acetyl cysteine will require surgery to remove the plugs
Biliary atresia
Jaundice > 14 days
Increased conjugated bilirubin
Urgent Kasai procedure
Necrotising enterocolitis
Prematurity is the main risk factor
Early features include abdominal distension and passage of bloody stools
X-Rays may show pneumatosis intestinalis and evidence of free air
Increased risk when empirical antibiotics are given to infants beyond 5 days
Treatment is with total gut rest and TPN, babies with perforations will require laparotomy

11
Q

A 7 month old girl presents with vomiting and diarrhoea. She is crying and drawing her legs up. There is a a sausage shaped mass in the abdomen.

A.	Meckel diverticulum
B.	Pyloric stenosis
C.	Acute appendicitis
D.	Mesenteric adenitis
E.	Intussusception
F.	Malrotation
G.	Hirschsprung disease
A

Intussusception

Sausage shaped mass (colon shaped) is common in intussusception. The other common sign is red jelly stool.

Paediatric Gastrointestinal disorders
Pyloric stenosis
M>F
5-10% Family history in parents
Projectile non bile stained vomiting at 4-6 weeks of life
Diagnosis is made by test feed or USS
Treatment: Ramstedt pyloromyotomy (open or laparoscopic)
Acute appendicitis
Uncommon under 3 years
When occurs may present atypically
Mesenteric adenitis
Central abdominal pain and URTI
Conservative management
Intussusception
Telescoping bowel
Proximal to or at the level of, ileocaecal valve
6-9 months age
Colicky pain, diarrhoea and vomiting, sausage shaped mass, red jelly stool.
Treatment: reduction with air insufflation
Malrotation
High caecum at the midline
Feature in exomphalos, congenital diaphragmatic hernia, intrinsic duodenal atresia
May be complicated by development of volvulus, infant with volvulus may have bile stained vomiting
Diagnosis is made by upper GI contrast study and USS
Treatment is by laparotomy, if volvulus is present (or at high risk of occurring then a ladds procedure is performed
Hirschsprung’s disease
Absence of ganglion cells from myenteric and submucosal plexuses
Occurs in 1/5000 births
Full thickness rectal biopsy for diagnosis
Delayed passage of meconium and abdominal distension
Treatment is with rectal washouts initially, thereafter an anorectal pull through procedure
Oesophageal atresia
Associated with tracheo-oesophageal fistula and polyhydramnios
May present with choking and cyanotic spells following aspiration
VACTERL associations
Meconium ileus
Usually delayed passage of meconium and abdominal distension
Majority have cystic fibrosis
X-Rays may not show a fluid level as the meconium is viscid (depends upon feeding), PR contrast studies may dislodge meconium plugs and be therapeutic
Infants who do not respond to PR contrast and NG N-acetyl cysteine will require surgery to remove the plugs
Biliary atresia
Jaundice > 14 days
Increased conjugated bilirubin
Urgent Kasai procedure
Necrotising enterocolitis
Prematurity is the main risk factor
Early features include abdominal distension and passage of bloody stools
X-Rays may show pneumatosis intestinalis and evidence of free air
Increased risk when empirical antibiotics are given to infants beyond 5 days
Treatment is with total gut rest and TPN, babies with perforations will require laparotomy

12
Q

A 1 month old baby girl presents with bile stained vomiting. She has an exomphalos and a congenital diaphragmatic hernia.

A.	Meckel diverticulum
B.	Pyloric stenosis
C.	Acute appendicitis
D.	Mesenteric adenitis
E.	Intussusception
F.	Malrotation
G.	Hirschsprung disease
A

Malrotation

Exomphalos and diaphragmatic herniae are commonly associated with malrotation.

Paediatric Gastrointestinal disorders

Pyloric stenosis
M>F
5-10% Family history in parents
Projectile non bile stained vomiting at 4-6 weeks of life
Diagnosis is made by test feed or USS
Treatment: Ramstedt pyloromyotomy (open or laparoscopic)

Acute appendicitis	
Uncommon under 3 years
When occurs may present atypically
Mesenteric adenitis	
Central abdominal pain and URTI
Conservative management

Intussusception
Telescoping bowel
Proximal to or at the level of, ileocaecal valve
6-9 months age
Colicky pain, diarrhoea and vomiting, sausage shaped mass, red jelly stool.
Treatment: reduction with air insufflation

Malrotation
High caecum at the midline
Feature in exomphalos, congenital diaphragmatic hernia, intrinsic duodenal atresia
May be complicated by development of volvulus, infant with volvulus may have bile stained vomiting
Diagnosis is made by upper GI contrast study and USS
Treatment is by laparotomy, if volvulus is present (or at high risk of occurring then a ladds procedure is performed

Hirschsprung’s disease
Absence of ganglion cells from myenteric and submucosal plexuses
Occurs in 1/5000 births
Full thickness rectal biopsy for diagnosis
Delayed passage of meconium and abdominal distension
Treatment is with rectal washouts initially, thereafter an anorectal pull through procedure

Oesophageal atresia
Associated with tracheo-oesophageal fistula and polyhydramnios
May present with choking and cyanotic spells following aspiration
VACTERL associations

Meconium ileus
Usually delayed passage of meconium and abdominal distension
Majority have cystic fibrosis
X-Rays may not show a fluid level as the meconium is viscid (depends upon feeding), PR contrast studies may dislodge meconium plugs and be therapeutic
Infants who do not respond to PR contrast and NG N-acetyl cysteine will require surgery to remove the plugs

Biliary atresia
Jaundice > 14 days
Increased conjugated bilirubin
Urgent Kasai procedure

Necrotising enterocolitis
Prematurity is the main risk factor
Early features include abdominal distension and passage of bloody stools
X-Rays may show pneumatosis intestinalis and evidence of free air
Increased risk when empirical antibiotics are given to infants beyond 5 days
Treatment is with total gut rest and TPN, babies with perforations will require laparotomy

13
Q

A 6 year old child develops ballooning of the foreskin on micturition and is brought to the clinic by his anxious mother. One examination the foreskin is non retractile but otherwise normal. By which age are 95% of all foreskins retractile

	2 years
	16 years
	8 years
	5 years
	10 years
A

By 16 years of age almost all foreskins should be retractile and if they are not circumcision should be considered at around this time.

Paediatric urology- foreskin disorders
Disorders of the foreskin
At birth and in the neonatal period the normal foreskin is non retractile due to the presence of adhesions between the foreskin and glans. In most cases these will separate spontaneously. By the end of puberty 95% of foreskins can be retracted. In some children the non-retractile foreskin may balloon during micturition. This is a normal variant and requires no specific treatment.

Balanitis This is inflammation of the glans penis. It may occur in both circumcised and non-circumcised individuals.
Posthitis This is inflammation of the foreskin. It may occur as a result of infections such as gonorrhoea and other STD’s. It may also complicate diabetes. Posthitis may progress to phimosis and as this may make cleaning of the glans difficult and allow progression to balanoposthitis.
Paraphimosis Prolonged retraction of the foreskin proximal to the glans may allow oedema to occur. This may then make foreskin manipulation difficult. It can usually be managed by compression to reduce the oedema and replacement of the foreskin. Where this fails a dorsal slit may be required and this followed by delayed circumcision.
Phimosis This is inability to retract the foreskin and may be partial or complete. It may occur secondary to balanoposthitis or balanitis xerotica obliterans. Depending upon the severity and symptoms treatment with circumcision may be required.
Balanitis xerotica obliterans This is a dermatological condition in which scarring of the foreskin occurs leading to phimosis. It is rare below the age of 5 years. Treatment is usually with circumcision.

14
Q

A neonate is noted to have colonic obstruction, what is the most likely cause?

	Imperforate anus
	Meconium plugs
	Colonic atresia
	Anal atresia
	Sigmoid volvulus
A

Cystic fibrosis is the most common cause of meconium plugs and such cases can be treated with intestinal lavage, this should be followed by formal testing for CF.

Paediatric Gastrointestinal disorders
Pyloric stenosis
M>F
5-10% Family history in parents
Projectile non bile stained vomiting at 4-6 weeks of life
Diagnosis is made by test feed or USS
Treatment: Ramstedt pyloromyotomy (open or laparoscopic)

Acute appendicitis
Uncommon under 3 years
When occurs may present atypically

Mesenteric adenitis
Central abdominal pain and URTI
Conservative management

Intussusception
Telescoping bowel
Proximal to or at the level of, ileocaecal valve
6-9 months age
Colicky pain, diarrhoea and vomiting, sausage shaped mass, red jelly stool.
Treatment: reduction with air insufflation

Malrotation
High caecum at the midline
Feature in exomphalos, congenital diaphragmatic hernia, intrinsic duodenal atresia
May be complicated by development of volvulus, infant with volvulus may have bile stained vomiting
Diagnosis is made by upper GI contrast study and USS
Treatment is by laparotomy, if volvulus is present (or at high risk of occurring then a ladds procedure is performed

Hirschsprung’s disease
Absence of ganglion cells from myenteric and submucosal plexuses
Occurs in 1/5000 births
Full thickness rectal biopsy for diagnosis
Delayed passage of meconium and abdominal distension
Treatment is with rectal washouts initially, thereafter an anorectal pull through procedure

Oesophageal atresia
Associated with tracheo-oesophageal fistula and polyhydramnios
May present with choking and cyanotic spells following aspiration
VACTERL associations

Meconium ileus
Usually delayed passage of meconium and abdominal distension
Majority have cystic fibrosis
X-Rays may not show a fluid level as the meconium is viscid (depends upon feeding), PR contrast studies may dislodge meconium plugs and be therapeutic
Infants who do not respond to PR contrast and NG N-acetyl cysteine will require surgery to remove the plugs

Biliary atresia
Jaundice > 14 days
Increased conjugated bilirubin
Urgent Kasai procedure

Necrotising enterocolitis
Prematurity is the main risk factor
Early features include abdominal distension and passage of bloody stools
X-Rays may show pneumatosis intestinalis and evidence of free air
Increased risk when empirical antibiotics are given to infants beyond 5 days
Treatment is with total gut rest and TPN, babies with perforations will require laparotomy

15
Q

A 42 year old teacher presents with an ulcer associated with varicose veins in the long saphenous vein territory. Apart from a DVT 1 year ago, she has no other past medical history.

A. No further management needed
B. Injection sclerotherapy using 0.5% Sodium tetradecyl sulphate
C. Injection sclerotherapy using 5% phenol
D. Long saphenous vein ligation
E. Long saphenous vein stripped to the ankle
F. Long saphenous vein stripped to the knee
G. Doppler scan
H. Duplex scan
I. Endothermal ablation

A

Duplex scan

This patient needs a duplex scan to assess the patency of her deep venous system before surgery can be undertaken. Other indications for duplex scan include recurrent varicose veins or complications.

Chronic venous insufficiency and varicose veins
Wide spectrum of disease ranging from minor cosmetic problem through to ulceration and disability. It is commoner in women than men and is worse during pregnancy. Varicose veins are best considered as being a saccular dilation of veins (WHO). Chronic venous insufficiency is a series of tissue changes which occur in relation to pooling of blood in the extremities with associated venous hypertension occurring as a result of incompetent deep vein valves.

The veins of the lower limb consist of an interconnected network of superficial and deep venous systems. Varices occur because of localised weakness in the vein wall resulting in dilatation and reflux of blood due to non union of valve cusps. Histologically the typical changes include fibrous scar tissue dividing smooth muscle within media in the vessel wall.

Tissue damage in chronic venous insufficiency occurs because of perivascular cytokine leakage resulting in localised tissue damage coupled with impaired lymphatic flow.

Diagnosis
Typical symptoms of varicose veins include:
Cosmetic appearance
Aching
Ankle swelling that worsens as the day progresses
Episodic thrombophlebitis
Bleeding
Itching
Symptoms of chronic venous insufficiency include:
Dependant leg pain
Prominent leg swelling
Oedema extending beyond the ankle
Venous stasis ulcers

The typical venous stasis ulcer is:
Located above the medial malleolus
Indolent appearance with basal granulation tissue
Variable degree of scarring
Non ischaemic edges
Haemosiderin deposition in the gaiter area (and also lipodermatosclerosis).

Differential diagnosis 
Lower limb arterial disease
Marjolins ulcer
Claudication
Spinal stenosis
Swelling due to medical causes e.g. CCF.

Exclusion of these differentials is by means of physical examination and ankle brachial pressure index measurement.

Examination
Assess for dilated short saphenous vein (popliteal fossa) and palpate for saphena varix medial to the femoral artery
Brodie-Trendelenburg test: to assess level of incompetence
Perthes’ walking test: assess if deep venous system competent

Investigation
Doppler exam: if incompetent a biphasic signal due to retrograde flow is detected
Duplex scanning: to ensure patent deep venous system (do if DVT or trauma)

All patients should have a Doppler assessment to assess for venous reflux and should be classified as having uncomplicated varicose veins or varicose veins with associated chronic venous insufficiency. In the history establishing a previous thrombotic event (DVT/ lower limb fracture) is important and patients with such a history and all who have evidence of chronic venous insufficiency should have a duplex scan performed.

Owing to litigation patients with saphenopopliteal incompetence should have a duplex scan performed and the site marked by scan on the day of surgery.

Treatment
Indications for surgery:
Cosmetic: majority
Lipodermatosclerosis causing venous ulceration
Recurrent superficial thrombophlebitis
Bleeding from ruptured varix

Condition Therapy
Minor varicose veins - no complications Reassure/ cosmetic therapy
Symptomatic uncomplicated varicose veins In those without deep venous insufficiency options include; endothermal ablation, foam sclerotherapy, saphenofemoral / popliteal disconnection, stripping and avulsions, compression stockings
Varicose veins with skin changes Therapy as above (if compression minimum is formal class I stockings)
Chronic venous insufficiency or ulcers Class 2-3 compression stockings (ensure no arterial disease).

Application of formal compression stockings (usually class II/III). In patients who have suffered ulceration, compression stockings should be worn long term. Where ulceration is present and established saphenofemoral reflux exists this should be addressed surgically for durable relief of symptoms, either at the outset or following ulcer healing.
Injection sclerotherapy (5% Ethanolamine oleate), foam is increasingly popular, though transient blindness has been reported. Endo venous laser therapy is another minimally invasive option
Sapheno-femoral or sapheno-popliteal ligation, in the case of the LSV; stripping and multiple phlebectomies

Current best practice guidance
In the United Kingdom the National Institute of Clinical Excellence guidance on varicose veins suggests that for patients with symptomatic varicose veins the first line procedure of choice should be endothermal ablation (see reference for more information). Where this is unavailable or unsuitable then foam sclerotherapy should be the second line option. Surgery is currently the third line treatment option.

Trendelenburg procedure (sapheno-femoral junction ligation)
Head tilt 15 degrees and legs abducted
Oblique incision 1cm medial from artery
Tributaries ligated (Superficial circumflex iliac vein, Superficial inferior epigastric vein, Superficial and deep external pudendal vein)
SF junction double ligated
Saphenous vein stripped to level of knee/upper calf. NB increased risk of saphenous neuralgia if stripped more distally

16
Q

A 42 year old accountant presents with thrombophlebitis of a long standing varicosity of the inner thigh. His past medical history is unremarkable apart from a conservatively managed tibial fracture of the ipsilateral limb 10 years ago. Doppler and clinical assessment demonstrate saphenofemoral junction incompetence.

A. No further management needed
B. Injection sclerotherapy using 0.5% Sodium tetradecyl sulphate
C. Injection sclerotherapy using 5% phenol
D. Long saphenous vein ligation
E. Long saphenous vein stripped to the ankle
F. Long saphenous vein stripped to the knee
G. Doppler scan
H. Duplex scan
I. Endothermal ablation

A

Duplex scan

Tibial fractures are a well recognised risk factor for occult lower limb deep venous thrombosis and most surgeons would perform a duplex scan to exclude deep venous insufficiency prior to surgery.

Chronic venous insufficiency and varicose veins
Wide spectrum of disease ranging from minor cosmetic problem through to ulceration and disability. It is commoner in women than men and is worse during pregnancy. Varicose veins are best considered as being a saccular dilation of veins (WHO). Chronic venous insufficiency is a series of tissue changes which occur in relation to pooling of blood in the extremities with associated venous hypertension occurring as a result of incompetent deep vein valves.

The veins of the lower limb consist of an interconnected network of superficial and deep venous systems. Varices occur because of localised weakness in the vein wall resulting in dilatation and reflux of blood due to non union of valve cusps. Histologically the typical changes include fibrous scar tissue dividing smooth muscle within media in the vessel wall.

Tissue damage in chronic venous insufficiency occurs because of perivascular cytokine leakage resulting in localised tissue damage coupled with impaired lymphatic flow.

Diagnosis
Typical symptoms of varicose veins include:
Cosmetic appearance
Aching
Ankle swelling that worsens as the day progresses
Episodic thrombophlebitis
Bleeding
Itching
Symptoms of chronic venous insufficiency include:
Dependant leg pain
Prominent leg swelling
Oedema extending beyond the ankle
Venous stasis ulcers

The typical venous stasis ulcer is:
Located above the medial malleolus
Indolent appearance with basal granulation tissue
Variable degree of scarring
Non ischaemic edges
Haemosiderin deposition in the gaiter area (and also lipodermatosclerosis).

Differential diagnosis 
Lower limb arterial disease
Marjolins ulcer
Claudication
Spinal stenosis
Swelling due to medical causes e.g. CCF.

Exclusion of these differentials is by means of physical examination and ankle brachial pressure index measurement.

Examination
Assess for dilated short saphenous vein (popliteal fossa) and palpate for saphena varix medial to the femoral artery
Brodie-Trendelenburg test: to assess level of incompetence
Perthes’ walking test: assess if deep venous system competent

Investigation
Doppler exam: if incompetent a biphasic signal due to retrograde flow is detected
Duplex scanning: to ensure patent deep venous system (do if DVT or trauma)

All patients should have a Doppler assessment to assess for venous reflux and should be classified as having uncomplicated varicose veins or varicose veins with associated chronic venous insufficiency. In the history establishing a previous thrombotic event (DVT/ lower limb fracture) is important and patients with such a history and all who have evidence of chronic venous insufficiency should have a duplex scan performed.

Owing to litigation patients with saphenopopliteal incompetence should have a duplex scan performed and the site marked by scan on the day of surgery.

Treatment
Indications for surgery:
Cosmetic: majority
Lipodermatosclerosis causing venous ulceration
Recurrent superficial thrombophlebitis
Bleeding from ruptured varix

Condition Therapy
Minor varicose veins - no complications Reassure/ cosmetic therapy
Symptomatic uncomplicated varicose veins In those without deep venous insufficiency options include; endothermal ablation, foam sclerotherapy, saphenofemoral / popliteal disconnection, stripping and avulsions, compression stockings
Varicose veins with skin changes Therapy as above (if compression minimum is formal class I stockings)
Chronic venous insufficiency or ulcers Class 2-3 compression stockings (ensure no arterial disease).

Application of formal compression stockings (usually class II/III). In patients who have suffered ulceration, compression stockings should be worn long term. Where ulceration is present and established saphenofemoral reflux exists this should be addressed surgically for durable relief of symptoms, either at the outset or following ulcer healing.
Injection sclerotherapy (5% Ethanolamine oleate), foam is increasingly popular, though transient blindness has been reported. Endo venous laser therapy is another minimally invasive option
Sapheno-femoral or sapheno-popliteal ligation, in the case of the LSV; stripping and multiple phlebectomies

Current best practice guidance
In the United Kingdom the National Institute of Clinical Excellence guidance on varicose veins suggests that for patients with symptomatic varicose veins the first line procedure of choice should be endothermal ablation (see reference for more information). Where this is unavailable or unsuitable then foam sclerotherapy should be the second line option. Surgery is currently the third line treatment option.

Trendelenburg procedure (sapheno-femoral junction ligation)
Head tilt 15 degrees and legs abducted
Oblique incision 1cm medial from artery
Tributaries ligated (Superficial circumflex iliac vein, Superficial inferior epigastric vein, Superficial and deep external pudendal vein)
SF junction double ligated
Saphenous vein stripped to level of knee/upper calf. NB increased risk of saphenous neuralgia if stripped more distally

17
Q

A 43 year old lady presents with a thigh varicosity in the territory of the long saphenous vein. She underwent endovenous laser therapy 5 years previously. On duplex assessment she has a patent deep venous system and sapheno-femoral junction incompetence.

A. No further management needed
B. Injection sclerotherapy using 0.5% Sodium tetradecyl sulphate
C. Injection sclerotherapy using 5% phenol
D. Long saphenous vein ligation
E. Long saphenous vein stripped to the ankle
F. Long saphenous vein stripped to the knee
G. Doppler scan
H. Duplex scan
I. Endothermal ablation

A

Long saphenous vein stripped to the knee

In the UK, NICE, suggest the use non operative measures such as endothermal ablation for first time varicose veins. Recurrences respond far less favourably and are best managed with surgery.

Chronic venous insufficiency and varicose veins
Wide spectrum of disease ranging from minor cosmetic problem through to ulceration and disability. It is commoner in women than men and is worse during pregnancy. Varicose veins are best considered as being a saccular dilation of veins (WHO). Chronic venous insufficiency is a series of tissue changes which occur in relation to pooling of blood in the extremities with associated venous hypertension occurring as a result of incompetent deep vein valves.

The veins of the lower limb consist of an interconnected network of superficial and deep venous systems. Varices occur because of localised weakness in the vein wall resulting in dilatation and reflux of blood due to non union of valve cusps. Histologically the typical changes include fibrous scar tissue dividing smooth muscle within media in the vessel wall.

Tissue damage in chronic venous insufficiency occurs because of perivascular cytokine leakage resulting in localised tissue damage coupled with impaired lymphatic flow.

Diagnosis
Typical symptoms of varicose veins include:
Cosmetic appearance
Aching
Ankle swelling that worsens as the day progresses
Episodic thrombophlebitis
Bleeding
Itching
Symptoms of chronic venous insufficiency include:
Dependant leg pain
Prominent leg swelling
Oedema extending beyond the ankle
Venous stasis ulcers

The typical venous stasis ulcer is:
Located above the medial malleolus
Indolent appearance with basal granulation tissue
Variable degree of scarring
Non ischaemic edges
Haemosiderin deposition in the gaiter area (and also lipodermatosclerosis).

Differential diagnosis 
Lower limb arterial disease
Marjolins ulcer
Claudication
Spinal stenosis
Swelling due to medical causes e.g. CCF.

Exclusion of these differentials is by means of physical examination and ankle brachial pressure index measurement.

Examination
Assess for dilated short saphenous vein (popliteal fossa) and palpate for saphena varix medial to the femoral artery
Brodie-Trendelenburg test: to assess level of incompetence
Perthes’ walking test: assess if deep venous system competent

Investigation
Doppler exam: if incompetent a biphasic signal due to retrograde flow is detected
Duplex scanning: to ensure patent deep venous system (do if DVT or trauma)

All patients should have a Doppler assessment to assess for venous reflux and should be classified as having uncomplicated varicose veins or varicose veins with associated chronic venous insufficiency. In the history establishing a previous thrombotic event (DVT/ lower limb fracture) is important and patients with such a history and all who have evidence of chronic venous insufficiency should have a duplex scan performed.

Owing to litigation patients with saphenopopliteal incompetence should have a duplex scan performed and the site marked by scan on the day of surgery.

Treatment
Indications for surgery:
Cosmetic: majority
Lipodermatosclerosis causing venous ulceration
Recurrent superficial thrombophlebitis
Bleeding from ruptured varix

Condition Therapy
Minor varicose veins - no complications Reassure/ cosmetic therapy
Symptomatic uncomplicated varicose veins In those without deep venous insufficiency options include; endothermal ablation, foam sclerotherapy, saphenofemoral / popliteal disconnection, stripping and avulsions, compression stockings
Varicose veins with skin changes Therapy as above (if compression minimum is formal class I stockings)
Chronic venous insufficiency or ulcers Class 2-3 compression stockings (ensure no arterial disease).

Application of formal compression stockings (usually class II/III). In patients who have suffered ulceration, compression stockings should be worn long term. Where ulceration is present and established saphenofemoral reflux exists this should be addressed surgically for durable relief of symptoms, either at the outset or following ulcer healing.
Injection sclerotherapy (5% Ethanolamine oleate), foam is increasingly popular, though transient blindness has been reported. Endo venous laser therapy is another minimally invasive option
Sapheno-femoral or sapheno-popliteal ligation, in the case of the LSV; stripping and multiple phlebectomies

Current best practice guidance
In the United Kingdom the National Institute of Clinical Excellence guidance on varicose veins suggests that for patients with symptomatic varicose veins the first line procedure of choice should be endothermal ablation (see reference for more information). Where this is unavailable or unsuitable then foam sclerotherapy should be the second line option. Surgery is currently the third line treatment option.

Trendelenburg procedure (sapheno-femoral junction ligation)
Head tilt 15 degrees and legs abducted
Oblique incision 1cm medial from artery
Tributaries ligated (Superficial circumflex iliac vein, Superficial inferior epigastric vein, Superficial and deep external pudendal vein)
SF junction double ligated
Saphenous vein stripped to level of knee/upper calf. NB increased risk of saphenous neuralgia if stripped more distally

18
Q

An elderly diabetic male presents with a severe deep seated otalgia and a facial nerve palsy, he has completed a course of amoxycillin with no benefit. What is the most likely diagnosis?

	Malignant otitis externa
	Otosclerosis
	Acoustic neuroma
	Meniers disease
	Viral illness
A

A combination of severe otalgia and facial nerve palsy in a diabetic should raise suspicion of malignant otitis externa. This is a condition caused by pseudomonas. It commences as otitis externa and then progresses to involve the temporal bone. Spread of the disease outside the external auditory canal occurs through the fissures of Santorini and the osseocartilaginous junction.

Acute otitis externa Boil in external auditory meatus Acute pain on moving the pinna
Conductive hearing loss if lesion is large
When rupture occurs pus will flow from ear Ear packs may be used
Topical antibiotics
Operative debridement may be needed in severe cases

Chronic otitis externa Chronic combined infection in the external auditory meatus usually combined staphylococcal and fungal infection Chronic discharge from affected ear, hearing loss and severe pain rare Cleansing of the external ear and treatment with antifungal and antibacterial ear drops

Acute suppurative otitis media Viral induced middle ear effusions secondary to eustacian tube dysfunction Most common in children and rare in adults
May present with symptoms elsewhere (e.g. vomiting) in children
Severe pain and sometimes fever
May present with discharge is tympanic rupture occurs Antibiotics (usually amoxycillin)

Chronic suppurative otitis media May occur with or without cholesteatoma
Those without cholesteatoma have a perforation of the pars tensa
Those with cholesteatoma have a perforation of the pars flaccida Those without cholesteatoma may complain of intermittent discharge (non offensive)
Those with cholesteatoma have impaired hearing and foul smelling discharge Simple pars tensa perforations may be managed non operatively or a myringoplasty considered if symptoms troublesome.
Pars flaccida perforations will usually require a radical mastoidectomy

Otosclerosis
Progressive conductive deafness
Secondary to fixation of the stapes in the oval window
Treatment is with stapedectomy and insertion of a prosthesis

Acoustic neuroma
Symptoms of gradually progressive unilateral perceptive deafness and tinnitus
Involvement of the vestibular nerve may cause vertigo
Extension to involve the facial nerve may cause weakness and then paralysis.

Pre auricular sinus
Common congenital condition in which an epithelial defect forms around the external ear
Small sinuses require no treatment
Deeper sinuses may become blocked and develop episodes of infection, they may be closely related to the facial nerve and are challenging to excise

19
Q

An 18 month old boy is brought to the emergency room by his parents. He was found in bed with a nappy filled with dark red blood. He is haemodynamically unstable and requires a blood transfusion. Prior to this episode he was well with no prior medical history. What is the most likely cause?

	Necrotising enterocolitis
	Anal fissure
	Oesophageal varices
	Meckels diverticulum
	Crohns disease
A

Meckels diverticulum is the number one cause of painless massive GI bleeding requiring a transfusion in children between the ages of 1 and 2 years.

Meckel’s diverticulum
Congenital abnormality resulting in incomplete obliteration of the vitello-intestinal duct
Normally, in the foetus, there is an attachment between the vitello-intestinal duct and the yolk sac.This disappears at 6 weeks gestation.
The tip is free in majority of cases.
Associated with enterocystomas, umbilical sinuses, and omphaloileal fistulas.
Arterial supply: omphalomesenteric artery.
2% of population, 2 inches long, 2 feet from the ileocaecal valve.
Typically lined by ileal mucosa but ectopic gastric mucosa can occur, with the risk of peptic ulceration. Pancreatic and jejunal mucosa can also occur.

Clinical
Normally asymptomatic and an incidental finding.
Complications are the result of obstruction, ectopic tissue, or inflammation.
Removal if narrow neck or symptomatic. Options are between wedge excision or formal small bowel resection and anastomosis.

20
Q

Which of the following statements relating to sebaceous cysts is false?

When infected are also known as Cocks peculiar tumour
Typically contain pus
Are usually associated with a central punctum
Most commonly occur on the scalp
They will typically have a cyst wall
A

Sebaceous cysts usually contain sebum, pus is only present in infected sebaceous cysts which should then be treated by surgical incision and drainage.

Sebaceous cysts
Originate from sebaceous glands and contain sebum.
Location: anywhere but most common scalp, ears, back, face, and upper arm (not palms of the hands and soles of the feet).
They will typically contain a punctum.
Excision of the cyst wall needs to be complete to prevent recurrence.
A Cock’s ‘Peculiar’ Tumour is a suppurating and ulcerated sebaceous cyst. It may resemble a squamous cell carcinoma- hence its name.

21
Q

A 42 year old lady who has systemic lupus erythematosus presents to the clinic with a 5 day history of a painful purple lesion on her index finger. On examination she has a tender red lesion on the index finger.

A.	Malignant fibrous histiocytoma
B.	Oslers nodes
C.	Heberdens nodes
D.	Bouchards nodes
E.	Carpal tunnel syndrome
F.	Complex regional pain syndrome
G.	Osteoclastoma
H.	Osteosarcoma
I.	Ganglion
A

Oslers nodes

Osler nodes are normally described as tender, purple/red raised lesions with a pale centre. These lesions occur as a result of immune complex deposition. These occur most often in association with endocarditis. However, other causes include SLE, gonorrhoea, typhoid and haemolytic anaemia.

Dupuytrens contracture
Fixed flexion contracture of the hand where the fingers bend towards the palm and cannot be fully extended.
Caused by underlying contractures of the palmar aponeurosis . The ring finger and little finger are the fingers most commonly affected. The middle finger may be affected in advanced cases, but the index finger and the thumb are nearly always spared.
Progresses slowly and is usually painless. In patients with this condition, the tissues under the skin on the palm of the hand thicken and shorten so that the tendons connected to the fingers cannot move freely. The palmar aponeurosis becomes hyperplastic and undergoes contracture.
Commonest in males over 40 years of age.
Association with liver cirrhosis and alcoholism. However, many cases are idiopathic.
Treatment is surgical and involves fasciectomy. However, the condition may recur and many surgical therapies are associated with risk of neurovascular damage to the digital nerves and arteries.

Carpel tunnel syndrome
Idiopathic median neuropathy at the carpal tunnel.
Characterised by altered sensation of the lateral 3 fingers.
The condition is commoner in females and is associated with other connective tissue disorders such as rheumatoid disease. It may also occur following trauma to the distal radius.
Symptoms occur mainly at night in early stages of the condition.
Examination may demonstrate wasting of the muscles of the thenar eminence and symptoms may be reproduced by Tinels test (compression of the contents of the carpal tunnel).
Formal diagnosis is usually made by electrophysiological studies.
Treatment is by surgical decompression of the carpal tunnel, a procedure achieved by division of the flexor retinaculum. Non - surgical options include splinting and bracing.

Osler’s nodes Osler’s nodes are painful, red, raised lesions found on the hands and feet. They are the result of the deposition of immune complexes.

Bouchards nodes Hard, bony outgrowths or gelatinous cysts on the proximal interphalangeal joints (the middle joints of fingers or toes.) They are a sign of osteoarthritis, and are caused by formation of calcific spurs of the articular cartilage.

Heberdens nodes Typically develop in middle age, beginning either with a chronic swelling of the affected joints or the sudden painful onset of redness, numbness, and loss of manual dexterity. This initial inflammation and pain eventually subsides, and the patient is left with a permanent bony outgrowth that often skews the fingertip sideways. It typically affects the DIP joint.

Ganglion Swelling in association with a tendon sheath commonly near a joint. They are common lesions in the wrist and hand. Usually they are asymptomatic and cause little in the way of functional compromise. They are fluid filled although the fluid is similar to synovial fluid it is slightly more viscous. When the cysts are troublesome they may be excised.

22
Q

A 62 year old lady presents with an non tender lump overlying the distal interphalangeal joint of the index finger. On examination she has a hard, non tender lump overlying the joint and deviation of the tip of the finger.

A.	Malignant fibrous histiocytoma
B.	Oslers nodes
C.	Heberdens nodes
D.	Bouchards nodes
E.	Carpal tunnel syndrome
F.	Complex regional pain syndrome
G.	Osteoclastoma
H.	Osteosarcoma
I.	Ganglion
A

Heberdens nodes

Heberdens nodes may produce swelling of the distal interphalangeal joint with deviation of the finger tip.

Dupuytrens contracture
Fixed flexion contracture of the hand where the fingers bend towards the palm and cannot be fully extended.
Caused by underlying contractures of the palmar aponeurosis . The ring finger and little finger are the fingers most commonly affected. The middle finger may be affected in advanced cases, but the index finger and the thumb are nearly always spared.
Progresses slowly and is usually painless. In patients with this condition, the tissues under the skin on the palm of the hand thicken and shorten so that the tendons connected to the fingers cannot move freely. The palmar aponeurosis becomes hyperplastic and undergoes contracture.
Commonest in males over 40 years of age.
Association with liver cirrhosis and alcoholism. However, many cases are idiopathic.
Treatment is surgical and involves fasciectomy. However, the condition may recur and many surgical therapies are associated with risk of neurovascular damage to the digital nerves and arteries.

Carpel tunnel syndrome
Idiopathic median neuropathy at the carpal tunnel.
Characterised by altered sensation of the lateral 3 fingers.
The condition is commoner in females and is associated with other connective tissue disorders such as rheumatoid disease. It may also occur following trauma to the distal radius.
Symptoms occur mainly at night in early stages of the condition.
Examination may demonstrate wasting of the muscles of the thenar eminence and symptoms may be reproduced by Tinels test (compression of the contents of the carpal tunnel).
Formal diagnosis is usually made by electrophysiological studies.
Treatment is by surgical decompression of the carpal tunnel, a procedure achieved by division of the flexor retinaculum. Non - surgical options include splinting and bracing.

Osler’s nodes Osler’s nodes are painful, red, raised lesions found on the hands and feet. They are the result of the deposition of immune complexes.

Bouchards nodes Hard, bony outgrowths or gelatinous cysts on the proximal interphalangeal joints (the middle joints of fingers or toes.) They are a sign of osteoarthritis, and are caused by formation of calcific spurs of the articular cartilage.

Heberdens nodes Typically develop in middle age, beginning either with a chronic swelling of the affected joints or the sudden painful onset of redness, numbness, and loss of manual dexterity. This initial inflammation and pain eventually subsides, and the patient is left with a permanent bony outgrowth that often skews the fingertip sideways. It typically affects the DIP joint.

Ganglion Swelling in association with a tendon sheath commonly near a joint. They are common lesions in the wrist and hand. Usually they are asymptomatic and cause little in the way of functional compromise. They are fluid filled although the fluid is similar to synovial fluid it is slightly more viscous. When the cysts are troublesome they may be excised.

23
Q

A 17 year old boy is brought to the clinic by his mother who is concerned about a lesion that has developed on the dorsal surface of his left hand. On examination he has a soft fluctuant swelling on the dorsal aspect of the hand, it is most obvious on making a fist.

A.	Malignant fibrous histiocytoma
B.	Oslers nodes
C.	Heberdens nodes
D.	Bouchards nodes
E.	Carpal tunnel syndrome
F.	Complex regional pain syndrome
G.	Osteoclastoma
H.	Osteosarcoma
I.	Ganglion
A

Ganglion

Ganglions commonly occur in the hand and are usually associated with tendons. They are typically soft and fluctuant. They do not require removal unless they are atypical or causing symptoms.

Dupuytrens contracture
Fixed flexion contracture of the hand where the fingers bend towards the palm and cannot be fully extended.
Caused by underlying contractures of the palmar aponeurosis . The ring finger and little finger are the fingers most commonly affected. The middle finger may be affected in advanced cases, but the index finger and the thumb are nearly always spared.
Progresses slowly and is usually painless. In patients with this condition, the tissues under the skin on the palm of the hand thicken and shorten so that the tendons connected to the fingers cannot move freely. The palmar aponeurosis becomes hyperplastic and undergoes contracture.
Commonest in males over 40 years of age.
Association with liver cirrhosis and alcoholism. However, many cases are idiopathic.
Treatment is surgical and involves fasciectomy. However, the condition may recur and many surgical therapies are associated with risk of neurovascular damage to the digital nerves and arteries.

Carpel tunnel syndrome
Idiopathic median neuropathy at the carpal tunnel.
Characterised by altered sensation of the lateral 3 fingers.
The condition is commoner in females and is associated with other connective tissue disorders such as rheumatoid disease. It may also occur following trauma to the distal radius.
Symptoms occur mainly at night in early stages of the condition.
Examination may demonstrate wasting of the muscles of the thenar eminence and symptoms may be reproduced by Tinels test (compression of the contents of the carpal tunnel).
Formal diagnosis is usually made by electrophysiological studies.
Treatment is by surgical decompression of the carpal tunnel, a procedure achieved by division of the flexor retinaculum. Non - surgical options include splinting and bracing.

Osler’s nodes Osler’s nodes are painful, red, raised lesions found on the hands and feet. They are the result of the deposition of immune complexes.

Bouchards nodes Hard, bony outgrowths or gelatinous cysts on the proximal interphalangeal joints (the middle joints of fingers or toes.) They are a sign of osteoarthritis, and are caused by formation of calcific spurs of the articular cartilage.

Heberdens nodes Typically develop in middle age, beginning either with a chronic swelling of the affected joints or the sudden painful onset of redness, numbness, and loss of manual dexterity. This initial inflammation and pain eventually subsides, and the patient is left with a permanent bony outgrowth that often skews the fingertip sideways. It typically affects the DIP joint.

Ganglion Swelling in association with a tendon sheath commonly near a joint. They are common lesions in the wrist and hand. Usually they are asymptomatic and cause little in the way of functional compromise. They are fluid filled although the fluid is similar to synovial fluid it is slightly more viscous. When the cysts are troublesome they may be excised.

24
Q

Which of the following statements relating to Keloid scars is untrue?

They have a predilection for sternal , mandibular and deltiod area wounds
They are confined to the margins of the original injury
They often recur following excision
May occur even after superficial injury
They may be treated by injection of triamcinolone
A

Hypertrophic scars remain confined to the wound edges.

Keloids (by definition) will tend to extend beyond the margins of the wound and in wounds of any depth.

Wound healing
Surgical wounds are either incisional or excisional and either clean, clean contaminated or dirty. Although the stages of wound healing are broadly similar their contributions will vary according to the wound type.

The main stages of wound healing include:

Haemostasis
Minutes to hours following injury
Vasospasm in adjacent vessels, platelet plug formation and generation of fibrin rich clot.

Inflammation
Typically days 1-5
Neutrophils migrate into wound (function impaired in diabetes).
Growth factors released, including basic fibroblast growth factor and vascular endothelial growth factor.
Fibroblasts replicate within the adjacent matrix and migrate into wound.
Macrophages and fibroblasts couple matrix regeneration and clot substitution.

Regeneration
Typically days 7 to 56
Platelet derived growth factor and transformation growth factors stimulate fibroblasts and epithelial cells.
Fibroblasts produce a collagen network.
Angiogenesis occurs and wound resembles granulation tissue.

Remodeling
From 6 weeks to 1 year
Longest phase of the healing process and may last up to one year (or longer).
During this phase fibroblasts become differentiated (myofibroblasts) and these facilitate wound contraction.
Collagen fibres are remodeled.
Microvessels regress leaving a pale scar.

The above description represents an idealised scenario. A number of diseases may distort this process. Neovascularisation is an important early process. Endothelial cells may proliferate in the wound bed and recanalise to form a vessel. Vascular disease, shock and sepsis can all compromise microvascular flow and impair healing.

Conditions such as jaundice will impair fibroblast synthetic function and immunity with a detrimental effect in most parts of the healing process.

Problems with scars:

Hypertrophic scars
Excessive amounts of collagen within a scar. Nodules may be present histologically containing randomly arranged fibrils within and parallel fibres on the surface. The tissue itself is confined to the extent of the wound itself and is usually the result of a full thickness dermal injury. They may go on to develop contractures.

Keloid scars
Excessive amounts of collagen within a scar. Typically a keloid scar will pass beyond the boundaries of the original injury. They do not contain nodules and may occur following even trivial injury. They do not regress over time and may recur following removal.

Drugs which impair wound healing:
Non steroidal anti inflammatory drugs
Steroids
Immunosupressive agents
Anti neoplastic drugs

Closure
Delayed primary closure is the anatomically precise closure that is delayed for a few days but before granulation tissue becomes macroscopically evident.

Secondary closure refers to either spontaneous closure or to surgical closure after granulation tissue has formed.

25
Q

What is the investigation of choice to look for renal scarring in a child with vesicoureteric reflux?

	Abdominal x-ray
	Ultrasound
	DMSA
	CT KUB
	Micturating cystourethrogram
A

DMSA

Vesicoureteric reflux (VUR) is the abnormal backflow of urine from the bladder into the ureter and kidney. It is relatively common abnormality of the urinary tract in children and predisposes to urinary tract infection (UTI), being found in around 30% of children who present with a UTI. As around 35% of children develop renal scarring it is important to investigate for VUR in children following a UTI

Pathophysiology of VUR
ureters are displaced laterally, entering the bladder in a more perpendicular fashion than at an angle
therefore shortened intramural course of ureter
vesicoureteric junction cannot therefore function adequately

The table below summarises the grading of VUR

{Grade}
I Reflux into the ureter only, no dilatation
II Reflux into the renal pelvis on micturition, no dilatation
III Mild/moderate dilatation of the ureter, renal pelvis and calyces
IV Dilation of the renal pelvis and calyces with moderate ureteral tortuosity
V Gross dilatation of the ureter, pelvis and calyces with ureteral tortuosity

Investigation
VUR is normally diagnosed following a micturating cystourethrogram
a DMSA scan may also be performed to look for renal scarring

26
Q

A 24-year-old female is referred to the acute surgical team as she is noted to have an absent left radial pulse. Apart from some dizziness and lethargy, the patient does not have any features suggestive of an acute ischaemic limb. Blood tests are as follows:

Na+ 136 mmol/l
K+ 4.1 mmol/l
Urea 2.3 mmol/l
Creatinine 77 µmol/l

ESR 66 mm/hr

What is the most likely diagnosis?

	Turner's syndrome
	Takayasu's arteritis
	Kawasaki disease
	Coarctation of the aorta
	Breast carcinoma with local spread
A

Takayasu’s arteritis

Inflammatory, obliterative arteritis affecting aorta and branches
Females> Males
Symptoms may include upper limb claudication
Clinical findings include diminished or absent pulses
ESR often affected during the acute phase

Takayasu’s arteritis is a large vessel vasculitis. It typically causes occlusion of the aorta and questions commonly refer to an absent limb pulse. It is more common in females and Asian people

Associations
renal artery stenosis

Management
steroids

27
Q

A 28 year old man is playing tennis when he suddenly collapses and has a GCS of 4 when examined.

A.	Acute sub dural haematoma
B.	Chronic sub dural haematoma
C.	Acute extradural haematoma
D.	Chronic extradural haematoma
E.	Intraventricular haemorrhage
F.	Sub arachnoid haemorrhage
A

Sub arachnoid haemorrhage

A sudden collapse and loss of consciousness is most likely to be due to a sub arachnoid haemorrhage. The other potential causes in the list usually occur as a sequel to a traumatic event, which has not occurred here.

Extradural haematoma Bleeding into the space between the dura mater and the skull. Often results from acceleration-deceleration trauma or a blow to the side of the head. The majority of extradural haematomas occur in the temporal region where skull fractures cause a rupture of the middle meningeal artery.

Features
Raised intracranial pressure
Some patients may exhibit a lucid interval
Subdural haematoma Bleeding into the outermost meningeal layer. Most commonly occur around the frontal and parietal lobes. May be either acute or chronic.

Risk factors include old age and alcoholism.

Slower onset of symptoms than a extradural haematoma.
Intracerebral haematoma Usually hyperdense lesions on CT scanning. Arise in areas of traumatic contusion with fuse to become a haematoma. Areas of clot and fresh blood may co-exist on the same CT scan (Swirl sign). Large haematomas and those associated with mass effect should be evacuated.
Subarachnoid haemorrhage Usually occurs spontaneously in the context of a ruptured cerebral aneurysm but may be seen in association with other injuries when a patient has sustained a traumatic brain injury
Intraventricular haemorrhage Haemorrhage that occurs into the ventricular system of the brain. It is relatively rare in adult surgical practice and when it does occur, it is typically associated with severe head injuries. In premature neonates it may occur spontaneously. The blood may clot and occlude CSF flow, hydrocephalus may result.
In neonatal practice the vast majority of IVH occur in the first 72 hours after birth, the aetiology is not well understood and it is suggested to occur as a result of birth trauma combined with cellular hypoxia, together with the delicate neonatal CNS.

28
Q

A 2 day old premature neonate is born by emergency cesarean section for maternal illness. The baby is noted to become floppy and unresponsive.

A.	Acute sub dural haematoma
B.	Chronic sub dural haematoma
C.	Acute extradural haematoma
D.	Chronic extradural haematoma
E.	Intraventricular haemorrhage
F.	Sub arachnoid haemorrhage
A

Intraventricular haemorrhage

Neonatal deterioration in premature babies is not infrequently due to intra ventricular haemorrhage. In extreme prematurity the prognosis can be very poor.

Extradural haematoma Bleeding into the space between the dura mater and the skull. Often results from acceleration-deceleration trauma or a blow to the side of the head. The majority of extradural haematomas occur in the temporal region where skull fractures cause a rupture of the middle meningeal artery.

Features
Raised intracranial pressure
Some patients may exhibit a lucid interval
Subdural haematoma Bleeding into the outermost meningeal layer. Most commonly occur around the frontal and parietal lobes. May be either acute or chronic.

Risk factors include old age and alcoholism.

Slower onset of symptoms than a extradural haematoma.
Intracerebral haematoma Usually hyperdense lesions on CT scanning. Arise in areas of traumatic contusion with fuse to become a haematoma. Areas of clot and fresh blood may co-exist on the same CT scan (Swirl sign). Large haematomas and those associated with mass effect should be evacuated.
Subarachnoid haemorrhage Usually occurs spontaneously in the context of a ruptured cerebral aneurysm but may be seen in association with other injuries when a patient has sustained a traumatic brain injury
Intraventricular haemorrhage Haemorrhage that occurs into the ventricular system of the brain. It is relatively rare in adult surgical practice and when it does occur, it is typically associated with severe head injuries. In premature neonates it may occur spontaneously. The blood may clot and occlude CSF flow, hydrocephalus may result.
In neonatal practice the vast majority of IVH occur in the first 72 hours after birth, the aetiology is not well understood and it is suggested to occur as a result of birth trauma combined with cellular hypoxia, together with the delicate neonatal CNS.

29
Q

A 78 year old man is brought to the emergency department by the police. He is found wandering around the town centre and is confused. His family report that he is usually well apart from a simple mechanical fall 3 weeks previously from which he sustained no obvious injuries.

A.	Acute sub dural haematoma
B.	Chronic sub dural haematoma
C.	Acute extradural haematoma
D.	Chronic extradural haematoma
E.	Intraventricular haemorrhage
F.	Sub arachnoid haemorrhage
A

Chronic sub dural haematoma

The injuries that are responsible for chronic sub dural haematomas are usually fairly trivial and forgotten by the patient and their families. The onset of symptoms can be insidious with vague symptomatology and confusion predominating.

Extradural haematoma Bleeding into the space between the dura mater and the skull. Often results from acceleration-deceleration trauma or a blow to the side of the head. The majority of extradural haematomas occur in the temporal region where skull fractures cause a rupture of the middle meningeal artery.

Features
Raised intracranial pressure
Some patients may exhibit a lucid interval
Subdural haematoma Bleeding into the outermost meningeal layer. Most commonly occur around the frontal and parietal lobes. May be either acute or chronic.

Risk factors include old age and alcoholism.

Slower onset of symptoms than a extradural haematoma.
Intracerebral haematoma Usually hyperdense lesions on CT scanning. Arise in areas of traumatic contusion with fuse to become a haematoma. Areas of clot and fresh blood may co-exist on the same CT scan (Swirl sign). Large haematomas and those associated with mass effect should be evacuated.
Subarachnoid haemorrhage Usually occurs spontaneously in the context of a ruptured cerebral aneurysm but may be seen in association with other injuries when a patient has sustained a traumatic brain injury
Intraventricular haemorrhage Haemorrhage that occurs into the ventricular system of the brain. It is relatively rare in adult surgical practice and when it does occur, it is typically associated with severe head injuries. In premature neonates it may occur spontaneously. The blood may clot and occlude CSF flow, hydrocephalus may result.
In neonatal practice the vast majority of IVH occur in the first 72 hours after birth, the aetiology is not well understood and it is suggested to occur as a result of birth trauma combined with cellular hypoxia, together with the delicate neonatal CNS.

30
Q

Which of the following statements relating to necrotising enterocolitis is false?

It has a mortality of 30%
Most frequently presents in premature neonates less than 32 weeks gestation.
Should be managed by early laparotomy and segmental resections in most cases.
Pneumostosis intestinalis may be visible on plain abdominal x-ray.
May be minimised by use of breast milk over formula feeds.
A

Most cases will settle with conservative management with NG decompression and appropriate support. Laparotomy should be undertaken in patients who progress despite conservative management or in whom compelling indications for surgery exist (eg free air).

Pyloric stenosis
M>F
5-10% Family history in parents
Projectile non bile stained vomiting at 4-6 weeks of life
Diagnosis is made by test feed or USS
Treatment: Ramstedt pyloromyotomy (open or laparoscopic)

Acute appendicitis
Uncommon under 3 years
When occurs may present atypically

Mesenteric adenitis
Central abdominal pain and URTI
Conservative management

Intussusception
Telescoping bowel
Proximal to or at the level of, ileocaecal valve
6-9 months age
Colicky pain, diarrhoea and vomiting, sausage shaped mass, red jelly stool.
Treatment: reduction with air insufflation

Malrotation
High caecum at the midline
Feature in exomphalos, congenital diaphragmatic hernia, intrinsic duodenal atresia
May be complicated by development of volvulus, infant with volvulus may have bile stained vomiting
Diagnosis is made by upper GI contrast study and USS
Treatment is by laparotomy, if volvulus is present (or at high risk of occurring then a ladds procedure is performed

Hirschsprung’s disease
Absence of ganglion cells from myenteric and submucosal plexuses
Occurs in 1/5000 births
Full thickness rectal biopsy for diagnosis
Delayed passage of meconium and abdominal distension
Treatment is with rectal washouts initially, thereafter an anorectal pull through procedure

Oesophageal atresia
Associated with tracheo-oesophageal fistula and polyhydramnios
May present with choking and cyanotic spells following aspiration
VACTERL associations

Meconium ileus
Usually delayed passage of meconium and abdominal distension
Majority have cystic fibrosis
X-Rays may not show a fluid level as the meconium is viscid (depends upon feeding), PR contrast studies may dislodge meconium plugs and be therapeutic
Infants who do not respond to PR contrast and NG N-acetyl cysteine will require surgery to remove the plugs

Biliary atresia
Jaundice > 14 days
Increased conjugated bilirubin
Urgent Kasai procedure

Necrotising enterocolitis
Prematurity is the main risk factor
Early features include abdominal distension and passage of bloody stools
X-Rays may show pneumatosis intestinalis and evidence of free air
Increased risk when empirical antibiotics are given to infants beyond 5 days
Treatment is with total gut rest and TPN, babies with perforations will require laparotomy

31
Q

A 4 year old child presents with a 4-5 day history of feeling generally unwell and also of having a sore throat. On examination, there is marked cervical lymphadenopathy, the oropharynx is covered with a thick grey membrane which bleeds following attempted removal. What is the most likely diagnosis?

	Acute streptococcal pharyngitis
	Infection with Epstein Barr virus
	Diptheria
	Ludwigs angina
	Ingestion of caustic soda
A

Infection with diphtheria classically causes a systemic illness that lasts several days. The tonsils or pharynx can be covered in a thick grey membrane which bleeds on attempted removal. There is often quite marked cervical adenopathy and some individuals can have a bulls neck appearance. Death can occur through airway compromise, which is why the often described attempted removal of the pseudomembrane so beloved of examiners, is, in practice rather a foolish thing to attempt in a young child!

Acute tonsillitis
Characterised by pharyngitis, fever, malaise and lymphadenopathy.
Over half of all cases are bacterial with Streptococcus pyogenes the most common organism
The tonsils are typically oedematous and yellow or white pustules may be present
Infectious mononucleosis may mimic the condition.
Treatment with penicillin type antibiotics is indicated for bacterial tonsillitis.
Bacterial tonsillitis may result in local abscess formation (quinsy)

32
Q

A 25 year old cyclist is hit by a bus traveling at 30mph. He is not wearing a helmet. He arrives with a GCS of 3/15 and is intubated. A CT scan shows evidence of cerebral contusion but no localising clinical signs are present

A.	Intravenous mannitol
B.	Parietotemporal craniotomy
C.	Burr Hole decompression
D.	Posterior fossa craniotomy
E.	Insertion of intracranial bolt monitor
F.	Discharge
G.	Intravenous frusemide
A

Insertion of intracranial bolt monitor

This patient may well develop raised ICP over the next few days and Intracranial pressure monitoring will help with management.

Head injury management- NICE Guidelines
Summary of guidelines
All patients should be assessed within 15 minutes on arrival to A&E
Document all 3 components of the GCS
If GCS <8 or = to 8, consider stabilising the airway
Treat pain with low dose IV opiates (if safe)
Full spine immobilisation until assessment if:

  • GCS < 15
  • neck pain/tenderness
  • paraesthesia extremities
  • focal neurological deficit
  • suspected c-spine injury

If a c-spine injury is suspected a 3 view c-spine x-ray is indicated. CT c-spine is preferred if:

  • Intubated
  • GCS <13
  • Normal x-ray but continued concerns regarding c-spine injury
  • Any focal neurology
  • A CT head scan is being performed
  • Initial plain films are abnormal
Immediate CT head (within 1 hour) if:
GCS < 13 on admission
GCS < 15 2 hours after admission
Suspected open or depressed skull fracture
Suspected skull base fracture (panda eyes, Battle's sign, CSF from nose/ear, bleeding ear)
Focal neurology
Vomiting > 1 episode
Post traumatic seizure
Coagulopathy
Contact neurosurgeon if:
Persistent GCS < 8 or = 8
Unexplained confusion > 4h
Reduced GCS after admission
Progressive neurological signs
Incomplete recovery post seizure
Penetrating injury
Cerebrospinal fluid leak

Observations
1/2 hourly GCS until 15

33
Q

A 32 year old rugby player is crushed in a scrum. He is briefly concussed but then regains consciousness. He then collapses and is brought to A+E. His GCS on arrival is 6/15 and his left pupil is dilated.

A.	Intravenous mannitol
B.	Parietotemporal craniotomy
C.	Burr Hole decompression
D.	Posterior fossa craniotomy
E.	Insertion of intracranial bolt monitor
F.	Discharge
G.	Intravenous frusemide
A

Parietotemporal craniotomy

This man needs urgent decompression and extradural haematoma is the most likely event, from a lacerated middle meningeal artery. The debate as to whether Burr Holes or craniotomy is the best option continues. Most neurosurgeons would perform a craniotomy. However, rural units and those units without neurosurgical kit facing this emergency may resort to Burr Holes.

Head injury management- NICE Guidelines
Summary of guidelines
All patients should be assessed within 15 minutes on arrival to A&E
Document all 3 components of the GCS
If GCS <8 or = to 8, consider stabilising the airway
Treat pain with low dose IV opiates (if safe)
Full spine immobilisation until assessment if:

  • GCS < 15
  • neck pain/tenderness
  • paraesthesia extremities
  • focal neurological deficit
  • suspected c-spine injury

If a c-spine injury is suspected a 3 view c-spine x-ray is indicated. CT c-spine is preferred if:

  • Intubated
  • GCS <13
  • Normal x-ray but continued concerns regarding c-spine injury
  • Any focal neurology
  • A CT head scan is being performed
  • Initial plain films are abnormal
Immediate CT head (within 1 hour) if:
GCS < 13 on admission
GCS < 15 2 hours after admission
Suspected open or depressed skull fracture
Suspected skull base fracture (panda eyes, Battle's sign, CSF from nose/ear, bleeding ear)
Focal neurology
Vomiting > 1 episode
Post traumatic seizure
Coagulopathy
Contact neurosurgeon if:
Persistent GCS < 8 or = 8
Unexplained confusion > 4h
Reduced GCS after admission
Progressive neurological signs
Incomplete recovery post seizure
Penetrating injury
Cerebrospinal fluid leak

Observations
1/2 hourly GCS until 15

34
Q

A 30 year old women is injured in a skiing accident. She suffers a blow to the occiput and is concussed for 5 minutes. On arrival in A+E she is confused with GCS 10/15. A CT scan shows no evidence of acute bleed or fracture but some evidence of oedema with the beginnings of mass effect

A.	Intravenous mannitol
B.	Parietotemporal craniotomy
C.	Burr Hole decompression
D.	Posterior fossa craniotomy
E.	Insertion of intracranial bolt monitor
F.	Discharge
G.	Intravenous frusemide
A

Intravenous mannitol

This women has raised ICP and mannitol will help reduce this in the acute phase.

Head injury management- NICE Guidelines
Summary of guidelines
All patients should be assessed within 15 minutes on arrival to A&E
Document all 3 components of the GCS
If GCS <8 or = to 8, consider stabilising the airway
Treat pain with low dose IV opiates (if safe)
Full spine immobilisation until assessment if:

  • GCS < 15
  • neck pain/tenderness
  • paraesthesia extremities
  • focal neurological deficit
  • suspected c-spine injury

If a c-spine injury is suspected a 3 view c-spine x-ray is indicated. CT c-spine is preferred if:

  • Intubated
  • GCS <13
  • Normal x-ray but continued concerns regarding c-spine injury
  • Any focal neurology
  • A CT head scan is being performed
  • Initial plain films are abnormal
Immediate CT head (within 1 hour) if:
GCS < 13 on admission
GCS < 15 2 hours after admission
Suspected open or depressed skull fracture
Suspected skull base fracture (panda eyes, Battle's sign, CSF from nose/ear, bleeding ear)
Focal neurology
Vomiting > 1 episode
Post traumatic seizure
Coagulopathy
Contact neurosurgeon if:
Persistent GCS < 8 or = 8
Unexplained confusion > 4h
Reduced GCS after admission
Progressive neurological signs
Incomplete recovery post seizure
Penetrating injury
Cerebrospinal fluid leak

Observations
1/2 hourly GCS until 15

35
Q

A 25-year-old female with a history of bilateral vitreous haemorrhage is referred with bilateral lesions in the cerebellar region. What is the likely diagnosis?

	Neurofibromatosis type I
	Neurofibromatosis type II
	Tuberous sclerosis
	Von Hippel-Lindau syndrome
	Sarcoidosis
A

Retinal and cerebellar haemangiomas are key features of Von Hippel-Lindau syndrome. Retinal haemangiomas are bilateral in 25% of patients and may lead to vitreous haemorrhage

on Hippel-Lindau (VHL) syndrome is an autosomal dominant condition predisposing to neoplasia. It is due to an abnormality in the VHL gene located on short arm of chromosome 3

Features
cerebellar haemangiomas
retinal haemangiomas: vitreous haemorrhage
renal cysts (premalignant)
phaeochromocytoma
extra-renal cysts: epididymal, pancreatic, hepatic
endolymphatic sac tumours

36
Q

The medical team refer a 72 year old lady with a bilateral swollen legs. Deep vein thrombosis has been excluded and there is no response to diuretics. On further questioning, the patient reveals that she was born with the swelling in both of her legs.

A.	Milroy's disease
B.	Meige's disease
C.	Lymphoedema tarda
D.	Filariasis
E.	Tuberculosis
F.	Locally advanced bladder carcinoma
G.	Malaria
A

Milroy’s disease

Milroy’s disease is present from birth and is due to failure of the lymphatic vessels to develop. Note that Meige’s disease develops AFTER birth.

Lymphoedema
Due to impaired lymphatic drainage in the presence of normal capillary function.
Lymphoedema causes the accumulation of protein rich fluid, subdermal fibrosis and dermal thickening.
Characteristically fluid is confined to the epifascial space (skin and subcutaneous tissues); muscle compartments are free of oedema. It involves the foot, unlike other forms of oedema. There may be a ‘buffalo hump’ on the dorsum of the foot and the skin cannot be pinched due to subcutaneous fibrosis.

Causes of lymphoedema

Primary
Congenital < 1 year: sporadic, Milroy’s disease
Onset 1-35 years: sporadic, Meige’s disease
> 35 years: Tarda
Secondary
Bacterial/fungal/parasitic infection (filariasis)
Lymphatic malignancy
Radiotherapy to lymph nodes
Surgical resection of lymph nodes
DVT
Thrombophlebitis

Indications for surgery
Marked disability or deformity from limb swelling
Lymphoedema caused by proximal lymphatic obstruction with patent distal lymphatics suitable for a lymphatic drainage procedure
Lymphocutaneous fistulae and megalymphatics

Procedures
Homans operation Reduction procedure with preservation of overlying skin (which must be in good condition). Skin flaps are raised and the underlying tissue excised. Limb circumference typically reduced by a third.
Charles operation All skin and subcutaneous tissue around the calf are excised down to the deep fascia. Split skin grafts are placed over the site. May be performed if overlying skin is not in good condition. Larger reduction in size than with Homans procedure.
Lymphovenous anastamosis Identifiable lymphatics are anastomosed to sub dermal venules. Usually indicated in 2% of patients with proximal lymphatic obstruction and normal distal lymphatics.

37
Q

A 52 year old woman presents with rapid swelling of the left leg. The swelling is greater in the thigh compared to the calf.

A.	Milroy's disease
B.	Meige's disease
C.	Lymphoedema tarda
D.	Filariasis
E.	Tuberculosis
F.	Locally advanced bladder carcinoma
G.	Malaria
A

Locally advanced bladder carcinoma

Always consider a malignancy in an older adult with new lymphoedema in a limb, especially if the swelling is greater proximally than distally. If malignancy is excluded consider the diagnosis of lymphoedema tarda.

Lymphoedema
Due to impaired lymphatic drainage in the presence of normal capillary function.
Lymphoedema causes the accumulation of protein rich fluid, subdermal fibrosis and dermal thickening.
Characteristically fluid is confined to the epifascial space (skin and subcutaneous tissues); muscle compartments are free of oedema. It involves the foot, unlike other forms of oedema. There may be a ‘buffalo hump’ on the dorsum of the foot and the skin cannot be pinched due to subcutaneous fibrosis.

Causes of lymphoedema

Primary
Congenital < 1 year: sporadic, Milroy’s disease
Onset 1-35 years: sporadic, Meige’s disease
> 35 years: Tarda
Secondary
Bacterial/fungal/parasitic infection (filariasis)
Lymphatic malignancy
Radiotherapy to lymph nodes
Surgical resection of lymph nodes
DVT
Thrombophlebitis

Indications for surgery
Marked disability or deformity from limb swelling
Lymphoedema caused by proximal lymphatic obstruction with patent distal lymphatics suitable for a lymphatic drainage procedure
Lymphocutaneous fistulae and megalymphatics

Procedures
Homans operation Reduction procedure with preservation of overlying skin (which must be in good condition). Skin flaps are raised and the underlying tissue excised. Limb circumference typically reduced by a third.
Charles operation All skin and subcutaneous tissue around the calf are excised down to the deep fascia. Split skin grafts are placed over the site. May be performed if overlying skin is not in good condition. Larger reduction in size than with Homans procedure.
Lymphovenous anastamosis Identifiable lymphatics are anastomosed to sub dermal venules. Usually indicated in 2% of patients with proximal lymphatic obstruction and normal distal lymphatics.

38
Q

A 34 year old African teacher attends A&E with a swollen leg. She has been in England for 2 weeks. She lives in an area prevalent with mosquitoes and where there is poor sanitation.

A.	Milroy's disease
B.	Meige's disease
C.	Lymphoedema tarda
D.	Filariasis
E.	Tuberculosis
F.	Locally advanced bladder carcinoma
G.	Malaria
A

Filariasis

Filariasis is caused by the nematode Wuchereria bancrofti, which is mainly spread by mosquito. The oedema can be gross leading to elephantitis. Treatment is with diethylcarbamazine.

Lymphoedema
Due to impaired lymphatic drainage in the presence of normal capillary function.
Lymphoedema causes the accumulation of protein rich fluid, subdermal fibrosis and dermal thickening.
Characteristically fluid is confined to the epifascial space (skin and subcutaneous tissues); muscle compartments are free of oedema. It involves the foot, unlike other forms of oedema. There may be a ‘buffalo hump’ on the dorsum of the foot and the skin cannot be pinched due to subcutaneous fibrosis.

Causes of lymphoedema

Primary
Congenital < 1 year: sporadic, Milroy’s disease
Onset 1-35 years: sporadic, Meige’s disease
> 35 years: Tarda
Secondary
Bacterial/fungal/parasitic infection (filariasis)
Lymphatic malignancy
Radiotherapy to lymph nodes
Surgical resection of lymph nodes
DVT
Thrombophlebitis

Indications for surgery
Marked disability or deformity from limb swelling
Lymphoedema caused by proximal lymphatic obstruction with patent distal lymphatics suitable for a lymphatic drainage procedure
Lymphocutaneous fistulae and megalymphatics

Procedures
Homans operation Reduction procedure with preservation of overlying skin (which must be in good condition). Skin flaps are raised and the underlying tissue excised. Limb circumference typically reduced by a third.
Charles operation All skin and subcutaneous tissue around the calf are excised down to the deep fascia. Split skin grafts are placed over the site. May be performed if overlying skin is not in good condition. Larger reduction in size than with Homans procedure.
Lymphovenous anastamosis Identifiable lymphatics are anastomosed to sub dermal venules. Usually indicated in 2% of patients with proximal lymphatic obstruction and normal distal lymphatics.

39
Q

A 21 year old post man notices leg pain after 5 minutes walking during his round. It improves 3 minutes after stopping. Clinically he is noted to have reduced hair of the lower limbs and his calf muscles appear atrophied. There is a weak popliteal pulse, but it is still present when the knee is fully extended. What is the most likely diagnosis?

	Occlusive arterial disease caused by atherosclerosis
	Popliteal fossa entrapment
	Cerebral vascular accident
	Diabetes mellitus
	Adductor canal compression syndrome
A

Adductor canal compression syndrome most commonly presents in young males and is an important differential diagnosis in men presenting with symptoms of acute limb ischaemia on exertion. It is caused by compression of the femoral artery by the musculotendinous band from adductor magnus muscle.
The treatment consists of the division of the abnormal band and restoration of the arterial circulation. Popliteal fossa entrapment is the main differential diagnosis, however the pulse disappears when the knee is fully extended.

Adductor canal
Also called Hunter’s or subsartorial canal
Immediately distal to the apex of the femoral triangle, lying in the middle third of the thigh. Canal terminates at the adductor hiatus.

Borders Contents
Laterally Vastus medialis muscle Saphenous nerve
Posteriorly Adductor longus, adductor magnus Superficial femoral artery
Roof Sartorius Superficial femoral vein

40
Q

A 18 year old boy is involved in a fall from a balcony whilst intoxicated. He has bruising over the mastoid area and is unconscious.

A.	Acute sub dural haematoma
B.	Intra cerebral haematoma
C.	Extra dural haematoma
D.	Chronic sub dural haematoma
E.	Basal skull fracture
F.	Subarachnoid haemorrhage
G.	Diffuse axonal injury
A

Basal skull fracture

Bruising over the mastoid process of the temporal bone is battle’s sign caused by a basal skull fracture

Head injury
Patients who suffer head injuries should be managed according to ATLS principles and extra cranial injuries should be managed alongside cranial trauma. Inadequate cardiac output will compromise CNS perfusion irrespective of the nature of the cranial injury.

Types of traumatic brain injury
Extradural haematoma Bleeding into the space between the dura mater and the skull. Often results from acceleration-deceleration trauma or a blow to the side of the head. The majority of extradural haematomas occur in the temporal region where skull fractures cause a rupture of the middle meningeal artery.

Features
Raised intracranial pressure
Some patients may exhibit a lucid interval
Subdural haematoma Bleeding into the outermost meningeal layer. Most commonly occur around the frontal and parietal lobes. May be either acute or chronic.

Risk factors include old age and alcoholism.

Slower onset of symptoms than a extradural haematoma.
Subarachnoid haemorrhage Usually occurs spontaneously in the context of a ruptured cerebral aneurysm, but may be seen in association with other injuries when a patient has sustained a traumatic brain injury.

Pathophysiology
Primary brain injury may be focal (contusion/ haematoma) or diffuse (diffuse axonal injury)
Diffuse axonal injury occurs as a result of mechanical shearing following deceleration, causing disruption and tearing of axons
Intra-cranial haematomas can be extradural, subdural or intracerebral, while contusions may occur adjacent to (coup) or contralateral (contre-coup) to the side of impact
Secondary brain injury occurs when cerebral oedema, ischaemia, infection, tonsillar or tentorial herniation exacerbates the original injury. The normal cerebral auto regulatory processes are disrupted following trauma rendering the brain more susceptible to blood flow changes and hypoxia
The Cushings reflex (hypertension and bradycardia) often occurs late and is usually a pre terminal event

Management
Where there is life threatening rising ICP such as in extra dural haematoma and whilst theatre is prepared or transfer arranged use of IV mannitol/ frusemide may be required.
Diffuse cerebral oedema may require decompressive craniotomy
Exploratory Burr Holes have little management in modern practice except where scanning may be unavailable and to thus facilitate creation of formal craniotomy flap
Depressed skull fractures that are open require formal surgical reduction and debridement, closed injuries may be managed non operatively if there is minimal displacement.
ICP monitoring is appropriate in those who have GCS 3-8 and normal CT scan.
ICP monitoring is mandatory in those who have GCS 3-8 and abnormal CT scan.
Hyponatraemia is most likely to be due to syndrome of inappropriate ADH secretion.
Minimum of cerebral perfusion pressure of 70mmHg in adults.
Minimum cerebral perfusion pressure of between 40 and 70 mmHg in children.

41
Q

A 18 year old boy falls off a balcony and hits the right side of the head. He is admitted to the emergency department and has a GCS of 8. He is admitted for observation, and over the following twelve hours develops an increasing headache and confusion. A CT scan shows a crescent shaped collection of fluid between the brain and the dura with associated midline shift.

A.	Acute sub dural haematoma
B.	Intra cerebral haematoma
C.	Extra dural haematoma
D.	Chronic sub dural haematoma
E.	Basal skull fracture
F.	Subarachnoid haemorrhage
G.	Diffuse axonal injury
A

Acute sub dural haematoma

Sub dural haematomas are the commonest intracranial mass lesions resulting from trauma. They are classified as acute, sub acute or chronic according to tempo of onset. Acute sub dural haematomas will present within 72 hours of the original injury and have hyperdense, crescent shaped appearances on CT scanning.

Head injury
Patients who suffer head injuries should be managed according to ATLS principles and extra cranial injuries should be managed alongside cranial trauma. Inadequate cardiac output will compromise CNS perfusion irrespective of the nature of the cranial injury.

Types of traumatic brain injury
Extradural haematoma Bleeding into the space between the dura mater and the skull. Often results from acceleration-deceleration trauma or a blow to the side of the head. The majority of extradural haematomas occur in the temporal region where skull fractures cause a rupture of the middle meningeal artery.

Features
Raised intracranial pressure
Some patients may exhibit a lucid interval
Subdural haematoma Bleeding into the outermost meningeal layer. Most commonly occur around the frontal and parietal lobes. May be either acute or chronic.

Risk factors include old age and alcoholism.

Slower onset of symptoms than a extradural haematoma.
Subarachnoid haemorrhage Usually occurs spontaneously in the context of a ruptured cerebral aneurysm, but may be seen in association with other injuries when a patient has sustained a traumatic brain injury.

Pathophysiology
Primary brain injury may be focal (contusion/ haematoma) or diffuse (diffuse axonal injury)
Diffuse axonal injury occurs as a result of mechanical shearing following deceleration, causing disruption and tearing of axons
Intra-cranial haematomas can be extradural, subdural or intracerebral, while contusions may occur adjacent to (coup) or contralateral (contre-coup) to the side of impact
Secondary brain injury occurs when cerebral oedema, ischaemia, infection, tonsillar or tentorial herniation exacerbates the original injury. The normal cerebral auto regulatory processes are disrupted following trauma rendering the brain more susceptible to blood flow changes and hypoxia
The Cushings reflex (hypertension and bradycardia) often occurs late and is usually a pre terminal event

Management
Where there is life threatening rising ICP such as in extra dural haematoma and whilst theatre is prepared or transfer arranged use of IV mannitol/ frusemide may be required.
Diffuse cerebral oedema may require decompressive craniotomy
Exploratory Burr Holes have little management in modern practice except where scanning may be unavailable and to thus facilitate creation of formal craniotomy flap
Depressed skull fractures that are open require formal surgical reduction and debridement, closed injuries may be managed non operatively if there is minimal displacement.
ICP monitoring is appropriate in those who have GCS 3-8 and normal CT scan.
ICP monitoring is mandatory in those who have GCS 3-8 and abnormal CT scan.
Hyponatraemia is most likely to be due to syndrome of inappropriate ADH secretion.
Minimum of cerebral perfusion pressure of 70mmHg in adults.
Minimum cerebral perfusion pressure of between 40 and 70 mmHg in children.

42
Q

A baby is brought to casualty unconscious and in a vegetative state. She has cigarette burns on her legs.

A.	Acute sub dural haematoma
B.	Intra cerebral haematoma
C.	Extra dural haematoma
D.	Chronic sub dural haematoma
E.	Basal skull fracture
F.	Subarachnoid haemorrhage
G.	Diffuse axonal injury
A

Diffuse axonal injury

The baby is likely to be a victim of shaken baby syndrome. This may result in diffuse axonal injury causing extensive lesions in the white matter.

Head injury
Patients who suffer head injuries should be managed according to ATLS principles and extra cranial injuries should be managed alongside cranial trauma. Inadequate cardiac output will compromise CNS perfusion irrespective of the nature of the cranial injury.

Types of traumatic brain injury
Extradural haematoma Bleeding into the space between the dura mater and the skull. Often results from acceleration-deceleration trauma or a blow to the side of the head. The majority of extradural haematomas occur in the temporal region where skull fractures cause a rupture of the middle meningeal artery.

Features
Raised intracranial pressure
Some patients may exhibit a lucid interval
Subdural haematoma Bleeding into the outermost meningeal layer. Most commonly occur around the frontal and parietal lobes. May be either acute or chronic.

Risk factors include old age and alcoholism.

Slower onset of symptoms than a extradural haematoma.
Subarachnoid haemorrhage Usually occurs spontaneously in the context of a ruptured cerebral aneurysm, but may be seen in association with other injuries when a patient has sustained a traumatic brain injury.

Pathophysiology
Primary brain injury may be focal (contusion/ haematoma) or diffuse (diffuse axonal injury)
Diffuse axonal injury occurs as a result of mechanical shearing following deceleration, causing disruption and tearing of axons
Intra-cranial haematomas can be extradural, subdural or intracerebral, while contusions may occur adjacent to (coup) or contralateral (contre-coup) to the side of impact
Secondary brain injury occurs when cerebral oedema, ischaemia, infection, tonsillar or tentorial herniation exacerbates the original injury. The normal cerebral auto regulatory processes are disrupted following trauma rendering the brain more susceptible to blood flow changes and hypoxia
The Cushings reflex (hypertension and bradycardia) often occurs late and is usually a pre terminal event

Management
Where there is life threatening rising ICP such as in extra dural haematoma and whilst theatre is prepared or transfer arranged use of IV mannitol/ frusemide may be required.
Diffuse cerebral oedema may require decompressive craniotomy
Exploratory Burr Holes have little management in modern practice except where scanning may be unavailable and to thus facilitate creation of formal craniotomy flap
Depressed skull fractures that are open require formal surgical reduction and debridement, closed injuries may be managed non operatively if there is minimal displacement.
ICP monitoring is appropriate in those who have GCS 3-8 and normal CT scan.
ICP monitoring is mandatory in those who have GCS 3-8 and abnormal CT scan.
Hyponatraemia is most likely to be due to syndrome of inappropriate ADH secretion.
Minimum of cerebral perfusion pressure of 70mmHg in adults.
Minimum cerebral perfusion pressure of between 40 and 70 mmHg in children.

43
Q

A 63 year old man presents with a non healing ulcer on his foot. His ABPI measurements are 0.35. A duplex scan shows a 4 cm 90% stenotic lesion of the superficial femoral artery with no distal disease.

A. Femoro-above knee popliteal bypass graft using PTFE
B. Femoro-above knee popliteal bypass graft using long saphenous vein
C. Femoro-distal bypass graft using PTFE
D. Femoro-distal bypass graft using PTFE with Miller Cuff
E. Femoro-distal bypass graft using long saphenous vein
F. Above knee amputation
G. Below knee amputation
H. Angioplasty

A

Angioplasty

Short occlusions are generally reasonable candidates for primary attempts at angioplasty.

Indications for surgery to revascularise the lower limb
Intermittent claudication
Critical ischaemia
Ulceration
Gangrene

Intermittent claudication that is not disabling may provide a relative indication, whilst the other complaints are often absolute indications depending upon the frailty of the patient.

Assessment
Clinical examination
Ankle brachial pressure index measurement
Duplex arterial ultrasound
Angiography (standard, CT or MRI): usually performed only if intervention being considered.

Angioplasty
In order for angioplasty to be undertaken successfully the artery has to be accessible. The lesion relatively short and reasonable distal vessel runoff. Longer lesions may be amenable to sub-intimal angioplasty.

Surgery
Surgery will be undertaken where attempts at angioplasty have either failed or are unsuitable. Bypass essentially involves bypassing the affected arterial segment by utilising a graft to run from above the disease to below the disease. As with angioplasty good runoff improves the outcome.

Some key concepts with bypass surgery

Superficial femoral artery occlusion to the above knee popliteal
In the ideal scenario, vein (either in situ or reversed LSV) would the used as a conduit. However, prosthetic material has reasonable 5 year patency rates and some would advocate using this in preference to vein so that vein can be used for other procedures in the future. In general terms either technique is usually associated with an excellent outcome (if run off satisfactory).

Procedure
Artery dissected out, IV heparin 3,000 units given and then the vessels are cross clamped
Longitudinal arteriotomy
Graft cut to size and tunneled to arteriotomy sites
Anastomosis to femoral artery usually with 5/0 ‘double ended’ Prolene suture
Distal anastomosis usually using 6/0 ‘double ended’ Prolene

Distal disease
Femoro-distal bypass surgery takes longer to perform, is more technically challenging and has higher failure rates.
In elderly diabetic patients with poor runoff a primary amputation may well be a safer and more effective option. There is no point in embarking on this type of surgery in patients who are wheelchair bound.
In femorodistal bypasses vein gives superior outcomes to PTFE.

Rules
Vein mapping 1st to see whether there is suitable vein (the preferred conduit). Sub intimal hyperplasia occurs early when PTFE is used for the distal anastomosis and will lead to early graft occlusion and failure.
Essential operative procedure as for above knee fem-pop.
If there is insufficient vein for the entire conduit then vein can be attached to the end of the PTFE graft and then used for the distal anastomosis. This type of ‘vein boot’ is technically referred to as a Miller Cuff and is associated with better patency rates than PTFE alone.
Remember the more distal the arterial anastomosis the lower the success rate.

44
Q

A 63 year old man who previously smoked 20 cigarettes a day and has newly diagnosed type II diabetes. He presents with rest pain. A diagnostic angiogram demonstrates occlusion of the distal superficial femoral artery continuing below the knee. He has reasonable posterior tibial artery below this level which branches to give good runoff to the foot. He has varicose veins.

A. Femoro-above knee popliteal bypass graft using PTFE
B. Femoro-above knee popliteal bypass graft using long saphenous vein
C. Femoro-distal bypass graft using PTFE
D. Femoro-distal bypass graft using PTFE with Miller Cuff
E. Femoro-distal bypass graft using long saphenous vein
F. Above knee amputation
G. Below knee amputation
H. Angioplasty

A

Femoro-distal bypass graft using PTFE with Miller Cuff

This man needs a bypass operation. Using PTFE alone will not give a good result as sub intimal hyperplasia will give poor outcome early. Using a vein cuff (Miller cuff) at the end of a PTFE graft will improve the situation.

Indications for surgery to revascularise the lower limb
Intermittent claudication
Critical ischaemia
Ulceration
Gangrene

Intermittent claudication that is not disabling may provide a relative indication, whilst the other complaints are often absolute indications depending upon the frailty of the patient.

Assessment
Clinical examination
Ankle brachial pressure index measurement
Duplex arterial ultrasound
Angiography (standard, CT or MRI): usually performed only if intervention being considered.

Angioplasty
In order for angioplasty to be undertaken successfully the artery has to be accessible. The lesion relatively short and reasonable distal vessel runoff. Longer lesions may be amenable to sub-intimal angioplasty.

Surgery
Surgery will be undertaken where attempts at angioplasty have either failed or are unsuitable. Bypass essentially involves bypassing the affected arterial segment by utilising a graft to run from above the disease to below the disease. As with angioplasty good runoff improves the outcome.

Some key concepts with bypass surgery

Superficial femoral artery occlusion to the above knee popliteal
In the ideal scenario, vein (either in situ or reversed LSV) would the used as a conduit. However, prosthetic material has reasonable 5 year patency rates and some would advocate using this in preference to vein so that vein can be used for other procedures in the future. In general terms either technique is usually associated with an excellent outcome (if run off satisfactory).

Procedure
Artery dissected out, IV heparin 3,000 units given and then the vessels are cross clamped
Longitudinal arteriotomy
Graft cut to size and tunneled to arteriotomy sites
Anastomosis to femoral artery usually with 5/0 ‘double ended’ Prolene suture
Distal anastomosis usually using 6/0 ‘double ended’ Prolene

Distal disease
Femoro-distal bypass surgery takes longer to perform, is more technically challenging and has higher failure rates.
In elderly diabetic patients with poor runoff a primary amputation may well be a safer and more effective option. There is no point in embarking on this type of surgery in patients who are wheelchair bound.
In femorodistal bypasses vein gives superior outcomes to PTFE.

Rules
Vein mapping 1st to see whether there is suitable vein (the preferred conduit). Sub intimal hyperplasia occurs early when PTFE is used for the distal anastomosis and will lead to early graft occlusion and failure.
Essential operative procedure as for above knee fem-pop.
If there is insufficient vein for the entire conduit then vein can be attached to the end of the PTFE graft and then used for the distal anastomosis. This type of ‘vein boot’ is technically referred to as a Miller Cuff and is associated with better patency rates than PTFE alone.
Remember the more distal the arterial anastomosis the lower the success rate.

45
Q

A wheelchair bound 78 year old women with ischaemic heart disease secondary to long smoking history and longstanding type II diabetes presents with rest pain and a non healing ulcer on the dorsum of her foot. Angiogram shows reasonable superficial femoral artery and iliacs. At the level of the popliteal there is an occlusion. Below this there is a short area of patent posterior tibial and this reconstitutes lower down the leg to flow to the foot.

A. Femoro-above knee popliteal bypass graft using PTFE
B. Femoro-above knee popliteal bypass graft using long saphenous vein
C. Femoro-distal bypass graft using PTFE
D. Femoro-distal bypass graft using PTFE with Miller Cuff
E. Femoro-distal bypass graft using long saphenous vein
F. Above knee amputation
G. Below knee amputation
H. Angioplasty

A

Above knee amputation

A femoro-distal bypass graft would carry a high risk of failure and risk of peri-operative myocardial infarct. This lady would be well suited to primary amputation.

Indications for surgery to revascularise the lower limb
Intermittent claudication
Critical ischaemia
Ulceration
Gangrene

Intermittent claudication that is not disabling may provide a relative indication, whilst the other complaints are often absolute indications depending upon the frailty of the patient.

Assessment
Clinical examination
Ankle brachial pressure index measurement
Duplex arterial ultrasound
Angiography (standard, CT or MRI): usually performed only if intervention being considered.

Angioplasty
In order for angioplasty to be undertaken successfully the artery has to be accessible. The lesion relatively short and reasonable distal vessel runoff. Longer lesions may be amenable to sub-intimal angioplasty.

Surgery
Surgery will be undertaken where attempts at angioplasty have either failed or are unsuitable. Bypass essentially involves bypassing the affected arterial segment by utilising a graft to run from above the disease to below the disease. As with angioplasty good runoff improves the outcome.

Some key concepts with bypass surgery

Superficial femoral artery occlusion to the above knee popliteal
In the ideal scenario, vein (either in situ or reversed LSV) would the used as a conduit. However, prosthetic material has reasonable 5 year patency rates and some would advocate using this in preference to vein so that vein can be used for other procedures in the future. In general terms either technique is usually associated with an excellent outcome (if run off satisfactory).

Procedure
Artery dissected out, IV heparin 3,000 units given and then the vessels are cross clamped
Longitudinal arteriotomy
Graft cut to size and tunneled to arteriotomy sites
Anastomosis to femoral artery usually with 5/0 ‘double ended’ Prolene suture
Distal anastomosis usually using 6/0 ‘double ended’ Prolene

Distal disease
Femoro-distal bypass surgery takes longer to perform, is more technically challenging and has higher failure rates.
In elderly diabetic patients with poor runoff a primary amputation may well be a safer and more effective option. There is no point in embarking on this type of surgery in patients who are wheelchair bound.
In femorodistal bypasses vein gives superior outcomes to PTFE.

Rules
Vein mapping 1st to see whether there is suitable vein (the preferred conduit). Sub intimal hyperplasia occurs early when PTFE is used for the distal anastomosis and will lead to early graft occlusion and failure.
Essential operative procedure as for above knee fem-pop.
If there is insufficient vein for the entire conduit then vein can be attached to the end of the PTFE graft and then used for the distal anastomosis. This type of ‘vein boot’ is technically referred to as a Miller Cuff and is associated with better patency rates than PTFE alone.
Remember the more distal the arterial anastomosis the lower the success rate.

46
Q

Which of the following does not cause parotid enlargement?

	Liver cirrhosis
	Myxoedema
	Amiodarone
	Tuberculosis
	Sjogrens syndrome
A

Drugs commonly implicated in parotid gland enlargement include:
Thiouracil, isoprenaline, phenylbutazone, high oestrogen dose contraceptive pills.

Parotid gland clinical
Benign neoplasms
Up to 80% of all salivary gland tumours occur in the parotid gland and up to 80% of these are benign. There is no consistent correlation between the rate of growth and the malignant potential of the lesion. However, benign tumours should not invade structures such as the facial nerve.
With the exception of Warthins tumours, they are commoner in women than men. The median age of developing a lesion is in the 5th decade of life.

Benign tumour types

Benign pleomorphic adenoma or benign mixed tumor Most common parotid neoplasm (80%)
Proliferation of epithelial and myoepithelial cells of the ducts and an increase in stromal components
Slow growing, lobular, and not well encapsulated
Recurrence rate of 1-5% with appropriate excision (parotidectomy)
Recurrence possibly secondary to capsular disruption during surgery
Malignant degeneration occurring in 2-10% of adenomas observed for long periods, with carcinoma ex-pleomorphic adenoma occurring most frequently as adenocarcinoma

Warthin tumor (papillary cystadenoma lymphoma or adenolymphoma) Second most common benign parotid tumor (5%)
Most common bilateral benign neoplasm of the parotid
Marked male as compared to female predominance
Occurs later in life (sixth and seventh decades)
Presents as a lymphocytic infiltrate and cystic epithelial proliferation
May represent heterotopic salivary gland epithelial tissue trapped within intraparotid lymph nodes
Incidence of bilaterality and multicentricity of 10%
Malignant transformation rare (almost unheard of)

Monomorphic adenoma Account for less than 5% of tumours
Slow growing
Consist of only one morphological cell type (hence term mono)
Include; basal cell adenoma, canalicular adenoma, oncocytoma, myoepitheliomas

Haemangioma Should be considered in the differential of a parotid mass in a child
Accounts for 90% of parotid tumours in children less than 1 year of age
Hypervascular on imaging
Spontaneous regression may occur and malignant transformation is almost unheard of

Malignant salivary gland tumours

Mucoepidermoid carcinoma 30% of all parotid malignancies
Usually low potential for local invasiveness and metastasis (depends mainly on grade)

Adenoid cystic carcinoma Unpredictable growth pattern
Tendency for perineural spread
Nerve growth may display skip lesions resulting in incomplete excision
Distant metastasis more common (visceral rather than nodal spread)
5 year survival 35%
Mixed tumours Often a malignancy occurring in a previously benign parotid lesion

Acinic cell carcinoma Intermediate grade malignancy
May show perineural invasion
Low potential for distant metastasis
5 year survival 80%

Adenocarcinoma Develops from secretory portion of gland
Risk of regional nodal and distant metastasis
5 year survival depends upon stage at presentation, may be up to 75% with small lesions with no nodal involvement

Lymphoma Large rubbery lesion, may occur in association with Warthins tumours
Diagnosis should be based on regional nodal biopsy rather than parotid resection
Treatment is with chemotherapy (and radiotherapy)

Diagnostic evaluation
Plain x-rays may be used to exclude calculi
Sialography may be used to delineate ductal anatomy
FNAC is used in most cases
Superficial parotidectomy may be either diagnostic of therapeutic depending upon the nature of the lesion
Where malignancy is suspected the primary approach should be definitive resection rather than excisional biopsy
CT/ MRI may be used in cases of malignancy for staging primary disease

Treatment
For nearly all lesions this consists of surgical resection, for benign disease this will usually consist of a superficial parotidectomy. For malignant disease a radical or extended radical parotidectomy is performed. The facial nerve is included in the resection if involved. The need for neck dissection is determined by the potential for nodal involvement.

Other parotid disorders
HIV infection
Lymphoepithelial cysts associated with HIV occur almost exclusively in the parotid
Typically presents as bilateral, multicystic, symmetrical swelling
Risk of malignant transformation is low and management usually conservative

Sjogren syndrome
Autoimmune disorder characterised by parotid enlargement, xerostomia and keratoconjunctivitis sicca
90% of cases occur in females
Second most common connective tissue disorder
Bilateral, non tender enlargement of the gland is usual
Histologically, the usual findings are of a lymphocytic infiltrate in acinar units and epimyoepithelial islands surrounded by lymphoid stroma
Treatment is supportive
There is an increased risk of subsequent lymphoma

Sarcoid
Parotid involvement occurs in 6% of patients with sarcoid
Bilateral in most cases
Gland is not tender
Xerostomia may occur
Management of isolated parotid disease is usually conservative

47
Q

At which of the following time frames is secondary haemorrhage most likely to occur following tonsillectomy?

	Between 5 and 10 days after surgery
	During the first 6 hours after surgery
	Between 6 and 12 hours after surgery
	Upon resumption of normal feeding
	Between 2 and 3 days post operatively
A

Between 5 and 10 days after surgery

Haemorrhage in the first 6 hours after surgery is termed reactionary haemorrhage. Feeding does not increase the risk and may actually lower the risks of infection developing.

Secondary haemorrhage after tonsillectomy
Haemorrhage is a feared complication following tonsillectomy. Primary, or reactionary haemorrhage most commonly occurs in the first 6-8 hours following surgery. It is managed by immediate return to theatre.
Secondary haemorrhage occurs between 5 and 10 days after surgery, it is often associated with a wound infection. Treatment is usually with admission and antibiotics. Severe bleeding may require surgery. Secondary haemorrhage occurs in 3% of all tonsillectomies.

48
Q

A 6 week old baby boy is brought to the clinic by his mother. She is concerned because although the left testis is present in the scrotum the right testis is absent. She reports that it is sometimes palpable when she bathes the child. on examination the right testis is palpable at the level of the superficial inguinal ring. What is the most appropriate management?

	Discharge
	Re-assess in 5 years
	Laparoscopy
	Re-assess in 6 months
	Orchidopexy
A

Re-assess in 6 months

Undescended testes are not uncommon in young children. They may be present in 4% of term infants, but only in 1.3% children at 3 months of age. In this scenario the testis is retractile and can be managed expectantly.

Cryptorchidism
A congenital undescended testis is one that has failed to reach the bottom of the scrotum by 3 months of age. At birth up to 5% of boys will have an undescended testis, post natal descent occurs in most and by 3 months the incidence of cryptorchidism falls to 1-2%. In the vast majority of cases the cause of the maldescent is unknown. A proportion may be associated with other congenital defects including:

Patent processus vaginalis
Abnormal epididymis
Cerebral palsy
Mental retardation
Wilms tumour
Abdominal wall defects (e.g. gastroschisis, prune belly syndrome)

Differential diagnosis
These include retractile testes and, in the case of absent bilateral testes the possibility of intersex conditions. A retractile testis can be brought into the scrotum by the clinician and when released remains in the scrotum. If the examining clinician notes the testis to return rapidly into the inguinal canal when released then surgery is probably indicated.

Reasons for correction of cryptorchidism
Reduce risk of infertility
Allows the testes to be examined for testicular cancer
Avoid testicular torsion
Cosmetic appearance
Males with undescended testis are 40 times as likely to develop testicular cancer (seminoma) as males without undescended testis
The location of the undescended testis affects the relative risk of testicular cancer (50% intra-abdominal testes)

Treatment
Orchidopexy at 6- 18 months of age. The operation usually consists of inguinal exploration, mobilisation of the testis and implantation into a dartos pouch.
Intra-abdominal testis should be evaluated laparoscopically and mobilised. Whether this is a single stage or two stage procedure depends upon the exact location.
After the age of 2 years in untreated individuals the Sertoli cells will degrade and those presenting late in teenage years may be better served by orchidectomy than to try and salvage a non functioning testis with an increased risk of malignancy.

49
Q

A 28 year old man falls and hits his head against a wall. There is a brief loss of consciousness. When assessed in accident and emergency he is alert and orientated with a GCS of 15, imaging shows no fracture of the skull. What is his risk of having an intracranial haematoma requiring removal?

	1 in 6000
	1 in 40
	1 in 4
	1 in 50,000
	1 in 120
A

1 in 6000

Risk of haematoma (requiring removal) in adults attending accident and emergency units following head injury.

  • Concussion, no skull fracture Orientated 1 in 6000
  • Concussion, no skull fracture Not orientated 1 in 120
  • Skull fracture Orientated 1 in 32
  • Skull fracture Not orientated 1 in 4
50
Q

With which of the conditions listed below is Boas’ sign classically associated?

	Perforation of the thoracic oesophagus
	Acute cholecystitis
	Hepatocellular carcinoma
	Closed loop small bowel obstruction
	Acute diverticulitis
A

Boas’ sign refers to hyperaesthesia of the tip of the right scapula and is seen classically in association with acute cholecystitis.

A number of eponymous abdominal signs are noted. These include:
Rovsings sign- appendicitis
Boas sign -cholecystitis
Murphys sign- cholecystitis
Cullens sign- pancreatitis (other intraabdominal haemorrhage)
Grey-Turners sign- pancreatitis (or other retroperitoneal haemorrhage)

51
Q

A 56 year old man presents with episodes of pruritus ani and bright red rectal bleeding. On examination there is a mass in the ano rectal region and biopsies confirm squamous cell cancer. What is the most likely cause?

	Anal cancer
	Rectal cancer
	Soft tissue sarcoma
	Retro-rectal cyst
	Pilonidal sinus disease
A

These are features of anal cancer. Anal cancers arise from the cutaneous epithelium and are therefore typically squamous cell. They are usually sensitive to chemoradiotherapy.

Rectal bleeding is a common cause for patients to be referred to the surgical clinic. In the clinical history it is useful to try and localise the anatomical source of the blood. Bright red blood is usually of rectal anal canal origin, whilst dark red blood is more suggestive of a proximally sited bleeding source. Blood which has entered the GI tract from a gastro-duodenal source will typically resemble malaena due to the effects of the digestive enzymes on the blood itself.

In the table below we give some typical bleeding scenarios together with physical examination findings and causation.

  • Fissure in ano Bright red rectal bleeding Painful bleeding that occurs post defecation in small volumes. Usually antecedent features of constipation Muco-epithelial defect usually in the midline posteriorly (anterior fissures more likely to be due to underlying disease)
  • Haemorroids Bright red rectal bleeding Post defecation bleeding noted both on toilet paper and drips into pan. May be alteration of bowel habit and history of straining. No blood mixed with stool. No local pain. Normal colon and rectum. Proctoscopy may show internal haemorrhoids. Internal haemorrhoids are usually impalpable.
  • Crohns disease Bright red or mixed blood Bleeding that is accompanied by other symptoms such as altered bowel habit, malaise, history of fissures (especially anterior) and abscesses. Perineal inspection may show fissures or fistulae. Proctoscopy may demonstrate indurated mucosa and possibly strictures. Skip lesions may be noted at colonoscopy.
  • Ulcerative colitis Bright red bleeding often mixed with stool Diarrhoea, weight loss, nocturnal incontinence, passage of mucous PR. Proctitis is the most marked finding. Peri anal disease is usually absent. Colonoscopy will show continuous mucosal lesion.
  • Rectal cancer Bright red blood mixed volumes Alteration of bowel habit. Tenesmus may be present. Symptoms of metastatic disease. Usually obvious mucosal abnormality. Lesion may be fixed or mobile depending upon disease extent. Surrounding mucosa often normal, although polyps may be present.

Investigation
All patients presenting with rectal bleeding require digital rectal examination and procto-sigmoidoscopy as a minimal baseline.
Remember that haemorrhoids are typically impalpable and to attribute bleeding to these in the absence of accurate internal inspection is unsatisfactory.
In young patients with no other concerning features in the history a carefully performed sigmoidoscopy that demonstrates clear haemorrhoidal disease may be sufficient. If clear views cannot be obtained then patients require bowel preparation with an enema and a flexible sigmoidscopy performed.
In those presenting with features of altered bowel habit or suspicion of inflammatory bowel disease a colonoscopy is the best test.
Patients with excessive pain who are suspected of having a fissure may require an examination under general or local anaesthesia.
In young patients with external stigmata of fissure and a compatible history it is acceptable to treat medically and defer internal examination until the fissure is healed. If the fissure fails to heal then internal examination becomes necessary along the lines suggested above to exclude internal disease.

Special tests
In patients with a malignancy of the rectum the staging investigations comprise an MRI of the rectum to identify circumferential resection margin compromise and to identify mesorectal nodal disease. In addition to this CT scanning of the chest abdomen and pelvis is necessary to stage for more distant disease. Some centres will still stage the mesorectum with endo rectal ultrasound but this is becoming far less common.

Patients with fissure in ano who are being considered for surgical sphincterotomy and are females who have an obstetric history should probably have ano rectal manometry testing performed together with endo anal ultrasound. As this service is not universally available it is not mandatory but in the absence of such information there are continence issues that may arise following sphincterotomy.

Management

  • Fissure in ano GTN ointment 0.2% or diltiazem cream applied topically is the usual first line treatment. Botulinum toxin for those who fail to respond. Internal sphincterotomy for those who fail with botox, can be considered earlier in males.
  • Haemorroids Lifestyle advice, for small internal haemorrhoids can consider injection sclerotherapy or rubber band ligation. For external haemorrhoids consider haemorrhoidectomy. Modern options include HALO procedure and stapled haemorrhoidectomy.
    Inflammatory bowel disease Medical management- although surgery may be needed for fistulating Crohns (setons).
  • Rectal cancer Anterior resection or abdomino-perineal excision of the colon and rectum. Total mesorectal excision is now standard of care. Most resections below the peritoneal reflection will require defunctioning ileostomy. Most patients will require preoperative radiotherapy.
52
Q

A 52 year old man falls off his bike. He is found to have a pelvic fracture. On examination, he is found to have perineal oedema and on PR the prostate is not palpable. A urine dipstick shows blood. What is the most likely underlying problem?

	Bulbar urethral rupture
	Bladder outflow obstruction
	Bladder rupture
	Bladder contusion
	Membranous urethral rupture
A

A pelvic fracture and highly displaced prostate should indicate a diagnosis of membranous urethral rupture.

Lower genitourinary tract trauma
Most bladder injuries occur due to blunt trauma
85% associated with pelvic fractures
Easily overlooked during assessment in trauma
Up to 10% of male pelvic fractures are associated with urethral or bladder injuries

Urethral injury	
Mainly in males
Blood at the meatus (50% cases)
There are 2 types:
i.Bulbar rupture
- most common
- straddle type injury e.g. bicycles
- triad signs: urinary retention, perineal haematoma, blood at the meatus
ii. Membranous rupture
- can be extra or intraperitoneal
- commonly due to pelvic fracture
- Penile or perineal oedema/ hematoma 
- PR: prostate displaced upwards (beware co-existing retroperitoneal haematomas as they may make examination difficult)
  • Investigation: ascending urethrogram
  • Management: suprapubic catheter (surgical placement, not percutaneously)
    External genitalia injuries (i.e., the penis and the scrotum)
    Secondary to injuries caused by penetration, blunt trauma, continence- or sexual pleasure-enhancing devices, and mutilation

Bladder injury
rupture is intra or extraperitoneal
presents with haematuria or suprapubic pain
history of pelvic fracture and inability to void: always suspect bladder or urethral injury
inability to retrieve all fluid used to irrigate the bladder through a Foley catheter indicates bladder injury
investigation- IVU or cystogram
management: laparotomy if intraperitoneal, conservative if extraperitoneal

53
Q

A 43 year old lady presents with 24 hour history of generalised right upper quadrant pain. On admission, she is septic and jaundiced and there is tenderness in the right upper quadrant. What is the most likely diagnosis?

	Cholecystitis
	Cholangitis
	Gallbladder empyema
	Gallbladder abscess
	Liver abscess
A

A combination of sepsis and jaundice generally favors a diagnosis of cholangitis. Conditions such as empyema may cause sepsis but not marked jaundice, the same is also true of the other differentials listed.

Gallstones
Up to 24% of women and 12% of men may have gallstones. Of these up to 30% may develop local infection and cholecystitis. In patients subjected to surgery 12% will have stones contained within the common bile duct. The majority of gallstones are of a mixed composition (50%) with pure cholesterol stones accounting for 20% of cases.
The aetiology of CBD stones differs in the world, in the West most CBD stones are the result of migration. In the East a far higher proportion arise in the CBD de novo.
The classical symptoms are of colicky right upper quadrant pain that occurs post prandially. The symptoms are usually worst following a fatty meal when cholecystokinin levels are highest and gallbladder contraction is maximal.

Investigation
In almost all suspected cases the standard diagnostic work up consists of abdominal ultrasound and liver function tests. Of patients who have stones within the bile duct, 60% will have at least one abnormal result on LFT’s. Ultrasound is an important test, but is operator dependent and therefore may occasionally need to be repeated if a negative result is at odds with the clinical picture. Where stones are suspected in the bile duct, the options lie between magnetic resonance cholangiography and intraoperative imaging. The choice between these two options is determined by the skills and experience of the surgeon. The advantages of intra operative imaging are less useful in making therapeutic decisions if the operator is unhappy about proceeding the bile duct exploration, and in such circumstances pre operative MRCP is probably a better option.

Biliary colic Colicky abdominal pain, worse post prandially, worse after fatty foods If imaging shows gallstones and history compatible then laparoscopic cholecystectomy

Acute cholecystitis
Right upper quadrant pain
Fever
Murphys sign on examination
Occasionally mildly deranged LFT’s (especially if Mirizzi syndrome)
Imaging (USS) and cholecystectomy (ideally within 48 hours of presentation) (2)

Gallbladder abscess
Usually prodromal illness and right upper quadrant pain
Swinging pyrexia
Patient may be systemically unwell
Generalised peritonism not present
Imaging with USS +/- CT Scanning
Ideally surgery, sub total cholecystectomy may be needed if Calots triangle is hostile
In unfit patients percutaneous drainage may be considered

Cholangitis	
Patient severely septic and unwell
Jaundice
Right upper quadrant pain
Fluid resuscitation
Broad spectrum intravenous antibiotics
Correct any coagulopathy
Early ERCP

Gallstone ileus
Patients may have a history of previous cholecystitis and known gallstones
Small bowel obstruction (may be intermittent)
Laparotomy and removal of gallstone from small bowel, the enterotomy must be made proximal to the site of obstruction and not at the site of obstruction. The fistula between the gallbladder and duodenum should not be interfered with.

Acalculous cholecystitis
Patients with inter current illness (e.g. diabetes, organ failure)
Patient of systemically unwell
Gallbladder inflammation in absence of stones
High fever
If patient fit then cholecystectomy, if unfit then percutaneous cholecystostomy

Treatment
Patients with asymptomatic gallstones rarely develop symptoms related to them (less than 2% per year) and may therefore be managed expectantly. In almost all cases of symptomatic gallstones the treatment of choice is cholecystectomy performed via the laparoscopic route. In the very frail patient there is sometimes a role for selective use of ultrasound guided cholecystostomy.

During the course of the procedure some surgeons will routinely perform either intra operative cholangiography or laparoscopic USS to either confirm anatomy or to exclude CBD stones. The latter may be more easily achieved by use of laparoscopic ultrasound. If stones are found then the options lie between early ERCP in the day or so following surgery or immediate surgical exploration of the bile duct. When performed via the trans cystic route this adds little in the way of morbidity and certainly results in faster recovery. Where transcystic exploration fails the alternative strategy is that of formal choledochotomy. The exploration of a small duct is challenging and ducts of less than 8mm should not be explored. Small stones that measure less than 5mm may be safely left and most will pass spontaneously.

Risks of ERCP
Bleeding 0.9% (rises to 1.5% if sphincterotomy performed)
Duodenal perforation 0.4%
Cholangitis 1.1%
Pancreatitis 1.5%
54
Q

A 23 year old lady is admitted with right upper quadrant pain. On examination, she is tender in the right upper quadrant. Imaging shows signs of acute cholecystitis due to gallstones. The CBD appears normal. Liver function tests are normal. What is the most appropriate course of action?

Laparoscopic cholecystectomy during the next 24-48 hours
Open cholecystectomy during the next 24-48 hours
Laparoscopic cholecystectomy 3 months following resolution of the attack
Open cholecystectomy 3 months following the attack
Laparoscopic cholecystectomy after 5 days of intravenous antibiotics
A

In most cases the treatment of choice for acute cholecystitis is an acute cholecystectomy performed early in the illness. Delayed surgery particularly around 5- 7 days after presentation is much more technically challenging and is often best deferred. In most cases the procedure can be performed laparoscopically, even when acute inflammation is present.

Up to 24% of women and 12% of men may have gallstones. Of these up to 30% may develop local infection and cholecystitis. In patients subjected to surgery 12% will have stones contained within the common bile duct. The majority of gallstones are of a mixed composition (50%) with pure cholesterol stones accounting for 20% of cases.
The aetiology of CBD stones differs in the world, in the West most CBD stones are the result of migration. In the East a far higher proportion arise in the CBD de novo.
The classical symptoms are of colicky right upper quadrant pain that occurs post prandially. The symptoms are usually worst following a fatty meal when cholecystokinin levels are highest and gallbladder contraction is maximal.

Investigation
In almost all suspected cases the standard diagnostic work up consists of abdominal ultrasound and liver function tests. Of patients who have stones within the bile duct, 60% will have at least one abnormal result on LFT’s. Ultrasound is an important test, but is operator dependent and therefore may occasionally need to be repeated if a negative result is at odds with the clinical picture. Where stones are suspected in the bile duct, the options lie between magnetic resonance cholangiography and intraoperative imaging. The choice between these two options is determined by the skills and experience of the surgeon. The advantages of intra operative imaging are less useful in making therapeutic decisions if the operator is unhappy about proceeding the bile duct exploration, and in such circumstances pre operative MRCP is probably a better option.

Specific gallstone and gallbladder related disease
Disease Features Management
Biliary colic Colicky abdominal pain, worse post prandially, worse after fatty foods If imaging shows gallstones and history compatible then laparoscopic cholecystectomy
Acute cholecystitis
Right upper quadrant pain
Fever
Murphys sign on examination
Occasionally mildly deranged LFT’s (especially if Mirizzi syndrome)
Imaging (USS) and cholecystectomy (ideally within 48 hours of presentation) (2)
Gallbladder abscess
Usually prodromal illness and right upper quadrant pain
Swinging pyrexia
Patient may be systemically unwell
Generalised peritonism not present
Imaging with USS +/- CT Scanning
Ideally surgery, sub total cholecystectomy may be needed if Calots triangle is hostile
In unfit patients percutaneous drainage may be considered
Cholangitis
Patient severely septic and unwell
Jaundice
Right upper quadrant pain
Fluid resuscitation
Broad spectrum intravenous antibiotics
Correct any coagulopathy
Early ERCP
Gallstone ileus
Patients may have a history of previous cholecystitis and known gallstones
Small bowel obstruction (may be intermittent)
Laparotomy and removal of gallstone from small bowel, the enterotomy must be made proximal to the site of obstruction and not at the site of obstruction. The fistula between the gallbladder and duodenum should not be interfered with.
Acalculous cholecystitis
Patients with inter current illness (e.g. diabetes, organ failure)
Patient of systemically unwell
Gallbladder inflammation in absence of stones
High fever
If patient fit then cholecystectomy, if unfit then percutaneous cholecystostomy

Treatment
Patients with asymptomatic gallstones rarely develop symptoms related to them (less than 2% per year) and may therefore be managed expectantly. In almost all cases of symptomatic gallstones the treatment of choice is cholecystectomy performed via the laparoscopic route. In the very frail patient there is sometimes a role for selective use of ultrasound guided cholecystostomy.

During the course of the procedure some surgeons will routinely perform either intra operative cholangiography or laparoscopic USS to either confirm anatomy or to exclude CBD stones. The latter may be more easily achieved by use of laparoscopic ultrasound. If stones are found then the options lie between early ERCP in the day or so following surgery or immediate surgical exploration of the bile duct. When performed via the trans cystic route this adds little in the way of morbidity and certainly results in faster recovery. Where transcystic exploration fails the alternative strategy is that of formal choledochotomy. The exploration of a small duct is challenging and ducts of less than 8mm should not be explored. Small stones that measure less than 5mm may be safely left and most will pass spontaneously.

Risks of ERCP(1)
Bleeding 0.9% (rises to 1.5% if sphincterotomy performed)
Duodenal perforation 0.4%
Cholangitis 1.1%
Pancreatitis 1.5%
55
Q

A 52 year old woman with known Hashimotos thyroiditis presents with a neck swelling. She describes it as rapidly increasing in size over 3 months and she complains of dysphagia to solids. On examination, there is an asymmetrical swelling of the thyroid gland. What is the most likely diagnosis?

	Anaplastic thyroid cancer
	Follicular thyroid cancer
	Papillary thyroid cancer
	Lymphoma
	Toxic adenoma
A

Thyroid lymphoma (Non Hodgkin’s B cell lymphoma) is rare. It should be considered in patients with a background of Hashimoto’s thyroiditis and a rapid growth in size of the thyroid gland. Diagnosis can be made with core needle biopsy; however an incisional biopsy may be needed. Radiotherapy is the main treatment option.

Thyroiditis
Sub acute thyroiditis
Subacute thyroiditis (also known as De Quervain’s thyroiditis) is thought to occur following viral infection and typically presents with hyperthyroidism

Features
Hyperthyroidism
Painful goitre
Raised ESR
Globally reduced uptake on iodine-131 scan

Management
Usually self-limiting - most patients do not require treatment
Thyroid pain may respond to aspirin or other NSAIDs
In more severe cases steroids are used, particularly if hypothyroidism develops

Hashimotos thyroiditis
Hashimotos thyroiditis is an immunological disorder in which lymphocytes become sensitised to thyroidal antigens. The three most important antibodies include; thyroglobulin, TPO and TSH-R. During the early phase of Hashimotos the the thyroglobulin antibody is markedly elevated and then declines.

Features
Goitre and either euthyroid or mild hypothyroidism
Progressive hypothyroidism (and associated symptoms)

Management
During the hyperthyroid phase of illness beta blockers may manage symptoms
As hypothyroidism develops patients may require thyroxine

56
Q

A 34 year old female presents with a thyroid nodule. She has a family history of thyroid disease and both her sisters have undergone total thyroidectomies. Her past medical history includes hypertension which has been difficult to manage. What is the most likely underlying thyroid lesion?

	Papillary thyroid cancer
	Follicular adenoma
	Medullary thyroid cancer
	Anaplastic thyroid cancer
	Lymphoma of the thyroid
A

This is a typical scenario for medullary carcinoma in which a phaeochromocytoma may also be present. It may be inherited in an autosomal dominant fashion and affected family members may be offered prophylactic thyroidectomy.

Thyroid disease
Patients may present with a number of different manifestations of thyroid disease. They can be broadly sub classified according to whether they are euthyroid or have clinical signs of thyroid dysfunction. In addition it needs to be established whether they have a mass or not.

Assessment
History
Examination including USS
If a nodule is identified then it should be sampled ideally via an image guided fine needle aspiration
Radionucleotide scanning is of limited use

Thyroid Tumours
Papillary carcinoma
Follicular carcinoma
Anaplastic carcinoma
Medullary carcinoma
Lymphoma's

Multinodular goitre
One of the most common reasons for presentation
Provided the patient is euthyroid and asymptomatic and no discrete nodules are seen, they can be reassured.
In those with compressive symptoms surgery is required and the best operation is a total thyroidectomy.
Sub total resections were practised in the past and simply result in recurrent disease that requires a difficult revisional resection.

Endocrine dysfunction
In general these patients are managed by physicians initially.
Surgery may be offered alongside radio iodine for patients with Graves disease that fails with medical management or in patients who would prefer not to be irradiated (e.g. pregnant women).
Patients with hypothyroidism do not generally get offered a thyroidectomy. Sometimes people inadvertently get offered resections during the early phase of Hashimotos thyroiditis, however, with time the toxic phase passes and patients can simply be managed with thyroxine.

Complications following surgery
Anatomical such as recurrent laryngeal nerve damage.
Bleeding. Owing to the confined space haematoma’s may rapidly lead to respiratory compromise owing to laryngeal oedema.
Damage to the parathyroid glands resulting in hypocalcaemia.

57
Q

A 29 year old man presents to the clinic with a recurrent thyroid cyst. It has been drained now on three occasions. Each time the cyst is aspirated and cytology is reassuring. What is the most appropriate course of action?

	Reassure and discharge
	Resection of the ipsilateral thyroid lobe
	Enucleation of the cyst
	Total thyroidectomy
	Radio-isotope scan
A

Persist refilling cysts may be associated with a well differentiated tumour and should be removed by lobectomy.

Large multinodular goitre Surgery for pressure symptoms. Total thyroidectomy is treatment of choice

Toxic nodule Hemithyroidectomy

Follicular lesion (THY 3f) Hemithyroidectomy to establish diagnosis

Papillary thyroid cancer Total thyroidectomy and central compartment nodal dissection (extended lymphadenectomy as required)

Follicular thyroid cancer Total thyroidectomy (usually completion as already had hemithyroidectomy)

Anaplastic thyroid cancer Palliative radiotherapy

Medullary thyroid cancer Total thyroidectomy (screen for other MEN tumours)

Lymphoma of the thyroid Consider core biopsy

Persistent refilling cysts Hemithyroidectomy

Graves disease with significant eye signs Total thyroidectomy

Graves disease without significant eye signs Patient choice radioiodine Vs surgery

58
Q

A 22 year old man presents with a 6 day history of passage of bloody diarrhoea with passage of mucous and slime. He is passing an average of 8 to 9 bowel movements per day. On digital rectal examination there is no discrete abnormality to feel, but there is some blood stained mucous on the glove. What is the most likely diagnosis?

	Solitary rectal ulcer syndrome
	Ulcerative colitis
	Irritable bowel syndrome
	Rectal cancer
	Diverticulitis
A

The passage of bloody diarrhoea together with mucus and a short history makes this a likely first presentation of inflammatory bowel disease. A rectal malignancy in a 22 year old would be a very unlikely event. The history is too short to be consistent with solitary rectal ulcer.

Rectal bleeding is a common cause for patients to be referred to the surgical clinic. In the clinical history it is useful to try and localise the anatomical source of the blood. Bright red blood is usually of rectal anal canal origin, whilst dark red blood is more suggestive of a proximally sited bleeding source. Blood which has entered the GI tract from a gastro-duodenal source will typically resemble malaena due to the effects of the digestive enzymes on the blood itself.

Fissure in ano Bright red rectal bleeding Painful bleeding that occurs post defecation in small volumes. Usually antecedent features of constipation Muco-epithelial defect usually in the midline posteriorly (anterior fissures more likely to be due to underlying disease)

Haemorroids Bright red rectal bleeding Post defecation bleeding noted both on toilet paper and drips into pan. May be alteration of bowel habit and history of straining. No blood mixed with stool. No local pain. Normal colon and rectum. Proctoscopy may show internal haemorrhoids. Internal haemorrhoids are usually impalpable.

Crohns disease Bright red or mixed blood Bleeding that is accompanied by other symptoms such as altered bowel habit, malaise, history of fissures (especially anterior) and abscesses. Perineal inspection may show fissures or fistulae. Proctoscopy may demonstrate indurated mucosa and possibly strictures. Skip lesions may be noted at colonoscopy.

Ulcerative colitis Bright red bleeding often mixed with stool Diarrhoea, weight loss, nocturnal incontinence, passage of mucous PR. Proctitis is the most marked finding. Peri anal disease is usually absent. Colonoscopy will show continuous mucosal lesion.

Rectal cancer Bright red blood mixed volumes Alteration of bowel habit. Tenesmus may be present. Symptoms of metastatic disease. Usually obvious mucosal abnormality. Lesion may be fixed or mobile depending upon disease extent. Surrounding mucosa often normal, although polyps may be present.

Investigation
All patients presenting with rectal bleeding require digital rectal examination and procto-sigmoidoscopy as a minimal baseline.
Remember that haemorrhoids are typically impalpable and to attribute bleeding to these in the absence of accurate internal inspection is unsatisfactory.
In young patients with no other concerning features in the history a carefully performed sigmoidoscopy that demonstrates clear haemorrhoidal disease may be sufficient. If clear views cannot be obtained then patients require bowel preparation with an enema and a flexible sigmoidscopy performed.
In those presenting with features of altered bowel habit or suspicion of inflammatory bowel disease a colonoscopy is the best test.
Patients with excessive pain who are suspected of having a fissure may require an examination under general or local anaesthesia.
In young patients with external stigmata of fissure and a compatible history it is acceptable to treat medically and defer internal examination until the fissure is healed. If the fissure fails to heal then internal examination becomes necessary along the lines suggested above to exclude internal disease.

Special tests
In patients with a malignancy of the rectum the staging investigations comprise an MRI of the rectum to identify circumferential resection margin compromise and to identify mesorectal nodal disease. In addition to this CT scanning of the chest abdomen and pelvis is necessary to stage for more distant disease. Some centres will still stage the mesorectum with endo rectal ultrasound but this is becoming far less common.

Patients with fissure in ano who are being considered for surgical sphincterotomy and are females who have an obstetric history should probably have ano rectal manometry testing performed together with endo anal ultrasound. As this service is not universally available it is not mandatory but in the absence of such information there are continence issues that may arise following sphincterotomy.

Management

Fissure in ano GTN ointment 0.2% or diltiazem cream applied topically is the usual first line treatment. Botulinum toxin for those who fail to respond. Internal sphincterotomy for those who fail with botox, can be considered earlier in males.

Haemorroids Lifestyle advice, for small internal haemorrhoids can consider injection sclerotherapy or rubber band ligation. For external haemorrhoids consider haemorrhoidectomy. Modern options include HALO procedure and stapled haemorrhoidectomy.
Inflammatory bowel disease Medical management- although surgery may be needed for fistulating Crohns (setons).

Rectal cancer Anterior resection or abdomino-perineal excision of the colon and rectum. Total mesorectal excision is now standard of care. Most resections below the peritoneal reflection will require defunctioning ileostomy. Most patients will require preoperative radiotherapy.

59
Q

In which of the conditions described below is Rovsing’s sign most likely to be absent?

	Locally advanced caecal cancer
	Para ileal appendicitis
	Right sided colonic diverticulitis
	Retrocaecal appendicitis
	Severe terminal ileal Crohns disease
A

Any advanced right iliac fossa pathology can result in a positive Rovsings sign. However, in retrocaecal appendicitis, it may be absent and this fact can contribute to a delayed diagnosis if undue weight is placed on the presence of the sign in making the diagnosis.

Rovsings sign- appendicitis
Boas sign -cholecystitis
Murphys sign- cholecystitis
Cullens sign- pancreatitis (other intraabdominal haemorrhage)
Grey-Turners sign- pancreatitis (or other retroperitoneal haemorrhage)

60
Q

An 8 year old boy presents with a 4 hour history of right iliac fossa pain with nausea and vomiting. He has been back at school for two days after being kept home with a flu like illness. On examination, he is tender in the right iliac fossa, although his abdomen is soft. Temperature is 39.3oC. Blood tests show a CRP of 40 and a WCC of 8.1. What is the most appropriate course of action?

	Abdominal MRI scan
	Abdominal CT scan
	Diagnostic laparoscopy
	Active observation
	Colonoscopy
A

The key point in the history is the preceding flu like illness and absence of abdominal signs. These make mesenteric adenitis the most likely diagnosis. The patient should have a period of active observation, if this were to represent early appendicitis, then the clinical picture may change and this would be detected with serial examination.

Appendicitis	
Pain radiating to right iliac fossa
Anorexia (very common)
Short history
Diarrhoea and profuse vomiting rare
Crohn's disease	
Often long history
Signs of malnutrition
Change in bowel habit, especially diarrhoea

Mesenteric adenitis
Mainly affects children
Causes include Adenoviruses, Epstein Barr Virus, beta-haemolytic Streptococcus, Staphylococcus spp., Escherichia coli, Streptococcus viridans and Yersinia spp.
Patients have a higher temperature than those with appendicitis
If laparotomy is performed, enlarged mesenteric lymph nodes will be present

Diverticulitis
Both left and right sided disease may present with right iliac fossa pain
Clinical history may be similar, although some change in bowel habit is usual
When suspected, a CT scan may help in refining the diagnosis

Meckel’s diverticulitis
A Meckel’s diverticulum is a congenital abnormality that is present in about 2% of the population
Typically 2 feet proximal to the ileocaecal valve
May be lined by ectopic gastric mucosal tissue and produce bleeding

Perforated peptic ulcer
This usually produces upper quadrant pain but pain may be lower
Perforations typically have a sharp sudden onset of pain in the history

Incarcerated right inguinal or femoral hernia
Usually only right iliac fossa pain if right sided or bowel obstruction.
Bowel perforation secondary to caecal or colon carcinoma
Seldom localised to right iliac fossa, although complete large bowel obstruction with caecal distension may cause pain prior to perforation.

Gynaecological causes
Pelvic inflammatory disease/salpingitis/pelvic abscess/Ectopic pregnancy/Ovarian torsion/Threatened or complete abortion/Mittelschmerz

Urological causes
Ureteric colic/UTI/Testicular torsion
Other causes
TB/Typhoid/Herpes Zoster/AAA/Situs inversus

61
Q

A 6 day old child is suspected of having a malrotation and requires urgent abdominal exploration. What is the most appropriate surgical approach?

Midline abdominal incision
Paramedian incision
Transverse supra umbilical abdominal incision
Transverse infra umbilical abdominal incision
Battle incision
A

In young children, laparotomy is performed via transverse supra umbilical incision. Access via midline incisions is very poor and they should not be used.

Midline incision
Commonest approach to the abdomen
Structures divided: linea alba, transversalis fascia, extraperitoneal fat, peritoneum (avoid falciform ligament above the umbilicus)
Bladder can be accessed via an extraperitoneal approach through the space of Retzius

Paramedian incision
Parallel to the midline (about 3-4cm)
Structures divided/retracted: anterior rectus sheath, rectus (retracted), posterior rectus sheath, transversalis fascia, extraperitoneal fat, peritoneum
Incision is closed in layers

Battle
Similar location to paramedian but rectus displaced medially (and thus denervated)
Now seldom used

Kocher’s Incision under right subcostal margin e.g. Cholecystectomy (open)

Lanz Incision in right iliac fossa e.g. Appendicectomy

Gridiron Oblique incision centered over McBurneys point- usually appendicectomy (less cosmetically acceptable than Lanz

Gable Rooftop incision

Pfannenstiel’s Transverse supra pubic, primarily used to access pelvic organs

McEvedy’s Groin incision e.g. Emergency repair strangulated femoral hernia

Rutherford Morrison Extraperitoneal approach to left or right lower quadrants. Gives excellent access to iliac vessels and is the approach of choice for first time renal transplantation.

62
Q

A 14 year old female is admitted with sudden onset right iliac fossa pain. She is otherwise well and on examination has some right iliac fossa tenderness but no guarding. She is afebrile. Urinary dipstick is normal. Her previous menstrual period two weeks ago was normal and pregnancy test is negative. What is the most likely underlying diagnosis?

	Mittelschmerz
	Endometriosis
	Appendicitis
	Crohns disease
	Pelvic abscess
A

The timing of the pain and the fact that it is mid cycle makes Mittelschmerz the most likely cause. When follicular cysts rupture, there is sometimes associated bleeding of small volume. The cyst contents and blood if present can be very irritant and pain can sometimes mimic appendicitis or pelvic inflammatory disease. However, normal inflammatory markers makes the former less likely and unless the 14 year old was sexually active, PID is also relatively rare in this group. Crohns disease and pelvic abscesses would typically present with a more protracted history and raised inflammatory markers and fever.

Appendicitis	
Pain radiating to right iliac fossa
Anorexia (very common)
Short history
Diarrhoea and profuse vomiting rare

Crohn’s disease
Often long history
Signs of malnutrition
Change in bowel habit, especially diarrhoea

Mesenteric adenitis
Mainly affects children
Causes include Adenoviruses, Epstein Barr Virus, beta-haemolytic Streptococcus, Staphylococcus spp., Escherichia coli, Streptococcus viridans and Yersinia spp.
Patients have a higher temperature than those with appendicitis
If laparotomy is performed, enlarged mesenteric lymph nodes will be present

Diverticulitis
Both left and right sided disease may present with right iliac fossa pain
Clinical history may be similar, although some change in bowel habit is usual
When suspected, a CT scan may help in refining the diagnosis

Meckel’s diverticulitis
A Meckel’s diverticulum is a congenital abnormality that is present in about 2% of the population
Typically 2 feet proximal to the ileocaecal valve
May be lined by ectopic gastric mucosal tissue and produce bleeding

Perforated peptic ulcer
This usually produces upper quadrant pain but pain may be lower
Perforations typically have a sharp sudden onset of pain in the history

Incarcerated right inguinal or femoral hernia
Usually only right iliac fossa pain if right sided or bowel obstruction.

Bowel perforation secondary to caecal or colon carcinoma
Seldom localised to right iliac fossa, although complete large bowel obstruction with caecal distension may cause pain prior to perforation.

Gynaecological causes
Pelvic inflammatory disease/salpingitis/pelvic abscess/Ectopic pregnancy/Ovarian torsion/Threatened or complete abortion/Mittelschmerz

Urological causes
Ureteric colic/UTI/Testicular torsion
Other causes
TB/Typhoid/Herpes Zoster/AAA/Situs inversus

63
Q

A 56 year old man is involved in a road traffic accident. He is found to have a pelvic fracture. He reports that he has some lower abdominal pain. He has peritonism in the lower abdomen. The nursing staff report that he has not passed any urine. A CT scan shows evidence of free fluid. What is the most likely diagnosis?

	Membranous urethral injury
	Bladder rupture
	Bladder contusion
	Bulbar urethral injury
	Urinary tract infection
A

A pelvic fracture and lower abdominal peritonism should raise suspicions of bladder rupture (especially as this man cannot pass urine).

Lower genitourinary tract trauma
Most bladder injuries occur due to blunt trauma
85% associated with pelvic fractures
Easily overlooked during assessment in trauma
Up to 10% of male pelvic fractures are associated with urethral or bladder injuries

Urethral injury	
Mainly in males
Blood at the meatus (50% cases)
There are 2 types:
i.Bulbar rupture
- most common
- straddle type injury e.g. bicycles
- triad signs: urinary retention, perineal haematoma, blood at the meatus
ii. Membranous rupture
- can be extra or intraperitoneal
- commonly due to pelvic fracture
- Penile or perineal oedema/ hematoma 
- PR: prostate displaced upwards (beware co-existing retroperitoneal haematomas as they may make examination difficult)
  • Investigation: ascending urethrogram
  • Management: suprapubic catheter (surgical placement, not percutaneously)
    External genitalia injuries (i.e., the penis and the scrotum)
    Secondary to injuries caused by penetration, blunt trauma, continence- or sexual pleasure-enhancing devices, and mutilation

Bladder injury
rupture is intra or extraperitoneal
presents with haematuria or suprapubic pain
history of pelvic fracture and inability to void: always suspect bladder or urethral injury
inability to retrieve all fluid used to irrigate the bladder through a Foley catheter indicates bladder injury
investigation- IVU or cystogram
management: laparotomy if intraperitoneal, conservative if extraperitoneal

64
Q

What is the most likely explanation for a 63 year old male to complain of a painless blood stained mucous rectal discharge 6 months following a Hartmann’s procedure?

	Pelvic abscess
	Crohns disease
	Dysplasia of the rectal stump
	Diversion proctitis
	Fissure in ano
A

Once the bowel has been disconnected, a degree of inflammation is commonly seen in the quiescent bowel. This is typically referred to as diversion colitis. Dysplasia is not usually seen in this context as a Hartmanns procedure is not usually a treatment modality used for IBD (which is the main risk factor for dysplasia).

Rectal bleeding is a common cause for patients to be referred to the surgical clinic. In the clinical history it is useful to try and localise the anatomical source of the blood. Bright red blood is usually of rectal anal canal origin, whilst dark red blood is more suggestive of a proximally sited bleeding source. Blood which has entered the GI tract from a gastro-duodenal source will typically resemble malaena due to the effects of the digestive enzymes on the blood itself.

  • Fissure in ano Bright red rectal bleeding Painful bleeding that occurs post defecation in small volumes. Usually antecedent features of constipation Muco-epithelial defect usually in the midline posteriorly (anterior fissures more likely to be due to underlying disease)
  • Haemorroids Bright red rectal bleeding Post defecation bleeding noted both on toilet paper and drips into pan. May be alteration of bowel habit and history of straining. No blood mixed with stool. No local pain. Normal colon and rectum. Proctoscopy may show internal haemorrhoids. Internal haemorrhoids are usually impalpable.
  • Crohns disease Bright red or mixed blood Bleeding that is accompanied by other symptoms such as altered bowel habit, malaise, history of fissures (especially anterior) and abscesses. Perineal inspection may show fissures or fistulae. Proctoscopy may demonstrate indurated mucosa and possibly strictures. Skip lesions may be noted at colonoscopy.
  • Ulcerative colitis Bright red bleeding often mixed with stool Diarrhoea, weight loss, nocturnal incontinence, passage of mucous PR. Proctitis is the most marked finding. Peri anal disease is usually absent. Colonoscopy will show continuous mucosal lesion.
  • Rectal cancer Bright red blood mixed volumes Alteration of bowel habit. Tenesmus may be present. Symptoms of metastatic disease. Usually obvious mucosal abnormality. Lesion may be fixed or mobile depending upon disease extent. Surrounding mucosa often normal, although polyps may be present.

Investigation
All patients presenting with rectal bleeding require digital rectal examination and procto-sigmoidoscopy as a minimal baseline.
Remember that haemorrhoids are typically impalpable and to attribute bleeding to these in the absence of accurate internal inspection is unsatisfactory.
In young patients with no other concerning features in the history a carefully performed sigmoidoscopy that demonstrates clear haemorrhoidal disease may be sufficient. If clear views cannot be obtained then patients require bowel preparation with an enema and a flexible sigmoidscopy performed.
In those presenting with features of altered bowel habit or suspicion of inflammatory bowel disease a colonoscopy is the best test.
Patients with excessive pain who are suspected of having a fissure may require an examination under general or local anaesthesia.
In young patients with external stigmata of fissure and a compatible history it is acceptable to treat medically and defer internal examination until the fissure is healed. If the fissure fails to heal then internal examination becomes necessary along the lines suggested above to exclude internal disease.

Special tests
In patients with a malignancy of the rectum the staging investigations comprise an MRI of the rectum to identify circumferential resection margin compromise and to identify mesorectal nodal disease. In addition to this CT scanning of the chest abdomen and pelvis is necessary to stage for more distant disease. Some centres will still stage the mesorectum with endo rectal ultrasound but this is becoming far less common.

Patients with fissure in ano who are being considered for surgical sphincterotomy and are females who have an obstetric history should probably have ano rectal manometry testing performed together with endo anal ultrasound. As this service is not universally available it is not mandatory but in the absence of such information there are continence issues that may arise following sphincterotomy.

Management
Fissure in ano GTN ointment 0.2% or diltiazem cream applied topically is the usual first line treatment. Botulinum toxin for those who fail to respond. Internal sphincterotomy for those who fail with botox, can be considered earlier in males.

Haemorroids Lifestyle advice, for small internal haemorrhoids can consider injection sclerotherapy or rubber band ligation. For external haemorrhoids consider haemorrhoidectomy. Modern options include HALO procedure and stapled haemorrhoidectomy.
Inflammatory bowel disease Medical management- although surgery may be needed for fistulating Crohns (setons).

Rectal cancer Anterior resection or abdomino-perineal excision of the colon and rectum. Total mesorectal excision is now standard of care. Most resections below the peritoneal reflection will require defunctioning ileostomy. Most patients will require preoperative radiotherapy.

65
Q

Which abdominal sign is described as being present when a patient with cholecystitis experiences pain on palpation of the right upper quadrant most marked on inspiration?

	Murphy's sign
	Boas' sign
	Rovsing's sign
	Cullens sign
	Grey Turners sign
A

Murphy’s sign

Rovsings sign- appendicitis
Boas sign -cholecystitis
Murphys sign- cholecystitis
Cullens sign- pancreatitis (other intraabdominal haemorrhage)
Grey-Turners sign- pancreatitis (or other retroperitoneal haemorrhage)

66
Q

A 19 year old man presents with diarrhea and rectal bleeding that has been present for the past two weeks. In addition, he has noticed that he has had incontinence at night. What is the most likely cause?

Viral gastroenteritis
Inflammatory bowel disease
Intersphincteric abscess
Irritable bowel syndrome and haemorrhoids
Irritable bowel syndrome and a fissure in ano
A

Nocturnal diarrhea and incontinence is a key feature in the history and is strongly suggestive of a diagnosis of IBD. More benign IBS presentations seldom have nocturnal events or a short history.

Rectal bleeding is a common cause for patients to be referred to the surgical clinic. In the clinical history it is useful to try and localise the anatomical source of the blood. Bright red blood is usually of rectal anal canal origin, whilst dark red blood is more suggestive of a proximally sited bleeding source. Blood which has entered the GI tract from a gastro-duodenal source will typically resemble malaena due to the effects of the digestive enzymes on the blood itself.

  • Fissure in ano Bright red rectal bleeding Painful bleeding that occurs post defecation in small volumes. Usually antecedent features of constipation Muco-epithelial defect usually in the midline posteriorly (anterior fissures more likely to be due to underlying disease)
  • Haemorroids Bright red rectal bleeding Post defecation bleeding noted both on toilet paper and drips into pan. May be alteration of bowel habit and history of straining. No blood mixed with stool. No local pain. Normal colon and rectum. Proctoscopy may show internal haemorrhoids. Internal haemorrhoids are usually impalpable.
  • Crohns disease Bright red or mixed blood Bleeding that is accompanied by other symptoms such as altered bowel habit, malaise, history of fissures (especially anterior) and abscesses. Perineal inspection may show fissures or fistulae. Proctoscopy may demonstrate indurated mucosa and possibly strictures. Skip lesions may be noted at colonoscopy.
  • Ulcerative colitis Bright red bleeding often mixed with stool Diarrhoea, weight loss, nocturnal incontinence, passage of mucous PR. Proctitis is the most marked finding. Peri anal disease is usually absent. Colonoscopy will show continuous mucosal lesion.
  • Rectal cancer Bright red blood mixed volumes Alteration of bowel habit. Tenesmus may be present. Symptoms of metastatic disease. Usually obvious mucosal abnormality. Lesion may be fixed or mobile depending upon disease extent. Surrounding mucosa often normal, although polyps may be present.

Investigation
All patients presenting with rectal bleeding require digital rectal examination and procto-sigmoidoscopy as a minimal baseline.
Remember that haemorrhoids are typically impalpable and to attribute bleeding to these in the absence of accurate internal inspection is unsatisfactory.
In young patients with no other concerning features in the history a carefully performed sigmoidoscopy that demonstrates clear haemorrhoidal disease may be sufficient. If clear views cannot be obtained then patients require bowel preparation with an enema and a flexible sigmoidscopy performed.
In those presenting with features of altered bowel habit or suspicion of inflammatory bowel disease a colonoscopy is the best test.
Patients with excessive pain who are suspected of having a fissure may require an examination under general or local anaesthesia.
In young patients with external stigmata of fissure and a compatible history it is acceptable to treat medically and defer internal examination until the fissure is healed. If the fissure fails to heal then internal examination becomes necessary along the lines suggested above to exclude internal disease.

Special tests
In patients with a malignancy of the rectum the staging investigations comprise an MRI of the rectum to identify circumferential resection margin compromise and to identify mesorectal nodal disease. In addition to this CT scanning of the chest abdomen and pelvis is necessary to stage for more distant disease. Some centres will still stage the mesorectum with endo rectal ultrasound but this is becoming far less common.

Patients with fissure in ano who are being considered for surgical sphincterotomy and are females who have an obstetric history should probably have ano rectal manometry testing performed together with endo anal ultrasound. As this service is not universally available it is not mandatory but in the absence of such information there are continence issues that may arise following sphincterotomy.

Management

Fissure in ano GTN ointment 0.2% or diltiazem cream applied topically is the usual first line treatment. Botulinum toxin for those who fail to respond. Internal sphincterotomy for those who fail with botox, can be considered earlier in males.

Haemorroids Lifestyle advice, for small internal haemorrhoids can consider injection sclerotherapy or rubber band ligation. For external haemorrhoids consider haemorrhoidectomy. Modern options include HALO procedure and stapled haemorrhoidectomy.
Inflammatory bowel disease Medical management- although surgery may be needed for fistulating Crohns (setons).

Rectal cancer Anterior resection or abdomino-perineal excision of the colon and rectum. Total mesorectal excision is now standard of care. Most resections below the peritoneal reflection will require defunctioning ileostomy. Most patients will require preoperative radiotherapy.

67
Q

Of the surgical incisions listed below, which is most suitable for a 45 year old female undergoing a first time renal transplant?

	Abdominal midline
	Paramedian
	Battle
	Rutherford Morrison
	Pfannenstiel
A

A Rutherford Morrison incision is the traditional approach for a renal transplant and provides extra peritoneal access to the iliac vessels.

Midline incision
Commonest approach to the abdomen
Structures divided: linea alba, transversalis fascia, extraperitoneal fat, peritoneum (avoid falciform ligament above the umbilicus)
Bladder can be accessed via an extraperitoneal approach through the space of Retzius

Paramedian incision
Parallel to the midline (about 3-4cm)
Structures divided/retracted: anterior rectus sheath, rectus (retracted), posterior rectus sheath, transversalis fascia, extraperitoneal fat, peritoneum
Incision is closed in layers

Battle
Similar location to paramedian but rectus displaced medially (and thus denervated)
Now seldom used

Kocher’s Incision under right subcostal margin e.g.
Cholecystectomy (open)

Lanz Incision in right iliac fossa e.g. Appendicectomy

Gridiron Oblique incision centered over McBurneys point- usually appendicectomy (less cosmetically acceptable than Lanz

Gable Rooftop incision

Pfannenstiel’s Transverse supra pubic, primarily used to access pelvic organs

McEvedy’s Groin incision e.g. Emergency repair strangulated femoral hernia
Rutherford Morrison Extraperitoneal approach to left or right lower quadrants. Gives excellent access to iliac vessels and is the approach of choice for first time renal transplantation.

68
Q

A 56 year old man is investigated with an abdominal CT scan for a change of bowel habit towards constipation. It shows no colonic lesions. However, a right sided adrenal lesion is noted and measures 2.5cm in diameter. What is the most appropriate course of action?

	Arrange an MRI of the adrenal gland
	Arrange an adrenal USS
	Arrange an image guided core biopsy
	Undertake an adrenalectomy
	Arrange a hormonal assay
A

The vast majority of small adrenal lesions are incidental, benign and non functioning adenomas. Apart from minimal workup, no further investigation is needed. Of note, if there are concerns about malignancy, the only surgical option is adrenalectomy.

Adrenal lesions- Incidental
Incidentaloma of the adrenal glands have become increasingly common as CT scanning of the abdomen is widely undertaken. Prevalences range from 1.5-9% in autopsy studies. Overall, 75% will be non functioning adenomas. However, a thorough diagnostic work up is required to exclude a more significant lesion.

Investigation
Morning and midnight plasma cortisol measurements
Dexamethasone suppression test
24 hour urinary cortisol excretion
24 hour urinary excretion of catecholamines
Serum potassium, aldosterone and renin levels

Management
The risk of malignancy is related to the size of the lesion and 25% of all masses greater than 4cm will be malignant. Such lesions should usually be excised. Where a lesion is a suspected metastatic deposit a biopsy may be considered.

69
Q

An 8 year old boy is examined by his doctor as part of a routine clinical examination. The doctor notices a smooth swelling in the right iliac fossa. It is mobile and he is otherwise well. What is the most likely underlying cause?

	Meckels diverticulum
	Spigelian hernia
	Mesenteric cyst
	Appendix mass
	Liposarcoma
A

Mesenteric cysts are often smooth. Imaging with ultrasound and CT is usually sufficient. Although rare, they most often occur in young children (up to 30% present before the age of 15). Many are asymptomatic and discovered incidentally. Acute presentations are recognised and may occur following cyst torsion, infarction or rupture. Most cysts will be surgically resected.
Spigelian hernias are very rare in children, liposarcomas are not smooth swellings. An appendix mass will usually produce systemic illness.

Appendicitis	
Pain radiating to right iliac fossa
Anorexia (very common)
Short history
Diarrhoea and profuse vomiting rare
Crohn's disease	
Often long history
Signs of malnutrition
Change in bowel habit, especially diarrhoea

Mesenteric adenitis
Mainly affects children
Causes include Adenoviruses, Epstein Barr Virus, beta-haemolytic Streptococcus, Staphylococcus spp., Escherichia coli, Streptococcus viridans and Yersinia spp.
Patients have a higher temperature than those with appendicitis
If laparotomy is performed, enlarged mesenteric lymph nodes will be present

Diverticulitis
Both left and right sided disease may present with right iliac fossa pain
Clinical history may be similar, although some change in bowel habit is usual
When suspected, a CT scan may help in refining the diagnosis
Meckel’s diverticulitis
A Meckel’s diverticulum is a congenital abnormality that is present in about 2% of the population
Typically 2 feet proximal to the ileocaecal valve
May be lined by ectopic gastric mucosal tissue and produce bleeding

Perforated peptic ulcer
This usually produces upper quadrant pain but pain may be lower
Perforations typically have a sharp sudden onset of pain in the history

Incarcerated right inguinal or femoral hernia
Usually only right iliac fossa pain if right sided or bowel obstruction.
Bowel perforation secondary to caecal or colon carcinoma
Seldom localised to right iliac fossa, although complete large bowel obstruction with caecal distension may cause pain prior to perforation.

Gynaecological causes
Pelvic inflammatory disease/salpingitis/pelvic abscess/Ectopic pregnancy/Ovarian torsion/Threatened or complete abortion/Mittelschmerz

Urological causes
Ureteric colic/UTI/Testicular torsion
Other causes
TB/Typhoid/Herpes Zoster/AAA/Situs inversus

70
Q

An 84 year old lady presents with a tender painful lump in the right groin and signs of small bowel obstruction. What is the most appropriate surgical incision to address this problem?

	Lothessien
	McEvedy
	Midline abdominal
	Paramedian
	Kockers
A

A McEvedy incision is traditionally used to approach incarcerated femoral hernias. The disadvantage of the Lothessien approach is that it weakens the inguinal canal and predisposes to inguinal hernia formation. The other incisions would not usually address femoral hernias. Given the features of bowel obstruction, a low approach would be inappropriate.

Midline incision
Commonest approach to the abdomen
Structures divided: linea alba, transversalis fascia, extraperitoneal fat, peritoneum (avoid falciform ligament above the umbilicus)
Bladder can be accessed via an extraperitoneal approach through the space of Retzius

Paramedian incision
Parallel to the midline (about 3-4cm)
Structures divided/retracted: anterior rectus sheath, rectus (retracted), posterior rectus sheath, transversalis fascia, extraperitoneal fat, peritoneum
Incision is closed in layers

Battle
Similar location to paramedian but rectus displaced medially (and thus denervated)
Now seldom used
Kocher’s Incision under right subcostal margin e.g. Cholecystectomy (open)

Lanz Incision in right iliac fossa e.g. Appendicectomy

Gridiron Oblique incision centered over McBurneys point- usually appendicectomy (less cosmetically acceptable than Lanz

Gable Rooftop incision

Pfannenstiel’s Transverse supra pubic, primarily used to access pelvic organs

McEvedy’s Groin incision e.g. Emergency repair strangulated femoral hernia
Rutherford Morrison Extraperitoneal approach to left or right lower quadrants. Gives excellent access to iliac vessels and is the approach of choice for first time renal transplantation.

71
Q

A 59 year old man is referred with symptoms of dysphagia. On examination, he has a large goitre and on imaging there is significant retrosternal extension and features of a multinodular goitre. What is the most appropriate course of action?

	Total thyroidectomy
	Sub total thyroidectomy
	Thyroid lobectomy
	Radio-iodine
	Conservative management
A

Total thyroidectomy

Sub total thyroidectomy is no longer routinely undertaken in this group.
Large multinodular goitre Surgery for pressure symptoms. Total thyroidectomy is treatment of choice
Toxic nodule Hemithyroidectomy
Follicular lesion (THY 3f) Hemithyroidectomy to establish diagnosis
Papillary thyroid cancer Total thyroidectomy and central compartment nodal dissection (extended lymphadenectomy as required)
Follicular thyroid cancer Total thyroidectomy (usually completion as already had hemithyroidectomy)
Anaplastic thyroid cancer Palliative radiotherapy
Medullary thyroid cancer Total thyroidectomy (screen for other MEN tumours)
Lymphoma of the thyroid Consider core biopsy
Persistent refilling cysts Hemithyroidectomy
Graves disease with significant eye signs Total thyroidectomy
Graves disease without significant eye signs Patient choice radioiodine Vs surgery

72
Q

A 21 year old women is admitted with a 48 hour history of worsening right iliac fossa pain. She has been nauseated and vomited twice. On examination, she is markedly tender in the right iliac fossa with localised guarding. Vaginal examination is unremarkable. Urine dipstick (including beta HCG) is negative. Blood tests show a WCC of 13.5 and CRP 70. What is the most appropriate course of action?

	Open appendicectomy
	Laparotomy
	Abdominal ultrasound
	Laparoscopic appendicectomy
	Abdominal CT scan
A

Laparoscopic appendicectomy

The most likely diagnosis is appendicitis. The negative vaginal examination (and therefore by definition the absence of cervical excitation) makes pelvic inflammatory disease unlikely. Given the raised inflammatory markers, the correct course of action is to proceed with surgery. In females, there are considerable advantages of undertaking this laparoscopically as it allows evaluation of the pelvic viscera. Imaging with USS is unlikely to alter management as it has a false negative rate and given the context of the clinical findings, surgery is likely to occur in any case. Whilst a CT scan would allow for an accurate pre-operative diagnosis, it carries a significant radiation dose, and again, is unlikely to alter management.

Appendicitis	
Pain radiating to right iliac fossa
Anorexia (very common)
Short history
Diarrhoea and profuse vomiting rare

Crohn’s disease
Often long history
Signs of malnutrition
Change in bowel habit, especially diarrhoea
Mesenteric adenitis
Mainly affects children
Causes include Adenoviruses, Epstein Barr Virus, beta-haemolytic Streptococcus, Staphylococcus spp., Escherichia coli, Streptococcus viridans and Yersinia spp.
Patients have a higher temperature than those with appendicitis
If laparotomy is performed, enlarged mesenteric lymph nodes will be present

Diverticulitis
Both left and right sided disease may present with right iliac fossa pain
Clinical history may be similar, although some change in bowel habit is usual
When suspected, a CT scan may help in refining the diagnosis

Meckel’s diverticulitis
A Meckel’s diverticulum is a congenital abnormality that is present in about 2% of the population
Typically 2 feet proximal to the ileocaecal valve
May be lined by ectopic gastric mucosal tissue and produce bleeding

Perforated peptic ulcer
This usually produces upper quadrant pain but pain may be lower
Perforations typically have a sharp sudden onset of pain in the history

Incarcerated right inguinal or femoral hernia
Usually only right iliac fossa pain if right sided or bowel obstruction.
Bowel perforation secondary to caecal or colon carcinoma
Seldom localised to right iliac fossa, although complete large bowel obstruction with caecal distension may cause pain prior to perforation.

Gynaecological causes
Pelvic inflammatory disease/salpingitis/pelvic abscess/Ectopic pregnancy/Ovarian torsion/Threatened or complete abortion/Mittelschmerz

Urological causes
Ureteric colic/UTI/Testicular torsion
Other causes
TB/Typhoid/Herpes Zoster/AAA/Situs inversus

73
Q

Which of the following signs is seen in patients who have a significant retroperitoneal haemorrhage?

	Boas' sign
	Pembertons sign
	Grey Turners sign
	Cullens sign
	Rovsing's sign
A

Grey Turner:

Rovsings sign- appendicitis
Boas sign -cholecystitis
Murphys sign- cholecystitis
Cullens sign- pancreatitis (other intraabdominal haemorrhage)
Grey-Turners sign- pancreatitis (or other retroperitoneal haemorrhage)

74
Q

What is the most likely diagnosis in a 17 year old man who presents with painful bright red rectal bleeding that has been noticed to occur in past 2 weeks?

	Fistula in ano
	Fissure in ano
	External haemorrhoids
	Internal haemorrhoids
	Ulcerative colitis
A

Painful rectal bleeding is typically seen with fissure in ano (most will be posterior). The initial history is often short (as in this case). A fistula is more likely to present with discharge than just blood. Haemorrhoidal disease bleeding is usually painless. Although thrombosed haemorrhoids may be painful, they typically occur in patients with a longer history.

Rectal bleeding is a common cause for patients to be referred to the surgical clinic. In the clinical history it is useful to try and localise the anatomical source of the blood. Bright red blood is usually of rectal anal canal origin, whilst dark red blood is more suggestive of a proximally sited bleeding source. Blood which has entered the GI tract from a gastro-duodenal source will typically resemble malaena due to the effects of the digestive enzymes on the blood itself.

In the table below we give some typical bleeding scenarios together with physical examination findings and causation.

Fissure in ano Bright red rectal bleeding Painful bleeding that occurs post defecation in small volumes. Usually antecedent features of constipation Muco-epithelial defect usually in the midline posteriorly (anterior fissures more likely to be due to underlying disease)

Haemorroids Bright red rectal bleeding Post defecation bleeding noted both on toilet paper and drips into pan. May be alteration of bowel habit and history of straining. No blood mixed with stool. No local pain. Normal colon and rectum. Proctoscopy may show internal haemorrhoids. Internal haemorrhoids are usually impalpable.

Crohns disease Bright red or mixed blood Bleeding that is accompanied by other symptoms such as altered bowel habit, malaise, history of fissures (especially anterior) and abscesses. Perineal inspection may show fissures or fistulae. Proctoscopy may demonstrate indurated mucosa and possibly strictures. Skip lesions may be noted at colonoscopy.

Ulcerative colitis Bright red bleeding often mixed with stool Diarrhoea, weight loss, nocturnal incontinence, passage of mucous PR. Proctitis is the most marked finding. Peri anal disease is usually absent. Colonoscopy will show continuous mucosal lesion.

Rectal cancer Bright red blood mixed volumes Alteration of bowel habit. Tenesmus may be present. Symptoms of metastatic disease. Usually obvious mucosal abnormality. Lesion may be fixed or mobile depending upon disease extent. Surrounding mucosa often normal, although polyps may be present.

Fissure in ano GTN ointment 0.2% or diltiazem cream applied topically is the usual first line treatment. Botulinum toxin for those who fail to respond. Internal sphincterotomy for those who fail with botox, can be considered earlier in males.

Haemorroids Lifestyle advice, for small internal haemorrhoids can consider injection sclerotherapy or rubber band ligation. For external haemorrhoids consider haemorrhoidectomy. Modern options include HALO procedure and stapled haemorrhoidectomy.
Inflammatory bowel disease Medical management- although surgery may be needed for fistulating Crohns (setons).

Rectal cancer Anterior resection or abdomino-perineal excision of the colon and rectum. Total mesorectal excision is now standard of care. Most resections below the peritoneal reflection will require defunctioning ileostomy. Most patients will require preoperative radiotherapy.

75
Q

A 34 year old lady is undergoing a laparoscopic cholecystectomy for cholecystitis. She has been unwell for the past 10 days. On attempting to dissect the gallbladder (which is distended), all that can be seen are the gallbladder fundus and dense adhesions make it difficult to dissect Calots triangle. What is the best course of action?

Perform an operative cholecystostomy
Dissect the adhesions off Calots triangle and continue with the cholecystectomy
Dissect out the bile duct and perform a cholangiogram
Arrange an ERCP
Arrange a PTC
A

Perform operative cholecystectomy

The timeframe of 10 days makes attempts at proceeding with surgery hazardous even in experienced hands. However, the patient is unwell and this will not settle without some form of intervention. If only the fundus can be seen, then it may be difficult to even proceed with a sub total cholecystectomy. Therefore, a cholecystostomy can be performed and this will usually allow the situation to settle. Definitive surgery can then be undertaken in more favourable circumstances.

Gallstones Typically history of biliary colic or episodes of chlolecystitis. Obstructive type history and test results. Usually small calibre gallstones which can pass through the cystic duct. In Mirizzi syndrome the stone may compress the bile duct directly- one of the rare times that cholecystitis may present with jaundice

Cholangitis Usually obstructive and will have Charcot’s triad of symptoms (pain, fever, jaundice) Ascending infection of the bile ducts usually by E. coli and by definition occurring in a pool of stagnant bile.

Pancreatic cancer Typically painless jaundice with palpable gallbladder (Courvoisier’s Law) Direct occlusion of distal bile duct or pancreatic duct by tumour. Sometimes nodal disease at the portal hepatis may be the culprit in which case the bile duct may be of normal calibre.

TPN (total parenteral nutrition) associated jaundice Usually follows long term use and is usually painless with non obstructive features Often due to hepatic dysfunction and fatty liver which may occur with long term TPN usage.

Bile duct injury Depending upon the type of injury may be of sudden or gradual onset and is usually of obstructive type Often due to a difficult laparoscopic cholecystectomy when anatomy in Calots triangle is not appreciated. In the worst scenario the bile duct is excised and jaundice develops rapidly post operatively. More insidious is that of bile duct stenosis which may be caused by clips or diathermy injury.

Cholangiocarcinoma Gradual onset obstructive pattern Direct occlusion by disease and also extrinsic compression by nodal disease at the porta hepatis.
Septic surgical patient Usually hepatic features Combination of impaired biliary excretion and drugs such as ciprofloxacin which may cause cholestasis.

Metastatic disease Mixed hepatic and post hepatic Combination of liver synthetic failure (late) and extrinsic compression by nodal disease and anatomical compression of intra hepatic structures (earlier)

76
Q

A 17 year old male is admitted with lower abdominal discomfort. He has been suffering from intermittent right iliac fossa pain for the past few months. His past medical history includes a negative colonoscopy and gastroscopy for iron deficiency anaemia. The pain is worse after meals. Inflammatory markers are normal. What is the most likely cause?

	Appendicitis
	Crohns disease
	Peptic ulcer disease
	Meckels diverticulum
	Irritable bowel syndrome
A

This scenario should raise suspicion for Meckels as these may contain ectopic gastric mucosa which may secrete acid with subsequent bleeding and ulceration. The iron deficiency anaemia is makes a Meckels more likely than IBD.

Appendicitis	
Pain radiating to right iliac fossa
Anorexia (very common)
Short history
Diarrhoea and profuse vomiting rare

Crohn’s disease
Often long history
Signs of malnutrition
Change in bowel habit, especially diarrhoea

Mesenteric adenitis
Mainly affects children
Causes include Adenoviruses, Epstein Barr Virus, beta-haemolytic Streptococcus, Staphylococcus spp., Escherichia coli, Streptococcus viridans and Yersinia spp.
Patients have a higher temperature than those with appendicitis
If laparotomy is performed, enlarged mesenteric lymph nodes will be present

Diverticulitis
Both left and right sided disease may present with right iliac fossa pain
Clinical history may be similar, although some change in bowel habit is usual
When suspected, a CT scan may help in refining the diagnosis

Meckel’s diverticulitis
A Meckel’s diverticulum is a congenital abnormality that is present in about 2% of the population
Typically 2 feet proximal to the ileocaecal valve
May be lined by ectopic gastric mucosal tissue and produce bleeding

Perforated peptic ulcer
This usually produces upper quadrant pain but pain may be lower
Perforations typically have a sharp sudden onset of pain in the history

Incarcerated right inguinal or femoral hernia
Usually only right iliac fossa pain if right sided or bowel obstruction.
Bowel perforation secondary to caecal or colon carcinoma
Seldom localised to right iliac fossa, although complete large bowel obstruction with caecal distension may cause pain prior to perforation.

Gynaecological causes
Pelvic inflammatory disease/salpingitis/pelvic abscess/Ectopic pregnancy/Ovarian torsion/Threatened or complete abortion/Mittelschmerz

Urological causes
Ureteric colic/UTI/Testicular torsion
Other causes
TB/Typhoid/Herpes Zoster/AAA/Situs inversus

77
Q

A 68 year old man with type 2 diabetes is admitted to hospital unwell. On examination, he has features of septic shock and right upper quadrant tenderness. He is not jaundiced. Imaging shows a normal calibre bile duct and no stones in the gallbladder. What is the most likely diagnosis?

	Acute calculous cholecystitis
	Acute acalculous cholecystitis
	Cholangitis
	Mirizzi syndrome
	Sphincter of oddi dysfunction
A

This is the classic description of acalculous cholecystitis and its commonest in patients with type 2 diabetes. If you answered it incorrectly, ensure that you were not caught out by the acute calculous cholecystitis as this is a common exam mistake if options are mis read in a rush.

Up to 24% of women and 12% of men may have gallstones. Of these up to 30% may develop local infection and cholecystitis. In patients subjected to surgery 12% will have stones contained within the common bile duct. The majority of gallstones are of a mixed composition (50%) with pure cholesterol stones accounting for 20% of cases.
The aetiology of CBD stones differs in the world, in the West most CBD stones are the result of migration. In the East a far higher proportion arise in the CBD de novo.
The classical symptoms are of colicky right upper quadrant pain that occurs post prandially. The symptoms are usually worst following a fatty meal when cholecystokinin levels are highest and gallbladder contraction is maximal.

Investigation
In almost all suspected cases the standard diagnostic work up consists of abdominal ultrasound and liver function tests. Of patients who have stones within the bile duct, 60% will have at least one abnormal result on LFT’s. Ultrasound is an important test, but is operator dependent and therefore may occasionally need to be repeated if a negative result is at odds with the clinical picture. Where stones are suspected in the bile duct, the options lie between magnetic resonance cholangiography and intraoperative imaging. The choice between these two options is determined by the skills and experience of the surgeon. The advantages of intra operative imaging are less useful in making therapeutic decisions if the operator is unhappy about proceeding the bile duct exploration, and in such circumstances pre operative MRCP is probably a better option.

Biliary colic Colicky abdominal pain, worse post prandially, worse after fatty foods If imaging shows gallstones and history compatible then laparoscopic cholecystectomy

Acute cholecystitis
Right upper quadrant pain
Fever
Murphys sign on examination
Occasionally mildly deranged LFT’s (especially if Mirizzi syndrome)
Imaging (USS) and cholecystectomy (ideally within 48 hours of presentation) (2)

Gallbladder abscess
Usually prodromal illness and right upper quadrant pain
Swinging pyrexia
Patient may be systemically unwell
Generalised peritonism not present
Imaging with USS +/- CT Scanning
Ideally surgery, sub total cholecystectomy may be needed if Calots triangle is hostile
In unfit patients percutaneous drainage may be considered

Cholangitis	
Patient severely septic and unwell
Jaundice
Right upper quadrant pain
Fluid resuscitation
Broad spectrum intravenous antibiotics
Correct any coagulopathy
Early ERCP

Gallstone ileus
Patients may have a history of previous cholecystitis and known gallstones
Small bowel obstruction (may be intermittent)
Laparotomy and removal of gallstone from small bowel, the enterotomy must be made proximal to the site of obstruction and not at the site of obstruction. The fistula between the gallbladder and duodenum should not be interfered with.

Acalculous cholecystitis
Patients with inter current illness (e.g. diabetes, organ failure)
Patient of systemically unwell
Gallbladder inflammation in absence of stones
High fever
If patient fit then cholecystectomy, if unfit then percutaneous cholecystostomy

Treatment
Patients with asymptomatic gallstones rarely develop symptoms related to them (less than 2% per year) and may therefore be managed expectantly. In almost all cases of symptomatic gallstones the treatment of choice is cholecystectomy performed via the laparoscopic route. In the very frail patient there is sometimes a role for selective use of ultrasound guided cholecystostomy.

During the course of the procedure some surgeons will routinely perform either intra operative cholangiography or laparoscopic USS to either confirm anatomy or to exclude CBD stones. The latter may be more easily achieved by use of laparoscopic ultrasound. If stones are found then the options lie between early ERCP in the day or so following surgery or immediate surgical exploration of the bile duct. When performed via the trans cystic route this adds little in the way of morbidity and certainly results in faster recovery. Where transcystic exploration fails the alternative strategy is that of formal choledochotomy. The exploration of a small duct is challenging and ducts of less than 8mm should not be explored. Small stones that measure less than 5mm may be safely left and most will pass spontaneously.

Risks of ERCP(1)
Bleeding 0.9% (rises to 1.5% if sphincterotomy performed)
Duodenal perforation 0.4%
Cholangitis 1.1%
Pancreatitis 1.5%
78
Q

A 24 year old woman presents with a long history of obstructed defecation and chronic constipation. She often strains to open her bowels for long periods and occasionally notices that she has passed a small amount of blood. On examination, she has an indurated area located anteriorly approximately 3cm proximal to the anal verge. What is the most likely diagnosis?

	Haemorrhoids
	Rectal cancer
	Ulcerative colitis
	Solitary rectal ulcer syndrome
	Fissure in ano
A

Solitary rectal ulcers are associated with chronic constipation and straining. It will need to be biopsied to exclude malignancy (the histological appearances are characteristic). Diagnostic work up should include endoscopy and probably defecating proctogram and ano-rectal manometry studies.

Rectal bleeding is a common cause for patients to be referred to the surgical clinic. In the clinical history it is useful to try and localise the anatomical source of the blood. Bright red blood is usually of rectal anal canal origin, whilst dark red blood is more suggestive of a proximally sited bleeding source. Blood which has entered the GI tract from a gastro-duodenal source will typically resemble malaena due to the effects of the digestive enzymes on the blood itself.

Fissure in ano Bright red rectal bleeding Painful bleeding that occurs post defecation in small volumes. Usually antecedent features of constipation Muco-epithelial defect usually in the midline posteriorly (anterior fissures more likely to be due to underlying disease)

Haemorroids Bright red rectal bleeding Post defecation bleeding noted both on toilet paper and drips into pan. May be alteration of bowel habit and history of straining. No blood mixed with stool. No local pain. Normal colon and rectum. Proctoscopy may show internal haemorrhoids. Internal haemorrhoids are usually impalpable.

Crohns disease Bright red or mixed blood Bleeding that is accompanied by other symptoms such as altered bowel habit, malaise, history of fissures (especially anterior) and abscesses. Perineal inspection may show fissures or fistulae. Proctoscopy may demonstrate indurated mucosa and possibly strictures. Skip lesions may be noted at colonoscopy.

Ulcerative colitis Bright red bleeding often mixed with stool Diarrhoea, weight loss, nocturnal incontinence, passage of mucous PR. Proctitis is the most marked finding. Peri anal disease is usually absent. Colonoscopy will show continuous mucosal lesion.

Rectal cancer Bright red blood mixed volumes Alteration of bowel habit. Tenesmus may be present. Symptoms of metastatic disease. Usually obvious mucosal abnormality. Lesion may be fixed or mobile depending upon disease extent. Surrounding mucosa often normal, although polyps may be present.

Investigation
All patients presenting with rectal bleeding require digital rectal examination and procto-sigmoidoscopy as a minimal baseline.
Remember that haemorrhoids are typically impalpable and to attribute bleeding to these in the absence of accurate internal inspection is unsatisfactory.
In young patients with no other concerning features in the history a carefully performed sigmoidoscopy that demonstrates clear haemorrhoidal disease may be sufficient. If clear views cannot be obtained then patients require bowel preparation with an enema and a flexible sigmoidscopy performed.
In those presenting with features of altered bowel habit or suspicion of inflammatory bowel disease a colonoscopy is the best test.
Patients with excessive pain who are suspected of having a fissure may require an examination under general or local anaesthesia.
In young patients with external stigmata of fissure and a compatible history it is acceptable to treat medically and defer internal examination until the fissure is healed. If the fissure fails to heal then internal examination becomes necessary along the lines suggested above to exclude internal disease.

Special tests
In patients with a malignancy of the rectum the staging investigations comprise an MRI of the rectum to identify circumferential resection margin compromise and to identify mesorectal nodal disease. In addition to this CT scanning of the chest abdomen and pelvis is necessary to stage for more distant disease. Some centres will still stage the mesorectum with endo rectal ultrasound but this is becoming far less common.

Patients with fissure in ano who are being considered for surgical sphincterotomy and are females who have an obstetric history should probably have ano rectal manometry testing performed together with endo anal ultrasound. As this service is not universally available it is not mandatory but in the absence of such information there are continence issues that may arise following sphincterotomy.

Management

Fissure in ano GTN ointment 0.2% or diltiazem cream applied topically is the usual first line treatment. Botulinum toxin for those who fail to respond. Internal sphincterotomy for those who fail with botox, can be considered earlier in males.

Haemorroids Lifestyle advice, for small internal haemorrhoids can consider injection sclerotherapy or rubber band ligation. For external haemorrhoids consider haemorrhoidectomy. Modern options include HALO procedure and stapled haemorrhoidectomy.
Inflammatory bowel disease Medical management- although surgery may be needed for fistulating Crohns (setons).

Rectal cancer Anterior resection or abdomino-perineal excision of the colon and rectum. Total mesorectal excision is now standard of care. Most resections below the peritoneal reflection will require defunctioning ileostomy. Most patients will require preoperative radiotherapy.

79
Q

A 46 year old man is admitted to hospital with a femoral shaft fracture that occurred suddenly whilst he was out walking his dog. On examination, there is no neurovascular deficit distal to the fracture site. He has a large firm nodule in the left lobe of the thyroid, there is no associated lymphadenopathy. What is the most likely underlying cause?

	Papillary thyroid cancer
	Follicular thyroid cancer
	Thyroid gland lymphoma
	Anaplastic thyroid cancer
	Medullary thyroid cancer
A

Follicular carcinomas may metastasise haematogenously (often to bone) where they may give rise to pathological fractures as in this case.

Multinodular goitre
One of the most common reasons for presentation
Provided the patient is euthyroid and asymptomatic and no discrete nodules are seen, they can be reassured.
In those with compressive symptoms surgery is required and the best operation is a total thyroidectomy.
Sub total resections were practised in the past and simply result in recurrent disease that requires a difficult revisional resection.

Endocrine dysfunction
In general these patients are managed by physicians initially.
Surgery may be offered alongside radio iodine for patients with Graves disease that fails with medical management or in patients who would prefer not to be irradiated (e.g. pregnant women).
Patients with hypothyroidism do not generally get offered a thyroidectomy. Sometimes people inadvertently get offered resections during the early phase of Hashimotos thyroiditis, however, with time the toxic phase passes and patients can simply be managed with thyroxine.

Complications following surgery
Anatomical such as recurrent laryngeal nerve damage.
Bleeding. Owing to the confined space haematoma’s may rapidly lead to respiratory compromise owing to laryngeal oedema.
Damage to the parathyroid glands resulting in hypocalcaemia.

80
Q

A 21 year old women presents with right iliac fossa pain. She reports some bloodstained vaginal discharge. On examination, she is afebrile and has a pulse rate of 97 bpm, normal blood pressure. She has diffuse lower abdominal tenderness. What is the most appropriate course of action?

	Laparotomy
	Laparoscopy
	Abdominal and pelvic MRI scan
	Abdominal and pelvic CT scan
	Abdominal and pelvic USS
A

Abdominal and pelvic USS
The history of blood stained discharge and tenderness makes an ectopic pregnancy a strong possibility, a USS should be performed and a pregnancy test undertaken. If the beta HCG is high then an intra uterine pregnancy should be found. If it is not, then an ectopic pregnancy is likely and surgery should be considered.

Appendicitis	
Pain radiating to right iliac fossa
Anorexia (very common)
Short history
Diarrhoea and profuse vomiting rare

Crohn’s disease
Often long history
Signs of malnutrition
Change in bowel habit, especially diarrhoea

Mesenteric adenitis
Mainly affects children
Causes include Adenoviruses, Epstein Barr Virus, beta-haemolytic Streptococcus, Staphylococcus spp., Escherichia coli, Streptococcus viridans and Yersinia spp.
Patients have a higher temperature than those with appendicitis
If laparotomy is performed, enlarged mesenteric lymph nodes will be present

Diverticulitis
Both left and right sided disease may present with right iliac fossa pain
Clinical history may be similar, although some change in bowel habit is usual
When suspected, a CT scan may help in refining the diagnosis

Meckel’s diverticulitis
A Meckel’s diverticulum is a congenital abnormality that is present in about 2% of the population
Typically 2 feet proximal to the ileocaecal valve
May be lined by ectopic gastric mucosal tissue and produce bleeding

Perforated peptic ulcer
This usually produces upper quadrant pain but pain may be lower
Perforations typically have a sharp sudden onset of pain in the history

Incarcerated right inguinal or femoral hernia
Usually only right iliac fossa pain if right sided or bowel obstruction.
Bowel perforation secondary to caecal or colon carcinoma
Seldom localised to right iliac fossa, although complete large bowel obstruction with caecal distension may cause pain prior to perforation.

Gynaecological causes
Pelvic inflammatory disease/salpingitis/pelvic abscess/Ectopic pregnancy/Ovarian torsion/Threatened or complete abortion/Mittelschmerz

Urological causes
Ureteric colic/UTI/Testicular torsion

Other causes
TB/Typhoid/Herpes Zoster/AAA/Situs inversus

81
Q

A 43 year old lady presents with an attack of acute pancreatitis. It is classified as a mild attack on severity scoring. Imaging identifies gallstones but a normal calibre bile duct, and a peripancreatic fluid collection. Which of the following management options is most appropriate?

	Intravenous octreotide
	Cholecystectomy within 4 weeks
	Nasogastric tube drainage of the stomach
	Insertion of a radiological drain
	Avoidance of enteral feeding
A

Patients with gallstone pancreatitis should undergo early cholecystectomy.
Enteral feeding helps minimise gut bacterial translocation and should be given to most patients with pancreatitis. Many studies have evaluated the role of octreotide in reducing pancreatic secretions and shown no benefit
The use of antibiotics in pancreatitis is controversial. However, a recent Cochrane review has presented reasonable evidence in favor of administration of imipenem to prevent infection in established necrosis.

Management of Acute Pancreatitis in the UK

Diagnosis
Traditionally hyperamylasaemia has been utilised with amylase being elevated three times the normal range.
However, amylase may give both false positive and negative results.
Serum lipase is both more sensitive and specific than serum amylase. It also has a longer half life.
Serum amylase levels do not correlate with disease severity.

Differential causes of hyperamylasaemia
Acute pancreatitis
Pancreatic pseudocyst
Mesenteric infarct
Perforated viscus
Acute cholecystitis
Diabetic ketoacidosis

Assessment of severity
Glasgow, Ranson scoring systems and APACHE II
Biochemical scoring e.g. using CRP

Features that may predict a severe attack within 48 hours of admission to hospital

Initial assessment	
Clinical impression of severity
Body mass index >30
Pleural effusion
APACHE score >8
24 hours after admission	
Clinical impression of severity
APACHE II >8
Glasgow score of 3 or more
Persisting multiple organ failure
CRP>150
48 hours after admission	
Glasgow Score of >3
CRP >150
Persisting or progressive organ failure
Table adapted from UK guidelines for management of acute pancreatitis. GUT 2005, 54 suppl III

Management

Nutrition
There is reasonable evidence to suggest that the use of enteral nutrition does not worsen the outcome in pancreatitis
Most trials to date were underpowered to demonstrate a conclusive benefit.
The rationale behind feeding is that it helps to prevent bacterial translocation from the gut, thereby contributing to the development of infected pancreatic necrosis.

Use of antibiotic therapy
Many UK surgeons administer antibiotics to patients with acute pancreatitis. However, there is very little evidence to support this practice.
A recent Cochrane review highlights the potential benefits of administering Imipenem to patients with established pancreatic necrosis in the hope of averting the progression to infection.
There are concerns that the administration of antibiotics in mild attacks of pancreatitis will not affect outcome and may contribute to antibiotic resistance and increase the risks of antibiotic associated diarrhoea.

Surgery
Patients with acute pancreatitis due to gallstones should undergo early cholecystectomy.
Patients with obstructed biliary system due to stones should undergo early ERCP.
Patients with extensive necrosis where infection is suspected should usually undergo FNA for culture.
Patients with infected necrosis should undergo either radiological drainage or surgical necrosectomy. The choice of procedure depends upon local expertise.

82
Q

A 21 year old male is admitted with a 3 month history of intermittent right iliac fossa pain. He suffers from episodic diarrhoea and has lost 2 kilos in weight. On examination, he has some right iliac fossa tenderness and is febrile. What is the most likely cause?

	Appendicitis
	Irritable bowel syndrome
	Inflammatory bowel disease
	Infective gastroenteritis
	Meckels diverticulum
A

The history of weight loss and intermittent diarrhea makes inflammatory bowel disease the most likely diagnosis. Conditions such as appendicitis and infections have a much shorter history. Although Meckels can bleed and cause inflammation, they seldom cause marked weight loss.

Appendicitis	
Pain radiating to right iliac fossa
Anorexia (very common)
Short history
Diarrhoea and profuse vomiting rare

Crohn’s disease
Often long history
Signs of malnutrition
Change in bowel habit, especially diarrhoea

Mesenteric adenitis
Mainly affects children
Causes include Adenoviruses, Epstein Barr Virus, beta-haemolytic Streptococcus, Staphylococcus spp., Escherichia coli, Streptococcus viridans and Yersinia spp.
Patients have a higher temperature than those with appendicitis
If laparotomy is performed, enlarged mesenteric lymph nodes will be present

Diverticulitis
Both left and right sided disease may present with right iliac fossa pain
Clinical history may be similar, although some change in bowel habit is usual
When suspected, a CT scan may help in refining the diagnosis

Meckel’s diverticulitis
A Meckel’s diverticulum is a congenital abnormality that is present in about 2% of the population
Typically 2 feet proximal to the ileocaecal valve
May be lined by ectopic gastric mucosal tissue and produce bleeding

Perforated peptic ulcer
This usually produces upper quadrant pain but pain may be lower
Perforations typically have a sharp sudden onset of pain in the history

Incarcerated right inguinal or femoral hernia
Usually only right iliac fossa pain if right sided or bowel obstruction.
Bowel perforation secondary to caecal or colon carcinoma
Seldom localised to right iliac fossa, although complete large bowel obstruction with caecal distension may cause pain prior to perforation.

Gynaecological causes
Pelvic inflammatory disease/salpingitis/pelvic abscess/Ectopic pregnancy/Ovarian torsion/Threatened or complete abortion/Mittelschmerz

Urological causes
Ureteric colic/UTI/Testicular torsion

Other causes
TB/Typhoid/Herpes Zoster/AAA/Situs inversus

83
Q

A 48 year old woman with end stage renal failure is undergoing a live donor renal transplant. The surgeon decides to implant the kidney in the left iliac fossa via a Rutherford Morrison incision. To which of the following vessels should the transplanted kidney be anastomosed?

	Aorta and inferior vena cava
	Internal iliac artery and vein
	Common iliac artery and vein
	External iliac artery and vein
	Inferior epigastric artery and vein
A

First time renal tranplants and typically implanted in the left or right iliac fossae. The vessels are usually joined to the external iliac artery and vein as these are the most easily accessible. The Rutherford Morrison incision provides access to the external iliac vessels.

A number of different organ and tissue transplants are now available. In many cases an allograft is performed, where an organ is transplanted from one individual to another. Allografts will elicit an immune response and this is one of the main reasons for organ rejection.

Graft rejection occurs because allografts have allelic differences at genes that code immunohistocompatability complex genes. The main antigens that give rise to rejection are:
ABO blood group
Human leucocyte antigens (HLA)
Minor histocompatability antigens

ABO Matching
ABO incompatibility will result in early organ rejection (hyperacute) because of pre existing antibodies to other groups. Group O donors can give organs to any type of ABO recipient whereas group AB donor can only donate to AB recipient.

HLA System
The four most important HLA alleles are:

HLA A
HLA B
HLA C
HLA DR

An ideal organ match would be one in which all 8 alleles are matched (remember 2 from each parent, four each = 8 alleles). Modern immunosuppressive regimes help to manage the potential rejection due to HLA mismatching. However, the greater the number of mismatches the worse the long term outcome will be. T lymphocytes will recognise antigens bound to HLA molecules and will then become activated. Clonal expansion then occurs with a response directed against that antigen.

Types of organ rejection
Hyperacute. This occurs immediately through presence of pre formed antibodies (such as ABO incompatibility).
Acute. Occurs during the first 6 months and is usually T cell mediated. Usually tissue infiltrates and vascular lesions.
Chronic. Occurs after the first 6 months. Vascular changes predominate.

Hyperacute
Renal transplants at greatest risk and liver transplants at least risk. Although ABO incompatibility and HLA Class I incompatible transplants will all fare worse in long term.

Acute
All organs may undergo acute rejection. Mononuclear cell infiltrates predominate. All types of transplanted organ are susceptible and it may occur in up to 50% cases.

Chronic
Again all transplants with HLA mismatch may suffer this fate. Previous acute rejections and other immunosensitising events all increase the risk. Vascular changes are most prominent with myointimal proliferation leading to organ ischaemia. Organ specific changes are also seen such as loss of acinar cells in pancreas transplants and rapidly progressive coronary artery disease in cardiac transplants.

Surgical overview-Renal transplantation
A brief overview of the steps involved in renal transplantation is given.
Patients with end stage renal failure who are dialysis dependent or likely to become so in the immediate future are considered for transplant. Exclusion criteria include; active malignancy, old age (due to limited organ availability). Patients are medically optimised.
Donor kidneys, these may be taken from live related donors and close family, members may have less HLA mismatch than members of the general population. Laparoscopic donor nephrectomy further minimises the operative morbidity for the donor. Other organs are typically taken from brain dead or dying patients who have a cardiac arrest and in whom resuscitation is futile. The key event is to minimise the warm ischaemic time in the donor phase.

The kidney once removed is usually prepared on the bench in theatre by the transplant surgeon immediately prior to implantation and factors such as accessory renal arteries and vessel length are assessed and managed.

For first time recipients the operation is performed under general anaesthesia. A Rutherford-Morrison incision is made on the preferred side. This provides excellent extraperitoneal access to the iliac vessels. The external iliac artery and vein are dissected out and following systemic heparinisation are cross clamped. The vein and artery are anastamosed to the iliacs and the clamps removed. The ureter is then implanted into the bladder and a stent is usually placed to maintain patency. The wounds are then closed and the patient recovered from surgery.

In the immediate phase a common problem encountered in cadaveric kidneys is acute tubular necrosis and this tends to resolve.

Graft survival times from cadaveric donors are typically of the order of 9 years and monozygotic twin transplant (live donor) may survive as long as 25 years.

84
Q

A 65 year old man presents with significant lower urinary tract symptoms and is diagnosed as having benign prostatic hyperplasia. Which of the following drug treatments will produce the slowest clinical response?

	Tamsulosin
	Alfuzosin
	Doxazosin
	Finasteride
	Terazosin
A

5 alpha reductase inhibitors have a more favorable side effect profile than α blockers.

Alpha blockers have a faster onset of action (but lower reduction of complications from BPH) than 5 α reductase inhibitors.

Benign prostatic hyperplasia occurs via an increase in the epithelial and stromal cell numbers in the peri-urethral zone of the prostate. BPH is very common and 90% of men aged over 80 will have at least microscopic evidence of benign prostatic hyperplasia. The causes of BPH are still not well understood, but the importance of androgens remains appreciated even if the exact role by which they induce BPH is elusive.

Presentation
The vast majority of men will present with lower urinary tract symptoms. These will typically be:

Poor flow
Nocturia
Hesitancy
Incomplete and double voiding
Terminal dribbling
Urgency
Incontinence

Investigation
Digital rectal examination to assess prostatic size and morphology.
Urine dipstick for infections and haematuria.
Uroflowmetry (a flow rate of >15ml/second helps to exclude BOO)
Bladder pressure studies may help identify detrusor failure and whilst may not form part of first line investigations should be included in those with atypical symptoms and prior to redo surgery.
Bladder scanning to demonstrate residual volumes. USS if high pressure chronic retention.

Management
Lifestyle changes such as stopping smoking and altering fluid intake may help those with mild symptoms.
Medical therapy includes alpha blockers and 5 α reductase inhibitors. The former work quickly on receptor zones located at the bladder neck. Cardiovascular side effects are well documented. The latter work on testosterone metabolising enzymes. Although they have a slower onset of action, the 5 α reductase inhibitors may prevent acute urinary retention.
Surgical therapy includes transurethral resection of the prostate and is the treatment of choice in those with severe symptoms and those who fail to respond to medical therapy. More tailored bladder neck incision procedures may be considered in those with small prostates. Retrograde ejaculation may occur following surgery. The change in the type of irrigation solutions used has helped to minimise the TURP syndrome of electrolyte disturbances.

85
Q

A 45 year old man develops a colocutaneous fistulae following reversal of a loop colostomy fashioned for the defunctioning of an anterior resection. Pre-operative gastrograffin enema showed no distal obstruction or anastamotic stricture.

A.	No further action needed
B.	Intravenous fluids
C.	Intravenous fluids and nasogastric tube
D.	Total parenteral nutrition and octreotide
E.	Defunctioning stoma
F.	Insertion of seton
G.	Intravenous octreotide
H.	Lay open fistula
A

No further action needed

Colocutaneous fistulae may occur as a result of anastomotic leakage following loop colostomy reversal. In the absence of abdominal signs a laparotomy is not necessarily required. Signs of wound sepsis may require antibiotics. Because there is not any distal obstruction (note normal pre-operative gastrograffin enema) these fistulae will usually close spontaneously.

A fistula is defined as an abnormal connection between two epithelial surfaces.
There are many types ranging from Branchial fistulae in the neck to entero-cutaneous fistulae abdominally.
In general surgical practice the abdominal cavity generates the majority and most of these arise from diverticular disease and Crohn’s.
As a general rule all fistulae will resolve spontaneously as long as there is no distal obstruction. This is particularly true of intestinal fistulae.

The four types of fistulae are:

Enterocutaneous
These link the intestine to the skin. They may be high (>500ml) or low output (<250ml) depending upon source. Duodenal /jejunal fistulae will tend to produce high volume, electrolyte rich secretions which can lead to severe excoriation of the skin. Colo-cutaneous fistulae will tend to leak faeculent material. Both fistulae may result from the spontaneous rupture of an abscess cavity onto the skin (such as following perianal abscess drainage) or may occur as a result of iatrogenic input. In some cases it may even be surgically desirable e.g. mucous fistula following sub total colectomy for colitis.

Suspect if there is excess fluid in the drain.

Enteroenteric or Enterocolic
This is a fistula that involves the large or small intestine. They may originate in a similar manner to enterocutaneous fistulae. A particular problem with this fistula type is that bacterial overgrowth may precipitate malabsorption syndromes. This may be particularly serious in inflammatory bowel disease.

Enterovaginal
Aetiology as above.

Enterovesicular
This type of fistula goes to the bladder. These fistulas may result in frequent urinary tract infections, or the passage of gas from the urethra during urination.

Management
Some rules relating to fistula management:
They will heal provided there is no underlying inflammatory bowel disease and no distal obstruction, so conservative measures may be the best option
Where there is skin involvement, protect the overlying skin, often using a well fitted stoma bag- skin damage is difficult to treat
A high output fistula may be rendered more easily managed by the use of octreotide, this will tend to reduce the volume of pancreatic secretions.
Nutritional complications are common especially with high fistula (e.g. high jejunal or duodenal) these may necessitate the use of TPN to provide nutritional support together with the concomitant use of octreotide to reduce volume and protect skin.
When managing perianal fistulae surgeons should avoid probing the fistula where acute inflammation is present, this almost always worsens outcomes.
When perianal fistulae occur secondary to Crohn’s disease the best management option is often to drain acute sepsis and maintain that drainage through the judicious use of setons whilst medical management is implemented.
Always attempt to delineate the fistula anatomy, for abscesses and fistulae that have an intra abdominal source the use of barium and CT studies should show a track. For perianal fistulae surgeons should recall Goodsall’s rule in relation to internal and external openings.

86
Q

A 43 year old man has suffered from small bowel Crohns disease for 15 years. Following a recent stricturoplasty he develops an enterocutaneous fistula which is high output. Small bowel follow through shows it to be 15 cm from the DJ flexure. His overlying skin is becoming excoriated.

A.	No further action needed
B.	Intravenous fluids
C.	Intravenous fluids and nasogastric tube
D.	Total parenteral nutrition and octreotide
E.	Defunctioning stoma
F.	Insertion of seton
G.	Intravenous octreotide
H.	Lay open fistula
A

Total parenteral nutrition and octreotide

This man has a high output and anatomically high fistula. Drying up the fistula with octreotide will not suffice, his nutrition is compromised and TPN will help.

A fistula is defined as an abnormal connection between two epithelial surfaces.
There are many types ranging from Branchial fistulae in the neck to entero-cutaneous fistulae abdominally.
In general surgical practice the abdominal cavity generates the majority and most of these arise from diverticular disease and Crohn’s.
As a general rule all fistulae will resolve spontaneously as long as there is no distal obstruction. This is particularly true of intestinal fistulae.

The four types of fistulae are:

Enterocutaneous
These link the intestine to the skin. They may be high (>500ml) or low output (<250ml) depending upon source. Duodenal /jejunal fistulae will tend to produce high volume, electrolyte rich secretions which can lead to severe excoriation of the skin. Colo-cutaneous fistulae will tend to leak faeculent material. Both fistulae may result from the spontaneous rupture of an abscess cavity onto the skin (such as following perianal abscess drainage) or may occur as a result of iatrogenic input. In some cases it may even be surgically desirable e.g. mucous fistula following sub total colectomy for colitis.

Suspect if there is excess fluid in the drain.

Enteroenteric or Enterocolic
This is a fistula that involves the large or small intestine. They may originate in a similar manner to enterocutaneous fistulae. A particular problem with this fistula type is that bacterial overgrowth may precipitate malabsorption syndromes. This may be particularly serious in inflammatory bowel disease.

Enterovaginal
Aetiology as above.

Enterovesicular
This type of fistula goes to the bladder. These fistulas may result in frequent urinary tract infections, or the passage of gas from the urethra during urination.

Management
Some rules relating to fistula management:
They will heal provided there is no underlying inflammatory bowel disease and no distal obstruction, so conservative measures may be the best option
Where there is skin involvement, protect the overlying skin, often using a well fitted stoma bag- skin damage is difficult to treat
A high output fistula may be rendered more easily managed by the use of octreotide, this will tend to reduce the volume of pancreatic secretions.
Nutritional complications are common especially with high fistula (e.g. high jejunal or duodenal) these may necessitate the use of TPN to provide nutritional support together with the concomitant use of octreotide to reduce volume and protect skin.
When managing perianal fistulae surgeons should avoid probing the fistula where acute inflammation is present, this almost always worsens outcomes.
When perianal fistulae occur secondary to Crohn’s disease the best management option is often to drain acute sepsis and maintain that drainage through the judicious use of setons whilst medical management is implemented.
Always attempt to delineate the fistula anatomy, for abscesses and fistulae that have an intra abdominal source the use of barium and CT studies should show a track. For perianal fistulae surgeons should recall Goodsall’s rule in relation to internal and external openings.

87
Q

A 33 year old lady presented with jaundice secondary to common bile duct stones. A cholecystectomy and common bile duct exploration is performed and the bile duct closed over a T tube. Six weeks post operatively a T tube cholangiogram is performed and shows no residual stones. The T tube is removed and five hours after removal a small amount of bile is noted to be draining from the T tube site.

A.	No further action needed
B.	Intravenous fluids
C.	Intravenous fluids and nasogastric tube
D.	Total parenteral nutrition and octreotide
E.	Defunctioning stoma
F.	Insertion of seton
G.	Intravenous octreotide
H.	Lay open fistula
A

No further action needed

When the bile duct is closed over a T Tube the latex in the T tube encourages tract fibrosis. This actually encourages a fistula to develop. The result is that when the tube is removed any bile which leaks will usually drain through the tract. Provided that there are no residual stones in the duct the fistula will slowly close. Persistent high volume drainage may be managed with ERCP and sphincterotomy.

A fistula is defined as an abnormal connection between two epithelial surfaces.
There are many types ranging from Branchial fistulae in the neck to entero-cutaneous fistulae abdominally.
In general surgical practice the abdominal cavity generates the majority and most of these arise from diverticular disease and Crohn’s.
As a general rule all fistulae will resolve spontaneously as long as there is no distal obstruction. This is particularly true of intestinal fistulae.

The four types of fistulae are:

Enterocutaneous
These link the intestine to the skin. They may be high (>500ml) or low output (<250ml) depending upon source. Duodenal /jejunal fistulae will tend to produce high volume, electrolyte rich secretions which can lead to severe excoriation of the skin. Colo-cutaneous fistulae will tend to leak faeculent material. Both fistulae may result from the spontaneous rupture of an abscess cavity onto the skin (such as following perianal abscess drainage) or may occur as a result of iatrogenic input. In some cases it may even be surgically desirable e.g. mucous fistula following sub total colectomy for colitis.

Suspect if there is excess fluid in the drain.

Enteroenteric or Enterocolic
This is a fistula that involves the large or small intestine. They may originate in a similar manner to enterocutaneous fistulae. A particular problem with this fistula type is that bacterial overgrowth may precipitate malabsorption syndromes. This may be particularly serious in inflammatory bowel disease.

Enterovaginal
Aetiology as above.

Enterovesicular
This type of fistula goes to the bladder. These fistulas may result in frequent urinary tract infections, or the passage of gas from the urethra during urination.

Management
Some rules relating to fistula management:
They will heal provided there is no underlying inflammatory bowel disease and no distal obstruction, so conservative measures may be the best option
Where there is skin involvement, protect the overlying skin, often using a well fitted stoma bag- skin damage is difficult to treat
A high output fistula may be rendered more easily managed by the use of octreotide, this will tend to reduce the volume of pancreatic secretions.
Nutritional complications are common especially with high fistula (e.g. high jejunal or duodenal) these may necessitate the use of TPN to provide nutritional support together with the concomitant use of octreotide to reduce volume and protect skin.
When managing perianal fistulae surgeons should avoid probing the fistula where acute inflammation is present, this almost always worsens outcomes.
When perianal fistulae occur secondary to Crohn’s disease the best management option is often to drain acute sepsis and maintain that drainage through the judicious use of setons whilst medical management is implemented.
Always attempt to delineate the fistula anatomy, for abscesses and fistulae that have an intra abdominal source the use of barium and CT studies should show a track. For perianal fistulae surgeons should recall Goodsall’s rule in relation to internal and external openings.

88
Q

A 52 year old woman presents with a neck swelling. Her GP reports that her TSH value is low at 0.01. A scintigraphy demonstrates a hot nodule. What is the most likely diagnosis?

	Graves disease
	Toxic adenoma
	Papillary thyroid cancer
	Hashimotos thyroiditis
	De Quervains thyroiditis
A

This lady has thyrotoxicosis (low TSH) and a hot solitary nodule indicating a toxic adenoma. Thyroid cancer rarely causes thyrotoxicosis or hot nodules.

Causes of hyperthyroidism include:
Diffuse toxic goitre (Graves Disease)
Toxic nodular goitre
Toxic nodule
Rare causes

Graves disease
Graves disease is characterised by a diffuse vascular goitre that appears at the same time as the clinical manifestations of hyperthyroidism. It is commonest in younger females and may be associated with eye signs. Thyrotoxic symptoms will predominate. Up to 50% of patients will have a familial history of autoimmune disorders. The glandular hypertrophy and hyperplasia occur as a result of the thyroid stimulating effects of the TSH receptor antibodies.

Toxic nodular goitre
In this disorder the goitre is present for a long period of time prior to the development of clinical symptoms. In most goitres the nodules are inactive and in some cases it is the internodular tissue that is responsible for the hyperthyroidism.

Toxic nodule
Overactive, autonomously functioning nodule. It may occur as part of generalised nodularity or be a true toxic adenoma. The TSH levels are usually low as the autonomously functioning thyroid tissue will exert a negative feedback effect.

Lethargy	Tachycardia
Emotionally labile	Agitation
Heat intolerance	Hot, moist palms
Weight loss	Exopthalmos
Excessive appetite	Thyroid goitre and bruit
Palpitations	Lid lag/retraction

Diagnosis
The most sensitive test for diagnosing hyperthyroidism is plasma T3 (which is raised). Note in hypothyroidism the plasma T4 and TSH are the most sensitive tests. A TSH level of <0.5U/L suggests hyperthyroidism. TSH receptor antibodies may be tested for in the diagnosis of Graves.

Treatment
First line treatment for Graves disease is usually medical and the block and replace regime is the favored option. Carbimazole is administered at higher doses and thyroxine is administered orally. Patient are maintained on this regime for between 6 and 12 months. Attempts are then made to wean off medication. Where relapse then occurs the options are between ongoing medical therapy, radioiodine or surgery.

89
Q

A 20 year old complains of severe pain and swelling of the scrotum after a cystoscopy. He had mumps as a child. The testis is tender. The urine dipstick is positive for leucocytes.

A.	Haematocele
B.	Epididymal cyst
C.	Hydrocele
D.	Testicular torsion
E.	Orchitis
F.	Epididymo-orchitis
A

Epididymo-orchitis

Epididymo-orchitis: acute pain and swelling after urological intervention. To differentiate from testicular torsion there is usually pyrexia and positive urine dipstick.

Inguinal hernia If inguinoscrotal swelling; cannot “get above it” on examination
Cough impulse may be present
May be reducible

Testicular tumours Often discrete testicular nodule (may have associated hydrocele)
Symptoms of metastatic disease may be present
USS scrotum and serum AFP and β HCG required

Acute epididymo-orchitis Often history of dysuria and urethral discharge
Swelling may be tender and eased by elevating testis
Most cases due to Chlamydia
Infections with other gram negative organisms may be associated with underlying structural abnormality

Epididymal cysts Single or multiple cysts
May contain clear or opalescent fluid (spermatoceles)
Usually occur over 40 years of age
Painless
Lie above and behind testis
It is usually possible to “get above the lump” on examination

Hydrocele Non painful, soft fluctuant swelling
Often possible to “get above it” on examination
Usually contain clear fluid
Will often transilluminate
May be presenting feature of testicular cancer in young men

Testicular torsion Severe, sudden onset testicular pain
Risk factors include abnormal testicular lie
Typically affects adolescents and young males
On examination testis is tender and pain not eased by elevation
Urgent surgery is indicated, the contra lateral testis should also be fixed

Varicocele Varicosities of the pampiniform plexus
Typically occur on left (because testicular vein drains into renal vein)
May be presenting feature of renal cell carcinoma
Affected testis may be smaller and bilateral varicoceles may affect fertility

Management
Testicular malignancy is always treated with orchidectomy via an inguinal approach. This allows high ligation of the testicular vessels and avoids exposure of another lymphatic field to the tumour.
Torsion is commonest in young teenagers and the history in older children can be difficult to elicit. Intermittent torsion is a recognised problem. The treatment is prompt surgical exploration and testicular fixation. This can be achieved using sutures or by placement of the testis in a Dartos pouch.
Varicoceles are usually managed conservatively. If there are concerns about testicular function of infertility then surgery or radiological management can be considered.
Epididymal cysts can be excised using a scrotal approach
Hydroceles are managed differently in children where the underlying pathology is a patent processus vaginalis and therefore an inguinal approach is used in children so that the processus can be ligated. In adults a scrotal approach is preferred and the hydrocele sac excised or plicated.

90
Q

A 20 year old complains of severe pain in the right scrotal area after jumping onto his moped. He has also noticed discomfort in this area over the past few months. On examination there is a swollen, painful testis that is drawn up into the groin.

A.	Haematocele
B.	Epididymal cyst
C.	Hydrocele
D.	Testicular torsion
E.	Orchitis
F.	Epididymo-orchitis
A

Testicular torsion

Testicular torsion: Severe pain which can be spontaneous or precipitated by minor trauma. There is usually severe pain and the patient will often not tolerate the testis being touched. Urgent scrotal exploration is indicated. It is associated with a high investment of the the tunica vaginalis with horizontal testicular lie, or when the epididymis and testis are separated by a mesorchium, in which case the twist occurs at that point.

Inguinal hernia If inguinoscrotal swelling; cannot “get above it” on examination
Cough impulse may be present
May be reducible
Testicular tumours Often discrete testicular nodule (may have associated hydrocele)
Symptoms of metastatic disease may be present
USS scrotum and serum AFP and β HCG required
Acute epididymo-orchitis Often history of dysuria and urethral discharge
Swelling may be tender and eased by elevating testis
Most cases due to Chlamydia
Infections with other gram negative organisms may be associated with underlying structural abnormality
Epididymal cysts Single or multiple cysts
May contain clear or opalescent fluid (spermatoceles)
Usually occur over 40 years of age
Painless
Lie above and behind testis
It is usually possible to “get above the lump” on examination
Hydrocele Non painful, soft fluctuant swelling
Often possible to “get above it” on examination
Usually contain clear fluid
Will often transilluminate
May be presenting feature of testicular cancer in young men
Testicular torsion Severe, sudden onset testicular pain
Risk factors include abnormal testicular lie
Typically affects adolescents and young males
On examination testis is tender and pain not eased by elevation
Urgent surgery is indicated, the contra lateral testis should also be fixed
Varicocele Varicosities of the pampiniform plexus
Typically occur on left (because testicular vein drains into renal vein)
May be presenting feature of renal cell carcinoma
Affected testis may be smaller and bilateral varicoceles may affect fertility

Management
Testicular malignancy is always treated with orchidectomy via an inguinal approach. This allows high ligation of the testicular vessels and avoids exposure of another lymphatic field to the tumour.
Torsion is commonest in young teenagers and the history in older children can be difficult to elicit. Intermittent torsion is a recognised problem. The treatment is prompt surgical exploration and testicular fixation. This can be achieved using sutures or by placement of the testis in a Dartos pouch.
Varicoceles are usually managed conservatively. If there are concerns about testicular function of infertility then surgery or radiological management can be considered.
Epididymal cysts can be excised using a scrotal approach
Hydroceles are managed differently in children where the underlying pathology is a patent processus vaginalis and therefore an inguinal approach is used in children so that the processus can be ligated. In adults a scrotal approach is preferred and the hydrocele sac excised or plicated.

91
Q

An 8 year old presents with scrotal swelling. He has just recovered from an acute viral illness with swelling of the parotid glands. On examination both testes are tender and slightly swollen.

A.	Haematocele
B.	Epididymal cyst
C.	Hydrocele
D.	Testicular torsion
E.	Orchitis
F.	Epididymo-orchitis
A

Orchitis

Orchitis may be associated with mumps viral infections.

Inguinal hernia If inguinoscrotal swelling; cannot “get above it” on examination
Cough impulse may be present
May be reducible
Testicular tumours Often discrete testicular nodule (may have associated hydrocele)
Symptoms of metastatic disease may be present
USS scrotum and serum AFP and β HCG required
Acute epididymo-orchitis Often history of dysuria and urethral discharge
Swelling may be tender and eased by elevating testis
Most cases due to Chlamydia
Infections with other gram negative organisms may be associated with underlying structural abnormality
Epididymal cysts Single or multiple cysts
May contain clear or opalescent fluid (spermatoceles)
Usually occur over 40 years of age
Painless
Lie above and behind testis
It is usually possible to “get above the lump” on examination
Hydrocele Non painful, soft fluctuant swelling
Often possible to “get above it” on examination
Usually contain clear fluid
Will often transilluminate
May be presenting feature of testicular cancer in young men
Testicular torsion Severe, sudden onset testicular pain
Risk factors include abnormal testicular lie
Typically affects adolescents and young males
On examination testis is tender and pain not eased by elevation
Urgent surgery is indicated, the contra lateral testis should also be fixed
Varicocele Varicosities of the pampiniform plexus
Typically occur on left (because testicular vein drains into renal vein)
May be presenting feature of renal cell carcinoma
Affected testis may be smaller and bilateral varicoceles may affect fertility

Management
Testicular malignancy is always treated with orchidectomy via an inguinal approach. This allows high ligation of the testicular vessels and avoids exposure of another lymphatic field to the tumour.
Torsion is commonest in young teenagers and the history in older children can be difficult to elicit. Intermittent torsion is a recognised problem. The treatment is prompt surgical exploration and testicular fixation. This can be achieved using sutures or by placement of the testis in a Dartos pouch.
Varicoceles are usually managed conservatively. If there are concerns about testicular function of infertility then surgery or radiological management can be considered.
Epididymal cysts can be excised using a scrotal approach
Hydroceles are managed differently in children where the underlying pathology is a patent processus vaginalis and therefore an inguinal approach is used in children so that the processus can be ligated. In adults a scrotal approach is preferred and the hydrocele sac excised or plicated.

92
Q

Which of these factors does not increase the risk of abdominal wound dehiscence following laparotomy?

	Jaundice
	Abdominal compartment syndrome
	Poorly controlled diabetes mellitus
	Administration of intravenous steroids
	Use of Ketamine as an anaesthetic agent
A

Ketamine does not affect healing. All the other situations in the list carry a strong association with poor healing and risk of dehisence

Abdominal wound dehiscence
This is a significant problem facing all surgeons who undertake abdominal surgery on a regular basis. Traditionally, it is said to occur when all layers of an abdominal mass closure fail and the viscera protrude externally (associated with 30% mortality).
It can be subdivided into superficial, in which the skin wound alone fails and complete, implying failure of all layers.

Factors which increase the risk are:

  • Malnutrition
  • Vitamin deficiencies
  • Jaundice
  • Steroid use
  • Major wound contamination (e.g. faecal peritonitis)
  • Poor surgical technique (Mass closure technique is the preferred method-Jenkins Rule)

When sudden full dehiscence occurs the management is as follows:

  • Analgesia
  • Intravenous fluids
  • Intravenous broad spectrum antibiotics
  • Coverage of the wound with saline impregnated gauze (on the ward)
  • Arrangements made for a return to theatre

Surgical strategy
Correct the underlying cause (e.g. TPN or NG feed if malnourished)
Determine the most appropriate strategy for managing the wound

Options
Resuturing of the wound This may be an option if the wound edges are healthy and there is enough tissue for sufficient coverage. Deep tension sutures are traditionally used for this purpose.
Application of a wound manager This is a clear dressing with removable front. Particularly suitable when some granulation tissue is present over the viscera or where there is a high output bowel fistula present in the dehisced wound.
Application of a ‘Bogota bag’ This is a clear plastic bag that is cut and sutured to the wound edges and is only a temporary measure to be adopted when the wound cannot be closed and will necessitate a return to theatre for definitive management.
Application of a VAC dressing system These can be safely used BUT ONLY if the correct layer is interposed between the suction device and the bowel. Failure to adhere to this absolute rule will almost invariably result in the development of multiple bowel fistulae and create an extremely difficult management problem.

93
Q

A 23 year old lady has suffered from diarrhoea for 8 months, she has also lost 2 Kg in weight. At colonoscopy appearances of melanosis coli are identified and confirmed on biopsy

A.	Campylobacter jejuni infection
B.	Salmonella gastroenteritis infection
C.	Crohns disease
D.	Ulcerative colitis
E.	Irritable bowel syndrome
F.	Ischaemic colitis
G.	Laxative abuse
H.	Clostridium difficile infection
A

Laxative abuse

This may occur as a result of laxative abuse and consists of lipofuschin laden marcophages that appear brown.

World Health Organisation definitions
Diarrhoea: > 3 loose or watery stool per day
Acute diarrhoea < 14 days
Chronic diarrhoea > 14 days

Acute Diarrhoea
Gastroenteritis May be accompanied by abdominal pain or nausea/vomiting
Diverticulitis Classically causes left lower quadrant pain, diarrhoea and fever
Antibiotic therapy More common with broad spectrum antibiotics
Clostridium difficile is also seen with antibiotic use
Constipation causing overflow A history of alternating diarrhoea and constipation may be given
May lead to faecal incontinence in the elderly

Chronic Diarrhoea
Irritable bowel syndrome Extremely common. The most consistent features are abdominal pain, bloating and change in bowel habit. Patients may be divided into those with diarrhoea predominant IBS and those with constipation predominant IBS.
Features such as lethargy, nausea, backache and bladder symptoms may also be present
Ulcerative colitis Bloody diarrhoea may be seen. Crampy abdominal pain and weight loss are also common. Faecal urgency and tenesmus may occur
Crohn’s disease Crampy abdominal pains and diarrhoea. Bloody diarrhoea less common than in ulcerative colitis. Other features include malabsorption, mouth ulcers perianal disease and intestinal obstruction
Colorectal cancer Symptoms depend on the site of the lesion but include diarrhoea, rectal bleeding, anaemia and constitutional symptoms e.g. Weight loss and anorexia
Coeliac disease
In children may present with failure to thrive, diarrhoea and abdominal distension
In adults lethargy, anaemia, diarrhoea and weight loss are seen. Other autoimmune conditions may coexist

Other conditions associated with diarrhoea include:
Thyrotoxicosis
Laxative abuse
Appendicitis with pelvic abscess or pelvic appendix
Radiation enteritis

Diagnosis
Stool culture
Abdominal and digital rectal examination
Consider colonoscopy (radiological studies unhelpful)
Thyroid function tests, serum calcium, anti endomysial antibodies, glucose

94
Q

A 68 year old lady has recently undergone an abdominal aortic aneurysm repair. The operation was performed electively and was uncomplicated. Since surgery she has had repeated episodes of diarrhoea.

A.	Campylobacter jejuni infection
B.	Salmonella gastroenteritis infection
C.	Crohns disease
D.	Ulcerative colitis
E.	Irritable bowel syndrome
F.	Ischaemic colitis
G.	Laxative abuse
H.	Clostridium difficile infection
A

Ischaemic colitis

The IMA is commonly ligated during an AAA repair and this may then render the left colon relatively ischaemic, thereby causing mesenteric colitis. Treatment is supportive and most cases will settle with conservative management.

World Health Organisation definitions
Diarrhoea: > 3 loose or watery stool per day
Acute diarrhoea < 14 days
Chronic diarrhoea > 14 days

Acute Diarrhoea
Gastroenteritis May be accompanied by abdominal pain or nausea/vomiting
Diverticulitis Classically causes left lower quadrant pain, diarrhoea and fever
Antibiotic therapy More common with broad spectrum antibiotics
Clostridium difficile is also seen with antibiotic use
Constipation causing overflow A history of alternating diarrhoea and constipation may be given
May lead to faecal incontinence in the elderly

Chronic Diarrhoea
Irritable bowel syndrome Extremely common. The most consistent features are abdominal pain, bloating and change in bowel habit. Patients may be divided into those with diarrhoea predominant IBS and those with constipation predominant IBS.
Features such as lethargy, nausea, backache and bladder symptoms may also be present
Ulcerative colitis Bloody diarrhoea may be seen. Crampy abdominal pain and weight loss are also common. Faecal urgency and tenesmus may occur
Crohn’s disease Crampy abdominal pains and diarrhoea. Bloody diarrhoea less common than in ulcerative colitis. Other features include malabsorption, mouth ulcers perianal disease and intestinal obstruction
Colorectal cancer Symptoms depend on the site of the lesion but include diarrhoea, rectal bleeding, anaemia and constitutional symptoms e.g. Weight loss and anorexia
Coeliac disease
In children may present with failure to thrive, diarrhoea and abdominal distension
In adults lethargy, anaemia, diarrhoea and weight loss are seen. Other autoimmune conditions may coexist

Other conditions associated with diarrhoea include:
Thyrotoxicosis
Laxative abuse
Appendicitis with pelvic abscess or pelvic appendix
Radiation enteritis

Diagnosis
Stool culture
Abdominal and digital rectal examination
Consider colonoscopy (radiological studies unhelpful)
Thyroid function tests, serum calcium, anti endomysial antibodies, glucose

95
Q

A 23 year old man is admitted to hospital with diarrhoea and severe abdominal pain. He was previously well and his illness has lasted 18 hours.

A.	Campylobacter jejuni infection
B.	Salmonella gastroenteritis infection
C.	Crohns disease
D.	Ulcerative colitis
E.	Irritable bowel syndrome
F.	Ischaemic colitis
G.	Laxative abuse
H.	Clostridium difficile infection
A

Campylobacter jejuni infection

Severe abdominal pain tends to favour Campylobacter infection.

World Health Organisation definitions
Diarrhoea: > 3 loose or watery stool per day
Acute diarrhoea < 14 days
Chronic diarrhoea > 14 days

Acute Diarrhoea
Gastroenteritis May be accompanied by abdominal pain or nausea/vomiting
Diverticulitis Classically causes left lower quadrant pain, diarrhoea and fever
Antibiotic therapy More common with broad spectrum antibiotics
Clostridium difficile is also seen with antibiotic use
Constipation causing overflow A history of alternating diarrhoea and constipation may be given
May lead to faecal incontinence in the elderly

Chronic Diarrhoea
Irritable bowel syndrome Extremely common. The most consistent features are abdominal pain, bloating and change in bowel habit. Patients may be divided into those with diarrhoea predominant IBS and those with constipation predominant IBS.
Features such as lethargy, nausea, backache and bladder symptoms may also be present
Ulcerative colitis Bloody diarrhoea may be seen. Crampy abdominal pain and weight loss are also common. Faecal urgency and tenesmus may occur
Crohn’s disease Crampy abdominal pains and diarrhoea. Bloody diarrhoea less common than in ulcerative colitis. Other features include malabsorption, mouth ulcers perianal disease and intestinal obstruction
Colorectal cancer Symptoms depend on the site of the lesion but include diarrhoea, rectal bleeding, anaemia and constitutional symptoms e.g. Weight loss and anorexia
Coeliac disease
In children may present with failure to thrive, diarrhoea and abdominal distension
In adults lethargy, anaemia, diarrhoea and weight loss are seen. Other autoimmune conditions may coexist

Other conditions associated with diarrhoea include:
Thyrotoxicosis
Laxative abuse
Appendicitis with pelvic abscess or pelvic appendix
Radiation enteritis

Diagnosis
Stool culture
Abdominal and digital rectal examination
Consider colonoscopy (radiological studies unhelpful)
Thyroid function tests, serum calcium, anti endomysial antibodies, glucose

96
Q

A 6 year old child presents with colicky abdominal pain, vomiting and the passage of red current jelly stool per rectum. On examination the child has a tender abdomen and a palpable mass in the right upper quadrant. Imaging shows an intussusception. Which of the conditions below is least recognised as a precipitant?

	Inflammation of Peyers patches
	Cystic fibrosis
	Meckels diverticulum
	Mesenteric cyst
	Mucosal polyps
A

Mesenteric cysts may be associated with intra abdominal catastrophes where these occur they are typically either intestinal volvulus or intestinal infarction. They seldom cause intussusception. Cystic fibrosis may lead to the formation of meconium ileus equivalent and plugs may occasionally serve as the lead points for an intussusception.

Intussusception typically presents with colicky abdominal pain and vomiting. The telescoping of the bowel produces mucosal ischaemia and bleeding may occur resulting in the passage of “red current jelly” stools. Recognised causes include lumenal pathologies such as polyps, lymphadenopathy and diseases such as cystic fibrosis. Idiopathic intussceception of the ileocaecal valve and terminal ileum is the most common variant and typically affects young children and toddlers.
The diagnosis is usually made by abdominal ultrasound investigation. The decision as to the optimal treatment is dictated by the patients physiological status and abdominal signs. In general, children who are unstable with localising peritoneal signs should undergo laparotomy, as should those in whom attempted radiological reduction has failed.
In relatively well children without localising signs attempted pneumatic reduction under fluroscopic guidance is the usual treatment.

97
Q

A 43 year old female has undergone a renal transplant 12 months previously. Over the past few weeks there have been concerns about deteriorating renal function.

A.	Non contrast abdominal CT scan
B.	DMSA scan
C.	PET/CT scan
D.	MAG 3 Renogram
E.	Renal ultrasound scan
F.	DTPA Scan
G.	Micturating cystourethrogram
H.	Intra venous urography
A

MAG 3 Renogram

Because it is excreted by renal tubular cells a MAG 3 renogram provides excellent imaging of renal function and is often used in investigating failing transplants.

DMSA scan
Dimercaptosuccinic acid (DMSA) scintigraphy
DMSA localises to the renal cortex with little accumulation in the renal papilla and medulla. It is useful for the identification of cortical defects and ectopic or aberrant kidneys. It does not provide useful information on the ureter of collecting system.

Diethylene-triamine-penta-acetic acid (DTPA)
This is primarily a glomerular filtration agent. It is most useful for the assessment of renal function. Because it is filtered at the level of the glomerulus it provides useful information about the GFR. Image quality may be degraded in patients with chronic renal impairment and derangement of GFR.

MAG 3 renogram
Mercaptoacetyle triglycine is an is extensively protein bound and is primarily secreted by tubular cells rather than filtered at the glomerulus. This makes it the agent of choice for imaging the kidneys of patients with existing renal impairment (where GFR is impaired).

Micturating cystourethrogram (MCUG scan)
This scan provides information relating to bladder reflux and is obtained by filling the bladder with contrast media (via a catheter) and asking the child to void. Images are taken during this phase and the degree of reflux can be calculated

Intra venous urography
This examination is conducted by the administration of intravenous iodinated contrast media. The agent is filtered by the kidneys and excreted and may provide evidence of renal stones or other structural lesions. A rough approximation of renal function may be obtained using the technique. But it is not primarily a technique to be used for this purpose. With the advent of widespread non contrast CT scan protocols for the detection of urinary tract calculi it is now rarely used.

PET/CT
This may be used to evaluate structurally indeterminate lesions in the staging of malignancy.

98
Q

A 5 year old boy presents with recurrent urinary tract infections and left sided loin pain. On investigation he is found to have a left sided PUJ obstruction, there are concerns that he may have developed renal scarring.

A.	Non contrast abdominal CT scan
B.	DMSA scan
C.	PET/CT scan
D.	MAG 3 Renogram
E.	Renal ultrasound scan
F.	DTPA Scan
G.	Micturating cystourethrogram
H.	Intra venous urography
A

DMSA scan

Although MAG 3 renograms may provide some information relating to the structural integrity of the kidney, many still consider a DMSA scan to be the gold standard for the detection of renal scarring (which is the main concern in PUJ obstruction and infections).

DMSA scan
Dimercaptosuccinic acid (DMSA) scintigraphy
DMSA localises to the renal cortex with little accumulation in the renal papilla and medulla. It is useful for the identification of cortical defects and ectopic or aberrant kidneys. It does not provide useful information on the ureter of collecting system.

Diethylene-triamine-penta-acetic acid (DTPA)
This is primarily a glomerular filtration agent. It is most useful for the assessment of renal function. Because it is filtered at the level of the glomerulus it provides useful information about the GFR. Image quality may be degraded in patients with chronic renal impairment and derangement of GFR.

MAG 3 renogram
Mercaptoacetyle triglycine is an is extensively protein bound and is primarily secreted by tubular cells rather than filtered at the glomerulus. This makes it the agent of choice for imaging the kidneys of patients with existing renal impairment (where GFR is impaired).

Micturating cystourethrogram (MCUG scan)
This scan provides information relating to bladder reflux and is obtained by filling the bladder with contrast media (via a catheter) and asking the child to void. Images are taken during this phase and the degree of reflux can be calculated

Intra venous urography
This examination is conducted by the administration of intravenous iodinated contrast media. The agent is filtered by the kidneys and excreted and may provide evidence of renal stones or other structural lesions. A rough approximation of renal function may be obtained using the technique. But it is not primarily a technique to be used for this purpose. With the advent of widespread non contrast CT scan protocols for the detection of urinary tract calculi it is now rarely used.

PET/CT
This may be used to evaluate structurally indeterminate lesions in the staging of malignancy.

99
Q

A 17 year old man is referred to the urology clinic. As a child he was diagnosed as having a right sided PUJ obstruction. However, he was lost to follow up. Over the past 7 months he has been complaining of recurrent episodes of right loin pain. A CT scan shows considerable renal scarring.

 A.	Non contrast abdominal CT scan
B.	DMSA scan
C.	PET/CT scan
D.	MAG 3 Renogram
E.	Renal ultrasound scan
F.	DTPA Scan
G.	Micturating cystourethrogram
H.	Intra venous urography
A

MAG 3 Renogram

In patients with long standing PUJ obstruction and renal scarring the main diagnostic question is whether the individual has sufficient renal function to consider a pyeloplasty or whether a primary nephrectomy is preferable. Since the CT has demonstrated scarring there is no use in obtaining a DMSA scan. Of the investigations listed both a DTPA and MAG 3 renogram will allow assessment of renal function. However, MAG 3 is superior in the assessment of renal function in damaged kidneys (as it is subjected to tubular secretion).

DMSA scan
Dimercaptosuccinic acid (DMSA) scintigraphy
DMSA localises to the renal cortex with little accumulation in the renal papilla and medulla. It is useful for the identification of cortical defects and ectopic or aberrant kidneys. It does not provide useful information on the ureter of collecting system.

Diethylene-triamine-penta-acetic acid (DTPA)
This is primarily a glomerular filtration agent. It is most useful for the assessment of renal function. Because it is filtered at the level of the glomerulus it provides useful information about the GFR. Image quality may be degraded in patients with chronic renal impairment and derangement of GFR.

MAG 3 renogram
Mercaptoacetyle triglycine is an is extensively protein bound and is primarily secreted by tubular cells rather than filtered at the glomerulus. This makes it the agent of choice for imaging the kidneys of patients with existing renal impairment (where GFR is impaired).

Micturating cystourethrogram (MCUG scan)
This scan provides information relating to bladder reflux and is obtained by filling the bladder with contrast media (via a catheter) and asking the child to void. Images are taken during this phase and the degree of reflux can be calculated

Intra venous urography
This examination is conducted by the administration of intravenous iodinated contrast media. The agent is filtered by the kidneys and excreted and may provide evidence of renal stones or other structural lesions. A rough approximation of renal function may be obtained using the technique. But it is not primarily a technique to be used for this purpose. With the advent of widespread non contrast CT scan protocols for the detection of urinary tract calculi it is now rarely used.

PET/CT
This may be used to evaluate structurally indeterminate lesions in the staging of malignancy.

100
Q

A 35-year-old female is admitted to hospital with hypovolaemic shock. CT abdomen reveals a haemorrhagic lesion in the right kidney. Following surgery and biopsy this is shown to be an angiomyolipomata. What is the most likely underlying diagnosis?

	Neurofibromatosis
	Budd-Chiari syndrome
	Hereditary haemorrhagic telangiectasia
	Von Hippel-Lindau syndrome
	Tuberous sclerosis
A
Tuberous sclerosis
Tuberous sclerosis (TS) is a genetic condition of autosomal dominant inheritance. Like neurofibromatosis, the majority of features seen in TS are neuro-cutaneous

Cutaneous features
depigmented ‘ash-leaf’ spots which fluoresce under UV light
roughened patches of skin over lumbar spine (Shagreen patches)
adenoma sebaceum: butterfly distribution over nose
fibromata beneath nails (subungual fibromata)
café-au-lait spots* may be seen

Neurological features
developmental delay
epilepsy (infantile spasms or partial)
intellectual impairment

Also
retinal hamartomas: dense white areas on retina (phakomata)
rhabdomyomas of the heart
gliomatous changes can occur in the brain lesions
polycystic kidneys, renal angiomyolipomata

*these of course are more commonly associated with neurofibromatosis. However a 1998 study of 106 children with TS found café-au-lait spots in 28% of patients

101
Q

A 22 year old man is participating in vigorous intercourse and suddenly feels a snap and his penis becomes swollen and painful. The admitting surgeon suspects a penile fracture. Which of the following is the most appropriate initial management?

	MRI scan of the penis
	Immediate surgical exploration
	CT scan of the penis
	USS of the penis
	Cystogram
A

Suspected penile fractures should be surgically explored and the injury repaired.

Penile fractures are a rare type of urological trauma that may be encountered. The injury is usually in the proximal part of the penile shaft and may involve the urethra. A classically history of a snapping sensation followed by immediate pain is usually given by the patient (usually during vigorous intercourse). On examination there is usually a tense haematoma and blood may be seen at the meatus if the urethra is injured.
When there is a a strong suspicion of the diagnosis the correct management is surgical and a circumferential incision made immediately inferior to the glans. The skin and superficial tissues are stripped back and the penile shaft inspected. Injuries are usually sutured and the urethra repaired over a catheter.

102
Q

Which of the following does not cause red urine?

	Rifampicin
	Phosphaturia
	Beetroot
	Rhubarb
	Blackberries
A

Phosphaturia causes cloudy urine not red urine

Causes of haematuria

Trauma
Injury to renal tract
Renal trauma commonly due to blunt injury (others penetrating injuries)
Ureter trauma rare: iatrogenic
Bladder trauma: due to RTA or pelvic fractures

Infection
Remember TB

Malignancy
Renal cell carcinoma (remember paraneoplastic syndromes): painful or painless
Urothelial malignancies: 90% are transitional cell carcinoma, can occur anywhere along the urinary tract. Painless haematuria.
Squamous cell carcinoma and adenocarcinoma: rare bladder tumours
Prostate cancer
Penile cancers: SCC

Renal disease
Glomerulonephritis
Stones
Microscopic haematuria common
Structural abnormalities
Benign prostatic hyperplasia (BPH) causes haematuria due to hypervascularity of the prostate gland
Cystic renal lesions e.g. polycystic kidney disease
Vascular malformations
Renal vein thrombosis due to renal cell carcinoma
Coagulopathy
Causes bleeding of underlying lesions

Drugs
Cause tubular necrosis or interstitial nephritis: aminoglycosides, chemotherapy
Interstitial nephritis: penicillin, sulphonamides, and NSAIDs
Anticoagulants

Benign
Exercise

Gynaecological
Endometriosis: flank pain, dysuria, and haematuria that is cyclical

Iatrogenic
Catheterisation
Radiotherapy; cystitis, severe haemorrhage, bladder necrosis

Pseudohaematuria For example following consumption of beetroot

103
Q

From the list below, which drug is known to cause haemorrhagic cystitis?

	Rifampicin
	Methotrexate
	Dexamethasone
	Leflunomide
	Cyclophosphamide
A

Cyclophosphamide is metabolised into a toxic metabolite acrolein. The effects may be attenuated by administration of large volumes of intravenous fluids and mesna (which neutralises the metabolite). The condition may be managed initially by bladder catheterisation and irrigation.

Causes of haematuria

Trauma
Injury to renal tract
Renal trauma commonly due to blunt injury (others penetrating injuries)
Ureter trauma rare: iatrogenic
Bladder trauma: due to RTA or pelvic fractures

Infection
Remember TB

Malignancy
Renal cell carcinoma (remember paraneoplastic syndromes): painful or painless
Urothelial malignancies: 90% are transitional cell carcinoma, can occur anywhere along the urinary tract. Painless haematuria.
Squamous cell carcinoma and adenocarcinoma: rare bladder tumours
Prostate cancer
Penile cancers: SCC

Renal disease
Glomerulonephritis
Stones
Microscopic haematuria common
Structural abnormalities
Benign prostatic hyperplasia (BPH) causes haematuria due to hypervascularity of the prostate gland
Cystic renal lesions e.g. polycystic kidney disease
Vascular malformations
Renal vein thrombosis due to renal cell carcinoma
Coagulopathy
Causes bleeding of underlying lesions

Drugs
Cause tubular necrosis or interstitial nephritis: aminoglycosides, chemotherapy
Interstitial nephritis: penicillin, sulphonamides, and NSAIDs
Anticoagulants

Benign
Exercise

Gynaecological
Endometriosis: flank pain, dysuria, and haematuria that is cyclical

Iatrogenic
Catheterisation
Radiotherapy; cystitis, severe haemorrhage, bladder necrosis

Pseudohaematuria For example following consumption of beetroot

104
Q

Which one of the following is least likely to cause malabsorption?

	Primary biliary cirrhosis
	Ileo-colic bypass
	Chronic pancreatitis
	Whipples disease
	Hartmans procedure
A

In a Hartmans procedure the sigmoid colon is removed and an end colostomy is fashioned. The bowel remains in continuity and no absorptive ability is lost.
An ileo-colic bypass leaves a redundant loop of small bowel in continuity, where the contents will stagnate and bacterial overgrowth will occur. Therefore this is recognised cause of malabsorption

Malabsorption is characterised by diarrhoea, steatorrhoea and weight loss. Causes may be broadly divided into intestinal (e.g. villous atrophy), pancreatic (deficiency of pancreatic enzyme production or secretion) and biliary (deficiency of bile-salts needed for emulsification of fats)

Intestinal causes of malabsorption
coeliac disease
Crohn's disease
tropical sprue
Whipple's disease
Giardiasis
brush border enzyme deficiencies (e.g. lactase insufficiency)

Pancreatic causes of malabsorption
chronic pancreatitis
cystic fibrosis
pancreatic cancer

Biliary causes of malabsorption
biliary obstruction
primary biliary cirrhosis

Other causes
bacterial overgrowth (e.g. systemic sclerosis, diverticulae, blind loop)
short bowel syndrome
lymphoma

105
Q

A 22 year old man presents with his first presentation of ulcerative colitis. Despite aggressive medical management with steroids, azathioprine and infliximab his symptoms remain unchanged and he has developed a megacolon.

A.	Proctectomy
B.	Anterior resection
C.	Panproctocolectomy
D.	Panproctocolectomy and ileoanal pouch
E.	Sub total colectomy
F.	Right hemicolectomy
A

Sub total colectomy

In patients with fulminant UC a sub total colectomy is the safest treatment option. The rectum will be left in situ as resection of the rectum in these acutely unwell patients carries an extremely high risk of complications.

Surgery for inflammatory bowel disease
Patients with inflammatory bowel disease (UC and Crohns) frequently present in surgical practice. Ulcerative colitis may be cured by surgical resection (Proctocolectomy), this is not the case in Crohns disease which may recur and affect other areas of the gastrointestinal tract.

Ulcerative colitis
Elective indications for surgery include disease that is requiring maximal therapy, or prolonged courses of steroids.
Longstanding UC is associated with a risk of malignant transformation. Dysplastic transformation of the colonic epithelium with associated mass lesions is an absolute indication for a proctocolectomy.
Emergency presentations of poorly controlled colitis that fails to respond to medical therapy should usually be managed with a sub total colectomy. Excision of the rectum is a procedure with a higher morbidity and is not generally performed in the emergency setting. An end ileostomy is usually created and the rectum either stapled off and left in situ, or, if the bowel is very oedematous, may be brought to the surface as a mucous fistula.
Patients with IBD have a high incidence of DVT and appropriate thromboprophylaxis is mandatory.
Restorative options in UC include an ileoanal pouch. This procedure can only be performed whilst the rectum is in situ and cannot usually be undertaken as a delayed procedure following proctectomy.
Ileoanal pouch complications include, anastomotic dehiscence, pouchitis and poor physiological function with seepage and soiling.

Crohns disease
Surgical resection of Crohns disease does not equate with cure, but may produce substantial symptomatic improvement.
Indications for surgery include complications such as fistulae, abscess formation and strictures.
Extensive small bowel resections may result in short bowel syndrome and localised stricturoplasty may allow preservation of intestinal length.
Staging of Crohns will usually involve colonoscopy and a small bowel study (e.g. MRI enteroclysis).
Complex perianal fistulae are best managed with long term draining seton sutures, complex attempts at fistula closure e.g. advancement flaps, may be complicated by non healing and fistula recurrence.
Severe perianal and / or rectal Crohns may require proctectomy. Ileoanal pouch reconstruction in Crohns carries a high risk of fistula formation and pouch failure and is not recommended.
Terminal ileal Crohns remains the commonest disease site and these patients may be treated with limited ileocaecal resections.
Terminal ileal Crohns may affect enterohepatic bile salt recycling and increase the risk of gallstones.

106
Q

A 22 year old lady has a long history of severe perianal Crohns disease with multiple fistulae. She is keen to avoid a stoma. However, she has progressive disease and multiple episodes of rectal bleeding. A colonoscopy shows rectal disease only and a small bowel study shows no involvement with Crohns.

A.	Proctectomy
B.	Anterior resection
C.	Panproctocolectomy
D.	Panproctocolectomy and ileoanal pouch
E.	Sub total colectomy
F.	Right hemicolectomy
A

Proctectomy

Severe rectal Crohns that has developed complications such as haemorrhage and multiple fistulae is usually best managed with proctectomy. Although a diverting stoma may reduce the risk of local sepsis it is unlikely to reduce the bleeding. She is keen to conserve a rectum, however, an ileoanal pouch in this setting is unwise.

Surgery for inflammatory bowel disease
Patients with inflammatory bowel disease (UC and Crohns) frequently present in surgical practice. Ulcerative colitis may be cured by surgical resection (Proctocolectomy), this is not the case in Crohns disease which may recur and affect other areas of the gastrointestinal tract.

Ulcerative colitis
Elective indications for surgery include disease that is requiring maximal therapy, or prolonged courses of steroids.
Longstanding UC is associated with a risk of malignant transformation. Dysplastic transformation of the colonic epithelium with associated mass lesions is an absolute indication for a proctocolectomy.
Emergency presentations of poorly controlled colitis that fails to respond to medical therapy should usually be managed with a sub total colectomy. Excision of the rectum is a procedure with a higher morbidity and is not generally performed in the emergency setting. An end ileostomy is usually created and the rectum either stapled off and left in situ, or, if the bowel is very oedematous, may be brought to the surface as a mucous fistula.
Patients with IBD have a high incidence of DVT and appropriate thromboprophylaxis is mandatory.
Restorative options in UC include an ileoanal pouch. This procedure can only be performed whilst the rectum is in situ and cannot usually be undertaken as a delayed procedure following proctectomy.
Ileoanal pouch complications include, anastomotic dehiscence, pouchitis and poor physiological function with seepage and soiling.

Crohns disease
Surgical resection of Crohns disease does not equate with cure, but may produce substantial symptomatic improvement.
Indications for surgery include complications such as fistulae, abscess formation and strictures.
Extensive small bowel resections may result in short bowel syndrome and localised stricturoplasty may allow preservation of intestinal length.
Staging of Crohns will usually involve colonoscopy and a small bowel study (e.g. MRI enteroclysis).
Complex perianal fistulae are best managed with long term draining seton sutures, complex attempts at fistula closure e.g. advancement flaps, may be complicated by non healing and fistula recurrence.
Severe perianal and / or rectal Crohns may require proctectomy. Ileoanal pouch reconstruction in Crohns carries a high risk of fistula formation and pouch failure and is not recommended.
Terminal ileal Crohns remains the commonest disease site and these patients may be treated with limited ileocaecal resections.
Terminal ileal Crohns may affect enterohepatic bile salt recycling and increase the risk of gallstones.

107
Q

A 22 year old man has a long history of ulcerative colitis. His symptoms are well controlled with steroids. However, attempts at steroid weaning and use of steroid sparing drugs have repeatedly failed. He wishes to avoid a permanent stoma.

A.	Proctectomy
B.	Anterior resection
C.	Panproctocolectomy
D.	Panproctocolectomy and ileoanal pouch
E.	Sub total colectomy
F.	Right hemicolectomy
A

Panproctocolectomy and ileoanal pouch

In patients with UC where medical management is not successful, surgical resection may offer a chance of cure. Those patients wishing to avoid a permanent stoma may be considered for an ileoanal pouch. However, this procedure is only offered in the elective setting.

Surgery for inflammatory bowel disease
Patients with inflammatory bowel disease (UC and Crohns) frequently present in surgical practice. Ulcerative colitis may be cured by surgical resection (Proctocolectomy), this is not the case in Crohns disease which may recur and affect other areas of the gastrointestinal tract.

Ulcerative colitis
Elective indications for surgery include disease that is requiring maximal therapy, or prolonged courses of steroids.
Longstanding UC is associated with a risk of malignant transformation. Dysplastic transformation of the colonic epithelium with associated mass lesions is an absolute indication for a proctocolectomy.
Emergency presentations of poorly controlled colitis that fails to respond to medical therapy should usually be managed with a sub total colectomy. Excision of the rectum is a procedure with a higher morbidity and is not generally performed in the emergency setting. An end ileostomy is usually created and the rectum either stapled off and left in situ, or, if the bowel is very oedematous, may be brought to the surface as a mucous fistula.
Patients with IBD have a high incidence of DVT and appropriate thromboprophylaxis is mandatory.
Restorative options in UC include an ileoanal pouch. This procedure can only be performed whilst the rectum is in situ and cannot usually be undertaken as a delayed procedure following proctectomy.
Ileoanal pouch complications include, anastomotic dehiscence, pouchitis and poor physiological function with seepage and soiling.

Crohns disease
Surgical resection of Crohns disease does not equate with cure, but may produce substantial symptomatic improvement.
Indications for surgery include complications such as fistulae, abscess formation and strictures.
Extensive small bowel resections may result in short bowel syndrome and localised stricturoplasty may allow preservation of intestinal length.
Staging of Crohns will usually involve colonoscopy and a small bowel study (e.g. MRI enteroclysis).
Complex perianal fistulae are best managed with long term draining seton sutures, complex attempts at fistula closure e.g. advancement flaps, may be complicated by non healing and fistula recurrence.
Severe perianal and / or rectal Crohns may require proctectomy. Ileoanal pouch reconstruction in Crohns carries a high risk of fistula formation and pouch failure and is not recommended.
Terminal ileal Crohns remains the commonest disease site and these patients may be treated with limited ileocaecal resections.
Terminal ileal Crohns may affect enterohepatic bile salt recycling and increase the risk of gallstones.

108
Q

A 32 year old man is admitted with a distended tense abdomen. He previously underwent a difficult appendicectomy 1 year previously and was discharged. At laparotomy the abdomen is filled with a gelatinous substance.

A.	Metastatic adenocarcinoma of the pancreas
B.	Metastatic appendiceal carcinoid
C.	Metastatic colonic cancer
D.	Pseudomyxoma peritonei
E.	MALT lymphoma
F.	Retroperitoneal liposarcoma
G.	Retroperitoneal fibrosis
A

Pseudomyxoma peritonei

Pseudomyxoma is classically associated with mucin production and the appendix is the commonest source.

Pseudomyxoma peritoneii- Curative treatment is peritonectomy (Sugarbaker procedure) and heated intra peritoneal chemotherapy.

Pseudomyxoma Peritonei
Rare mucinous tumour
Most commonly arising from the appendix (other abdominal viscera are also recognised as primary sites)
Incidence of 1-2/1,000,000 per year
The disease is characterised by the accumulation of large amounts of mucinous material in the abdominal cavity

Treatment
Is usually surgical and consists of cytoreductive surgery (and often peritonectomy c.f Sugarbaker procedure) combined with intra peritoneal chemotherapy with mitomycin C.

Survival is related to the quality of primary treatment and in Sugarbakers own centre 5 year survival rates of 75% have been quoted. Patients with disseminated intraperitoneal malignancy from another source fare far worse.
In selected patients a second look laparotomy is advocated and some practice this routinely.

109
Q

A 62 year old man is admitted with dull lower back pain and abdominal discomfort. On examination he is hypertensive and a lower abdominal fullness is elicited on examination. An abdominal ultrasound demonstrates hydronephrosis and intravenous urography demonstrated medially displaced ureters. A CT scan shows a periaortic mass.

A.	Metastatic adenocarcinoma of the pancreas
B.	Metastatic appendiceal carcinoid
C.	Metastatic colonic cancer
D.	Pseudomyxoma peritonei
E.	MALT lymphoma
F.	Retroperitoneal liposarcoma
G.	Retroperitoneal fibrosis
A

Retroperitoneal fibrosis

Retroperitoneal fibrosis is an uncommon condition and its aetiology is poorly understood. In a significant proportion the ureters are displaced medially. In most retroperitoneal malignancies they are displaced laterally. Hypertension is another common finding. A CT scan will often show a para-aortic mass

110
Q

A 48 year old lady is admitted with abdominal distension. On examination she is cachectic and has ascites. Her CA19-9 returns highly elevated.

A.	Metastatic adenocarcinoma of the pancreas
B.	Metastatic appendiceal carcinoid
C.	Metastatic colonic cancer
D.	Pseudomyxoma peritonei
E.	MALT lymphoma
F.	Retroperitoneal liposarcoma
G.	Retroperitoneal fibrosis
A

Metastatic adenocarcinoma of the pancreas

Although not specific CA 19-9 in the context of this history is highly suggestive of pancreatic cancer over the other scenarios.

111
Q

Which of the following are not true of follicular thyroid cancer?

They often appear to be encapsulated.
Those with a Hurthle cell subtype have an excellent prognosis.
Haematogenous metastasis is more common than in Papillary carcinoma.
The overall mortality rate is 24%.
Vascular invasion is seen in up to 60% of cases.
A

The Hurthle cell subtype have a worse prognosis.

Thyroid malignancy
Papillary carcinoma
Commonest sub-type
Accurately diagnosed on fine needle aspiration cytology
Histologically, they may demonstrate psammoma bodies (areas of calcification) and so called ‘orphan Annie’ nuclei
They typically metastasise via the lymphatics and thus laterally located apparently ectopic thyroid tissue is usually a metastasis from a well differentiated papillary carcinoma

Follicular carcinoma
Are less common than papillary lesions
Like papillary tumours, they may present as a discrete nodule. Although they appear to be well encapsulated macroscopically there is invasion on microscopic evaluation
Lymph node metastases are uncommon and these tumours tend to spread haematogenously. This translates into a higher mortality rate
Follicular lesions cannot be accurately diagnosed on fine needle aspiration cytology and thus all follicular FNA’s (THY 3f) will require at least a hemi thyroidectomy

Anaplastic carcinoma
Less common and tend to occur in elderly females
Disease is usually advanced at presentation and often only palliative decompression and radiotherapy can be offered.

Medullary carcinoma
These are tumours of the parafollicular cells ( C Cells) and are of neural crest origin.
The serum calcitonin may be elevated which is of use when monitoring for recurrence.
They may be familial and occur as part of the MEN -2A disease spectrum.
Spread may be either lymphatic or haematogenous and as these tumours are not derived primarily from thyroid cells they are not responsive to radioiodine.

Lymphoma
These respond well to radiotherapy
Radical surgery is unnecessary once the disease has been diagnosed on biopsy material. Such biopsy material is not generated by an FNA and thus a core biopsy has to be obtained (with care!).

112
Q

A 28-year-old female undergoes a renal transplant for focal segmental glomerulosclerosis. Within hours of the operation the patient becomes unwell with features consistent with severe systemic inflammatory response syndrome. The patient is immediately taken back to theatre and the transplanted kidney is removed. What type of immunoglobulins are responsible for the graft rejection?

	IgE
	IgM
	IgG
	IgD
	IgA
A

Hyperacute graft rejection is due to pre-existent antibodies to HLA antigens and is therefore IgG mediated

Renal transplant:HLA typing and graft failure
The human leucocyte antigen (HLA) system is the name given to the major histocompatibility complex (MHC) in humans. It is coded for on chromosome 6.

Some basic points on the HLA system
Class 1 antigens include A, B and C. Class 2 antigens include DP,DQ and DR
When HLA matching for a renal transplant the relative importance of the HLA antigens are as follows DR > B > A

Graft survival
1 year = 90%, 10 years = 60% for cadaveric transplants
1 year = 95%, 10 years = 70% for living-donor transplants

Post-op problems
ATN of graft
Vascular thrombosis
Urine leakage
UTI

Hyperacute acute rejection
Due to antibodies against donor HLA type 1 antigens
Rarely seen due to HLA matching

Acute graft failure (< 6 months)
Usually due to mismatched HLA
Other causes include cytomegalovirus infection
Management: give steroids, if resistant use monoclonal antibodies

Causes of chronic graft failure (> 6 months)
Chronic allograft nephropathy
Ureteric obstruction
Recurrence of original renal disease (MCGN > IgA > FSGS)

113
Q

Which of the following procedures is not performed for obesity?

	Sleeve gastrectomy
	Gastric band
	Intra gastric balloon
	Mckeown procedure
	Small bowel bypass
A

A McKeown procedure is a total oesophagectomy

Bariatric surgery
Obesity is a major health problem in the Western world. Surgical solutions to the problem have evolved dramatically over the past few years. Randomised controlled trials have shown that dramatic weight loss can be achieved following surgical interventions compared with standard medical therapy. The weight loss process is also more durable following surgery than with non surgical interventions.

Case selection
BMI >/= 40 kg/m2 or between 35-40 kg/m2 and other significant disease (for example, type 2 diabetes, hypertension) that could be improved with weight loss.

Pre-requisites to surgery (NICE UK Guidelines)
All non-surgical measures have failed to achieve or maintain adequate clinically beneficial weight loss for at least 6 months.
Will receive intensive specialist management
They are generally fit for anaesthesia and surgery
They commit to the need for long-term follow-up
First-line option for adults with a BMI > 40 kg/m2 in whom surgical intervention is considered appropriate; consider orlistat if there is a long waiting list.

Surgical options
Adjustable gastric band
Laparoscopic placement of adjustable band around proximal stomach.
Contains an adjustable filling port
Effective method for lifestyle control
Reversible
Takes longer to achieve target weight
Complications such as band erosion (rare), slippage or loss of efficacy may require re-intervention
Gastric bypass
Combines changes to reservoir size with malabsorptive procedure for more enduring weight loss.
Technically more challenging
Risks related to anastomoses (2% leak rate)
Irreversible
Up to 50% may become B12 deficient
Sleeve gastrectomy
Resection of stomach using stapling devices
Less popular now as initial promising results not sustained

114
Q

A 52 year old woman presents with an acute ischaemic right arm. She is found to have fast atrial fibrillation. Her blood results reveal a free T4 level of 20 and a TSH of < 0.01.

A.	Thyrotoxicosis
B.	Sick euthyroid syndrome
C.	Hypothyroidism
D.	Poor compliance with thyroid medication
E.	Hashimotos thyroiditis
F.	Multinodular goitre
A

Thyrotoxicosis

The diagnosis is thyrotoxicosis. An elevated T4 and a low TSH should indicate this diagnosis. Remember atrial fibrillation and its complications i.e acute ischaemic limbs can be precipitated by hyperthyroid disorders.

115
Q

A 42 year old woman presents with a goitre. On examination the goitre feels ‘lumpy’. The blood results reveal a TSH of 12 and a free T4 of 2. Antithyroid peroxidase antibodies are high.

A.	Thyrotoxicosis
B.	Sick euthyroid syndrome
C.	Hypothyroidism
D.	Poor compliance with thyroid medication
E.	Hashimotos thyroiditis
F.	Multinodular goitre
A

Hashimotos thyroiditis

Hashimotos usually presents in women aged between 30- 50 years. They are normally associated with a goitre. To differentiate from hypothyroidism, the antithyroid peroxidase antibodies will be elevated.

116
Q

A 55 year old man is on the intensive care unit for many months after open aortic surgery. He is maintained on total parenteral nutrition. Clinically he is euthyroid, but his thyroid function tests reveal a low TSH and low T4.

A.	Thyrotoxicosis
B.	Sick euthyroid syndrome
C.	Hypothyroidism
D.	Poor compliance with thyroid medication
E.	Hashimotos thyroiditis
F.	Multinodular goitre
A

Sick euthyroid syndrome

Sick euthyroid syndrome is most commonly seen in chronically ill patients or those with starvation. The thyroid function tests are often low and the patient clinically euthyroid.

117
Q

Which of the following variables is not included in the Rockall score?

	Congestive cardiac failure
	Liver failure
	Systolic blood pressure < 100mmHg
	Aspirin usage
	Age
A

A patients should have their Rockall score calculated following endoscopy for upper GI haemorrhage

Mnemonic for Rockall score
ABCDE 
A: Age 
B: Blood pressure drop (Shock) 
C: Co-morbidity 
D: Diagnosis 
E: Evidence of bleeding

Rockall Score

Applies to upper gastrointestinal bleeding

Score <3 = Good prognosis (mortality approx. 2%)

Score >8= High mortality (Mortality approx. 40%)

Upper gastrointestinal bleeding
Patients may present with the following:
Haematemesis and/ or malaena
Epigastric discomfort
Sudden collapse

The extent to which these will occur will depend upon the source. Mortality is higher in patients presenting with haematemesis than malaena alone.

Oesophageal bleeding

Oesophagitis Small volume of fresh blood, often streaking vomit. Malaena rare. Often ceases spontaneously. Usually history of antecedent GORD type symptoms.

Cancer Usually small volume of blood, except as pre terminal event with erosion of major vessels. Often associated symptoms of dysphagia and constitutional symptoms such as weight loss. May be recurrent until malignancy managed.

Mallory Weiss Tear Typically brisk small to moderate volume of bright red blood following bout of repeated vomiting. Malaena rare. Usually ceases spontaneously.

Varices Usually large volume of fresh blood. Swallowed blood may cause malaena. Often associated with haemodynamic compromise. May stop spontaneously but re-bleeds are common until appropriately managed.

Gastric Bleeding

Gastric cancer May be frank haematemesis or altered blood mixed with vomit. Usually prodromal features of dyspepsia and may have constitutional symptoms. Amount of bleeding variable but erosion of major vessel may produce considerable haemorrhage.

Dieulafoy Lesion Often no prodromal features prior to haematemesis and malaena, but this arteriovenous malformation may produce quite considerable haemorrhage and may be difficult to detect endoscopically.

Diffuse erosive gastritis Usually haematemesis and epigastric discomfort. Usually there is an underlying cause such as recent NSAID usage. Large volume haemorrhage may occur with considerable haemodynamic compromise.

Gastric ulcer Small low volume bleeds more common so would tend to present as iron deficiency anaemia. Erosion into a significant vessel may produce considerable haemorrhage and haematemesis.

Duodenum
Most common cause of major haemorrhage is a posteriorly sited duodenal ulcer. However, ulcers at any site in the duodenum may present with haematemesis, malaena and epigastric discomfort. The pain of duodenal ulcer is slightly different to that of gastric ulcers and often occurs several hours after eating. Peri ampullary tumours may bleed but these are rare. In patients with previous abdominal aortic aneurysm surgery aorto-enteric fistulation remains a rare but important cause of major haemorrhage associated with high mortality.

Management
Admission to hospital careful monitoring, cross match blood, check FBC, LFTs, U+E and Clotting (as a minimum)
Patients with on-going bleeding and haemodynamic instability are likely to require O negative blood pending cross matched blood
Early control of airway is vital (e.g. Drowsy patient with liver failure)
Patients with suspected varices should receive terlipressin prior to endoscopy
Ideally all patients admitted with upper gastrointestinal haemorrhage should undergo Upper GI endoscopy within 24 hours of admission. In those who are unstable this should occur immediately after resuscitation or in tandem with it. The endoscopy department is a potentially dangerous place for unstable patients and it may be safer to perform the endoscopy in theatre with an anaesthetist present.
Varices should be banded or subjected to sclerotherapy. If this is not possible owing to active bleeding then a Sengaksten- Blakemore tube (or Minnesota tube) should be inserted. This should be done with care; gastric balloon should be inflated first and oesophageal balloon second. Remember the balloon will need deflating after 12 hours (ideally sooner) to prevent necrosis. Portal pressure should be lowered by combination of medical therapy +/- TIPSS.
Patients with erosive oesophagitis / gastritis should receive a proton pump inhibitor.
Mallory Weiss tears will typically resolve spontaneously
Identifiable bleeding points should receive combination therapy of injection of adrenaline and either a thermal or mechanical treatment. All who have received intervention should receive a continuous infusion of a proton pump inhibitor (IV omeprazole for 72 hours) to reduce the re-bleeding rate.
Patients with diffuse erosive gastritis who cannot be managed endoscopically and continue to bleed may require gastrectomy
Bleeding ulcers that cannot be controlled endoscopically may require laparotomy and ulcer underruning

Indications for surgery
Patients > 60 years
Continued bleeding despite endoscopic intervention
Recurrent bleeding
Known cardiovascular disease with poor response to hypotension

Surgery
Duodenal ulcer
Laparotomy, duodenotomy and under running of the ulcer. If bleeding is brisk then the ulcer is almost always posteriorly sited and will have invaded the gastroduodenal artery. Large bites using 0 Vicryl are taken above and below the ulcer base to occlude the vessel. The duodenotomy should be longitudinal but closed transversely to avoid stenosis.

For gastric ulcer
Under-running of the bleeding site 
Partial gastrectomy-antral ulcer
Partial gastrectomy or under running the ulcer- lesser curve ulcer (involving left gastric artery)
Total gastrectomy if bleeding persists

Summary of Acute Upper GI bleeding recommendations:
The need for admission and timing of endoscopic intervention may be predicted by using the Blatchford score. This considers a patients Hb, serum urea, pulse rate and blood pressure. Those patients with a score of 0 are low risk, all others are considered high risk and require admission and endoscopy.
The requirement for pre endoscopic proton pump inhibition is contentious. In the UK the National Institute of Clinical Excellence guidelines suggest the pre endoscopic PPI therapy is unnecessary. Whilst it is accepted that such treatment has no impact on mortality or morbidity a Cochrane review of this practice in 2007 did suggest that it reduced the stigmata of recent haemorrhage at endoscopy. As a result many will still administer PPI to patients prior to endoscopic intervention.
Following endoscopy it is important to calculate the Rockall score for patients to determine their risk of rebleeding and mortality. A score of 3 or less is associated with a rebleeding rate of 4% and a very low risk of mortality and identifies a group of patients suitable for early discharge.

118
Q

What is the earliest complication that can occur following construction of an ileostomy?

	Prolapse
	Retraction
	Necrosis
	Parastomal hernia
	Dermatitis
A

Necrosis

Construction of a stoma may be complicated by several factors. Necrosis may occur because of technical errors in mesenteric division, excessive tension or failure to construct a fascial defect of adequate size to permit safe passage of the mesentery and the bowel.

Ileostomy

Ileostomies are generally fashioned in the right iliac fossa in a triangle between the anterior superior iliac spine, symphysis pubis and umbilicus. They should lie one-third of the distance between the umbilicus and anterior superior iliac spine. A 2cm skin incision is made and dissection continued through the rectus muscle. A cruciate incision should be made, and generally dilated to admit two fingers. The ileum is brought through the incisions and should generally be spouted to a final length of 2.5cm. Ileostomies that are too short may cause problems with appliance fixation and those which are too long may cause problems with tension and subsequent ulceration or prolapse.

Complications following ileostomy construction include dermatitis (most common), bowel obstruction (usually adhesional) and prolapse.

Ileostomy output is roughly in the range of 5-10ml/Kg/ 24 hours. Output in excess of 20ml/Kg/24 hours usually requires supplementary intravenous fluids. Excessive fluid losses are generally managed by administration of oral loperamide (up to 4mg QDS) to try and slow the output. Foods containing gelatine may also thicken output. Early high output is not uncommon and most patients (50%) will respond to conservative management.

119
Q

A 43 year old lady is diagnosed as having a malignant lesion in the inferior aspect of her left breast. There is palpable axillary lymphadenopathy. What is the most appropriate course of action?

Mastectomy and axillary node clearance
Wide local excision and axillary node clearance
Wide local excision and sentinel lymph node biopsy
Image guided fine needle aspiration of the axillary nodes
CT scanning of the chest, abdomen and pelvis
A

Image guided fine needle aspiration of the axillary nodes

Where axillary nodal involvement is suspected from the outset it is important to establish whether this is the case prior to surgery. This is because, if axillary metastatic disease is present then the correct management would be an axillary node clearance and this is irrespective of the surgical plans for the breast primary. In the case of breast cancer, image guided FNAC is acceptable as it is accurate and if carcinoma cells are identified at FNA then axillary node clearance can be performed. If FNAC is negative then a sentinel node biopsy should accompany excision of the primary tumour.
Where the axilla is clinically clear on palpation and imaging then a sentinel lymph node biopsy should accompany excision of the primary tumour.

Management of the axilla- breast cancer

Lymph node stage
Accurate staging of the axilla is an essential component of breast cancer management. Involvement of the axillary nodes has an adverse effect on prognosis with 10 year survival reduced from 75% to 25%[1]. Involvement of level 3 nodes carries the worst prognosis[2]. Historically, management of the axilla ranged from limited level 1 axillary node excision through to full level 3 axillary nodal clearances. Attempts to mimimise the morbidity of axillary node clearance led to targeted operations including axillary nodal sampling and sentinel lymph node biopsy. The focus on sentinel lymph node biopsy has led to more detailed pathological analysis of excised lymph nodes (e.g. using immunohistochemistry). This has led to increasing focus on the develop of axillary nodal micrometastasis. The presence of micrometastasis and its impact on survival is debated. In some studies it seems to confer an increased risk of locoregional recurrence [3] and a reduction of disease free survival [4], whilst in others it shows no overall impact[5]. It is important to distinguish between micrometastasis and isolated tumour cells, as the latter do not have an adverse impact on prognosis[6]. The need for definitive treatment of the axilla in women with positive sentinel nodes was addressed by the ASCOG Z0011trial. In this trial women were randomised to either undergo axillary node clearance or observation, groups were adjusted for other prognostic factors and treatments. The investigators found no survival benefit in routinely undertaking axillary node clearance where axillary nodal disease was limited in its extent.
Regardless of the options in women with a low risk axilla, those individuals who have overt evidence of axillary nodal involvement either through positive SLNB or preoperative USS and FNA, should still receive axillary clearance as a standard of care.

120
Q

A male infant, born at term appears well following delivery. Six hours later, he is noted to have bilious vomiting by the paediatricians. On examination he seems well and his abdomen is soft and non tender.

A.	Liver transplant
B.	Rectal biopsy
C.	Roux-en-Y portojejunostomy
D.	Ramstedt pyloromyotomy
E.	Abdominal CT scan
F.	Upper GI contrast study
G.	Laparotomy and formation of stoma
H.	Laparotomy
I.	Colonoscopy
J.	Gastroscopy
A

Upper GI contrast study

Bilious vomiting in neonates is a surgical emergency and is intestinal malrotation and volvulus until otherwise proven. It is investigated with an upper GI contrast study. Contrast should be seen to exit the stomach and the location of the DJ flexure is noted (it lies to the left of the midline). If this is not the case, or the study is inconclusive, a laparotomy is performed.

Acute appendicitis	
Uncommon under 3 years
When occurs may present atypically
Mesenteric adenitis	
Central abdominal pain and URTI
Conservative management

Intussusception
Telescoping bowel
Proximal to or at the level of, ileocaecal valve
6-9 months age
Colicky pain, diarrhoea and vomiting, sausage shaped mass, red jelly stool.
Treatment: reduction with air insufflation

Malrotation
High caecum at the midline
Feature in exomphalos, congenital diaphragmatic hernia, intrinsic duodenal atresia
May be complicated by development of volvulus, infant with volvulus may have bile stained vomiting
Diagnosis is made by upper GI contrast study and USS
Treatment is by laparotomy, if volvulus is present (or at high risk of occurring then a ladds procedure is performed

Hirschsprung’s disease
Absence of ganglion cells from myenteric and submucosal plexuses
Occurs in 1/5000 births
Full thickness rectal biopsy for diagnosis
Delayed passage of meconium and abdominal distension
Treatment is with rectal washouts initially, thereafter an anorectal pull through procedure

Oesophageal atresia
Associated with tracheo-oesophageal fistula and polyhydramnios
May present with choking and cyanotic spells following aspiration
VACTERL associations

Meconium ileus
Usually delayed passage of meconium and abdominal distension
Majority have cystic fibrosis
X-Rays may not show a fluid level as the meconium is viscid (depends upon feeding), PR contrast studies may dislodge meconium plugs and be therapeutic
Infants who do not respond to PR contrast and NG N-acetyl cysteine will require surgery to remove the plugs

Biliary atresia
Jaundice > 14 days
Increased conjugated bilirubin
Urgent Kasai procedure

Necrotising enterocolitis
Prematurity is the main risk factor
Early features include abdominal distension and passage of bloody stools
X-Rays may show pneumatosis intestinalis and evidence of free air
Increased risk when empirical antibiotics are given to infants beyond 5 days
Treatment is with total gut rest and TPN, babies with perforations will require laparotomy

121
Q

A 2 month old baby presents with jaundice. He has an elevated conjugated bilirubin level. Diagnosis is confirmed by cholangiography during surgery.

A.	Liver transplant
B.	Rectal biopsy
C.	Roux-en-Y portojejunostomy
D.	Ramstedt pyloromyotomy
E.	Abdominal CT scan
F.	Upper GI contrast study
G.	Laparotomy and formation of stoma
H.	Laparotomy
I.	Colonoscopy
J.	Gastroscopy
A

Roux-en-Y portojejunostomy

This child has biliary atresia. The aim is to avoid liver transplantation (however, most will come to transplant in time).

Pyloric stenosis
M>F
5-10% Family history in parents
Projectile non bile stained vomiting at 4-6 weeks of life
Diagnosis is made by test feed or USS
Treatment: Ramstedt pyloromyotomy (open or laparoscopic)

Acute appendicitis	
Uncommon under 3 years
When occurs may present atypically
Mesenteric adenitis	
Central abdominal pain and URTI
Conservative management

Intussusception
Telescoping bowel
Proximal to or at the level of, ileocaecal valve
6-9 months age
Colicky pain, diarrhoea and vomiting, sausage shaped mass, red jelly stool.
Treatment: reduction with air insufflation

Malrotation
High caecum at the midline
Feature in exomphalos, congenital diaphragmatic hernia, intrinsic duodenal atresia
May be complicated by development of volvulus, infant with volvulus may have bile stained vomiting
Diagnosis is made by upper GI contrast study and USS
Treatment is by laparotomy, if volvulus is present (or at high risk of occurring then a ladds procedure is performed

Hirschsprung’s disease
Absence of ganglion cells from myenteric and submucosal plexuses
Occurs in 1/5000 births
Full thickness rectal biopsy for diagnosis
Delayed passage of meconium and abdominal distension
Treatment is with rectal washouts initially, thereafter an anorectal pull through procedure

Oesophageal atresia
Associated with tracheo-oesophageal fistula and polyhydramnios
May present with choking and cyanotic spells following aspiration
VACTERL associations

Meconium ileus
Usually delayed passage of meconium and abdominal distension
Majority have cystic fibrosis
X-Rays may not show a fluid level as the meconium is viscid (depends upon feeding), PR contrast studies may dislodge meconium plugs and be therapeutic
Infants who do not respond to PR contrast and NG N-acetyl cysteine will require surgery to remove the plugs

Biliary atresia
Jaundice > 14 days
Increased conjugated bilirubin
Urgent Kasai procedure

Necrotising enterocolitis
Prematurity is the main risk factor
Early features include abdominal distension and passage of bloody stools
X-Rays may show pneumatosis intestinalis and evidence of free air
Increased risk when empirical antibiotics are given to infants beyond 5 days
Treatment is with total gut rest and TPN, babies with perforations will require laparotomy

122
Q

A male infant, born at term by normal vaginal delivery is well. However, 72 hours following delivery, he has still not passed meconium. On examination, his abdomen is soft and not particularly distended. He has a normally sited anus.

A.	Liver transplant
B.	Rectal biopsy
C.	Roux-en-Y portojejunostomy
D.	Ramstedt pyloromyotomy
E.	Abdominal CT scan
F.	Upper GI contrast study
G.	Laparotomy and formation of stoma
H.	Laparotomy
I.	Colonoscopy
J.	Gastroscopy
A

Rectal biopsy

Delayed passage of meconium is suggestive of Hirschsprung’s disease and the investigation of choice is full thickness suction rectal biopsy. If Hirschprungs is confirmed, then the correct treatment is laparotomy and stoma formation. At between 9 and 12 months of age, definitive surgery (usually resection and primary anastomosis) is performed.

Acute appendicitis	
Uncommon under 3 years
When occurs may present atypically
Mesenteric adenitis	
Central abdominal pain and URTI
Conservative management

Intussusception
Telescoping bowel
Proximal to or at the level of, ileocaecal valve
6-9 months age
Colicky pain, diarrhoea and vomiting, sausage shaped mass, red jelly stool.
Treatment: reduction with air insufflation

Malrotation
High caecum at the midline
Feature in exomphalos, congenital diaphragmatic hernia, intrinsic duodenal atresia
May be complicated by development of volvulus, infant with volvulus may have bile stained vomiting
Diagnosis is made by upper GI contrast study and USS
Treatment is by laparotomy, if volvulus is present (or at high risk of occurring then a ladds procedure is performed

Hirschsprung’s disease
Absence of ganglion cells from myenteric and submucosal plexuses
Occurs in 1/5000 births
Full thickness rectal biopsy for diagnosis
Delayed passage of meconium and abdominal distension
Treatment is with rectal washouts initially, thereafter an anorectal pull through procedure

Oesophageal atresia
Associated with tracheo-oesophageal fistula and polyhydramnios
May present with choking and cyanotic spells following aspiration
VACTERL associations

Meconium ileus
Usually delayed passage of meconium and abdominal distension
Majority have cystic fibrosis
X-Rays may not show a fluid level as the meconium is viscid (depends upon feeding), PR contrast studies may dislodge meconium plugs and be therapeutic
Infants who do not respond to PR contrast and NG N-acetyl cysteine will require surgery to remove the plugs

Biliary atresia
Jaundice > 14 days
Increased conjugated bilirubin
Urgent Kasai procedure

Necrotising enterocolitis
Prematurity is the main risk factor
Early features include abdominal distension and passage of bloody stools
X-Rays may show pneumatosis intestinalis and evidence of free air
Increased risk when empirical antibiotics are given to infants beyond 5 days
Treatment is with total gut rest and TPN, babies with perforations will require laparotomy

123
Q

A 55 year old man presents with a history of progressive dysphagia over the past 4 weeks. For the preceding 5 years he had regularly attended his general practitioner with symptoms of dyspepsia and reflux.

A.	Benign oesophageal stricture
B.	Globus
C.	Carcinoma of the oesophagus
D.	Achalasia
E.	Hiatus hernia
F.	Pharyngeal pouch
G.	Oesophageal web
H.	Oesophageal atresia
A

Carcinoma of the oesophagus

A short history of progressive dysphagia in a middle aged man who has a background history of reflux is strongly suggestive of malignancy. Long standing reflux symptoms may be suggestive of a increased risk of developing Barretts oesophagus. Note that not all patients with Barretts transformation alone are symptomatic.

Dysphagia

Causes of dysphagia

Extrinsic	
Mediastinal masses
Cervical spondylosis
Oesophageal wall	
Achalasia
Diffuse oesophageal spasm
Hypertensive lower oesophageal sphincter
Intrinsic	
Tumours
Strictures
Oesophageal web
Schatzki rings

Neurological
CVA
Parkinson’s disease
Multiple Sclerosis

Brainstem pathology
Myasthenia Gravis

Investigation
All patients require an upper GI endoscopy unless there are compelling reasons for this not to be performed. Motility disorders may be best appreciated by undertaking fluoroscopic swallowing studies.

A full blood count should be performed.

Ambulatory oesophageal pH and manometry studies will be required to evaluate conditions such as achalasia and patients with GORD being considered for fundoplication surgery.

124
Q

A 40 year old man presents with symptoms of dysphagia that have been present for many months. His investigations demonstrate lack of relaxation of the lower oesophageal sphincter during swallowing.

A.	Benign oesophageal stricture
B.	Globus
C.	Carcinoma of the oesophagus
D.	Achalasia
E.	Hiatus hernia
F.	Pharyngeal pouch
G.	Oesophageal web
H.	Oesophageal atresia
A

Achalasia

Patients with dysphagia will usually undergo an upper GI endoscopy as a first line investigation. Where this investigation is normal, the next stage is to perform studies assessing oesophageal motility. These comprise fluroscopic barium swallows and oesophageal manometry and pH studies. Lack of sphincter relaxation suggests achalasia (pressures are usually high).

Dysphagia

Causes of dysphagia

Extrinsic	
Mediastinal masses
Cervical spondylosis
Oesophageal wall	
Achalasia
Diffuse oesophageal spasm
Hypertensive lower oesophageal sphincter
Intrinsic	
Tumours
Strictures
Oesophageal web
Schatzki rings

Neurological
CVA
Parkinson’s disease
Multiple Sclerosis

Brainstem pathology
Myasthenia Gravis

Investigation
All patients require an upper GI endoscopy unless there are compelling reasons for this not to be performed. Motility disorders may be best appreciated by undertaking fluoroscopic swallowing studies.

A full blood count should be performed.

Ambulatory oesophageal pH and manometry studies will be required to evaluate conditions such as achalasia and patients with GORD being considered for fundoplication surgery.

125
Q

A 4 year old presents with sudden onset of dysphagia. He undergoes an upper GI endoscopy and a large bolus of food is identified in the mid oesophagus. He has no significant history, other than a tracheo-oesophageal fistula repair soon after birth.

A.	Benign oesophageal stricture
B.	Globus
C.	Carcinoma of the oesophagus
D.	Achalasia
E.	Hiatus hernia
F.	Pharyngeal pouch
G.	Oesophageal web
H.	Oesophageal atresia
A

Benign oesophageal stricture

Children with tracheo-oesophageal fistulas will commonly develop oesophageal strictures following repair. These may require regular dilations throughout childhood.

Dysphagia

Causes of dysphagia

Extrinsic	
Mediastinal masses
Cervical spondylosis
Oesophageal wall	
Achalasia
Diffuse oesophageal spasm
Hypertensive lower oesophageal sphincter
Intrinsic	
Tumours
Strictures
Oesophageal web
Schatzki rings

Neurological
CVA
Parkinson’s disease
Multiple Sclerosis

Brainstem pathology
Myasthenia Gravis

Investigation
All patients require an upper GI endoscopy unless there are compelling reasons for this not to be performed. Motility disorders may be best appreciated by undertaking fluoroscopic swallowing studies.

A full blood count should be performed.

Ambulatory oesophageal pH and manometry studies will be required to evaluate conditions such as achalasia and patients with GORD being considered for fundoplication surgery.

126
Q

A 20 year old lady presents with a mobile lump in the upper outer aspect of her right breast. On examination she has a firm mobile mass in the upper outer quadrant of her right breast.

A.	Mondors disease
B.	Duct ectasia
C.	Periductal mastitis
D.	Lactational breast abscess
E.	Fibroadenoma
F.	Breast cyst
G.	Intraductal papilloma
H.	Atypical ductal hyperplasia
I.	Radial scar
A

Fibroadenoma

Fibroadenomas account for 60% of discrete breast lesions in the 18-25 year age group. They are mobile lesions. Core biopsy should be performed in lesions measuring more than 4cm in diameter.

Fibroadenoma
Under the age of 25 years the breast is usually classified as undergoing development. Lobular units are being formed and a dense stroma is formed within the breast tissue. This may result in the development of fibroadenomas.
As a group, fibroadenomas account for 13% of all palpable breast lesions. However, in women aged 18-25 they constitute up to 60% of all palpable breast lesions. The are classified as juvenile, common and giant. The former occur in early adolescence and the latter are characterised by a size greater than 4cm. In young females with small fibroadenomas (less than 3cm on imaging) a policy of watchful waiting without biopsy may be adopted. A size of greater than 4cm attracts a recommendation for core biopsy to exclude a phyllodes tumour. The natural history of fibroadenomas is that 10% will increase in size, 30% regress and the remainder stay the same. This does not apply during pregnancy and lactation when they may increase in size substantially and subsequently sequester milk.
Some women may wish to have their fibroadenomas excised, they can usually be shelled out through a circumareolar incision. Smaller lesions may be removed using a mammotome.

Breast cysts
Palpable cysts constitute 15% of all breast lumps. They occur most frequently in perimenopausal females and are caused by distended and involuted lobules.
They may be readily apparent on clinical examination as soft, fluctuant swellings. It is important to exclude the presence of an underlying mass lesion. On imaging they will usually show a “halo appearance” on mammography. Ultrasound will confirm the fluid filled nature of the cyst. Symptomatic cysts may be aspirated and following aspiration the breast re-examined to ensure that the lump has gone.

Duct ectasia
As women progress through the menopause the breast ducts shorten and dilate. In some women this may cause a cheese like nipple discharge and slit like retraction of the nipple. No specific treatment is required.

127
Q

A 55 year old women presents with nipple discharge. On examination she has a slit like retraction of the nipple in the centre of this area is a small amount of cheese like material. No discrete mass lesion is palpable in the underlying breast.

A.	Mondors disease
B.	Duct ectasia
C.	Periductal mastitis
D.	Lactational breast abscess
E.	Fibroadenoma
F.	Breast cyst
G.	Intraductal papilloma
H.	Atypical ductal hyperplasia
I.	Radial scar
A

Duct ectasia

Duct ectasia is a common alteration in the breast that occurs with aging. As the ducts shorten and dilate a degree of symmetrical slit like retraction occurs. A small amount of cheese like discharge may occur.

Fibroadenoma
Under the age of 25 years the breast is usually classified as undergoing development. Lobular units are being formed and a dense stroma is formed within the breast tissue. This may result in the development of fibroadenomas.
As a group, fibroadenomas account for 13% of all palpable breast lesions. However, in women aged 18-25 they constitute up to 60% of all palpable breast lesions. The are classified as juvenile, common and giant. The former occur in early adolescence and the latter are characterised by a size greater than 4cm. In young females with small fibroadenomas (less than 3cm on imaging) a policy of watchful waiting without biopsy may be adopted. A size of greater than 4cm attracts a recommendation for core biopsy to exclude a phyllodes tumour. The natural history of fibroadenomas is that 10% will increase in size, 30% regress and the remainder stay the same. This does not apply during pregnancy and lactation when they may increase in size substantially and subsequently sequester milk.
Some women may wish to have their fibroadenomas excised, they can usually be shelled out through a circumareolar incision. Smaller lesions may be removed using a mammotome.

Breast cysts
Palpable cysts constitute 15% of all breast lumps. They occur most frequently in perimenopausal females and are caused by distended and involuted lobules.
They may be readily apparent on clinical examination as soft, fluctuant swellings. It is important to exclude the presence of an underlying mass lesion. On imaging they will usually show a “halo appearance” on mammography. Ultrasound will confirm the fluid filled nature of the cyst. Symptomatic cysts may be aspirated and following aspiration the breast re-examined to ensure that the lump has gone.

Duct ectasia
As women progress through the menopause the breast ducts shorten and dilate. In some women this may cause a cheese like nipple discharge and slit like retraction of the nipple. No specific treatment is required.

128
Q

A 48 year old lady presents with discomfort in the right breast. On examination she has a discrete soft fluctuant area in the upper outer quadrant of her right breast. A mammogram is performed and a “halo sign” is seen by the radiologist.

A.	Mondors disease
B.	Duct ectasia
C.	Periductal mastitis
D.	Lactational breast abscess
E.	Fibroadenoma
F.	Breast cyst
G.	Intraductal papilloma
H.	Atypical ductal hyperplasia
I.	Radial scar
A

Breast cyst

Lesions such as breast cysts compress the underlying fat and produce a radiolucent area (halo sign). If symptomatic, these cysts should be aspirated.

Fibroadenoma
Under the age of 25 years the breast is usually classified as undergoing development. Lobular units are being formed and a dense stroma is formed within the breast tissue. This may result in the development of fibroadenomas.
As a group, fibroadenomas account for 13% of all palpable breast lesions. However, in women aged 18-25 they constitute up to 60% of all palpable breast lesions. The are classified as juvenile, common and giant. The former occur in early adolescence and the latter are characterised by a size greater than 4cm. In young females with small fibroadenomas (less than 3cm on imaging) a policy of watchful waiting without biopsy may be adopted. A size of greater than 4cm attracts a recommendation for core biopsy to exclude a phyllodes tumour. The natural history of fibroadenomas is that 10% will increase in size, 30% regress and the remainder stay the same. This does not apply during pregnancy and lactation when they may increase in size substantially and subsequently sequester milk.
Some women may wish to have their fibroadenomas excised, they can usually be shelled out through a circumareolar incision. Smaller lesions may be removed using a mammotome.

Breast cysts
Palpable cysts constitute 15% of all breast lumps. They occur most frequently in perimenopausal females and are caused by distended and involuted lobules.
They may be readily apparent on clinical examination as soft, fluctuant swellings. It is important to exclude the presence of an underlying mass lesion. On imaging they will usually show a “halo appearance” on mammography. Ultrasound will confirm the fluid filled nature of the cyst. Symptomatic cysts may be aspirated and following aspiration the breast re-examined to ensure that the lump has gone.

Duct ectasia
As women progress through the menopause the breast ducts shorten and dilate. In some women this may cause a cheese like nipple discharge and slit like retraction of the nipple. No specific treatment is required.

129
Q

A 53 year old man undergoes a reversal of a loop colostomy. He recovers well and is discharged home. He is readmitted 10 days later with symptoms of vomiting and colicky abdominal pain. On examination he has a swelling of the loop colostomy site and it is tender. What is the most likely underlying diagnosis?

	Haematoma
	Intra abdominal adhesions
	Anastomotic leak
	Anastomotic stricture
	Obstructed incisional hernia
A

Obstructed incisional hernia

In this scenario the most likely diagnosis would be obstructed incisional hernia. The tender swelling coupled with symptoms of obstruction point to this diagnosis. Prompt surgical exploration is warranted. Loop colostomy reversals are at high risk of this complication as the operative site is at increased risk of the development of post operative wound infections.

Acute incisional hernia

Any surgical procedure involving entry into a cavity containing viscera may be complicated by post operative hernia
The abdomen is the commonest site
The deep layer of the wound has usually broken down, allowing internal viscera to protrude through
Management is dictated by the patients clinical status and the timing of the hernia in relation to recent surgery
Bowel obstruction or tenderness at the hernia site both mandate early surgical intervention to reduce the risk of bowel necrosis
Mature incisional hernias with a wide neck, and no symptoms, may be either left or listed for elective repair
Risk factors for the development of post operative incisional hernias include; post operative wound infections, long term steroid use, obesity and chronic cough

130
Q

A 43 year old lady has recently undergone a wide local excision and sentinel lymph node biopsy for carcinoma of the breast. Of the factors listed below, which will provide the most important prognostic information?

	Mitotic number
	Grade
	Nodal status
	Size
	Oestrogen receptor status
A

Nodal status is the single most important prognostic factor in breast cancer.

Nodal status is important because it serves as a marker of tumour metastatic potential. This translates to survival advantages of up to 40% at five years. Both grade and size are of secondary importance as they both less concerning in the absence of nodal involvement.

Breast cancer

Commoner in the older age group
Invasive ductal carcinomas are the most common type. Some may arise as a result of ductal carcinoma in situ (DCIS). There are associated carcinomas of special type e.g. Tubular that may carry better prognosis.
The pathological assessment involves assessment of the tumour and lymph nodes, sentinel lymph node biopsy is often used to minimise the morbidity of an axillary dissection.
Treatment, typically this is either wide local excision or mastectomy. There are many sub types of both of these that fall outside of the MRCS. Some key rules to bear in mind.
Whatever operation is contemplated the final cosmetic outcome does have a bearing. A woman with small breasts and a large tumour will tend to fare better with mastectomy, even if clear pathological and clinical margins can be obtained. Conversely a women with larger breasts may be able to undergo breast conserving surgery even with a relatively large primary lesion (NB tumours >4cm used to attract recommendation for mastectomy). For screen detected and impalpable tumour image guidance will be necessary.
Reconstruction is always an option following any resectional procedure. However, its exact type must be tailored to age and co-morbidities of the patient. The main operations in common use include latissimus dorsi myocutaneous flap and sub pectoral implants. Women wishing to avoid a prosthesis may be offered TRAM or DIEP flaps.

Surgical options
Mastectomy vs Wide local excision

Mastectomy: Multifocal tumour, Central tumour, Large lesion in small breast, DCIS >4cm

Wide Local Excision: Solitary lesion, Peripheral tumour, Small lesion in large breast, DCIS <4cm

Central lesions may be managed using breast conserving surgery where an acceptable cosmetic result may be obtained, this is rarely the case in small breasts

A compelling indication for mastectomy, a larger tumour that would be unsuitable for breast conserving surgery

Whatever surgical option is chosen the aim should be to have a local recurrence rate of 5% or less at 5 years .

Nottingham Prognostic Index
The Nottingham Prognostic Index can be used to give an indication of survival. In this system the tumour size is weighted less heavily than other major prognostic parameters.

Calculation of NPI
Tumour Size x 0.2 + Lymph node score(From table below)+Grade score(From table below).

Score Lymph nodes involved Grade
1 0 1
2 1-3 2
3 >3 3

Prognosis

Score	Percentage 5 year survival
2.0 to 2.4	93%
2.5 to 3.4	85%
3.5 to 5.4	70%
>5.4	50%
131
Q

A 56 year old man is undergoing a low anterior resection for carcinoma of the rectum. A primary anastomosis is planned.

A.	End ileostomy
B.	End colostomy
C.	Loop ileostomy
D.	Loop colostomy
E.	End jejunostomy
F.	Loop jejunostomy
G.	Caecostomy
A

Loop ileostomy

Colonic resections with an anastomosis below the peritoneal reflection may have an anastomotic leak rate (both clinical and radiological) of up to 15%. Therefore most surgeons will defunction such an anastomosis to reduce the clinical severity of an anastomotic leak. A loop ileostomy will achieve this end point and is relatively easy to reverse.

Types of stomas

Gastrostomy
Gastric decompression or fixation
Feeding
Epigastrium

Loop jejunostomy
Seldom used as very high output
May be used following emergency laparotomy with planned early closure
Any location according to need

Percutaneous jejunostomy
Usually performed for feeding purposes and site in the proximal bowel
Usually left upper quadrant

Loop ileostomy
Defunctioning of colon e.g. following rectal cancer surgery
Does not decompress colon (if ileocaecal valve competent)
Usually right iliac fossa

End ilestomy
Usually following complete excision of colon or where ileo-colic anastomosis is not planned
May be used to defunction colon, but reversal is more difficult
Usually right iliac fossa

End colostomy Where a colon is diverted or resected and anastomosis is not primarily achievable or desirable Either left or right iliac fossa

Loop colostomy
To defunction a distal segment of colon
Since both lumens are present the distal lumen acts as a vent
May be located in any region of the abdomen, depending upon colonic segment used

Caecostomy Stoma of last resort where loop colostomy is not possible Right iliac fossa

Mucous fistula
To decompress a distal segment of bowel following colonic division or resection
Where closure of a distal resection margin is not safe or achievable
May be located in any region of the abdomen according to clinical need

132
Q

A 23 year old man with uncontrolled ulcerative colitis is undergoing an emergency sub total colectomy.

A.	End ileostomy
B.	End colostomy
C.	Loop ileostomy
D.	Loop colostomy
E.	End jejunostomy
F.	Loop jejunostomy
G.	Caecostomy
A

End ileostomy

Following a sub total colectomy the immediate surgical options include an end ileostomy or ileorectal anastomosis. In the emergency setting an ileorectal anastomosis would be unsafe.

Types of stomas

Gastrostomy
Gastric decompression or fixation
Feeding
Epigastrium

Loop jejunostomy
Seldom used as very high output
May be used following emergency laparotomy with planned early closure
Any location according to need

Percutaneous jejunostomy
Usually performed for feeding purposes and site in the proximal bowel
Usually left upper quadrant

Loop ileostomy
Defunctioning of colon e.g. following rectal cancer surgery
Does not decompress colon (if ileocaecal valve competent)
Usually right iliac fossa

End ilestomy
Usually following complete excision of colon or where ileo-colic anastomosis is not planned
May be used to defunction colon, but reversal is more difficult
Usually right iliac fossa

End colostomy Where a colon is diverted or resected and anastomosis is not primarily achievable or desirable Either left or right iliac fossa

Loop colostomy
To defunction a distal segment of colon
Since both lumens are present the distal lumen acts as a vent
May be located in any region of the abdomen, depending upon colonic segment used

Caecostomy Stoma of last resort where loop colostomy is not possible Right iliac fossa

Mucous fistula
To decompress a distal segment of bowel following colonic division or resection
Where closure of a distal resection margin is not safe or achievable
May be located in any region of the abdomen according to clinical need

133
Q

A 63 year old women presents with large bowel obstruction. On examination she has a carcinoma 10cm from the anal verge.

A.	End ileostomy
B.	End colostomy
C.	Loop ileostomy
D.	Loop colostomy
E.	End jejunostomy
F.	Loop jejunostomy
G.	Caecostomy
A

Loop colostomy

Large bowel obstruction resulting from carcinoma should be resected, stented or defunctioned. The first two options typically apply to tumours above the peritoneal reflection. Lower tumours should be defunctioned with a loop colostomy and then formal staging undertaken prior to definitive surgery. An emergency attempted rectal resection carries a high risk of involvement of the circumferential resection margin and is not recommended.

Abdominal stomas

Stomas may be sited during a range of abdominal procedures and involve bringing the lumen or visceral contents onto the skin. In most cases this applies to the bowel. However, other organs or their contents may be diverted in case of need.

With bowel stomas the type method of construction and to a lesser extent the site will be determined by the contents of the bowel. In practice, small bowel stomas should be spouted so that their irritant contents are not in contact with the skin. Colonic stomas do not need to be spouted as their contents are less irritant.

In the ideal situation the site of the stoma should be marked with the patient prior to surgery. Stoma siting is important as it will ultimately influence the ability of the patient to manage their stoma and also reduce the risk of leakage. Leakage of stoma contents and subsequent maceration of the surrounding skin can rapidly progress into a spiraling loss of control of stoma contents.

Types of stomas

Gastrostomy
Gastric decompression or fixation
Feeding
Epigastrium

Loop jejunostomy
Seldom used as very high output
May be used following emergency laparotomy with planned early closure
Any location according to need

Percutaneous jejunostomy
Usually performed for feeding purposes and site in the proximal bowel
Usually left upper quadrant

Loop ileostomy
Defunctioning of colon e.g. following rectal cancer surgery
Does not decompress colon (if ileocaecal valve competent)
Usually right iliac fossa

End ilestomy
Usually following complete excision of colon or where ileo-colic anastomosis is not planned
May be used to defunction colon, but reversal is more difficult
Usually right iliac fossa

End colostomy Where a colon is diverted or resected and anastomosis is not primarily achievable or desirable Either left or right iliac fossa

Loop colostomy
To defunction a distal segment of colon
Since both lumens are present the distal lumen acts as a vent
May be located in any region of the abdomen, depending upon colonic segment used

Caecostomy Stoma of last resort where loop colostomy is not possible Right iliac fossa

Mucous fistula
To decompress a distal segment of bowel following colonic division or resection
Where closure of a distal resection margin is not safe or achievable
May be located in any region of the abdomen according to clinical need

134
Q

A 23 year old male is admitted with left sided loin pain and fever. His investigations demonstrate a left sided ureteric calculi that measures 0.7cm in diameter and associated hydronephrosis.

A.	Nephrectomy
B.	Open ureteric exploration
C.	Extra corporeal shock wave lithotripsy
D.	Percutaneous nephrostomy
E.	Pyeloplasty
F.	Conservative management
G.	Percutaneous nephrolithotomy
A

Percutaneous nephrostomy

An obstructed, infected system is an indication for urgent decompression. This may be achieved by ureteroscopy or nephrostomy. In addition to this the patient should also receive broad spectrum, intravenous antibiotics.

Management of renal stones

Urolithiasis will affect up to 15% of the worldwide population. The development of sudden onset loin to groin pain which is colicky in nature is a classic feature in the history. It is nearly always associated with haematuria that is either micro or macroscopic.
Where the diagnosis is suspected the most sensitive and specific diagnostic test is helical, non contrast, computerised tomographic (CT) scanning.

Management
Most renal stones measuring less than 5mm in maximum diameter will typically pass within 4 weeks of symptom onset. More intensive and urgent treatment is indicated in the presence of ureteric obstruction, renal developmental abnormality such as horseshoe kidney and previous renal transplant. Ureteric obstruction due to stones together with infection is a surgical emergency and the system must be decompressed. Options include nephrostomy tube placement, insertion of ureteric catheters and ureteric stent placement.
In the non emergency setting the preferred options for treatment of stone disease include extra corporeal shock wave lithotripsy, percutaneous nephrolithotomy, ureteroscopy, open surgery remains an option for selected cases. However, minimally invasive options are the most popular first line treatment.

Shock wave lithotripsy
A shock wave is generated external to the patient, internally cavitation bubbles and mechanical stress lead to stone fragmentation. The passage of shock waves can result in the development of solid organ injury. Fragmentation of larger stones may result in the development of ureteric obstruction. The procedure is uncomfortable for patients and analgesia is required during the procedure and afterwards.

Ureteroscopy
A ureteroscope is passed retrograde through the ureter and into the renal pelvis. It is indicated in individuals (e.g. pregnant females) where lithotripsy is contraindicated and in complex stone disease. In most cases a stent is left in situ for 4 weeks after the procedure.

Percutaneous nephrolithotomy
In this procedure access is gained to the renal collecting system. Once access is achieved, intra corporeal lithotripsy or stone fragmentation is performed and stone fragments removed.

Therapeutic selection

  • Stone burden of less than 2cm in aggregate: Lithotripsy
  • Stone burden of less than 2cm in pregnant females: Ureteroscopy
  • Complex renal calculi and staghorn calculi: Percutaneous nephrolithotomy
  • Ureteric calculi less than 5mm: Manage expectantly
135
Q

A 23 year old man is admitted with left sided loin pain that radiates to his groin. His investigations demonstrate a 1cm left sided calculus within the renal pelvis.

A.	Nephrectomy
B.	Open ureteric exploration
C.	Extra corporeal shock wave lithotripsy
D.	Percutaneous nephrostomy
E.	Pyeloplasty
F.	Conservative management
G.	Percutaneous nephrolithotomy
A

Extra corporeal shock wave lithotripsy

Stones with a total volume of less than 2cm can be considered for lithotripsy. If it is impacted in the upper ureter then some may consider a ureteroscopy.

Management of renal stones

Urolithiasis will affect up to 15% of the worldwide population. The development of sudden onset loin to groin pain which is colicky in nature is a classic feature in the history. It is nearly always associated with haematuria that is either micro or macroscopic.
Where the diagnosis is suspected the most sensitive and specific diagnostic test is helical, non contrast, computerised tomographic (CT) scanning.

Management
Most renal stones measuring less than 5mm in maximum diameter will typically pass within 4 weeks of symptom onset. More intensive and urgent treatment is indicated in the presence of ureteric obstruction, renal developmental abnormality such as horseshoe kidney and previous renal transplant. Ureteric obstruction due to stones together with infection is a surgical emergency and the system must be decompressed. Options include nephrostomy tube placement, insertion of ureteric catheters and ureteric stent placement.
In the non emergency setting the preferred options for treatment of stone disease include extra corporeal shock wave lithotripsy, percutaneous nephrolithotomy, ureteroscopy, open surgery remains an option for selected cases. However, minimally invasive options are the most popular first line treatment.

Shock wave lithotripsy
A shock wave is generated external to the patient, internally cavitation bubbles and mechanical stress lead to stone fragmentation. The passage of shock waves can result in the development of solid organ injury. Fragmentation of larger stones may result in the development of ureteric obstruction. The procedure is uncomfortable for patients and analgesia is required during the procedure and afterwards.

Ureteroscopy
A ureteroscope is passed retrograde through the ureter and into the renal pelvis. It is indicated in individuals (e.g. pregnant females) where lithotripsy is contraindicated and in complex stone disease. In most cases a stent is left in situ for 4 weeks after the procedure.

Percutaneous nephrolithotomy
In this procedure access is gained to the renal collecting system. Once access is achieved, intra corporeal lithotripsy or stone fragmentation is performed and stone fragments removed.

Therapeutic selection

  • Stone burden of less than 2cm in aggregate: Lithotripsy
  • Stone burden of less than 2cm in pregnant females: Ureteroscopy
  • Complex renal calculi and staghorn calculi: Percutaneous nephrolithotomy
  • Ureteric calculi less than 5mm: Manage expectantly
136
Q

A 30 year old male presents with left sided loin pain. His investigations demonstrate a large left sided staghorn calculus that measures 2.3cm in diameter.

A.	Nephrectomy
B.	Open ureteric exploration
C.	Extra corporeal shock wave lithotripsy
D.	Percutaneous nephrostomy
E.	Pyeloplasty
F.	Conservative management
G.	Percutaneous nephrolithotomy
A

Percutaneous nephrolithotomy

Large, proximal stones are generally best managed with a percutaneous nephrolithotomy. The use of lithotripsy has low clearance rates. Where stones remain after the initial procedure a repeat percutaneous nephrolithotomy is generally preferred over follow up lithotripsy.

Management of renal stones

Urolithiasis will affect up to 15% of the worldwide population. The development of sudden onset loin to groin pain which is colicky in nature is a classic feature in the history. It is nearly always associated with haematuria that is either micro or macroscopic.
Where the diagnosis is suspected the most sensitive and specific diagnostic test is helical, non contrast, computerised tomographic (CT) scanning.

Management
Most renal stones measuring less than 5mm in maximum diameter will typically pass within 4 weeks of symptom onset. More intensive and urgent treatment is indicated in the presence of ureteric obstruction, renal developmental abnormality such as horseshoe kidney and previous renal transplant. Ureteric obstruction due to stones together with infection is a surgical emergency and the system must be decompressed. Options include nephrostomy tube placement, insertion of ureteric catheters and ureteric stent placement.
In the non emergency setting the preferred options for treatment of stone disease include extra corporeal shock wave lithotripsy, percutaneous nephrolithotomy, ureteroscopy, open surgery remains an option for selected cases. However, minimally invasive options are the most popular first line treatment.

Shock wave lithotripsy
A shock wave is generated external to the patient, internally cavitation bubbles and mechanical stress lead to stone fragmentation. The passage of shock waves can result in the development of solid organ injury. Fragmentation of larger stones may result in the development of ureteric obstruction. The procedure is uncomfortable for patients and analgesia is required during the procedure and afterwards.

Ureteroscopy
A ureteroscope is passed retrograde through the ureter and into the renal pelvis. It is indicated in individuals (e.g. pregnant females) where lithotripsy is contraindicated and in complex stone disease. In most cases a stent is left in situ for 4 weeks after the procedure.

Percutaneous nephrolithotomy
In this procedure access is gained to the renal collecting system. Once access is achieved, intra corporeal lithotripsy or stone fragmentation is performed and stone fragments removed.

Therapeutic selection

  • Stone burden of less than 2cm in aggregate: Lithotripsy
  • Stone burden of less than 2cm in pregnant females: Ureteroscopy
  • Complex renal calculi and staghorn calculi: Percutaneous nephrolithotomy
  • Ureteric calculi less than 5mm: Manage expectantly
137
Q

A 72 year old lady underwent an open cholecystectomy 12 years previously. She has been admitted since with 2 episodes of cholangitis and stones were retrieved at ERCP. She has just recovered from a further episode of sepsis and MRCP has shown further biliary stones.

A. Acute laparoscopic cholecystectomy
B. Delayed laparoscopic cholecystectomy
C. Percutaneous cholecystostomy
D. Elective cholecystectomy and intra operative cholangiogram
E. Endoscopic retrograde cholangiopancreatography
F. Choledochoduodenostomy
G. Bile duct excision and hepatico-jejunostomy
H. Operative cholecystostomy

A

Choledochoduodenostomy

A patient with long standing common bile duct stones is at risk of developing duct fibrosis and ductal disproportion. This can result in impaired biliary drainage. Not only may further stones be formed in the bile that is present, but because of the ductal disproportion the tendency will be for the stones to accumulate (rather than pass spontaneously, as would usually be the case post ERCP and sphincterotomy). A biliary bypass procedure is the standard method dealing with this and a choledochoduodenstomy is one procedure that can be used.

Gallstones Typically history of biliary colic or episodes of chlolecystitis. Obstructive type history and test results. Usually small calibre gallstones which can pass through the cystic duct. In Mirizzi syndrome the stone may compress the bile duct directly- one of the rare times that cholecystitis may present with jaundice

Cholangitis Usually obstructive and will have Charcot’s triad of symptoms (pain, fever, jaundice) Ascending infection of the bile ducts usually by E. coli and by definition occurring in a pool of stagnant bile.

Pancreatic cancer Typically painless jaundice with palpable gallbladder (Courvoisier’s Law) Direct occlusion of distal bile duct or pancreatic duct by tumour. Sometimes nodal disease at the portal hepatis may be the culprit in which case the bile duct may be of normal calibre.
TPN (total parenteral nutrition) associated jaundice Usually follows long term use and is usually painless with non obstructive features Often due to hepatic dysfunction and fatty liver which may occur with long term TPN usage.

Bile duct injury Depending upon the type of injury may be of sudden or gradual onset and is usually of obstructive type Often due to a difficult laparoscopic cholecystectomy when anatomy in Calots triangle is not appreciated. In the worst scenario the bile duct is excised and jaundice develops rapidly post operatively. More insidious is that of bile duct stenosis which may be caused by clips or diathermy injury.

Cholangiocarcinoma Gradual onset obstructive pattern Direct occlusion by disease and also extrinsic compression by nodal disease at the porta hepatis.

Septic surgical patient Usually hepatic features Combination of impaired biliary excretion and drugs such as ciprofloxacin which may cause cholestasis.

Metastatic disease Mixed hepatic and post hepatic Combination of liver synthetic failure (late) and extrinsic compression by nodal disease and anatomical compression of intra hepatic structures (earlier)

A gallbladder may develop a thickened wall in chronic cholecystitis, microscopically Roikitansky-Aschoff Sinuses may be seen

138
Q

A 26 year old women is admitted with acute cholecystitis of 24 hours duration. LFT’s are normal and Ultrasound shows a thick walled gallbladder containing stones.

A. Acute laparoscopic cholecystectomy
B. Delayed laparoscopic cholecystectomy
C. Percutaneous cholecystostomy
D. Elective cholecystectomy and intra operative cholangiogram
E. Endoscopic retrograde cholangiopancreatography
F. Choledochoduodenostomy
G. Bile duct excision and hepatico-jejunostomy
H. Operative cholecystostomy

A

Acute laparoscopic cholecystectomy

This is an ideal case for an acute cholecystectomy, provided that surgery can be undertaken promptly. After 48 -72 hours the patient should receive parenteral antibiotics and delayed cholecystectomy performed.

Gallstones Typically history of biliary colic or episodes of chlolecystitis. Obstructive type history and test results. Usually small calibre gallstones which can pass through the cystic duct. In Mirizzi syndrome the stone may compress the bile duct directly- one of the rare times that cholecystitis may present with jaundice

Cholangitis Usually obstructive and will have Charcot’s triad of symptoms (pain, fever, jaundice) Ascending infection of the bile ducts usually by E. coli and by definition occurring in a pool of stagnant bile.

Pancreatic cancer Typically painless jaundice with palpable gallbladder (Courvoisier’s Law) Direct occlusion of distal bile duct or pancreatic duct by tumour. Sometimes nodal disease at the portal hepatis may be the culprit in which case the bile duct may be of normal calibre.
TPN (total parenteral nutrition) associated jaundice Usually follows long term use and is usually painless with non obstructive features Often due to hepatic dysfunction and fatty liver which may occur with long term TPN usage.

Bile duct injury Depending upon the type of injury may be of sudden or gradual onset and is usually of obstructive type Often due to a difficult laparoscopic cholecystectomy when anatomy in Calots triangle is not appreciated. In the worst scenario the bile duct is excised and jaundice develops rapidly post operatively. More insidious is that of bile duct stenosis which may be caused by clips or diathermy injury.

Cholangiocarcinoma Gradual onset obstructive pattern Direct occlusion by disease and also extrinsic compression by nodal disease at the porta hepatis.

Septic surgical patient Usually hepatic features Combination of impaired biliary excretion and drugs such as ciprofloxacin which may cause cholestasis.

Metastatic disease Mixed hepatic and post hepatic Combination of liver synthetic failure (late) and extrinsic compression by nodal disease and anatomical compression of intra hepatic structures (earlier)

A gallbladder may develop a thickened wall in chronic cholecystitis, microscopically Roikitansky-Aschoff Sinuses may be seen

139
Q

A 32 year old lady is seen in the outpatients. She has had multiple episodes of biliary colic and ultrasound shows thin walled gallbladder with multiple calculi. Her ALT is slightly raised but other parameters are normal.

A. Acute laparoscopic cholecystectomy
B. Delayed laparoscopic cholecystectomy
C. Percutaneous cholecystostomy
D. Elective cholecystectomy and intra operative cholangiogram
E. Endoscopic retrograde cholangiopancreatography
F. Choledochoduodenostomy
G. Bile duct excision and hepatico-jejunostomy
H. Operative cholecystostomy

A

Elective cholecystectomy and intra operative cholangiogram

The easiest option is to perform an intraoperative cholangiogram. It is unlikely to reveal any stones. If is does then either laparoscopic bile duct exploration or urgent ERCP can be performed. An MRCP pre op is an alternative strategy.

Gallstones Typically history of biliary colic or episodes of chlolecystitis. Obstructive type history and test results. Usually small calibre gallstones which can pass through the cystic duct. In Mirizzi syndrome the stone may compress the bile duct directly- one of the rare times that cholecystitis may present with jaundice

Cholangitis Usually obstructive and will have Charcot’s triad of symptoms (pain, fever, jaundice) Ascending infection of the bile ducts usually by E. coli and by definition occurring in a pool of stagnant bile.

Pancreatic cancer Typically painless jaundice with palpable gallbladder (Courvoisier’s Law) Direct occlusion of distal bile duct or pancreatic duct by tumour. Sometimes nodal disease at the portal hepatis may be the culprit in which case the bile duct may be of normal calibre.
TPN (total parenteral nutrition) associated jaundice Usually follows long term use and is usually painless with non obstructive features Often due to hepatic dysfunction and fatty liver which may occur with long term TPN usage.

Bile duct injury Depending upon the type of injury may be of sudden or gradual onset and is usually of obstructive type Often due to a difficult laparoscopic cholecystectomy when anatomy in Calots triangle is not appreciated. In the worst scenario the bile duct is excised and jaundice develops rapidly post operatively. More insidious is that of bile duct stenosis which may be caused by clips or diathermy injury.

Cholangiocarcinoma Gradual onset obstructive pattern Direct occlusion by disease and also extrinsic compression by nodal disease at the porta hepatis.

Septic surgical patient Usually hepatic features Combination of impaired biliary excretion and drugs such as ciprofloxacin which may cause cholestasis.

Metastatic disease Mixed hepatic and post hepatic Combination of liver synthetic failure (late) and extrinsic compression by nodal disease and anatomical compression of intra hepatic structures (earlier)

A gallbladder may develop a thickened wall in chronic cholecystitis, microscopically Roikitansky-Aschoff Sinuses may be seen

140
Q

A 19 year old female presents with severe anal pain and bleeding which typically occurs post defecation. On examination she has a large posteriorly sited fissure in ano.

A.	Excision and primary closure
B.	Incision and drainage
C.	Topical steroids
D.	Topical diltiazem
E.	Steroid injections
F.	Haemorroidectomy
G.	Manual anal dilation
H.	Injection with 88% aqueous phenol
I.	Sphincterotomy
A

Topical diltiazem

Initial therapy should be with pharmacological agents to relax the sphincter and facilitate healing. This is particularly true in females presenting for the first time.

Fissure in ano: Painful, bright red rectal bleeding Stool softeners, topical diltiazem or GTN, botulinum toxin, Sphincterotomy

Haemorroids: Painless, bright red rectal bleeding occurs following defecation and bleeds onto the toilet paper and into the toilet pan Stool softeners, avoid straining, surgery (see below)

Fistula in ano: May initially present with an abscess and then persisting discharge onto the perineum, separate from the anus Lay open if low, no sphincter involvement or IBD, if complex, high or IBD insert seton and consider other options (see below)

Peri anal abscess: Peri anal swelling and surrounding erythema Incision and drainage, leave the cavity open to heal by secondary intention

Pruritus ani: Peri anal itching, occasional mild bleeding (if severe skin damage) Avoid scented products, use wet wipes rather than tissue, avoidance of scratching, ensure no underlying faecal incontinence

Overview of surgical therapies
Haemorroidal disease
The treatment of haemorroids is usually conservative. Acutely thrombosed haemorroids may be extremely painful. Treatment of this acute condition is usually conservative and consists of stool softeners, ice compressions and topical GTN or diltiazem to reduce sphincter spasm. Most cases managed with this approach will settle over the next 5-7 days. After this period there may be residual skin tags that merit surgical excision or indeed residual haemorroidal disease that may necessitate haemorroidectomy.
Patients with more chronic symptoms are managed according to the stage of their disease, small mild internal haemorroids causing little symptoms are best managed conservatively. More marked symptoms of bleeding and occasional prolapse, where the haemorroidal complex is largely internal may benefit from stapled haemorroidopexy. This procedure excises rectal tissue above the dentate line and disrupts the haemorroidal blood supply. At the same time the excisional component of the procedure means that the haemorroids are less prone to prolapse. Adverse effects of this procedure include urgency, which can affect up to 40% of patients (but settles over 6-12 months) and recurrence. The procedure does not address skin tags and therefore this procedure is unsuitable if this is the dominant symptom.
Large haemorroids with a substantial external component may be best managed with a Milligan Morgan style conventional haemorroidectomy. In this procedure three haemorroidal cushions are excised, together with their vascular pedicle. Excision of excessive volumes of tissue may result in anal stenosis. The procedure is quite painful and most surgeons prescribe metronidazole post operatively as it decreases post operative pain.

Fissure in ano
Probably the most efficient and definitive treatment for fissure in ano is lateral internal sphincterotomy. The treatment is permanent and nearly all patients will recover. Up to 30% will develop incontinence to flatus. There are justifiable concerns about using this procedure in females as pregnancy and pelvic floor damage together with a sphincterotomy may result in faecal incontinence. The usual first line therapy is relaxation of the internal sphincter with either GTN or diltiazem (the latter being better tolerated) applied topically for 6 weeks. Treatment failures with topical therapy will usually go on to have treatment with botulinum toxin. This leads to more permanent changes in the sphincter and this may facilitate healing.
Typical fissures usually present in the posterior midline, multiple or unusually located fissures should prompt a search for an underlying cause such as inflammatory bowel disease or internal prolapse.
Refractory cases where the above treatments have failed may be considered for advancement flaps.

Fistula in ano
The most effective treatment for fistula is laying it open (fistulotomy). When the fistula is below the sphincter and uncomplicated, this is a reasonable option. Sphincter involvement and complex underlying disease should be assessed both surgically and ideally with imaging (either MRI or endoanal USS). Surgery is then usually staged, in the first instance a draining seton suture may be inserted. This avoids the development of recurrent sepsis and may allow resolution. In patients with Crohns disease the seton should be left in situ long term and the patient managed medically, as in these cases attempts at complex surgical repair nearly always fail. Fistulas not associated with IBD may be managed by advancement flaps, instillation of plugs and glue is generally unsuccessful. A newer technique of ligation of intersphincteric tract (LIFT procedure) is reported to have good results in selected centres.

141
Q

A 43 year old male has been troubled with symptoms of post defecation bleeding for many years. On examination he has large prolapsed haemorroids, colonoscopy shows no other disease.

A.	Excision and primary closure
B.	Incision and drainage
C.	Topical steroids
D.	Topical diltiazem
E.	Steroid injections
F.	Haemorroidectomy
G.	Manual anal dilation
H.	Injection with 88% aqueous phenol
I.	Sphincterotomy
A

Haemorroidectomy

Prolapsed haemorroids are best managed surgically if symptomatic. Note that phenol injections are usually only used for minor internal haemorroids. Where used low concentration phenol in oil is used, the phenolic solution above is used to ablate the nail bed in toe nail surgery!
Fissure in ano: Painful, bright red rectal bleeding Stool softeners, topical diltiazem or GTN, botulinum toxin, Sphincterotomy

Haemorroids: Painless, bright red rectal bleeding occurs following defecation and bleeds onto the toilet paper and into the toilet pan Stool softeners, avoid straining, surgery (see below)

Fistula in ano: May initially present with an abscess and then persisting discharge onto the perineum, separate from the anus Lay open if low, no sphincter involvement or IBD, if complex, high or IBD insert seton and consider other options (see below)

Peri anal abscess: Peri anal swelling and surrounding erythema Incision and drainage, leave the cavity open to heal by secondary intention

Pruritus ani: Peri anal itching, occasional mild bleeding (if severe skin damage) Avoid scented products, use wet wipes rather than tissue, avoidance of scratching, ensure no underlying faecal incontinence

Overview of surgical therapies
Haemorroidal disease
The treatment of haemorroids is usually conservative. Acutely thrombosed haemorroids may be extremely painful. Treatment of this acute condition is usually conservative and consists of stool softeners, ice compressions and topical GTN or diltiazem to reduce sphincter spasm. Most cases managed with this approach will settle over the next 5-7 days. After this period there may be residual skin tags that merit surgical excision or indeed residual haemorroidal disease that may necessitate haemorroidectomy.
Patients with more chronic symptoms are managed according to the stage of their disease, small mild internal haemorroids causing little symptoms are best managed conservatively. More marked symptoms of bleeding and occasional prolapse, where the haemorroidal complex is largely internal may benefit from stapled haemorroidopexy. This procedure excises rectal tissue above the dentate line and disrupts the haemorroidal blood supply. At the same time the excisional component of the procedure means that the haemorroids are less prone to prolapse. Adverse effects of this procedure include urgency, which can affect up to 40% of patients (but settles over 6-12 months) and recurrence. The procedure does not address skin tags and therefore this procedure is unsuitable if this is the dominant symptom.
Large haemorroids with a substantial external component may be best managed with a Milligan Morgan style conventional haemorroidectomy. In this procedure three haemorroidal cushions are excised, together with their vascular pedicle. Excision of excessive volumes of tissue may result in anal stenosis. The procedure is quite painful and most surgeons prescribe metronidazole post operatively as it decreases post operative pain.

Fissure in ano
Probably the most efficient and definitive treatment for fissure in ano is lateral internal sphincterotomy. The treatment is permanent and nearly all patients will recover. Up to 30% will develop incontinence to flatus. There are justifiable concerns about using this procedure in females as pregnancy and pelvic floor damage together with a sphincterotomy may result in faecal incontinence. The usual first line therapy is relaxation of the internal sphincter with either GTN or diltiazem (the latter being better tolerated) applied topically for 6 weeks. Treatment failures with topical therapy will usually go on to have treatment with botulinum toxin. This leads to more permanent changes in the sphincter and this may facilitate healing.
Typical fissures usually present in the posterior midline, multiple or unusually located fissures should prompt a search for an underlying cause such as inflammatory bowel disease or internal prolapse.
Refractory cases where the above treatments have failed may be considered for advancement flaps.

Fistula in ano
The most effective treatment for fistula is laying it open (fistulotomy). When the fistula is below the sphincter and uncomplicated, this is a reasonable option. Sphincter involvement and complex underlying disease should be assessed both surgically and ideally with imaging (either MRI or endoanal USS). Surgery is then usually staged, in the first instance a draining seton suture may be inserted. This avoids the development of recurrent sepsis and may allow resolution. In patients with Crohns disease the seton should be left in situ long term and the patient managed medically, as in these cases attempts at complex surgical repair nearly always fail. Fistulas not associated with IBD may be managed by advancement flaps, instillation of plugs and glue is generally unsuccessful. A newer technique of ligation of intersphincteric tract (LIFT procedure) is reported to have good results in selected centres.

142
Q

A 20 year old man presents with a 24 hour history of anal pain. On examination he has a peri anal abscess.

A.	Excision and primary closure
B.	Incision and drainage
C.	Topical steroids
D.	Topical diltiazem
E.	Steroid injections
F.	Haemorroidectomy
G.	Manual anal dilation
H.	Injection with 88% aqueous phenol
I.	Sphincterotomy
A

Incision and drainage

Abscesses require incision and drainage as a first line treatment.

Fissure in ano: Painful, bright red rectal bleeding Stool softeners, topical diltiazem or GTN, botulinum toxin, Sphincterotomy

Haemorroids: Painless, bright red rectal bleeding occurs following defecation and bleeds onto the toilet paper and into the toilet pan Stool softeners, avoid straining, surgery (see below)

Fistula in ano: May initially present with an abscess and then persisting discharge onto the perineum, separate from the anus Lay open if low, no sphincter involvement or IBD, if complex, high or IBD insert seton and consider other options (see below)

Peri anal abscess: Peri anal swelling and surrounding erythema Incision and drainage, leave the cavity open to heal by secondary intention

Pruritus ani: Peri anal itching, occasional mild bleeding (if severe skin damage) Avoid scented products, use wet wipes rather than tissue, avoidance of scratching, ensure no underlying faecal incontinence

Overview of surgical therapies
Haemorroidal disease
The treatment of haemorroids is usually conservative. Acutely thrombosed haemorroids may be extremely painful. Treatment of this acute condition is usually conservative and consists of stool softeners, ice compressions and topical GTN or diltiazem to reduce sphincter spasm. Most cases managed with this approach will settle over the next 5-7 days. After this period there may be residual skin tags that merit surgical excision or indeed residual haemorroidal disease that may necessitate haemorroidectomy.
Patients with more chronic symptoms are managed according to the stage of their disease, small mild internal haemorroids causing little symptoms are best managed conservatively. More marked symptoms of bleeding and occasional prolapse, where the haemorroidal complex is largely internal may benefit from stapled haemorroidopexy. This procedure excises rectal tissue above the dentate line and disrupts the haemorroidal blood supply. At the same time the excisional component of the procedure means that the haemorroids are less prone to prolapse. Adverse effects of this procedure include urgency, which can affect up to 40% of patients (but settles over 6-12 months) and recurrence. The procedure does not address skin tags and therefore this procedure is unsuitable if this is the dominant symptom.
Large haemorroids with a substantial external component may be best managed with a Milligan Morgan style conventional haemorroidectomy. In this procedure three haemorroidal cushions are excised, together with their vascular pedicle. Excision of excessive volumes of tissue may result in anal stenosis. The procedure is quite painful and most surgeons prescribe metronidazole post operatively as it decreases post operative pain.

Fissure in ano
Probably the most efficient and definitive treatment for fissure in ano is lateral internal sphincterotomy. The treatment is permanent and nearly all patients will recover. Up to 30% will develop incontinence to flatus. There are justifiable concerns about using this procedure in females as pregnancy and pelvic floor damage together with a sphincterotomy may result in faecal incontinence. The usual first line therapy is relaxation of the internal sphincter with either GTN or diltiazem (the latter being better tolerated) applied topically for 6 weeks. Treatment failures with topical therapy will usually go on to have treatment with botulinum toxin. This leads to more permanent changes in the sphincter and this may facilitate healing.
Typical fissures usually present in the posterior midline, multiple or unusually located fissures should prompt a search for an underlying cause such as inflammatory bowel disease or internal prolapse.
Refractory cases where the above treatments have failed may be considered for advancement flaps.

Fistula in ano
The most effective treatment for fistula is laying it open (fistulotomy). When the fistula is below the sphincter and uncomplicated, this is a reasonable option. Sphincter involvement and complex underlying disease should be assessed both surgically and ideally with imaging (either MRI or endoanal USS). Surgery is then usually staged, in the first instance a draining seton suture may be inserted. This avoids the development of recurrent sepsis and may allow resolution. In patients with Crohns disease the seton should be left in situ long term and the patient managed medically, as in these cases attempts at complex surgical repair nearly always fail. Fistulas not associated with IBD may be managed by advancement flaps, instillation of plugs and glue is generally unsuccessful. A newer technique of ligation of intersphincteric tract (LIFT procedure) is reported to have good results in selected centres.

143
Q

A 45 year old lady presented with a 2cm mobile breast mass. A mammogram is indeterminate (M3), USS shows benign changes (U2), clinical examination is also indeterminate (P3). What is the next most appropriate course of action?

	Re-assure and discharge
	Fine needle aspiration cytology
	Excision biopsy
	Image guided core biopsy
	Wide local excision
A

Image guided core biopsy

All discrete breast lumps, including those that seem benign, should have a confirmed histological diagnosis. In this case, a core biopsy has not yet been performed. This may yield a diagnosis that is concordant with imaging findings. In which case, this concludes the investigative process (if benign). If it remains unclear, excision biopsy will be needed.

Fibroadenoma
Develop from a whole lobule
Mobile, firm breast lumps
12% of all breast masses
Over a 2 year period up to 30% will get smaller
No increase in risk of malignancy
If >3cm surgical excision is usual, Phyllodes tumours should be widely excised (mastectomy if the lesion is large)

Breast cyst
7% of all Western females will present with a breast cyst
Usually presents as a smooth discrete lump (may be fluctuant)
Small increased risk of breast cancer (especially if younger)
Cysts should be aspirated, those which are blood stained or persistently refill should be biopsied or excised

Sclerosing adenosis, (radial scars and complex sclerosing lesions)
Usually presents as a breast lump or breast pain
Causes mammographic changes which may mimic carcinoma
Cause distortion of the distal lobular unit, without hyperplasia (complex lesions will show hyperplasia)
Considered a disorder of involution, no increase in malignancy risk
Lesions should be biopsied, excision is not mandatory

Epithelial hyperplasia
Variable clinical presentation ranging from generalised lumpiness through to discrete lump
Disorder consists of increased cellularity of terminal lobular unit, atypical features may be present
Atypical features and family history of breast cancer confers greatly increased risk of malignancy
If no atypical features then conservative, those with atypical features require either close monitoring or surgical resection

Fat necrosis
Up to 40% cases usually have a traumatic aetiology
Physical features usually mimic carcinoma
Mass may increase in size initially
Imaging and core biopsy

Duct papilloma	
Usually present with nipple discharge
Large papillomas may present with a mass
The discharge usually originates from a single duct
No increase risk of malignancy
Microdochectomy
144
Q

A 43 year old man is investigated for altered bowel habit. At colonoscopy he is found to have a 2cm polyp on a long stalk in the proximal sigmoid colon. The rest of the colonoscopy is normal. It bears no macroscopic features of malignancy.

A.	Reassure and discharge
B.	Pan proctocolectomy
C.	Hot biopsy
D.	Snare polypectomy
E.	Segmental colonic resection
F.	Repeat endoscopy at 3 years
G.	Repeat endoscopy at 1 year
H.	Repeat endoscopy at 5 years
A

Snare polypectomy

Polyps on long stalks are best managed by snare excision. It is important to retrieve the polyp for histology.

Colonic Polyps
May occur in isolation, or greater numbers as part of the polyposis syndromes. In FAP greater than 100 polyps are typically present. The risk of malignancy in association with adenomas is related to size, and is the order of 10% in a 1cm adenoma. Isolated adenomas seldom give risk of symptoms (unless large and distal). Distally sited villous lesions may produce mucous and if very large, electrolyte disturbances may occur.

Follow up of colonic polyps

  • Low risk 1 or 2 adenomas less than 1cm No follow up or re-colonoscopy at 5 years
  • Moderate risk 3 or 4 small adenomas or 1 adenoma greater than 1cm Re-scope at 3 years
  • High risk More than 5 small adenomas or more than 3 with 1 of them greater than 1cm Re scope at 1 year

Segmental resection or complete colectomy should be considered when:

  1. Incomplete excision of malignant polyp
  2. Malignant sessile polyp
  3. Malignant pedunculated polyp with submucosal invasion
  4. Polyps with poorly differentiated carcinoma
  5. Familial polyposis coli
    - Screening from teenager up to 40 years by 2 yearly sigmoidoscopy/colonoscopy
    - Panproctocolectomy and Ileostomy or Restorative Panproctocolectomy.

Rectal polypoidal lesions may be amenable to trans anal endoscopic microsurgery.

145
Q

A 60 year old lady is investigated for abdominal pain. A polyp is identified at the proximal descending colon, three small polyps are also noted in the sigmoid colon. The largest lesion is removed by snare polypectomy and the pathology report states that this polyp is a low grade dysplastic adenoma measuring 3cm in diameter. The remaining lesions are ablated using diathermy.

A.	Reassure and discharge
B.	Pan proctocolectomy
C.	Hot biopsy
D.	Snare polypectomy
E.	Segmental colonic resection
F.	Repeat endoscopy at 3 years
G.	Repeat endoscopy at 1 year
H.	Repeat endoscopy at 5 years
A

Repeat endoscopy at 1 year

She is at high risk of malignancy and should be closely followed up. Fulguration of polyps without histology is unhelpful.

Colonic Polyps
May occur in isolation, or greater numbers as part of the polyposis syndromes. In FAP greater than 100 polyps are typically present. The risk of malignancy in association with adenomas is related to size, and is the order of 10% in a 1cm adenoma. Isolated adenomas seldom give risk of symptoms (unless large and distal). Distally sited villous lesions may produce mucous and if very large, electrolyte disturbances may occur.

Follow up of colonic polyps

  • Low risk 1 or 2 adenomas less than 1cm No follow up or re-colonoscopy at 5 years
  • Moderate risk 3 or 4 small adenomas or 1 adenoma greater than 1cm Re-scope at 3 years
  • High risk More than 5 small adenomas or more than 3 with 1 of them greater than 1cm Re scope at 1 year

Segmental resection or complete colectomy should be considered when:

  1. Incomplete excision of malignant polyp
  2. Malignant sessile polyp
  3. Malignant pedunculated polyp with submucosal invasion
  4. Polyps with poorly differentiated carcinoma
  5. Familial polyposis coli
    - Screening from teenager up to 40 years by 2 yearly sigmoidoscopy/colonoscopy
    - Panproctocolectomy and Ileostomy or Restorative Panproctocolectomy.

Rectal polypoidal lesions may be amenable to trans anal endoscopic microsurgery.

146
Q

A 73 year old lady is investigated for anaemia. At colonoscopy she is found to have a flat broad based lesion in the caecum. This is biopsied and the histology report states that these have diagnostic features of an adenoma with high grade dysplasia.

 A.	Reassure and discharge
B.	Pan proctocolectomy
C.	Hot biopsy
D.	Snare polypectomy
E.	Segmental colonic resection
F.	Repeat endoscopy at 3 years
G.	Repeat endoscopy at 1 year
H.	Repeat endoscopy at 5 years
A

Segmental colonic resection

Management of right sided colonic polyps such as this is controversial. There is a high liklihood that this lesion will harbor an focus on invasive malignancy (which should generally be managed with resection). This is further suggested by the background history of anaemia. It is for this reason that resection over right sided colonic endoscopic mucosal resection is chosen.
Polypectomy of flat broad lesions in the right colon is difficult and where concern arises a right hemicolectomy is probably the safest option.

Colonic Polyps
May occur in isolation, or greater numbers as part of the polyposis syndromes. In FAP greater than 100 polyps are typically present. The risk of malignancy in association with adenomas is related to size, and is the order of 10% in a 1cm adenoma. Isolated adenomas seldom give risk of symptoms (unless large and distal). Distally sited villous lesions may produce mucous and if very large, electrolyte disturbances may occur.

Follow up of colonic polyps

  • Low risk 1 or 2 adenomas less than 1cm No follow up or re-colonoscopy at 5 years
  • Moderate risk 3 or 4 small adenomas or 1 adenoma greater than 1cm Re-scope at 3 years
  • High risk More than 5 small adenomas or more than 3 with 1 of them greater than 1cm Re scope at 1 year

Segmental resection or complete colectomy should be considered when:

  1. Incomplete excision of malignant polyp
  2. Malignant sessile polyp
  3. Malignant pedunculated polyp with submucosal invasion
  4. Polyps with poorly differentiated carcinoma
  5. Familial polyposis coli
    - Screening from teenager up to 40 years by 2 yearly sigmoidoscopy/colonoscopy
    - Panproctocolectomy and Ileostomy or Restorative Panproctocolectomy.

Rectal polypoidal lesions may be amenable to trans anal endoscopic microsurgery.

147
Q

A 75 year old man requires resection of an obstructing carcinoma of the splenic flexure.

A.	End ileostomy
B.	Loop ileostomy
C.	Ileo anal pouch
D.	Loop colostomy
E.	Pan proctocolectomy
F.	Extended right hemicolectomy
G.	Right hemicolectomy
H.	Anterior resection
I.	Anterior resection with covering loop ileostomy
A

Extended right hemicolectomy

Carcinoma of the splenic flexure requires extended right hemicolectomy. Or a left hemicolectomy. The ileocolic anastomosis has a lower leak rate, particularly when the bowel is obstructed.

Colorectal cancer treatment

Patients diagnosed as having colorectal cancer should be completely staged using CT of the chest/ abdomen and pelvis. Their entire colon should have been evaluated with colonoscopy or CT colonography. Patients whose tumours lie below the peritoneal reflection should have their mesorectum evaluated with MRI.

Once their staging is complete patients should be discussed within a dedicated colorectal MDT meeting and a treatment plan formulated.

Treatment of colonic cancer
Cancer of the colon is nearly always treated with surgery. Stents, surgical bypass and diversion stomas may all be used as palliative adjuncts. Resectional surgery is the only option for cure in patients with colon cancer. The procedure is tailored to the patient and the tumour location. The lymphatic drainage of the colon follows the arterial supply and therefore most resections are tailored around the resection of particular lymphatic chains (e.g. ileo-colic pedicle for right sided tumours). Some patients may have confounding factors that will govern the choice of procedure, for example a tumour in a patient from a HNPCC family may be better served with a panproctocolectomy rather than segmental resection. Following resection the decision has to be made regarding restoration of continuity. For an anastomosis to heal the key technical factors include; adequate blood supply, mucosal apposition and no tissue tension. Surrounding sepsis, unstable patients and inexperienced surgeons may compromise these key principles and in such circumstances it may be safer to construct an end stoma rather than attempting an anastomosis.
When a colonic cancer presents with an obstructing lesion; the options are to either stent it or resect. In modern practice it is unusual to simply defunction a colonic tumour with a proximal loop stoma. This differs from the situation in the rectum (see below).
Following resection patients with risk factors for disease recurrence are usually offered chemotherapy, a combination of 5FU and oxaliplatin is common.

Rectal cancer
The management of rectal cancer is slightly different to that of colonic cancer. This reflects the rectum’s anatomical location and the challenges posed as a result. Tumours located in the rectum can be surgically resected with either an anterior resection or an abdomino - perineal resection. The technical aspects governing the choice between these two procedures can be complex to appreciate and the main point to appreciate for the MRCS is that involvement of the sphincter complex or very low tumours require APER. In the rectum a 2cm distal clearance margin is required and this may also impact on the procedure chosen. In addition to excision of the rectal tube an integral part of the procedure is a meticulous dissection of the mesorectal fat and lymph nodes (total mesorectal excision/ TME). In rectal cancer surgery invovlement of the cirumferential resection margin carries a high risk of disease recurrence. Because the rectum is an extraperitoneal structure (until you remove it that is!) it is possible to irradiate it, something which cannot be offered for colonic tumours. This has a major impact in rectal cancer treatment and many patients will be offered neoadjuvent radiotherapy (both long and short course) prior to resectional surgery. Patients with T1, 2 and 3 /N0 disease on imaging do not require irradiation and should proceed straight to surgery. Patients with T4 disease will typically have long course chemo radiotherapy. Patients presenting with large bowel obstruction from rectal cancer should not undergo resectional surgery without staging as primary treatment (very different from colonic cancer). This is because rectal surgery is more technically demanding, the anastomotic leak rate is higher and the danger of a positive resection margin in an unstaged patient is high. Therefore patients with obstructing rectal cancer should have a defunctioning loop colostomy.

Summary of procedures
The operations for cancer are segmental resections based on blood supply and lymphatic drainage. These commonly performed procedures are core knowledge for the MRCS and should be understood.

Site of cancer // Type of resection // Anastomosis // Risk of leak

  • Right colon Right hemicolectomy Ileo-colic Low <5%
  • Transverse Extended right hemicolectomy Ileo-colic Low <5%
  • Splenic flexure Extended right hemicolectomy Ileo-colic Low <5%
  • Splenic flexure Left hemicolectomy Colo-colon 2-5%
  • Left colon Left hemicolectomy Colo-colon 2-5%
  • Sigmoid colon High anterior resection Colo-rectal 5%
  • Upper rectum Anterior resection (TME) Colo-rectal 5%
  • Low rectum Anterior resection (Low TME) Colo-rectal (+/- Defunctioning stoma) 10%
  • Anal verge Abdomino-perineal excision of colon and rectum None n/a

In the emergency setting, where the bowel has perforated, the risk of an anastomosis is much greater, particularly when the anastomosis is colon-colon. In this situation, an end colostomy is often safer and can be reversed later. When resection of the sigmoid colon is performed and an end colostomy is fashioned the operation is referred to as a Hartmans procedure. Whilst left sided resections are more risky, ileo-colic anastomoses are relatively safe even in the emergency setting and do not need to be defunctioned.

148
Q

A patient presenting with a large bowel obstruction from a low rectal cancer.

A.	End ileostomy
B.	Loop ileostomy
C.	Ileo anal pouch
D.	Loop colostomy
E.	Pan proctocolectomy
F.	Extended right hemicolectomy
G.	Right hemicolectomy
H.	Anterior resection
I.	Anterior resection with covering loop ileostomy
A

Loop colostomy

This patient should be defunctioned, definitive surgery should wait until staging is completed. A loop ileostomy will not satisfactorily decompress an acutely obstructed colon. Low rectal cancers that are obstructed should not usually be primarily resected. The obstructed colon that would be used for anastomosis would carry a high risk of anastomotic dehisence. In addition, as this is an emergency presentation, staging may not be completed, an attempted resection may therefore compromise the circumferential resection margin, with an associated risk of local recurrence.

Colorectal cancer treatment

Patients diagnosed as having colorectal cancer should be completely staged using CT of the chest/ abdomen and pelvis. Their entire colon should have been evaluated with colonoscopy or CT colonography. Patients whose tumours lie below the peritoneal reflection should have their mesorectum evaluated with MRI.

Once their staging is complete patients should be discussed within a dedicated colorectal MDT meeting and a treatment plan formulated.

Treatment of colonic cancer
Cancer of the colon is nearly always treated with surgery. Stents, surgical bypass and diversion stomas may all be used as palliative adjuncts. Resectional surgery is the only option for cure in patients with colon cancer. The procedure is tailored to the patient and the tumour location. The lymphatic drainage of the colon follows the arterial supply and therefore most resections are tailored around the resection of particular lymphatic chains (e.g. ileo-colic pedicle for right sided tumours). Some patients may have confounding factors that will govern the choice of procedure, for example a tumour in a patient from a HNPCC family may be better served with a panproctocolectomy rather than segmental resection. Following resection the decision has to be made regarding restoration of continuity. For an anastomosis to heal the key technical factors include; adequate blood supply, mucosal apposition and no tissue tension. Surrounding sepsis, unstable patients and inexperienced surgeons may compromise these key principles and in such circumstances it may be safer to construct an end stoma rather than attempting an anastomosis.
When a colonic cancer presents with an obstructing lesion; the options are to either stent it or resect. In modern practice it is unusual to simply defunction a colonic tumour with a proximal loop stoma. This differs from the situation in the rectum (see below).
Following resection patients with risk factors for disease recurrence are usually offered chemotherapy, a combination of 5FU and oxaliplatin is common.

Rectal cancer
The management of rectal cancer is slightly different to that of colonic cancer. This reflects the rectum’s anatomical location and the challenges posed as a result. Tumours located in the rectum can be surgically resected with either an anterior resection or an abdomino - perineal resection. The technical aspects governing the choice between these two procedures can be complex to appreciate and the main point to appreciate for the MRCS is that involvement of the sphincter complex or very low tumours require APER. In the rectum a 2cm distal clearance margin is required and this may also impact on the procedure chosen. In addition to excision of the rectal tube an integral part of the procedure is a meticulous dissection of the mesorectal fat and lymph nodes (total mesorectal excision/ TME). In rectal cancer surgery invovlement of the cirumferential resection margin carries a high risk of disease recurrence. Because the rectum is an extraperitoneal structure (until you remove it that is!) it is possible to irradiate it, something which cannot be offered for colonic tumours. This has a major impact in rectal cancer treatment and many patients will be offered neoadjuvent radiotherapy (both long and short course) prior to resectional surgery. Patients with T1, 2 and 3 /N0 disease on imaging do not require irradiation and should proceed straight to surgery. Patients with T4 disease will typically have long course chemo radiotherapy. Patients presenting with large bowel obstruction from rectal cancer should not undergo resectional surgery without staging as primary treatment (very different from colonic cancer). This is because rectal surgery is more technically demanding, the anastomotic leak rate is higher and the danger of a positive resection margin in an unstaged patient is high. Therefore patients with obstructing rectal cancer should have a defunctioning loop colostomy.

Summary of procedures
The operations for cancer are segmental resections based on blood supply and lymphatic drainage. These commonly performed procedures are core knowledge for the MRCS and should be understood.

Site of cancer // Type of resection // Anastomosis // Risk of leak

  • Right colon Right hemicolectomy Ileo-colic Low <5%
  • Transverse Extended right hemicolectomy Ileo-colic Low <5%
  • Splenic flexure Extended right hemicolectomy Ileo-colic Low <5%
  • Splenic flexure Left hemicolectomy Colo-colon 2-5%
  • Left colon Left hemicolectomy Colo-colon 2-5%
  • Sigmoid colon High anterior resection Colo-rectal 5%
  • Upper rectum Anterior resection (TME) Colo-rectal 5%
  • Low rectum Anterior resection (Low TME) Colo-rectal (+/- Defunctioning stoma) 10%
  • Anal verge Abdomino-perineal excision of colon and rectum None n/a

In the emergency setting, where the bowel has perforated, the risk of an anastomosis is much greater, particularly when the anastomosis is colon-colon. In this situation, an end colostomy is often safer and can be reversed later. When resection of the sigmoid colon is performed and an end colostomy is fashioned the operation is referred to as a Hartmans procedure. Whilst left sided resections are more risky, ileo-colic anastomoses are relatively safe even in the emergency setting and do not need to be defunctioned.

149
Q

A 45 year old man presents with a carcinoma 10cm from the anal verge, he has completed a long course of chemoradiotherapy and has achieved downstaging with no evidence of threatened circumferential margin on MRI scanning.

A.	End ileostomy
B.	Loop ileostomy
C.	Ileo anal pouch
D.	Loop colostomy
E.	Pan proctocolectomy
F.	Extended right hemicolectomy
G.	Right hemicolectomy
H.	Anterior resection
I.	Anterior resection with covering loop ileostomy
A

Anterior resection with covering loop ileostomy

Low rectal cancer is usually treated with a low anterior resection. Contraindications to this include involvement of the sphincters (unlikely here) and poor sphincter function that would lead to unsatisfactory function post resection. Most colorectal surgeons defunction resections below the peritoneal reflection as they have an intrinsically high risk of anastomotic leak. A loop ileostomy provides a safe an satisfactory method of defunctioning these patients. A contrast enema should be performed prior to stoma reversal.

Colorectal cancer treatment

Patients diagnosed as having colorectal cancer should be completely staged using CT of the chest/ abdomen and pelvis. Their entire colon should have been evaluated with colonoscopy or CT colonography. Patients whose tumours lie below the peritoneal reflection should have their mesorectum evaluated with MRI.

Once their staging is complete patients should be discussed within a dedicated colorectal MDT meeting and a treatment plan formulated.

Treatment of colonic cancer
Cancer of the colon is nearly always treated with surgery. Stents, surgical bypass and diversion stomas may all be used as palliative adjuncts. Resectional surgery is the only option for cure in patients with colon cancer. The procedure is tailored to the patient and the tumour location. The lymphatic drainage of the colon follows the arterial supply and therefore most resections are tailored around the resection of particular lymphatic chains (e.g. ileo-colic pedicle for right sided tumours). Some patients may have confounding factors that will govern the choice of procedure, for example a tumour in a patient from a HNPCC family may be better served with a panproctocolectomy rather than segmental resection. Following resection the decision has to be made regarding restoration of continuity. For an anastomosis to heal the key technical factors include; adequate blood supply, mucosal apposition and no tissue tension. Surrounding sepsis, unstable patients and inexperienced surgeons may compromise these key principles and in such circumstances it may be safer to construct an end stoma rather than attempting an anastomosis.
When a colonic cancer presents with an obstructing lesion; the options are to either stent it or resect. In modern practice it is unusual to simply defunction a colonic tumour with a proximal loop stoma. This differs from the situation in the rectum (see below).
Following resection patients with risk factors for disease recurrence are usually offered chemotherapy, a combination of 5FU and oxaliplatin is common.

Rectal cancer
The management of rectal cancer is slightly different to that of colonic cancer. This reflects the rectum’s anatomical location and the challenges posed as a result. Tumours located in the rectum can be surgically resected with either an anterior resection or an abdomino - perineal resection. The technical aspects governing the choice between these two procedures can be complex to appreciate and the main point to appreciate for the MRCS is that involvement of the sphincter complex or very low tumours require APER. In the rectum a 2cm distal clearance margin is required and this may also impact on the procedure chosen. In addition to excision of the rectal tube an integral part of the procedure is a meticulous dissection of the mesorectal fat and lymph nodes (total mesorectal excision/ TME). In rectal cancer surgery invovlement of the cirumferential resection margin carries a high risk of disease recurrence. Because the rectum is an extraperitoneal structure (until you remove it that is!) it is possible to irradiate it, something which cannot be offered for colonic tumours. This has a major impact in rectal cancer treatment and many patients will be offered neoadjuvent radiotherapy (both long and short course) prior to resectional surgery. Patients with T1, 2 and 3 /N0 disease on imaging do not require irradiation and should proceed straight to surgery. Patients with T4 disease will typically have long course chemo radiotherapy. Patients presenting with large bowel obstruction from rectal cancer should not undergo resectional surgery without staging as primary treatment (very different from colonic cancer). This is because rectal surgery is more technically demanding, the anastomotic leak rate is higher and the danger of a positive resection margin in an unstaged patient is high. Therefore patients with obstructing rectal cancer should have a defunctioning loop colostomy.

Summary of procedures
The operations for cancer are segmental resections based on blood supply and lymphatic drainage. These commonly performed procedures are core knowledge for the MRCS and should be understood.

Site of cancer // Type of resection // Anastomosis // Risk of leak

  • Right colon Right hemicolectomy Ileo-colic Low <5%
  • Transverse Extended right hemicolectomy Ileo-colic Low <5%
  • Splenic flexure Extended right hemicolectomy Ileo-colic Low <5%
  • Splenic flexure Left hemicolectomy Colo-colon 2-5%
  • Left colon Left hemicolectomy Colo-colon 2-5%
  • Sigmoid colon High anterior resection Colo-rectal 5%
  • Upper rectum Anterior resection (TME) Colo-rectal 5%
  • Low rectum Anterior resection (Low TME) Colo-rectal (+/- Defunctioning stoma) 10%
  • Anal verge Abdomino-perineal excision of colon and rectum None n/a

In the emergency setting, where the bowel has perforated, the risk of an anastomosis is much greater, particularly when the anastomosis is colon-colon. In this situation, an end colostomy is often safer and can be reversed later. When resection of the sigmoid colon is performed and an end colostomy is fashioned the operation is referred to as a Hartmans procedure. Whilst left sided resections are more risky, ileo-colic anastomoses are relatively safe even in the emergency setting and do not need to be defunctioned.

150
Q

A 52 year old man falls off his bike. He is found to have a pelvic fracture. On examination, he is found to have perineal oedema and on PR the prostate is not palpable. A urine dipstick shows blood. What is the most likely underlying problem?

	Bulbar urethral rupture
	Bladder outflow obstruction
	Bladder rupture
	Bladder contusion
	Membranous urethral rupture
A

A pelvic fracture and highly displaced prostate should indicate a diagnosis of membranous urethral rupture.

Lower genitourinary tract trauma

Most bladder injuries occur due to blunt trauma
85% associated with pelvic fractures
Easily overlooked during assessment in trauma
Up to 10% of male pelvic fractures are associated with urethral or bladder injuries

Types of injury

Urethral injury	
Mainly in males
Blood at the meatus (50% cases)
There are 2 types:
i.Bulbar rupture
- most common
- straddle type injury e.g. bicycles
- triad signs: urinary retention, perineal haematoma, blood at the meatus
ii. Membranous rupture
- can be extra or intraperitoneal
- commonly due to pelvic fracture
- Penile or perineal oedema/ hematoma 
- PR: prostate displaced upwards (beware co-existing retroperitoneal haematomas as they may make examination difficult)
  • Investigation: ascending urethrogram
  • Management: suprapubic catheter (surgical placement, not percutaneously)
    External genitalia injuries (i.e., the penis and the scrotum)
    Secondary to injuries caused by penetration, blunt trauma, continence- or sexual pleasure-enhancing devices, and mutilation
    Bladder injury
    rupture is intra or extraperitoneal
    presents with haematuria or suprapubic pain
    history of pelvic fracture and inability to void: always suspect bladder or urethral injury
    inability to retrieve all fluid used to irrigate the bladder through a Foley catheter indicates bladder injury
    investigation- IVU or cystogram
    management: laparotomy if intraperitoneal, conservative if extraperitoneal
151
Q

A 52 year old woman with known Hashimotos thyroiditis presents with a neck swelling. She describes it as rapidly increasing in size over 3 months and she complains of dysphagia to solids. On examination, there is an asymmetrical swelling of the thyroid gland. What is the most likely diagnosis?

	Anaplastic thyroid cancer
	Follicular thyroid cancer
	Papillary thyroid cancer
	Lymphoma
	Toxic adenoma
A

Thyroid lymphoma (Non Hodgkin’s B cell lymphoma) is rare. It should be considered in patients with a background of Hashimoto’s thyroiditis and a rapid growth in size of the thyroid gland. Diagnosis can be made with core needle biopsy; however an incisional biopsy may be needed. Radiotherapy is the main treatment option.

Thyroiditis

Sub acute thyroiditis
Subacute thyroiditis (also known as De Quervain's thyroiditis) is thought to occur following viral infection and typically presents with hyperthyroidism
Features
Hyperthyroidism
Painful goitre
Raised ESR
Globally reduced uptake on iodine-131 scan

Management
Usually self-limiting - most patients do not require treatment
Thyroid pain may respond to aspirin or other NSAIDs
In more severe cases steroids are used, particularly if hypothyroidism develops

Hashimotos thyroiditis
Hashimotos thyroiditis is an immunological disorder in which lymphocytes become sensitised to thyroidal antigens. The three most important antibodies include; thyroglobulin, TPO and TSH-R. During the early phase of Hashimotos the the thyroglobulin antibody is markedly elevated and then declines.

Features
Goitre and either euthyroid or mild hypothyroidism
Progressive hypothyroidism (and associated symptoms)

Management
During the hyperthyroid phase of illness beta blockers may manage symptoms
As hypothyroidism develops patients may require thyroxine

152
Q

A parent brings her 4 year old child to the surgical clinic. She has noticed an intermittent swelling in the right scrotum that is worse in the evening. On examination he has a soft fluctuant swelling in the right scrotum that cannot be separated from the testis. It transilluminates when a pen torch is held against it.

A.	Antibiotics
B.	Aspiration
C.	Testicular exploration after 6 hours
D.	Testicular exploration within 6 hours
E.	Orchidectomy via inguinal approach
F.	Orchidectomy via scrotal approach
G.	No treatment needed
H.	Ligation of patent processus vaginalis via inguinal approach
I.	Jaboulay procedure via scrotal approach
A

Ligation of patent processus vaginalis via inguinal approach

In children, a hydrocele is most commonly due to a persistent processus vaginalis. The swelling is intermittent and in most cases that are diagnosed in infancy the hydrocele resolves. Cases that persist beyond two years of age are best managed surgically and the surgical approach is usually made via the inguinal canal where the patent processus is identified and ligated.

Inguinal hernia If inguinoscrotal swelling; cannot “get above it” on examination
Cough impulse may be present
May be reducible

Testicular tumours Often discrete testicular nodule (may have associated hydrocele)
Symptoms of metastatic disease may be present
USS scrotum and serum AFP and β HCG required

Acute epididymo-orchitis Often history of dysuria and urethral discharge
Swelling may be tender and eased by elevating testis
Most cases due to Chlamydia
Infections with other gram negative organisms may be associated with underlying structural abnormality

Epididymal cysts Single or multiple cysts
May contain clear or opalescent fluid (spermatoceles)
Usually occur over 40 years of age
Painless
Lie above and behind testis
It is usually possible to “get above the lump” on examination

Hydrocele Non painful, soft fluctuant swelling
Often possible to “get above it” on examination
Usually contain clear fluid
Will often transilluminate
May be presenting feature of testicular cancer in young men

Testicular torsion Severe, sudden onset testicular pain
Risk factors include abnormal testicular lie
Typically affects adolescents and young males
On examination testis is tender and pain not eased by elevation
Urgent surgery is indicated, the contra lateral testis should also be fixed

Varicocele Varicosities of the pampiniform plexus
Typically occur on left (because testicular vein drains into renal vein)
May be presenting feature of renal cell carcinoma
Affected testis may be smaller and bilateral varicoceles may affect fertility

Management
Testicular malignancy is always treated with orchidectomy via an inguinal approach. This allows high ligation of the testicular vessels and avoids exposure of another lymphatic field to the tumour.
Torsion is commonest in young teenagers and the history in older children can be difficult to elicit. Intermittent torsion is a recognised problem. The treatment is prompt surgical exploration and testicular fixation. This can be achieved using sutures or by placement of the testis in a Dartos pouch.
Varicoceles are usually managed conservatively. If there are concerns about testicular function of infertility then surgery or radiological management can be considered.
Epididymal cysts can be excised using a scrotal approach
Hydroceles are managed differently in children where the underlying pathology is a patent processus vaginalis and therefore an inguinal approach is used in children so that the processus can be ligated. In adults a scrotal approach is preferred and the hydrocele sac excised or plicated.

153
Q

A 20 year old complains of severe pain in the right scrotal area after jumping onto his moped. He has noticed discomfort intermittently in this area over the past few months. Clinically the testis is tender.

A.	Antibiotics
B.	Aspiration
C.	Testicular exploration after 6 hours
D.	Testicular exploration within 6 hours
E.	Orchidectomy via inguinal approach
F.	Orchidectomy via scrotal approach
G.	No treatment needed
H.	Ligation of patent processus vaginalis via inguinal approach
I.	Jaboulay procedure via scrotal approach
A

Testicular exploration within 6 hours

Testicular torsion: Severe pain which can be spontaneous or precipitated by minor trauma. The patient may have noticed pain previously. Surgical intervention is needed as soon as possible to prevent the risk of loss of the testis.

Inguinal hernia If inguinoscrotal swelling; cannot “get above it” on examination
Cough impulse may be present
May be reducible

Testicular tumours Often discrete testicular nodule (may have associated hydrocele)
Symptoms of metastatic disease may be present
USS scrotum and serum AFP and β HCG required

Acute epididymo-orchitis Often history of dysuria and urethral discharge
Swelling may be tender and eased by elevating testis
Most cases due to Chlamydia
Infections with other gram negative organisms may be associated with underlying structural abnormality

Epididymal cysts Single or multiple cysts
May contain clear or opalescent fluid (spermatoceles)
Usually occur over 40 years of age
Painless
Lie above and behind testis
It is usually possible to “get above the lump” on examination

Hydrocele Non painful, soft fluctuant swelling
Often possible to “get above it” on examination
Usually contain clear fluid
Will often transilluminate
May be presenting feature of testicular cancer in young men

Testicular torsion Severe, sudden onset testicular pain
Risk factors include abnormal testicular lie
Typically affects adolescents and young males
On examination testis is tender and pain not eased by elevation
Urgent surgery is indicated, the contra lateral testis should also be fixed

Varicocele Varicosities of the pampiniform plexus
Typically occur on left (because testicular vein drains into renal vein)
May be presenting feature of renal cell carcinoma
Affected testis may be smaller and bilateral varicoceles may affect fertility

Management
Testicular malignancy is always treated with orchidectomy via an inguinal approach. This allows high ligation of the testicular vessels and avoids exposure of another lymphatic field to the tumour.
Torsion is commonest in young teenagers and the history in older children can be difficult to elicit. Intermittent torsion is a recognised problem. The treatment is prompt surgical exploration and testicular fixation. This can be achieved using sutures or by placement of the testis in a Dartos pouch.
Varicoceles are usually managed conservatively. If there are concerns about testicular function of infertility then surgery or radiological management can be considered.
Epididymal cysts can be excised using a scrotal approach
Hydroceles are managed differently in children where the underlying pathology is a patent processus vaginalis and therefore an inguinal approach is used in children so that the processus can be ligated. In adults a scrotal approach is preferred and the hydrocele sac excised or plicated.

154
Q

A 44 year old man is referred to the clinic because of a swelling and discomfort in the right scrotum. This is present most of the time and he is otherwise well with no urinary symptoms. On examination he has a soft, fluctuant swelling in the right scrotum that transilluminates easily. An ultrasound is performed that confirms that the underlying testicle is structurally normal.

A.	Antibiotics
B.	Aspiration
C.	Testicular exploration after 6 hours
D.	Testicular exploration within 6 hours
E.	Orchidectomy via inguinal approach
F.	Orchidectomy via scrotal approach
G.	No treatment needed
H.	Ligation of patent processus vaginalis via inguinal approach
I.	Jaboulay procedure via scrotal approach
A

Jaboulay procedure via scrotal approach

Adult hydroceles are less commonly due to the persistence of embryonic remnants and therefore can be managed via a scrotal approach. Both the Lords and Jaboulay procedures are reasonable options.

Inguinal hernia If inguinoscrotal swelling; cannot “get above it” on examination
Cough impulse may be present
May be reducible

Testicular tumours Often discrete testicular nodule (may have associated hydrocele)
Symptoms of metastatic disease may be present
USS scrotum and serum AFP and β HCG required

Acute epididymo-orchitis Often history of dysuria and urethral discharge
Swelling may be tender and eased by elevating testis
Most cases due to Chlamydia
Infections with other gram negative organisms may be associated with underlying structural abnormality

Epididymal cysts Single or multiple cysts
May contain clear or opalescent fluid (spermatoceles)
Usually occur over 40 years of age
Painless
Lie above and behind testis
It is usually possible to “get above the lump” on examination

Hydrocele Non painful, soft fluctuant swelling
Often possible to “get above it” on examination
Usually contain clear fluid
Will often transilluminate
May be presenting feature of testicular cancer in young men

Testicular torsion Severe, sudden onset testicular pain
Risk factors include abnormal testicular lie
Typically affects adolescents and young males
On examination testis is tender and pain not eased by elevation
Urgent surgery is indicated, the contra lateral testis should also be fixed

Varicocele Varicosities of the pampiniform plexus
Typically occur on left (because testicular vein drains into renal vein)
May be presenting feature of renal cell carcinoma
Affected testis may be smaller and bilateral varicoceles may affect fertility

Management
Testicular malignancy is always treated with orchidectomy via an inguinal approach. This allows high ligation of the testicular vessels and avoids exposure of another lymphatic field to the tumour.
Torsion is commonest in young teenagers and the history in older children can be difficult to elicit. Intermittent torsion is a recognised problem. The treatment is prompt surgical exploration and testicular fixation. This can be achieved using sutures or by placement of the testis in a Dartos pouch.
Varicoceles are usually managed conservatively. If there are concerns about testicular function of infertility then surgery or radiological management can be considered.
Epididymal cysts can be excised using a scrotal approach
Hydroceles are managed differently in children where the underlying pathology is a patent processus vaginalis and therefore an inguinal approach is used in children so that the processus can be ligated. In adults a scrotal approach is preferred and the hydrocele sac excised or plicated.

155
Q

A 29 year old man presents to the clinic with a recurrent thyroid cyst. It has been drained now on three occasions. Each time the cyst is aspirated and cytology is reassuring. What is the most appropriate course of action?

	Reassure and discharge
	Resection of the ipsilateral thyroid lobe
	Enucleation of the cyst
	Total thyroidectomy
	Radio-isotope scan
A

Resection of the ipsilateral thyroid lobe

Persist refilling cysts may be associated with a well differentiated tumour and should be removed by lobectomy.

Large multinodular goitre Surgery for pressure symptoms. Total thyroidectomy is treatment of choice

Toxic nodule Hemithyroidectomy

Follicular lesion (THY 3f) Hemithyroidectomy to establish diagnosis

Papillary thyroid cancer Total thyroidectomy and central compartment nodal dissection (extended lymphadenectomy as required)

Follicular thyroid cancer Total thyroidectomy (usually completion as already had hemithyroidectomy)

Anaplastic thyroid cancer Palliative radiotherapy

Medullary thyroid cancer Total thyroidectomy (screen for other MEN tumours)

Lymphoma of the thyroid Consider core biopsy

Persistent refilling cysts Hemithyroidectomy

Graves disease with significant eye signs Total thyroidectomy

Graves disease without significant eye signs Patient choice radioiodine Vs surgery

156
Q

A 45 year old man undergoes an upper gastrointestinal endoscopy for a benign oesophageal stricture. This is dilated and he suffers an iatrogenic perforation at the site. His imaging shows a small contained leak and a small amount of surgical emphysema. What is the most appropriate nutritional option?

	Nil by mouth and intravenous fluids alone
	Intravenous fluids and sips orally
	Total parenteral nutrition
	Nasogastric feeding
	PEG tube feeding
A

Iatrogenic perforations of the oesophagus may be managed non operatively. This usually involves a nil by mouth regime, tube thoracostomy may be needed. Total parenteral nutrition is the safest option. Insertion of NG feeding tubes and PEG tubes may complicate the process or allow feed to enter the perforation site.

Nutrition options in surgical patients

Oral intake
Easiest option
May be supplemented by calorie rich dietary supplements
May contra indicated following certain procedures

Naso gastric feeding
Usually administered via fine bore naso gastric feeding tube
Complications relate to aspiration of feed or misplaced tube
May be safe to use in patients with impaired swallow
Often contra indicated following head injury due to risks associated with tube insertion

Naso jejunal feeding
Avoids problems of feed pooling in stomach (and risk of aspiration)
Insertion of feeding tube more technically complicated (easiest if done intra operatively)
Safe to use following oesophagogastric surgery

Feeding jejunostomy
Surgically sited feeding tube
May be used for long term feeding
Low risk of aspiration and thus safe for long term feeding following upper GI surgery
Main risks are those of tube displacement and peritubal leakage immediately following insertion, which carries a risk of peritonitis

Percutaneous endoscopic gastrostomy
Combined endoscopic and percutaneous tube insertion
May not be technically possible in those patients who cannot undergo successful endoscopy
Risks include aspiration and leakage at the insertion site

Total parenteral nutrition
The definitive option in those patients in whom enteral feeding is contra indicated
Individualised prescribing and monitoring needed
Should be administered via a central vein as it is strongly phlebitic
Long term use is associated with fatty liver and deranged LFT’s

157
Q

A 40 year old lady presents with new onset dyspepsia. She is diagnosed as having a localised cacinoma of the pancreatic head.

A.	Gastrojejunostomy
B.	Pancreatoduodenectomy
C.	MRI guided pancreatic stent
D.	Endoscopic biliary stent
E.	Duodenoduodenostomy
F.	Pancreatic radiotherapy
A

Pancreatoduodenectomy

Localised carcinoma of the pancreas is treated with a pancreatoduodenectomy, the eponymous name for this is a Whipples procedure. Newer variants of the procedure include pylorus preservation. Adjuvent chemotherapy is often used.

Pancreatic cancer

Adenocarcinoma
Risk factors: Smoking, diabetes, adenoma, familial adenomatous polyposis
Mainly occur in the head of the pancreas (70%)
Spread locally and metastasizes to the liver
Carcinoma of the pancreas should be differentiated from other periampullary tumours with better prognosis

Clinical features
Weight loss
Painless jaundice
Epigastric discomfort (pain usually due to invasion of the coeliac plexus is a late feature)
Pancreatitis
Trousseau’s sign: migratory superficial thrombophlebitis

Investigations
USS: May miss small lesions
CT Scanning (pancreatic protocol). If unresectable on CT then no further staging needed
PET/CT for those with operable disease on CT alone
ERCP/ MRI for bile duct assessment
Staging laparoscopy to exclude peritoneal disease

Management
Head of pancreas: Whipple’s resection (SE dumping and ulcers). Newer techniques include pylorus preservation and SMA/ SMV resection
Carcinoma body and tail: poor prognosis, distal pancreatectomy, if operable
Usually adjuvent chemotherapy for resectable disease
ERCP and stent for jaundice and palliation
Surgical bypass may be needed for duodenal obstruction

158
Q

A 67 year old lady presents with jaundice and abdominal pain. Her investigations show a dilated common bile duct, a carcinoma of the pancreatic head compressing the pancreatic duct. Her liver contains bi-lobar metastasis.

A.	Gastrojejunostomy
B.	Pancreatoduodenectomy
C.	MRI guided pancreatic stent
D.	Endoscopic biliary stent
E.	Duodenoduodenostomy
F.	Pancreatic radiotherapy
A

Endoscopic biliary stent

Jaundice associated with pancreatic cancer is best managed with a stent. These are usually inserted at the time of ERCP. Consideration here should be given to the use of a metallic stent (which is contra indicated where resection is contemplated).

Pancreatic cancer

Adenocarcinoma
Risk factors: Smoking, diabetes, adenoma, familial adenomatous polyposis
Mainly occur in the head of the pancreas (70%)
Spread locally and metastasizes to the liver
Carcinoma of the pancreas should be differentiated from other periampullary tumours with better prognosis

Clinical features
Weight loss
Painless jaundice
Epigastric discomfort (pain usually due to invasion of the coeliac plexus is a late feature)
Pancreatitis
Trousseau’s sign: migratory superficial thrombophlebitis

Investigations
USS: May miss small lesions
CT Scanning (pancreatic protocol). If unresectable on CT then no further staging needed
PET/CT for those with operable disease on CT alone
ERCP/ MRI for bile duct assessment
Staging laparoscopy to exclude peritoneal disease

Management
Head of pancreas: Whipple’s resection (SE dumping and ulcers). Newer techniques include pylorus preservation and SMA/ SMV resection
Carcinoma body and tail: poor prognosis, distal pancreatectomy, if operable
Usually adjuvent chemotherapy for resectable disease
ERCP and stent for jaundice and palliation
Surgical bypass may be needed for duodenal obstruction

159
Q

A 67 year old lady presents with symptoms of persistent vomiting. Her investigations show gastric outlet obstruction from a carcinoma of the pancreatic head. Her liver contains bi-lobar metastases.

A.	Gastrojejunostomy
B.	Pancreatoduodenectomy
C.	MRI guided pancreatic stent
D.	Endoscopic biliary stent
E.	Duodenoduodenostomy
F.	Pancreatic radiotherapy
A

Gastrojejunostomy

Gastric outlet obstruction from pancreatic cancer is best managed with a surgical bypass procedure or a duodenal stent (if the disease is not resectable or curable).

Pancreatic cancer

Adenocarcinoma
Risk factors: Smoking, diabetes, adenoma, familial adenomatous polyposis
Mainly occur in the head of the pancreas (70%)
Spread locally and metastasizes to the liver
Carcinoma of the pancreas should be differentiated from other periampullary tumours with better prognosis

Clinical features
Weight loss
Painless jaundice
Epigastric discomfort (pain usually due to invasion of the coeliac plexus is a late feature)
Pancreatitis
Trousseau’s sign: migratory superficial thrombophlebitis

Investigations
USS: May miss small lesions
CT Scanning (pancreatic protocol). If unresectable on CT then no further staging needed
PET/CT for those with operable disease on CT alone
ERCP/ MRI for bile duct assessment
Staging laparoscopy to exclude peritoneal disease

Management
Head of pancreas: Whipple’s resection (SE dumping and ulcers). Newer techniques include pylorus preservation and SMA/ SMV resection
Carcinoma body and tail: poor prognosis, distal pancreatectomy, if operable
Usually adjuvent chemotherapy for resectable disease
ERCP and stent for jaundice and palliation
Surgical bypass may be needed for duodenal obstruction

160
Q

A 65 year old man is admitted with jaundice and investigations demonstrate a carcinoma of the pancreatic head. An ERCP is attempted but the surgeon is unable to cannulate the ampulla.

A. ERCP
B. MRCP
C. Percutaneous transhepatic cholangiogram
D. Laparotomy
E. Laparotomy and formation of hepatico-jejunostomy
F. Laparoscopic biliary bypass
G. CT scan

A

Percutaneous transhepatic cholangiogram

Cancer of the pancreatic head will cause obstructive jaundice and intrahepatic duct dilatation. When an ERCP has failed the most appropriate option is to attempt a PTC. This procedure is always preceded by an ultrasound (which presumably this patient has already had or they would not be undergoing an ERCP). Prior to performing the PTC it is important to stage the disease and establish resectability or not. This is because the PTC drains frequently dislodge and fall out. It is usually desirable to pass a stent at the time of doing the PTC to mitigate the effects of this problem.

Gallstones Typically history of biliary colic or episodes of chlolecystitis. Obstructive type history and test results. Usually small calibre gallstones which can pass through the cystic duct. In Mirizzi syndrome the stone may compress the bile duct directly- one of the rare times that cholecystitis may present with jaundice
Cholangitis Usually obstructive and will have Charcots triad of symptoms (pain, fever, jaundice) Ascending infection of the bile ducts usually by E. coli and by definition occurring in a pool of stagnant bile.
Pancreatic cancer Typically painless jaundice with palpable gallbladder (Courvoisier’s Law) Direct occlusion of distal bile duct or pancreatic duct by tumour. Sometimes nodal disease at the portal hepatis may be the culprit in which case the bile duct may be of normal calibre.
TPN associated jaundice Usually follows long term use and is usually painless with non obstructive features Often due to hepatic dysfunction and fatty liver which may occur with long term TPN usage.
Bile duct injury Depending upon the type of injury may be of sudden or gradual onset and is usually of obstructive type Often due to a difficult cholecystectomy when anatomy in Calots triangle is not appreciated. In the worst scenario the bile duct is excised and jaundice offers rapidly post operatively. More insidious is that of bile duct stenosis which may be caused by clips or diathermy injury.
Cholangiocarcinoma Gradual onset obstructive pattern Direct occlusion by disease and also extrinsic compression by nodal disease at the porta hepatis.
Septic surgical patient Usually hepatic features Combination of impaired biliary excretion and drugs such as ciprofloxacin which may cause cholestasis.
Metastatic disease Mixed hepatic and post hepatic Combination of liver synthetic failure (late) and extrinsic compression by nodal disease and anatomical compression of intra hepatic structures (earlier)

Diagnosis
An ultrasound of the liver and biliary tree is the most commonly used first line test. This will establish bile duct calibre, often ascertain the presence of gallstones, may visualise pancreatic masses and other lesions. The most important clinical question is essentially the extent of biliary dilatation and its distribution.

Where pancreatic neoplasia is suspected, the next test should be a pancreatic protocol CT scan. With liver tumours and cholangiocarcinoma an MRI/ MRCP is often the preferred option. PET scans may be used to stage a number of malignancies but do not routinely form part of first line testing.

Where MRCP fails to give adequate information an ERCP may be necessary. In many cases this may form part of patient management. It is however, invasive and certainly not without risk and highly operator dependent.

Management
Clearly this will depend to an extent upon the underlying cause but relief of jaundice is important, even if surgery forms part of the planned treatment. Patients with unrelieved jaundice have a much higher incidence of septic complications, bleeding and death.

Screen for and address any clotting irregularities

In patients with malignancy a stent will need to be inserted. These come in two main types; metal and plastic. Plastic stents are cheap and easy to replace and should be used if any surgical intervention (e.g. Whipples) is planned. However, they are prone to displacement and blockage. Metal stents are much more expensive and may compromise a surgical resection. However, they are far less prone to displacement and to a lesser extent blockage than their plastic counterparts.

If malignancy is in bile duct/ pancreatic head and stenting has been attempted and has failed, then an alternative strategy is to drain the biliary system percutaneously via a transhepatic route. It may also be possible to insert a stent in this way. One of the main problems with temporary PTC’s is their propensity to displacement, which may result in a bile leak.

In patients who have a bile duct injury surgery will be required to repair the defect. If the bile duct has been inadvertently excised then a hepatico-jejunostomy will need to be created (difficult!)

If gallstones are the culprit, then these may be removed by ERCP and a cholecystectomy performed. Where there is doubt about the efficacy of the ERCP an operative cholangiogram should be performed and bile duct exploration undertaken where stones remain. When the bile duct has been formally opened the options are between closure over a T tube, a choledochoduodenostomy or choledochojejunostomy.

Patients with cholangitis should receive high dose broad spectrum antibiotics via the intravenous route. Biliary decompression should follow soon afterwards, instrumenting the bile duct of these patients will often provoke a septic episode (but should be done anyway).

161
Q

A 48 year old lady is admitted with attacks of biliary colic and investigations show gallstones. A laparoscopic cholecystectomy is performed. The operation is technically challenging due to a large stone impacted in Hartmans pouch. Following the operation she fails to settle and becomes jaundiced and has bile draining into a drain placed at the surgical site.

A. ERCP
B. MRCP
C. Percutaneous transhepatic cholangiogram
D. Laparotomy
E. Laparotomy and formation of hepatico-jejunostomy
F. Laparoscopic biliary bypass
G. CT scan

A

ERCP

In this scenario it must be assumed that the bile duct has been damaged. In most cases an ERCP is the most appropriate investigation. This can also allow the passage of a stent if this is deemed to be safe and sensible.

Gallstones Typically history of biliary colic or episodes of chlolecystitis. Obstructive type history and test results. Usually small calibre gallstones which can pass through the cystic duct. In Mirizzi syndrome the stone may compress the bile duct directly- one of the rare times that cholecystitis may present with jaundice
Cholangitis Usually obstructive and will have Charcots triad of symptoms (pain, fever, jaundice) Ascending infection of the bile ducts usually by E. coli and by definition occurring in a pool of stagnant bile.
Pancreatic cancer Typically painless jaundice with palpable gallbladder (Courvoisier’s Law) Direct occlusion of distal bile duct or pancreatic duct by tumour. Sometimes nodal disease at the portal hepatis may be the culprit in which case the bile duct may be of normal calibre.
TPN associated jaundice Usually follows long term use and is usually painless with non obstructive features Often due to hepatic dysfunction and fatty liver which may occur with long term TPN usage.
Bile duct injury Depending upon the type of injury may be of sudden or gradual onset and is usually of obstructive type Often due to a difficult cholecystectomy when anatomy in Calots triangle is not appreciated. In the worst scenario the bile duct is excised and jaundice offers rapidly post operatively. More insidious is that of bile duct stenosis which may be caused by clips or diathermy injury.
Cholangiocarcinoma Gradual onset obstructive pattern Direct occlusion by disease and also extrinsic compression by nodal disease at the porta hepatis.
Septic surgical patient Usually hepatic features Combination of impaired biliary excretion and drugs such as ciprofloxacin which may cause cholestasis.
Metastatic disease Mixed hepatic and post hepatic Combination of liver synthetic failure (late) and extrinsic compression by nodal disease and anatomical compression of intra hepatic structures (earlier)

Diagnosis
An ultrasound of the liver and biliary tree is the most commonly used first line test. This will establish bile duct calibre, often ascertain the presence of gallstones, may visualise pancreatic masses and other lesions. The most important clinical question is essentially the extent of biliary dilatation and its distribution.

Where pancreatic neoplasia is suspected, the next test should be a pancreatic protocol CT scan. With liver tumours and cholangiocarcinoma an MRI/ MRCP is often the preferred option. PET scans may be used to stage a number of malignancies but do not routinely form part of first line testing.

Where MRCP fails to give adequate information an ERCP may be necessary. In many cases this may form part of patient management. It is however, invasive and certainly not without risk and highly operator dependent.

Management
Clearly this will depend to an extent upon the underlying cause but relief of jaundice is important, even if surgery forms part of the planned treatment. Patients with unrelieved jaundice have a much higher incidence of septic complications, bleeding and death.

Screen for and address any clotting irregularities

In patients with malignancy a stent will need to be inserted. These come in two main types; metal and plastic. Plastic stents are cheap and easy to replace and should be used if any surgical intervention (e.g. Whipples) is planned. However, they are prone to displacement and blockage. Metal stents are much more expensive and may compromise a surgical resection. However, they are far less prone to displacement and to a lesser extent blockage than their plastic counterparts.

If malignancy is in bile duct/ pancreatic head and stenting has been attempted and has failed, then an alternative strategy is to drain the biliary system percutaneously via a transhepatic route. It may also be possible to insert a stent in this way. One of the main problems with temporary PTC’s is their propensity to displacement, which may result in a bile leak.

In patients who have a bile duct injury surgery will be required to repair the defect. If the bile duct has been inadvertently excised then a hepatico-jejunostomy will need to be created (difficult!)

If gallstones are the culprit, then these may be removed by ERCP and a cholecystectomy performed. Where there is doubt about the efficacy of the ERCP an operative cholangiogram should be performed and bile duct exploration undertaken where stones remain. When the bile duct has been formally opened the options are between closure over a T tube, a choledochoduodenostomy or choledochojejunostomy.

Patients with cholangitis should receive high dose broad spectrum antibiotics via the intravenous route. Biliary decompression should follow soon afterwards, instrumenting the bile duct of these patients will often provoke a septic episode (but should be done anyway).

162
Q

A 34 year old lady is admitted with jaundice and undergoes an ERCP. The procedure is technically difficult and she is returned to the ward still jaundiced. Unfortunately she now has severe generalised abdominal pain.

A. ERCP
B. MRCP
C. Percutaneous transhepatic cholangiogram
D. Laparotomy
E. Laparotomy and formation of hepatico-jejunostomy
F. Laparoscopic biliary bypass
G. CT scan

A

CT scan

There are two main differential diagnoses here. One is pancreatitis, repeated trauma to the ampulla and duct (if partially cannulated) is a major risk factor for pancreatitis. The second is the possibility that the duodenum has been perforated. ERCP is performed using a side viewing endoscope, the manipulation of which can be technically challenging for the inexperienced operator in a patient with abnormal anatomy. A CT scan is the best investigation to distinguish between these two differential diagnoses.

Gallstones Typically history of biliary colic or episodes of chlolecystitis. Obstructive type history and test results. Usually small calibre gallstones which can pass through the cystic duct. In Mirizzi syndrome the stone may compress the bile duct directly- one of the rare times that cholecystitis may present with jaundice
Cholangitis Usually obstructive and will have Charcots triad of symptoms (pain, fever, jaundice) Ascending infection of the bile ducts usually by E. coli and by definition occurring in a pool of stagnant bile.
Pancreatic cancer Typically painless jaundice with palpable gallbladder (Courvoisier’s Law) Direct occlusion of distal bile duct or pancreatic duct by tumour. Sometimes nodal disease at the portal hepatis may be the culprit in which case the bile duct may be of normal calibre.
TPN associated jaundice Usually follows long term use and is usually painless with non obstructive features Often due to hepatic dysfunction and fatty liver which may occur with long term TPN usage.
Bile duct injury Depending upon the type of injury may be of sudden or gradual onset and is usually of obstructive type Often due to a difficult cholecystectomy when anatomy in Calots triangle is not appreciated. In the worst scenario the bile duct is excised and jaundice offers rapidly post operatively. More insidious is that of bile duct stenosis which may be caused by clips or diathermy injury.
Cholangiocarcinoma Gradual onset obstructive pattern Direct occlusion by disease and also extrinsic compression by nodal disease at the porta hepatis.
Septic surgical patient Usually hepatic features Combination of impaired biliary excretion and drugs such as ciprofloxacin which may cause cholestasis.
Metastatic disease Mixed hepatic and post hepatic Combination of liver synthetic failure (late) and extrinsic compression by nodal disease and anatomical compression of intra hepatic structures (earlier)

Diagnosis
An ultrasound of the liver and biliary tree is the most commonly used first line test. This will establish bile duct calibre, often ascertain the presence of gallstones, may visualise pancreatic masses and other lesions. The most important clinical question is essentially the extent of biliary dilatation and its distribution.

Where pancreatic neoplasia is suspected, the next test should be a pancreatic protocol CT scan. With liver tumours and cholangiocarcinoma an MRI/ MRCP is often the preferred option. PET scans may be used to stage a number of malignancies but do not routinely form part of first line testing.

Where MRCP fails to give adequate information an ERCP may be necessary. In many cases this may form part of patient management. It is however, invasive and certainly not without risk and highly operator dependent.

Management
Clearly this will depend to an extent upon the underlying cause but relief of jaundice is important, even if surgery forms part of the planned treatment. Patients with unrelieved jaundice have a much higher incidence of septic complications, bleeding and death.

Screen for and address any clotting irregularities

In patients with malignancy a stent will need to be inserted. These come in two main types; metal and plastic. Plastic stents are cheap and easy to replace and should be used if any surgical intervention (e.g. Whipples) is planned. However, they are prone to displacement and blockage. Metal stents are much more expensive and may compromise a surgical resection. However, they are far less prone to displacement and to a lesser extent blockage than their plastic counterparts.

If malignancy is in bile duct/ pancreatic head and stenting has been attempted and has failed, then an alternative strategy is to drain the biliary system percutaneously via a transhepatic route. It may also be possible to insert a stent in this way. One of the main problems with temporary PTC’s is their propensity to displacement, which may result in a bile leak.

In patients who have a bile duct injury surgery will be required to repair the defect. If the bile duct has been inadvertently excised then a hepatico-jejunostomy will need to be created (difficult!)

If gallstones are the culprit, then these may be removed by ERCP and a cholecystectomy performed. Where there is doubt about the efficacy of the ERCP an operative cholangiogram should be performed and bile duct exploration undertaken where stones remain. When the bile duct has been formally opened the options are between closure over a T tube, a choledochoduodenostomy or choledochojejunostomy.

Patients with cholangitis should receive high dose broad spectrum antibiotics via the intravenous route. Biliary decompression should follow soon afterwards, instrumenting the bile duct of these patients will often provoke a septic episode (but should be done anyway).

163
Q

A 6 day old child is suspected of having a malrotation and requires urgent abdominal exploration. What is the most appropriate surgical approach?

Midline abdominal incision
Paramedian incision
Transverse supra umbilical abdominal incision
Transverse infra umbilical abdominal incision
Battle incision
A

In young children, laparotomy is performed via transverse supra umbilical incision. Access via midline incisions is very poor and they should not be used.

Midline incision
Commonest approach to the abdomen
Structures divided: linea alba, transversalis fascia, extraperitoneal fat, peritoneum (avoid falciform ligament above the umbilicus)
Bladder can be accessed via an extraperitoneal approach through the space of Retzius

Paramedian incision
Parallel to the midline (about 3-4cm)
Structures divided/retracted: anterior rectus sheath, rectus (retracted), posterior rectus sheath, transversalis fascia, extraperitoneal fat, peritoneum
Incision is closed in layers

Battle
Similar location to paramedian but rectus displaced medially (and thus denervated)
Now seldom used

Kocher’s Incision under right subcostal margin e.g. Cholecystectomy (open)

Lanz Incision in right iliac fossa e.g. Appendicectomy

Gridiron Oblique incision centered over McBurneys point- usually appendicectomy (less cosmetically acceptable than Lanz

Gable Rooftop incision

Pfannenstiel’s Transverse supra pubic, primarily used to access pelvic organs

McEvedy’s Groin incision e.g. Emergency repair strangulated femoral hernia

Rutherford Morrison Extraperitoneal approach to left or right lower quadrants. Gives excellent access to iliac vessels and is the approach of choice for first time renal transplantation.

164
Q

Which of the following statements in relation to fistula in ano is untrue?

High fistulae are safest treated with a seton insertion
Low fistulae may be laid open
They are typically probed with Lockhart Mummary probes
When discovered during incision and drainage of peri anal abscess; should always be probed to locate the internal opening
When complicating Crohns disease, may respond to infliximab
A

False : When discovered during incision and drainage of peri anal abscess; should always be probed to locate the internal opening

Probing fistulae during acute sepsis is associated with a high complication rate and should not be undertaken routinely.

Fistulas

A fistula is defined as an abnormal connection between two epithelial surfaces.
There are many types ranging from Branchial fistulae in the neck to entero-cutaneous fistulae abdominally.
In general surgical practice the abdominal cavity generates the majority and most of these arise from diverticular disease and Crohn’s.
As a general rule all fistulae will resolve spontaneously as long as there is no distal obstruction. This is particularly true of intestinal fistulae.

The four types of fistulae are:

Enterocutaneous
These link the intestine to the skin. They may be high (>500ml) or low output (<250ml) depending upon source. Duodenal /jejunal fistulae will tend to produce high volume, electrolyte rich secretions which can lead to severe excoriation of the skin. Colo-cutaneous fistulae will tend to leak faeculent material. Both fistulae may result from the spontaneous rupture of an abscess cavity onto the skin (such as following perianal abscess drainage) or may occur as a result of iatrogenic input. In some cases it may even be surgically desirable e.g. mucous fistula following sub total colectomy for colitis.

Suspect if there is excess fluid in the drain.

Enteroenteric or Enterocolic
This is a fistula that involves the large or small intestine. They may originate in a similar manner to enterocutaneous fistulae. A particular problem with this fistula type is that bacterial overgrowth may precipitate malabsorption syndromes. This may be particularly serious in inflammatory bowel disease.

Enterovaginal
Aetiology as above.

Enterovesicular
This type of fistula goes to the bladder. These fistulas may result in frequent urinary tract infections, or the passage of gas from the urethra during urination.

Management
Some rules relating to fistula management:
They will heal provided there is no underlying inflammatory bowel disease and no distal obstruction, so conservative measures may be the best option
Where there is skin involvement, protect the overlying skin, often using a well fitted stoma bag- skin damage is difficult to treat
A high output fistula may be rendered more easily managed by the use of octreotide, this will tend to reduce the volume of pancreatic secretions.
Nutritional complications are common especially with high fistula (e.g. high jejunal or duodenal) these may necessitate the use of TPN to provide nutritional support together with the concomitant use of octreotide to reduce volume and protect skin.
When managing perianal fistulae surgeons should avoid probing the fistula where acute inflammation is present, this almost always worsens outcomes.
When perianal fistulae occur secondary to Crohn’s disease the best management option is often to drain acute sepsis and maintain that drainage through the judicious use of setons whilst medical management is implemented.
Always attempt to delineate the fistula anatomy, for abscesses and fistulae that have an intra abdominal source the use of barium and CT studies should show a track. For perianal fistulae surgeons should recall Goodsall’s rule in relation to internal and external openings.

165
Q

A 23 year old woman with greenish nipple discharge on one occasion. Clinical examination of the breast is normal. Ultrasound report is U1.

A.	Prescribe danazol
B.	Microdochectomy
C.	Total duct excision
D.	Cytology of duct fluid
E.	Core biopsy
F.	Prescribe co-amoxiclav
G.	Reassure and discharge
H.	Mastectomy
A

Reassure and discharge

This is likely to be simple duct ectasia and U1 (normal USS) coupled with normal examination would favor discharge from clinic. Mammography is generally unhelpful in this age group

Causes of nipple discharge
Physiological	During breast feeding
Galactorrhoea	Commonest cause may be response to emotional events, drugs such as histamine receptor antagonists are also implicated
Hyperprolactinaemia	
Commonest type of pituitary tumour
Microadenomas <1cm in diameter
Macroadenomas >1cm in diameter
Pressure on optic chiasm may cause bitemporal hemianopia
Mammary duct ectasia	
Dilatation breast ducts.
Most common in menopausal women
Discharge typically thick and green in colour
Most common in smokers
Carcinoma	
Often blood stained
May be underlying mass or axillary lymphadenopathy
Intraductal papilloma	
Commoner in younger patients
May cause blood stained discharge
There is usually no palpable lump

Assessment of patients
Examine breast and determine whether there is mass lesion present
All mass lesions should undergo Triple assessment.

Reporting of investigations
Where a mass lesion is suspected or investigations are requested these are prefixed using a system that denotes the investigation type e.g. M for mammography, followed by a numerical code as shown below:

1	No abnormality
2	Abnormality with benign features
3	Indeterminate probably benign
4	Indeterminate probably malignant
5	Malignant

Management of non malignant nipple discharge
Exclude endocrine disease
Nipple cytology unhelpful
Smoking cessation advice for duct ectasia
For duct ectasia with severe symptoms, total duct excision may be warranted.

166
Q

A 43 year old woman has had recurrent episodes of periductal mastitis. She has received multiple courses of antibiotics and is troubled by persisting green nipple discharge. Clinical examination reveals green nipple discharge, but no discrete lump. Imaging with mammography and ultrasound is reassuring (U2, M2)

A.	Prescribe danazol
B.	Microdochectomy
C.	Total duct excision
D.	Cytology of duct fluid
E.	Core biopsy
F.	Prescribe co-amoxiclav
G.	Reassure and discharge
H.	Mastectomy
A

Total duct excision

This woman has troublesome duct ectasia and total duct excision is warranted.

Causes of nipple discharge
Physiological	During breast feeding
Galactorrhoea	Commonest cause may be response to emotional events, drugs such as histamine receptor antagonists are also implicated
Hyperprolactinaemia	
Commonest type of pituitary tumour
Microadenomas <1cm in diameter
Macroadenomas >1cm in diameter
Pressure on optic chiasm may cause bitemporal hemianopia
Mammary duct ectasia	
Dilatation breast ducts.
Most common in menopausal women
Discharge typically thick and green in colour
Most common in smokers
Carcinoma	
Often blood stained
May be underlying mass or axillary lymphadenopathy
Intraductal papilloma	
Commoner in younger patients
May cause blood stained discharge
There is usually no palpable lump

Assessment of patients
Examine breast and determine whether there is mass lesion present
All mass lesions should undergo Triple assessment.

Reporting of investigations
Where a mass lesion is suspected or investigations are requested these are prefixed using a system that denotes the investigation type e.g. M for mammography, followed by a numerical code as shown below:

1	No abnormality
2	Abnormality with benign features
3	Indeterminate probably benign
4	Indeterminate probably malignant
5	Malignant

Management of non malignant nipple discharge
Exclude endocrine disease
Nipple cytology unhelpful
Smoking cessation advice for duct ectasia
For duct ectasia with severe symptoms, total duct excision may be warranted.

167
Q

A 55 year old woman complains of nipple discharge. This was blood stained on one occasion. But not subsequently. Clinical examination shows clear fluid but no discrete lump. Imaging with ultrasound and mammography is normal.

A.	Prescribe danazol
B.	Microdochectomy
C.	Total duct excision
D.	Cytology of duct fluid
E.	Core biopsy
F.	Prescribe co-amoxiclav
G.	Reassure and discharge
H.	Mastectomy
A

Microdochectomy

Although this is likely to be benign disease, her age coupled with an episode of blood stained discharge would attract a recommendation for microdochectomy. She may have an intraductal papilloma. But the concern would be DCIS.

Causes of nipple discharge
Physiological	During breast feeding
Galactorrhoea	Commonest cause may be response to emotional events, drugs such as histamine receptor antagonists are also implicated
Hyperprolactinaemia	
Commonest type of pituitary tumour
Microadenomas <1cm in diameter
Macroadenomas >1cm in diameter
Pressure on optic chiasm may cause bitemporal hemianopia
Mammary duct ectasia	
Dilatation breast ducts.
Most common in menopausal women
Discharge typically thick and green in colour
Most common in smokers
Carcinoma	
Often blood stained
May be underlying mass or axillary lymphadenopathy
Intraductal papilloma	
Commoner in younger patients
May cause blood stained discharge
There is usually no palpable lump

Assessment of patients
Examine breast and determine whether there is mass lesion present
All mass lesions should undergo Triple assessment.

Reporting of investigations
Where a mass lesion is suspected or investigations are requested these are prefixed using a system that denotes the investigation type e.g. M for mammography, followed by a numerical code as shown below:

1	No abnormality
2	Abnormality with benign features
3	Indeterminate probably benign
4	Indeterminate probably malignant
5	Malignant

Management of non malignant nipple discharge
Exclude endocrine disease
Nipple cytology unhelpful
Smoking cessation advice for duct ectasia
For duct ectasia with severe symptoms, total duct excision may be warranted.

168
Q

A 32 year old male presents with a swollen right scrotum which has developed over 3 weeks after being kicked in the groin area. There is a non tense swelling of the right scrotum and the underlying testis cannot be easily palpated. A dipstick is positive for nitrates only.

A.	Haematocele
B.	Epididymal cyst
C.	Hydrocele
D.	Testicular torsion
E.	Orchitis
F.	Epididymo-orchitis
A

Hydrocele

This is a secondary hydrocele which occurs in patients aged 20-40 years. It develops rapidly and there may not be a tense swelling. The underlying testis is NOT palpated therefore indicating a hydrocele. Causes include trauma, infection and tumour.

Inguinal hernia If inguinoscrotal swelling; cannot “get above it” on examination
Cough impulse may be present
May be reducible
Testicular tumours Often discrete testicular nodule (may have associated hydrocele)
Symptoms of metastatic disease may be present
USS scrotum and serum AFP and β HCG required
Acute epididymo-orchitis Often history of dysuria and urethral discharge
Swelling may be tender and eased by elevating testis
Most cases due to Chlamydia
Infections with other gram negative organisms may be associated with underlying structural abnormality
Epididymal cysts Single or multiple cysts
May contain clear or opalescent fluid (spermatoceles)
Usually occur over 40 years of age
Painless
Lie above and behind testis
It is usually possible to “get above the lump” on examination
Hydrocele Non painful, soft fluctuant swelling
Often possible to “get above it” on examination
Usually contain clear fluid
Will often transilluminate
May be presenting feature of testicular cancer in young men
Testicular torsion Severe, sudden onset testicular pain
Risk factors include abnormal testicular lie
Typically affects adolescents and young males
On examination testis is tender and pain not eased by elevation
Urgent surgery is indicated, the contra lateral testis should also be fixed
Varicocele Varicosities of the pampiniform plexus
Typically occur on left (because testicular vein drains into renal vein)
May be presenting feature of renal cell carcinoma
Affected testis may be smaller and bilateral varicoceles may affect fertility

Management
Testicular malignancy is always treated with orchidectomy via an inguinal approach. This allows high ligation of the testicular vessels and avoids exposure of another lymphatic field to the tumour.
Torsion is commonest in young teenagers and the history in older children can be difficult to elicit. Intermittent torsion is a recognised problem. The treatment is prompt surgical exploration and testicular fixation. This can be achieved using sutures or by placement of the testis in a Dartos pouch.
Varicoceles are usually managed conservatively. If there are concerns about testicular function of infertility then surgery or radiological management can be considered.
Epididymal cysts can be excised using a scrotal approach
Hydroceles are managed differently in children where the underlying pathology is a patent processus vaginalis and therefore an inguinal approach is used in children so that the processus can be ligated. In adults a scrotal approach is preferred and the hydrocele sac excised or plicated.

169
Q

A 40 year old male presents with a non painful, bilateral scrotal swellings over 3 years. The testis is felt separately and the swelling transilluminates.

A.	Haematocele
B.	Epididymal cyst
C.	Hydrocele
D.	Testicular torsion
E.	Orchitis
F.	Epididymo-orchitis
A

Epididymal cyst

The testis is palpated therefore this differentiates it from a hydrocele.

Inguinal hernia If inguinoscrotal swelling; cannot “get above it” on examination
Cough impulse may be present
May be reducible
Testicular tumours Often discrete testicular nodule (may have associated hydrocele)
Symptoms of metastatic disease may be present
USS scrotum and serum AFP and β HCG required
Acute epididymo-orchitis Often history of dysuria and urethral discharge
Swelling may be tender and eased by elevating testis
Most cases due to Chlamydia
Infections with other gram negative organisms may be associated with underlying structural abnormality
Epididymal cysts Single or multiple cysts
May contain clear or opalescent fluid (spermatoceles)
Usually occur over 40 years of age
Painless
Lie above and behind testis
It is usually possible to “get above the lump” on examination
Hydrocele Non painful, soft fluctuant swelling
Often possible to “get above it” on examination
Usually contain clear fluid
Will often transilluminate
May be presenting feature of testicular cancer in young men
Testicular torsion Severe, sudden onset testicular pain
Risk factors include abnormal testicular lie
Typically affects adolescents and young males
On examination testis is tender and pain not eased by elevation
Urgent surgery is indicated, the contra lateral testis should also be fixed
Varicocele Varicosities of the pampiniform plexus
Typically occur on left (because testicular vein drains into renal vein)
May be presenting feature of renal cell carcinoma
Affected testis may be smaller and bilateral varicoceles may affect fertility

Management
Testicular malignancy is always treated with orchidectomy via an inguinal approach. This allows high ligation of the testicular vessels and avoids exposure of another lymphatic field to the tumour.
Torsion is commonest in young teenagers and the history in older children can be difficult to elicit. Intermittent torsion is a recognised problem. The treatment is prompt surgical exploration and testicular fixation. This can be achieved using sutures or by placement of the testis in a Dartos pouch.
Varicoceles are usually managed conservatively. If there are concerns about testicular function of infertility then surgery or radiological management can be considered.
Epididymal cysts can be excised using a scrotal approach
Hydroceles are managed differently in children where the underlying pathology is a patent processus vaginalis and therefore an inguinal approach is used in children so that the processus can be ligated. In adults a scrotal approach is preferred and the hydrocele sac excised or plicated.

170
Q

A 32 year old male presents with a swollen, painful right scrotum after being kicked in the groin area 1 hour ago. There is a painful swelling of the right scrotum and the underlying testis cannot be easily palpated.

A.	Haematocele
B.	Epididymal cyst
C.	Hydrocele
D.	Testicular torsion
E.	Orchitis
F.	Epididymo-orchitis
A

Haematocele

Acute haematocele: tense, tender and non transilluminating mass post trauma. A chronic haematoma causes a blood clot to surround the testis. The blood clot hardens and contracts causing a hard mass which may be indistinguishable from a tumour. Therefore the testis will need surgical exploration.

Inguinal hernia If inguinoscrotal swelling; cannot “get above it” on examination
Cough impulse may be present
May be reducible
Testicular tumours Often discrete testicular nodule (may have associated hydrocele)
Symptoms of metastatic disease may be present
USS scrotum and serum AFP and β HCG required
Acute epididymo-orchitis Often history of dysuria and urethral discharge
Swelling may be tender and eased by elevating testis
Most cases due to Chlamydia
Infections with other gram negative organisms may be associated with underlying structural abnormality
Epididymal cysts Single or multiple cysts
May contain clear or opalescent fluid (spermatoceles)
Usually occur over 40 years of age
Painless
Lie above and behind testis
It is usually possible to “get above the lump” on examination
Hydrocele Non painful, soft fluctuant swelling
Often possible to “get above it” on examination
Usually contain clear fluid
Will often transilluminate
May be presenting feature of testicular cancer in young men
Testicular torsion Severe, sudden onset testicular pain
Risk factors include abnormal testicular lie
Typically affects adolescents and young males
On examination testis is tender and pain not eased by elevation
Urgent surgery is indicated, the contra lateral testis should also be fixed
Varicocele Varicosities of the pampiniform plexus
Typically occur on left (because testicular vein drains into renal vein)
May be presenting feature of renal cell carcinoma
Affected testis may be smaller and bilateral varicoceles may affect fertility

Management
Testicular malignancy is always treated with orchidectomy via an inguinal approach. This allows high ligation of the testicular vessels and avoids exposure of another lymphatic field to the tumour.
Torsion is commonest in young teenagers and the history in older children can be difficult to elicit. Intermittent torsion is a recognised problem. The treatment is prompt surgical exploration and testicular fixation. This can be achieved using sutures or by placement of the testis in a Dartos pouch.
Varicoceles are usually managed conservatively. If there are concerns about testicular function of infertility then surgery or radiological management can be considered.
Epididymal cysts can be excised using a scrotal approach
Hydroceles are managed differently in children where the underlying pathology is a patent processus vaginalis and therefore an inguinal approach is used in children so that the processus can be ligated. In adults a scrotal approach is preferred and the hydrocele sac excised or plicated.

171
Q

An 85 year old man is diagnosed as having prostate cancer and is considered suitable for hormonal ablation. However, he does not want the repeated injections of GnRH analogues.

A.	Fine needle aspiration cytology
B.	Excision biopsy
C.	Orchidectomy via an inguinal approach
D.	Lords procedure
E.	Orchidectomy via a scrotal approach
F.	Division of patent processus vaginalis via an inguinal approach
G.	Division of patent processus vaginalis via a scrotal approach
H.	Fowler Stephens procedure
I.	Immediate scrotal exploration
J.	Conservative management
A

Orchidectomy via a scrotal approach

At one time bilateral orchidectomy was performed routinely when prostate cancer was diagnosed (sometimes under the same anaesthetic). A combination of modern agents (GnRH analogues) and better consent processes have made this almost obsolete. Where required a scrotal approach should be used.

Testicular cancer is the most common malignancy in men aged 20-30 years. Around 95% of cases of testicular cancer are germ-cell tumours. Germ cell tumours may essentially be divided into:

Seminoma
Commonest subtype (50%)
Average age at diagnosis = 40
Even advanced disease associated with 5 year survival of 73%
AFP usually normal
HCG elevated in 10% seminomas
Lactate dehydrogenase; elevated in 10-20% seminomas (but also in many other conditions)
Sheet like lobular patterns of cells with substantial fibrous component. Fibrous septa contain lymphocytic inclusions and granulomas may be seen.

Non seminomatous germ cell tumours (42%)
(Teratoma, Yolk sac tumour, Choriocarcinoma, Mixed germ cell tumours (10%))
Younger age at presentation =20-30 years
Advanced disease carries worse prognosis (48% at 5 years)
Retroperitoneal lymph node dissection may be needed for residual disease after chemotherapy
AFP elevated in up to 70% of cases
HCG elevated in up to 40% of cases
Other markers rarely helpful
Heterogenous texture with occasional ectopic tissue such as hair

Risk factors for testicular cancer
Cryptorchidism
Infertility
Family history
Klinefelter's syndrome
Mumps orchitis

Features
A painless lump is the most common presenting symptom
Pain may also be present in a minority of men
Other possible features include hydrocele, gynaecomastia

Diagnosis
Ultrasound is first-line
CT scanning of the chest/ abdomen and pelvis is used for staging
Tumour markers (see above) should be measured

Management
Orchidectomy (Inguinal approach)
Chemotherapy and radiotherapy may be given depending on staging
Abdominal lesions >1cm following chemotherapy may require retroperitoneal lymph node dissection.

Prognosis is generally excellent
5 year survival for seminomas is around 95% if Stage I
5 year survival for teratomas is around 85% if Stage I

Benign disease

Epididymo-orchitis
Acute epididymitis is an acute inflammation of the epididymis, often involving the testis and usually caused by bacterial infection.
Infection spreads from the urethra or bladder. In men <35 years, gonorrhoea or chlamydia are the usual infections.
Amiodarone is a recognised non infective cause of epididymitis, which resolves on stopping the drug.
Tenderness is usually confined to the epididymis, which may facilitate differentiating it from torsion where pain usually affects the entire testis.

Testicular torsion
Twist of the spermatic cord resulting in testicular ischaemia and necrosis.
Most common in males aged between 10 and 30 (peak incidence 13-15 years)
Pain is usually severe and of sudden onset.
Cremasteric reflex is lost and elevation of the testis does not ease the pain.
Treatment is with surgical exploration. If a torted testis is identified then both testis should be fixed as the condition of bell clapper testis is often bilateral.

Hydrocele
Presents as a mass that transilluminates, usually possible to “get above” it on examination.
In younger men it should be investigated with USS to exclude tumour.
In children it may occur as a result of a patent processus vaginalis.
Treatment in adults is with a Lords or Jabouley procedure.
Treatment in children is with trans inguinal ligation of PPV.

172
Q

A 33 year old man presents with a painless lump in his left testes. USS and blood tests are suspicious for teratoma.

A.	Fine needle aspiration cytology
B.	Excision biopsy
C.	Orchidectomy via an inguinal approach
D.	Lords procedure
E.	Orchidectomy via a scrotal approach
F.	Division of patent processus vaginalis via an inguinal approach
G.	Division of patent processus vaginalis via a scrotal approach
H.	Fowler Stephens procedure
I.	Immediate scrotal exploration
J.	Conservative management
A

Orchidectomy via an inguinal approach

Oncological orchidectomy is routinely performed via an inguinal approach to avoid contamination of another lymphatic field.

Testicular cancer is the most common malignancy in men aged 20-30 years. Around 95% of cases of testicular cancer are germ-cell tumours. Germ cell tumours may essentially be divided into:

Seminoma
Commonest subtype (50%)
Average age at diagnosis = 40
Even advanced disease associated with 5 year survival of 73%
AFP usually normal
HCG elevated in 10% seminomas
Lactate dehydrogenase; elevated in 10-20% seminomas (but also in many other conditions)
Sheet like lobular patterns of cells with substantial fibrous component. Fibrous septa contain lymphocytic inclusions and granulomas may be seen.

Non seminomatous germ cell tumours (42%)
(Teratoma, Yolk sac tumour, Choriocarcinoma, Mixed germ cell tumours (10%))
Younger age at presentation =20-30 years
Advanced disease carries worse prognosis (48% at 5 years)
Retroperitoneal lymph node dissection may be needed for residual disease after chemotherapy
AFP elevated in up to 70% of cases
HCG elevated in up to 40% of cases
Other markers rarely helpful
Heterogenous texture with occasional ectopic tissue such as hair

Risk factors for testicular cancer
Cryptorchidism
Infertility
Family history
Klinefelter's syndrome
Mumps orchitis

Features
A painless lump is the most common presenting symptom
Pain may also be present in a minority of men
Other possible features include hydrocele, gynaecomastia

Diagnosis
Ultrasound is first-line
CT scanning of the chest/ abdomen and pelvis is used for staging
Tumour markers (see above) should be measured

Management
Orchidectomy (Inguinal approach)
Chemotherapy and radiotherapy may be given depending on staging
Abdominal lesions >1cm following chemotherapy may require retroperitoneal lymph node dissection.

Prognosis is generally excellent
5 year survival for seminomas is around 95% if Stage I
5 year survival for teratomas is around 85% if Stage I

Benign disease

Epididymo-orchitis
Acute epididymitis is an acute inflammation of the epididymis, often involving the testis and usually caused by bacterial infection.
Infection spreads from the urethra or bladder. In men <35 years, gonorrhoea or chlamydia are the usual infections.
Amiodarone is a recognised non infective cause of epididymitis, which resolves on stopping the drug.
Tenderness is usually confined to the epididymis, which may facilitate differentiating it from torsion where pain usually affects the entire testis.

Testicular torsion
Twist of the spermatic cord resulting in testicular ischaemia and necrosis.
Most common in males aged between 10 and 30 (peak incidence 13-15 years)
Pain is usually severe and of sudden onset.
Cremasteric reflex is lost and elevation of the testis does not ease the pain.
Treatment is with surgical exploration. If a torted testis is identified then both testis should be fixed as the condition of bell clapper testis is often bilateral.

Hydrocele
Presents as a mass that transilluminates, usually possible to “get above” it on examination.
In younger men it should be investigated with USS to exclude tumour.
In children it may occur as a result of a patent processus vaginalis.
Treatment in adults is with a Lords or Jabouley procedure.
Treatment in children is with trans inguinal ligation of PPV.

173
Q

A 4 year old boy is brought to the clinic by his mother. He has a swelling in his right hemiscrotum. On examination is transilluminates brilliantly.

A.	Fine needle aspiration cytology
B.	Excision biopsy
C.	Orchidectomy via an inguinal approach
D.	Lords procedure
E.	Orchidectomy via a scrotal approach
F.	Division of patent processus vaginalis via an inguinal approach
G.	Division of patent processus vaginalis via a scrotal approach
H.	Fowler Stephens procedure
I.	Immediate scrotal exploration
J.	Conservative management
A

Division of patent processus vaginalis via an inguinal approach

Ligation of the patent processus vaginalis is performed via an inguinal approach. There is no indication for scrotal surgery for hydrocele in young children.

Testicular cancer is the most common malignancy in men aged 20-30 years. Around 95% of cases of testicular cancer are germ-cell tumours. Germ cell tumours may essentially be divided into:

Seminoma
Commonest subtype (50%)
Average age at diagnosis = 40
Even advanced disease associated with 5 year survival of 73%
AFP usually normal
HCG elevated in 10% seminomas
Lactate dehydrogenase; elevated in 10-20% seminomas (but also in many other conditions)
Sheet like lobular patterns of cells with substantial fibrous component. Fibrous septa contain lymphocytic inclusions and granulomas may be seen.

Non seminomatous germ cell tumours (42%)
(Teratoma, Yolk sac tumour, Choriocarcinoma, Mixed germ cell tumours (10%))
Younger age at presentation =20-30 years
Advanced disease carries worse prognosis (48% at 5 years)
Retroperitoneal lymph node dissection may be needed for residual disease after chemotherapy
AFP elevated in up to 70% of cases
HCG elevated in up to 40% of cases
Other markers rarely helpful
Heterogenous texture with occasional ectopic tissue such as hair

Risk factors for testicular cancer
Cryptorchidism
Infertility
Family history
Klinefelter's syndrome
Mumps orchitis

Features
A painless lump is the most common presenting symptom
Pain may also be present in a minority of men
Other possible features include hydrocele, gynaecomastia

Diagnosis
Ultrasound is first-line
CT scanning of the chest/ abdomen and pelvis is used for staging
Tumour markers (see above) should be measured

Management
Orchidectomy (Inguinal approach)
Chemotherapy and radiotherapy may be given depending on staging
Abdominal lesions >1cm following chemotherapy may require retroperitoneal lymph node dissection.

Prognosis is generally excellent
5 year survival for seminomas is around 95% if Stage I
5 year survival for teratomas is around 85% if Stage I

Benign disease

Epididymo-orchitis
Acute epididymitis is an acute inflammation of the epididymis, often involving the testis and usually caused by bacterial infection.
Infection spreads from the urethra or bladder. In men <35 years, gonorrhoea or chlamydia are the usual infections.
Amiodarone is a recognised non infective cause of epididymitis, which resolves on stopping the drug.
Tenderness is usually confined to the epididymis, which may facilitate differentiating it from torsion where pain usually affects the entire testis.

Testicular torsion
Twist of the spermatic cord resulting in testicular ischaemia and necrosis.
Most common in males aged between 10 and 30 (peak incidence 13-15 years)
Pain is usually severe and of sudden onset.
Cremasteric reflex is lost and elevation of the testis does not ease the pain.
Treatment is with surgical exploration. If a torted testis is identified then both testis should be fixed as the condition of bell clapper testis is often bilateral.

Hydrocele
Presents as a mass that transilluminates, usually possible to “get above” it on examination.
In younger men it should be investigated with USS to exclude tumour.
In children it may occur as a result of a patent processus vaginalis.
Treatment in adults is with a Lords or Jabouley procedure.
Treatment in children is with trans inguinal ligation of PPV.

174
Q

An 18 year old man is involved in a road traffic accident. A CT scan shows disruption of the splenic hilum and a moderate sized perisplenic haematoma.

A.	Splenectomy
B.	Angiography
C.	CT Scan
D.	Admit for bed rest and observation
E.	Ultrasound scan
F.	Splenic conservation
G.	MRI of the abdomen
A

Splenectomy

Hilar injuries usually mandate splenectomy. The main risk with conservative management here is that he will rebleed and with hilar injuries this can be dramatic.

Splenic trauma

The spleen is one of the more commonly injured intra abdominal organs
In most cases the spleen can be conserved. The management is dictated by the associated injuries, haemodynamic status and extent of direct splenic injury.

Management of splenic trauma
Conservative	Small subcapsular haematoma
Minimal intra abdominal blood
No hilar disruption
Laparotomy with conservation	Increased amounts of intraabdominal blood
Moderate haemodynamic compromise
Tears or lacerations affecting <50%
Resection	Hilar injuries
Major haemorrhage
Major associated injuries

Splenectomy
Technique
Trauma
GA
Long midline incision
If time permits insert a self retaining retractor (e.g. Balfour/ omnitract)
Large amount of free blood is usually present. Pack all 4 quadrants of the abdomen. Allow the anaesthetist to ‘catch up’
Remove the packs and assess the viability of the spleen. Hilar injuries and extensive parenchymal lacerations will usually require splenectomy.
Divide the short gastric vessels and ligate them.
Clamp the splenic artery and vein. Two clamps on the patient side are better and allow for double ligation and serve as a safety net if your assistant does not release the clamp smoothly.
Be careful not to damage the tail of the pancreas, if you do then this will need to be formally removed and the pancreatic duct closed.
Wash out the abdomen and place a tube drain to the splenic bed.
Some surgeons implant a portion of spleen into the omentum, whether you decide to do this is a matter of personal choice.
Post operatively the patient will require prophylactic penicillin V and pneumococcal vaccine.

Elective
Elective splenectomy is a very different operation from that performed in the emergency setting. The spleen is often large (sometimes massive). Most cases can be performed laparoscopically. The spleen will often be macerated inside a specimen bag to facilitate extraction.

Complications
Haemorrhage (may be early and either from short gastrics or splenic hilar vessels
Pancreatic fistula (from iatrogenic damage to pancreatic tail)
Thrombocytosis: prophylactic aspirin
Encapsulated bacteria infection e.g. Strep. pneumoniae, Haemophilus influenzae and Neisseria meningitidis

175
Q

A 42 year old motorcyclist is involved in a road traffic accident. A FAST scan in the emergency department shows free intrabdominal fluid and a laparotomy is performed. At operation there is evidence of small liver laceration that has stopped bleeding and a tear to the inferior pole of the spleen.

A.	Splenectomy
B.	Angiography
C.	CT Scan
D.	Admit for bed rest and observation
E.	Ultrasound scan
F.	Splenic conservation
G.	MRI of the abdomen
A

Splenic conservation

As minimum damage, attempt conservation

Splenic trauma

The spleen is one of the more commonly injured intra abdominal organs
In most cases the spleen can be conserved. The management is dictated by the associated injuries, haemodynamic status and extent of direct splenic injury.

Management of splenic trauma
Conservative	Small subcapsular haematoma
Minimal intra abdominal blood
No hilar disruption
Laparotomy with conservation	Increased amounts of intraabdominal blood
Moderate haemodynamic compromise
Tears or lacerations affecting <50%
Resection	Hilar injuries
Major haemorrhage
Major associated injuries

Splenectomy
Technique
Trauma
GA
Long midline incision
If time permits insert a self retaining retractor (e.g. Balfour/ omnitract)
Large amount of free blood is usually present. Pack all 4 quadrants of the abdomen. Allow the anaesthetist to ‘catch up’
Remove the packs and assess the viability of the spleen. Hilar injuries and extensive parenchymal lacerations will usually require splenectomy.
Divide the short gastric vessels and ligate them.
Clamp the splenic artery and vein. Two clamps on the patient side are better and allow for double ligation and serve as a safety net if your assistant does not release the clamp smoothly.
Be careful not to damage the tail of the pancreas, if you do then this will need to be formally removed and the pancreatic duct closed.
Wash out the abdomen and place a tube drain to the splenic bed.
Some surgeons implant a portion of spleen into the omentum, whether you decide to do this is a matter of personal choice.
Post operatively the patient will require prophylactic penicillin V and pneumococcal vaccine.

Elective
Elective splenectomy is a very different operation from that performed in the emergency setting. The spleen is often large (sometimes massive). Most cases can be performed laparoscopically. The spleen will often be macerated inside a specimen bag to facilitate extraction.

Complications
Haemorrhage (may be early and either from short gastrics or splenic hilar vessels
Pancreatic fistula (from iatrogenic damage to pancreatic tail)
Thrombocytosis: prophylactic aspirin
Encapsulated bacteria infection e.g. Strep. pneumoniae, Haemophilus influenzae and Neisseria meningitidis

176
Q

A 7 year old boy falls off a wall the distance is 7 feet. He lands on his left side and there is left flank bruising. There is no haematuria. He is otherwise stable and haemoglobin is within normal limits.

A.	Splenectomy
B.	Angiography
C.	CT Scan
D.	Admit for bed rest and observation
E.	Ultrasound scan
F.	Splenic conservation
G.	MRI of the abdomen
A

Ultrasound scan

This will demonstrate any overt splenic injury. A CT scan carries a significant dose of radiation. In the absence of haemodynamic instability or other major associated injuries the use of USS to exclude intraabdominal free fluid (blood) would seem safe when coupled with active observation. An USS will also show splenic haematomas.

Splenic trauma

The spleen is one of the more commonly injured intra abdominal organs
In most cases the spleen can be conserved. The management is dictated by the associated injuries, haemodynamic status and extent of direct splenic injury.

Management of splenic trauma
Conservative	Small subcapsular haematoma
Minimal intra abdominal blood
No hilar disruption
Laparotomy with conservation	Increased amounts of intraabdominal blood
Moderate haemodynamic compromise
Tears or lacerations affecting <50%
Resection	Hilar injuries
Major haemorrhage
Major associated injuries

Splenectomy
Technique
Trauma
GA
Long midline incision
If time permits insert a self retaining retractor (e.g. Balfour/ omnitract)
Large amount of free blood is usually present. Pack all 4 quadrants of the abdomen. Allow the anaesthetist to ‘catch up’
Remove the packs and assess the viability of the spleen. Hilar injuries and extensive parenchymal lacerations will usually require splenectomy.
Divide the short gastric vessels and ligate them.
Clamp the splenic artery and vein. Two clamps on the patient side are better and allow for double ligation and serve as a safety net if your assistant does not release the clamp smoothly.
Be careful not to damage the tail of the pancreas, if you do then this will need to be formally removed and the pancreatic duct closed.
Wash out the abdomen and place a tube drain to the splenic bed.
Some surgeons implant a portion of spleen into the omentum, whether you decide to do this is a matter of personal choice.
Post operatively the patient will require prophylactic penicillin V and pneumococcal vaccine.

Elective
Elective splenectomy is a very different operation from that performed in the emergency setting. The spleen is often large (sometimes massive). Most cases can be performed laparoscopically. The spleen will often be macerated inside a specimen bag to facilitate extraction.

Complications
Haemorrhage (may be early and either from short gastrics or splenic hilar vessels
Pancreatic fistula (from iatrogenic damage to pancreatic tail)
Thrombocytosis: prophylactic aspirin
Encapsulated bacteria infection e.g. Strep. pneumoniae, Haemophilus influenzae and Neisseria meningitidis

177
Q

A 63 year old man undergoes an upper GI endoscopy and adrenaline injection for a large actively bleeding duodenal ulcer. He remains stable for 6 hours and the nurses then call because he has passed 400ml malaena and has become tachycardic (pulse rate 120) and hypotensive (Bp 80/40). What is the best option?

Reassure that blood trapped in the upper portion of the gastrointestinal system will pass and that this episode will resolve with phosphate enema
Perform a repeat upper GI endoscopy
Perform a laparotomy and under-running of the ulcer
Administer tranexamic acid and intravenous proton pump inhibitors
Insert a Minnesota tube
A

Perform a laparotomy and under-running of the ulcer

The decision as to how best to manage patients with re-bleeding is difficult. Whilst it is tempting to offer repeat endoscopy, this intervention is best used on those with small ulcers. Large posteriorly sited duodenal ulcers are at high risk for re-bleeding and the timeframe of this event suggests that primary endoscopic haemostasis was inadequate. Surgery thus represents the safest way forward.

Upper gastrointestinal bleeding

Patients may present with the following:
Haematemesis and/ or malaena
Epigastric discomfort
Sudden collapse

The extent to which these will occur will depend upon the source. Mortality is higher in patients presenting with haematemesis than malaena alone.

Oesophageal bleeding
Cause Presenting features
Oesophagitis Small volume of fresh blood, often streaking vomit. Malaena rare. Often ceases spontaneously. Usually history of antecedent GORD type symptoms.
Cancer Usually small volume of blood, except as pre terminal event with erosion of major vessels. Often associated symptoms of dysphagia and constitutional symptoms such as weight loss. May be recurrent until malignancy managed.
Mallory Weiss Tear Typically brisk small to moderate volume of bright red blood following bout of repeated vomiting. Malaena rare. Usually ceases spontaneously.
Varices Usually large volume of fresh blood. Swallowed blood may cause malaena. Often associated with haemodynamic compromise. May stop spontaneously but re-bleeds are common until appropriately managed.

Gastric Bleeding
Cause Presenting features
Gastric cancer May be frank haematemesis or altered blood mixed with vomit. Usually prodromal features of dyspepsia and may have constitutional symptoms. Amount of bleeding variable but erosion of major vessel may produce considerable haemorrhage.
Dieulafoy Lesion Often no prodromal features prior to haematemesis and malaena, but this arteriovenous malformation may produce quite considerable haemorrhage and may be difficult to detect endoscopically.
Diffuse erosive gastritis Usually haematemesis and epigastric discomfort. Usually there is an underlying cause such as recent NSAID usage. Large volume haemorrhage may occur with considerable haemodynamic compromise.
Gastric ulcer Small low volume bleeds more common so would tend to present as iron deficiency anaemia. Erosion into a significant vessel may produce considerable haemorrhage and haematemesis.

Duodenum
Most common cause of major haemorrhage is a posteriorly sited duodenal ulcer. However, ulcers at any site in the duodenum may present with haematemesis, malaena and epigastric discomfort. The pain of duodenal ulcer is slightly different to that of gastric ulcers and often occurs several hours after eating. Peri ampullary tumours may bleed but these are rare. In patients with previous abdominal aortic aneurysm surgery aorto-enteric fistulation remains a rare but important cause of major haemorrhage associated with high mortality.

Management
Admission to hospital careful monitoring, cross match blood, check FBC, LFTs, U+E and Clotting (as a minimum)
Patients with on-going bleeding and haemodynamic instability are likely to require O negative blood pending cross matched blood
Early control of airway is vital (e.g. Drowsy patient with liver failure)
Patients with suspected varices should receive terlipressin prior to endoscopy
Ideally all patients admitted with upper gastrointestinal haemorrhage should undergo Upper GI endoscopy within 24 hours of admission. In those who are unstable this should occur immediately after resuscitation or in tandem with it. The endoscopy department is a potentially dangerous place for unstable patients and it may be safer to perform the endoscopy in theatre with an anaesthetist present.
Varices should be banded or subjected to sclerotherapy. If this is not possible owing to active bleeding then a Sengaksten- Blakemore tube (or Minnesota tube) should be inserted. This should be done with care; gastric balloon should be inflated first and oesophageal balloon second. Remember the balloon will need deflating after 12 hours (ideally sooner) to prevent necrosis. Portal pressure should be lowered by combination of medical therapy +/- TIPSS.
Patients with erosive oesophagitis / gastritis should receive a proton pump inhibitor.
Mallory Weiss tears will typically resolve spontaneously
Identifiable bleeding points should receive combination therapy of injection of adrenaline and either a thermal or mechanical treatment. All who have received intervention should receive a continuous infusion of a proton pump inhibitor (IV omeprazole for 72 hours) to reduce the re-bleeding rate.
Patients with diffuse erosive gastritis who cannot be managed endoscopically and continue to bleed may require gastrectomy
Bleeding ulcers that cannot be controlled endoscopically may require laparotomy and ulcer underruning

Indications for surgery
Patients > 60 years
Continued bleeding despite endoscopic intervention
Recurrent bleeding
Known cardiovascular disease with poor response to hypotension

Surgery
Duodenal ulcer
Laparotomy, duodenotomy and under running of the ulcer. If bleeding is brisk then the ulcer is almost always posteriorly sited and will have invaded the gastroduodenal artery. Large bites using 0 Vicryl are taken above and below the ulcer base to occlude the vessel. The duodenotomy should be longitudinal but closed transversely to avoid stenosis.

For gastric ulcer
Under-running of the bleeding site 
Partial gastrectomy-antral ulcer
Partial gastrectomy or under running the ulcer- lesser curve ulcer (involving left gastric artery)
Total gastrectomy if bleeding persists

Summary of Acute Upper GI bleeding recommendations:
The need for admission and timing of endoscopic intervention may be predicted by using the Blatchford score. This considers a patients Hb, serum urea, pulse rate and blood pressure. Those patients with a score of 0 are low risk, all others are considered high risk and require admission and endoscopy.
The requirement for pre endoscopic proton pump inhibition is contentious. In the UK the National Institute of Clinical Excellence guidelines suggest the pre endoscopic PPI therapy is unnecessary. Whilst it is accepted that such treatment has no impact on mortality or morbidity a Cochrane review of this practice in 2007 did suggest that it reduced the stigmata of recent haemorrhage at endoscopy. As a result many will still administer PPI to patients prior to endoscopic intervention.
Following endoscopy it is important to calculate the Rockall score for patients to determine their risk of rebleeding and mortality. A score of 3 or less is associated with a rebleeding rate of 4% and a very low risk of mortality and identifies a group of patients suitable for early discharge.

178
Q

A 45 year old man with end stage renal failure undergoes a cadaveric renal transplant. The transplanted organ has a cold ischaemic time of 26 hours and a warm ischaemic time of 54 minutes. Post operatively the patient receives immunosuppressive therapy. Ten days later the patient has gained weight, becomes oliguric and feels systemically unwell. He also complains of swelling over the transplant site that is painful.

A.	Acute tubular necrosis
B.	Renal artery thrombosis
C.	Bladder occlusion
D.	Ureteric occlusion
E.	Acute rejection
F.	Acute on chronic rejection
G.	Hyperacute rejection
A

Acute rejection

The features described are those of worsening graft function and acute rejection. The fact that there is a 10 day delay goes against hyperacute rejection. Cold ischaemic times are a major factor for delayed graft function. However, even 26 hours is not incompatible with graft survival.

Complications following renal transplant

Renal transplantation is widely practised. The commonest technical related complications are related to the ureteric anastomosis. The warm ischaemic time is also of considerable importance and graft survival is directly related to this. Long warm ischaemic times increase the risk of acute tubular necrosis which may occur in all types of renal transplanation and provided other insults are minimised, will usually recover. Organ rejection may occur at any phase following the transplantation process.

Immunological complications
Types of organ rejection
Hyperacute. This occurs immediately through presence of pre formed antibody (such as ABO incompatibility).
Acute. Occurs during the first 6 months and is usually T cell mediated. Usually tissue infiltrates and vascular lesions.
Chronic. Occurs after the first 6 months. Vascular changes predominate.

Hyperacute
Renal transplants are most susceptible to this process. Risk factors include major HLA mismatch and ABO incompatibility. The rejection occurs almost immediately and the macroscopic features may become manifest following completion of the vascular anastomosis and removal of clamps. The kidney becomes mottled, dusky and the vessels will thrombose. The only treatment is removal of the graft, if left in situ it will result in abscess formation.

Acute
All organs may undergo acute rejection. Mononuclear cell infiltrates predominate. All types of transplanted organ are susceptible and it may occur in up to 50% cases. Most cases can be managed medically.

Chronic
Again all transplants with HLA mismatch may suffer this fate. Previous acute rejections and other immunosensitising events all increase the risk. Vascular changes are most prominent with myointimal proliferation leading to organ ischaemia. Organ specific changes are also seen such as loss of acinar cells in pancreas transplants and rapidly progressive coronary artery disease in cardiac transplants.

Technical complications

  • Renal artery thrombosis Sudden complete loss of urine output Immediate surgery may salvage the graft, delays beyond 30 minutes are associated with a high rate of graft loss
  • Renal artery stenosis Uncontrolled hypertension, allograft dysfunction and oedema Angioplasty is the treatment of choice
  • Renal vein thrombosis Pain and swelling over the graft site, haematuria and oliguria The graft is usually lost
  • Urine leaks Diminished urine output, rising creatinine, fever and abdominal pain USS shows perigraft collection, necrosis of ureter tip is the commonest cause and the anastomosis may need revision
  • Lymphocele Common complication (occurs in 15%), may present as a mass, if large may compress ureter May be drained with percutaneous technique and sclerotherapy, or intraperitoneal drainage
179
Q

A 44 year old man with end stage renal failure undergoes a live donor renal transplant. During the immediate post operative period a good urine output is recorded. However, on return to the ward the nursing staff notice that the urinary catheter is no longer draining. However, the urostomy is continuing to drain urine.

A.	Acute tubular necrosis
B.	Renal artery thrombosis
C.	Bladder occlusion
D.	Ureteric occlusion
E.	Acute rejection
F.	Acute on chronic rejection
G.	Hyperacute rejection
A

Bladder occlusion

The most likely explanation for this event is a blocked catheter. This may be the result of blood clot from the ureteric anastomosis. Bladder irrigation will usually resolve the problem.

Complications following renal transplant

Renal transplantation is widely practised. The commonest technical related complications are related to the ureteric anastomosis. The warm ischaemic time is also of considerable importance and graft survival is directly related to this. Long warm ischaemic times increase the risk of acute tubular necrosis which may occur in all types of renal transplanation and provided other insults are minimised, will usually recover. Organ rejection may occur at any phase following the transplantation process.

Immunological complications
Types of organ rejection
Hyperacute. This occurs immediately through presence of pre formed antibody (such as ABO incompatibility).
Acute. Occurs during the first 6 months and is usually T cell mediated. Usually tissue infiltrates and vascular lesions.
Chronic. Occurs after the first 6 months. Vascular changes predominate.

Hyperacute
Renal transplants are most susceptible to this process. Risk factors include major HLA mismatch and ABO incompatibility. The rejection occurs almost immediately and the macroscopic features may become manifest following completion of the vascular anastomosis and removal of clamps. The kidney becomes mottled, dusky and the vessels will thrombose. The only treatment is removal of the graft, if left in situ it will result in abscess formation.

Acute
All organs may undergo acute rejection. Mononuclear cell infiltrates predominate. All types of transplanted organ are susceptible and it may occur in up to 50% cases. Most cases can be managed medically.

Chronic
Again all transplants with HLA mismatch may suffer this fate. Previous acute rejections and other immunosensitising events all increase the risk. Vascular changes are most prominent with myointimal proliferation leading to organ ischaemia. Organ specific changes are also seen such as loss of acinar cells in pancreas transplants and rapidly progressive coronary artery disease in cardiac transplants.

Technical complications

  • Renal artery thrombosis Sudden complete loss of urine output Immediate surgery may salvage the graft, delays beyond 30 minutes are associated with a high rate of graft loss
  • Renal artery stenosis Uncontrolled hypertension, allograft dysfunction and oedema Angioplasty is the treatment of choice
  • Renal vein thrombosis Pain and swelling over the graft site, haematuria and oliguria The graft is usually lost
  • Urine leaks Diminished urine output, rising creatinine, fever and abdominal pain USS shows perigraft collection, necrosis of ureter tip is the commonest cause and the anastomosis may need revision
  • Lymphocele Common complication (occurs in 15%), may present as a mass, if large may compress ureter May be drained with percutaneous technique and sclerotherapy, or intraperitoneal drainage
180
Q

A 43 year old man undergoes a live donor renal transplant. The donor’s right kidney is anastomosed to the recipient. On removal of the arterial clamps there is good urinary flow noted and the wounds are closed. On return to the ward the nurses notice that the patient suddenly becomes anuric and irrigation of the bladder does not improve the situation.

A.	Acute tubular necrosis
B.	Renal artery thrombosis
C.	Bladder occlusion
D.	Ureteric occlusion
E.	Acute rejection
F.	Acute on chronic rejection
G.	Hyperacute rejection
A

Renal artery thrombosis
Right sided live donor transplants are extremely rare. This is because the vena cava precludes mobilisation of the right renal artery. The short right renal artery that is produced therefore presents a major challenge. The sudden cessation of urine output in this context is highly suggestive of an acute thrombosis. Delay in thrombectomy beyond 1 hour almost inevitably results in graft loss.

Complications following renal transplant

Renal transplantation is widely practised. The commonest technical related complications are related to the ureteric anastomosis. The warm ischaemic time is also of considerable importance and graft survival is directly related to this. Long warm ischaemic times increase the risk of acute tubular necrosis which may occur in all types of renal transplanation and provided other insults are minimised, will usually recover. Organ rejection may occur at any phase following the transplantation process.

Immunological complications
Types of organ rejection
Hyperacute. This occurs immediately through presence of pre formed antibody (such as ABO incompatibility).
Acute. Occurs during the first 6 months and is usually T cell mediated. Usually tissue infiltrates and vascular lesions.
Chronic. Occurs after the first 6 months. Vascular changes predominate.

Hyperacute
Renal transplants are most susceptible to this process. Risk factors include major HLA mismatch and ABO incompatibility. The rejection occurs almost immediately and the macroscopic features may become manifest following completion of the vascular anastomosis and removal of clamps. The kidney becomes mottled, dusky and the vessels will thrombose. The only treatment is removal of the graft, if left in situ it will result in abscess formation.

Acute
All organs may undergo acute rejection. Mononuclear cell infiltrates predominate. All types of transplanted organ are susceptible and it may occur in up to 50% cases. Most cases can be managed medically.

Chronic
Again all transplants with HLA mismatch may suffer this fate. Previous acute rejections and other immunosensitising events all increase the risk. Vascular changes are most prominent with myointimal proliferation leading to organ ischaemia. Organ specific changes are also seen such as loss of acinar cells in pancreas transplants and rapidly progressive coronary artery disease in cardiac transplants.

Technical complications

  • Renal artery thrombosis Sudden complete loss of urine output Immediate surgery may salvage the graft, delays beyond 30 minutes are associated with a high rate of graft loss
  • Renal artery stenosis Uncontrolled hypertension, allograft dysfunction and oedema Angioplasty is the treatment of choice
  • Renal vein thrombosis Pain and swelling over the graft site, haematuria and oliguria The graft is usually lost
  • Urine leaks Diminished urine output, rising creatinine, fever and abdominal pain USS shows perigraft collection, necrosis of ureter tip is the commonest cause and the anastomosis may need revision
  • Lymphocele Common complication (occurs in 15%), may present as a mass, if large may compress ureter May be drained with percutaneous technique and sclerotherapy, or intraperitoneal drainage
181
Q

A 56 year old man who drinks heavily is found collapsed by friends at his house. He was out drinking the previous night and following this was noted to have vomited repeatedly so his friends brought him home.

A.	Schatzki ring
B.	Plummer Vinson syndrome
C.	Squamous cell carcinoma
D.	Barretts oesophagus
E.	Pharyngeal pouch
F.	Adenocarcinoma
G.	Leiomyoma
H.	Oesophageal rupture
I.	Diffuse oesophageal spasm
J.	Hiatus hernia
A

Oesophageal rupture

Spontaneous rupture of the oesophagus may occur following an episode of vomiting. The subsequent mediastinitis can produce severe sepsis and death if not treated promptly. Adequate drainage of sepsis and early surgery are the cornerstones of management.

Mallory-Weiss Tear Usually history of antecedent vomiting. This is then followed by the vomiting of a small amount of blood. There is usually little in the way of systemic disturbance or prior symptoms.
Hiatus hernia of gastric cardia Often longstanding history of dyspepsia, patients are often overweight. Uncomplicated hiatus hernias should not be associated with dysphagia or haematemesis.
Oesophageal rupture Complete disruption of the oesophageal wall in absence of pre-existing pathology. Left postero-lateral oesophageal is commonest site (2-3cm from OG junction). Suspect in patients with severe chest pain without cardiac diagnosis and signs suggestive of pneumonia without convincing history, where there is history of vomiting. Erect CXR shows infiltrate or effusion in 90% of cases(1).
Squamous cell carcinoma of the oesophagus History of progressive dysphagia. Often signs of weight loss. Usually little or no history of previous GORD type symptoms.
Adenocarcinoma of the oesophagus Progressive dysphagia, may have previous symptoms of GORD or Barretts oesophagus.
Peptic stricture Longer history of dysphagia, often not progressive. Usually symptoms of GORD. Often lack systemic features seen with malignancy
Dysmotility disorder May have dysphagia that is episodic and non progressive. Retrosternal pain may accompany the episodes.

Diagnosis
Most of the differential diagnoses listed above can be accurately categorised by upper GI endoscopy (usually most patients). Where this fails to demonstrate a mechanical stricture the use of pH and manometry studies together with radiological contrast swallows will facilitate the diagnosis.

182
Q

A 43 year old man has been troubled with dysphagia for many years. He is known to have achalasia and has had numerous dilatations. Over the past 6 weeks his dysphagia has worsened. At endoscopy a friable mass is noted in the oesophagus.

A.	Schatzki ring
B.	Plummer Vinson syndrome
C.	Squamous cell carcinoma
D.	Barretts oesophagus
E.	Pharyngeal pouch
F.	Adenocarcinoma
G.	Leiomyoma
H.	Oesophageal rupture
I.	Diffuse oesophageal spasm
J.	Hiatus hernia
A

Squamous cell carcinoma

The risk of squamous cell carcinoma of the oesophagus is increased in people with achalasia. The condition often presents late and has a poor prognosis.

Mallory-Weiss Tear Usually history of antecedent vomiting. This is then followed by the vomiting of a small amount of blood. There is usually little in the way of systemic disturbance or prior symptoms.
Hiatus hernia of gastric cardia Often longstanding history of dyspepsia, patients are often overweight. Uncomplicated hiatus hernias should not be associated with dysphagia or haematemesis.
Oesophageal rupture Complete disruption of the oesophageal wall in absence of pre-existing pathology. Left postero-lateral oesophageal is commonest site (2-3cm from OG junction). Suspect in patients with severe chest pain without cardiac diagnosis and signs suggestive of pneumonia without convincing history, where there is history of vomiting. Erect CXR shows infiltrate or effusion in 90% of cases(1).
Squamous cell carcinoma of the oesophagus History of progressive dysphagia. Often signs of weight loss. Usually little or no history of previous GORD type symptoms.
Adenocarcinoma of the oesophagus Progressive dysphagia, may have previous symptoms of GORD or Barretts oesophagus.
Peptic stricture Longer history of dysphagia, often not progressive. Usually symptoms of GORD. Often lack systemic features seen with malignancy
Dysmotility disorder May have dysphagia that is episodic and non progressive. Retrosternal pain may accompany the episodes.

Diagnosis
Most of the differential diagnoses listed above can be accurately categorised by upper GI endoscopy (usually most patients). Where this fails to demonstrate a mechanical stricture the use of pH and manometry studies together with radiological contrast swallows will facilitate the diagnosis.

183
Q

A 73 year old lady is troubled by episodic swallowing difficulty and halitosis. An upper GI endoscopy is attempted and abandoned due to difficulty in achieving intubation.

A.	Schatzki ring
B.	Plummer Vinson syndrome
C.	Squamous cell carcinoma
D.	Barretts oesophagus
E.	Pharyngeal pouch
F.	Adenocarcinoma
G.	Leiomyoma
H.	Oesophageal rupture
I.	Diffuse oesophageal spasm
J.	Hiatus hernia
A

Pharyngeal pouch

Pharyngeal pouches occur when a defect occurs in killians dehiscence. Difficulty in intubation is a well recognised consequence and care must be taken to take the correct track during OGD to avoid perforation. Most cases are now treated with endoscopic stapling.

Mallory-Weiss Tear Usually history of antecedent vomiting. This is then followed by the vomiting of a small amount of blood. There is usually little in the way of systemic disturbance or prior symptoms.
Hiatus hernia of gastric cardia Often longstanding history of dyspepsia, patients are often overweight. Uncomplicated hiatus hernias should not be associated with dysphagia or haematemesis.
Oesophageal rupture Complete disruption of the oesophageal wall in absence of pre-existing pathology. Left postero-lateral oesophageal is commonest site (2-3cm from OG junction). Suspect in patients with severe chest pain without cardiac diagnosis and signs suggestive of pneumonia without convincing history, where there is history of vomiting. Erect CXR shows infiltrate or effusion in 90% of cases(1).
Squamous cell carcinoma of the oesophagus History of progressive dysphagia. Often signs of weight loss. Usually little or no history of previous GORD type symptoms.
Adenocarcinoma of the oesophagus Progressive dysphagia, may have previous symptoms of GORD or Barretts oesophagus.
Peptic stricture Longer history of dysphagia, often not progressive. Usually symptoms of GORD. Often lack systemic features seen with malignancy
Dysmotility disorder May have dysphagia that is episodic and non progressive. Retrosternal pain may accompany the episodes.

Diagnosis
Most of the differential diagnoses listed above can be accurately categorised by upper GI endoscopy (usually most patients). Where this fails to demonstrate a mechanical stricture the use of pH and manometry studies together with radiological contrast swallows will facilitate the diagnosis.

184
Q

A two week old child is brought to the emergency department by his parents. He was slow to establish on feeds but was discharged home three days following delivery. During the past 7 hours he has been vomiting and the vomit is largely bile stained. On examination he has a soft, scaphoid abdomen.

A.	Duodenal atresia
B.	Hypertrophy of the pyloric sphincter
C.	Budd Chiari Syndrome
D.	Intussceception
E.	Oesophageal atresia
F.	Congenital diaphragmatic hernia
G.	Cystic fibrosis
H.	Intestinal malrotation
I.	Gastroenteritis
A

Intestinal malrotation

The combination of scaphoid abdomen and bilious vomiting is highly suggestive of intestinal malrotation and volvulus. An urgent upper GI contrast study and ultrasound is required.

Pyloric stenosis
M>F
5-10% Family history in parents
Projectile non bile stained vomiting at 4-6 weeks of life
Diagnosis is made by test feed or USS
Treatment: Ramstedt pyloromyotomy (open or laparoscopic)
Acute appendicitis
Uncommon under 3 years
When occurs may present atypically
Mesenteric adenitis
Central abdominal pain and URTI
Conservative management
Intussusception
Telescoping bowel
Proximal to or at the level of, ileocaecal valve
6-9 months age
Colicky pain, diarrhoea and vomiting, sausage shaped mass, red jelly stool.
Treatment: reduction with air insufflation
Malrotation
High caecum at the midline
Feature in exomphalos, congenital diaphragmatic hernia, intrinsic duodenal atresia
May be complicated by development of volvulus, infant with volvulus may have bile stained vomiting
Diagnosis is made by upper GI contrast study and USS
Treatment is by laparotomy, if volvulus is present (or at high risk of occurring then a ladds procedure is performed
Hirschsprung’s disease
Absence of ganglion cells from myenteric and submucosal plexuses
Occurs in 1/5000 births
Full thickness rectal biopsy for diagnosis
Delayed passage of meconium and abdominal distension
Treatment is with rectal washouts initially, thereafter an anorectal pull through procedure
Oesophageal atresia
Associated with tracheo-oesophageal fistula and polyhydramnios
May present with choking and cyanotic spells following aspiration
VACTERL associations
Meconium ileus
Usually delayed passage of meconium and abdominal distension
Majority have cystic fibrosis
X-Rays may not show a fluid level as the meconium is viscid (depends upon feeding), PR contrast studies may dislodge meconium plugs and be therapeutic
Infants who do not respond to PR contrast and NG N-acetyl cysteine will require surgery to remove the plugs
Biliary atresia
Jaundice > 14 days
Increased conjugated bilirubin
Urgent Kasai procedure
Necrotising enterocolitis
Prematurity is the main risk factor
Early features include abdominal distension and passage of bloody stools
X-Rays may show pneumatosis intestinalis and evidence of free air
Increased risk when empirical antibiotics are given to infants beyond 5 days
Treatment is with total gut rest and TPN, babies with perforations will require laparotomy

185
Q

A 4 week old baby is developing well and develops profuse and projectile vomiting after feeds. He has been losing weight and the vomit is described as being non bilious.

A.	Duodenal atresia
B.	Hypertrophy of the pyloric sphincter
C.	Budd Chiari Syndrome
D.	Intussceception
E.	Oesophageal atresia
F.	Congenital diaphragmatic hernia
G.	Cystic fibrosis
H.	Intestinal malrotation
I.	Gastroenteritis
A

Hypertrophy of the pyloric sphincter

A history of projective vomiting and weight loss is a common story suggestive of pyloric stenosis. The vomit is often not bile stained. Diagnosis is further suggested by hypochloraemic metabolic alkalosis and a palpable tumour on test feeding.

Pyloric stenosis
M>F
5-10% Family history in parents
Projectile non bile stained vomiting at 4-6 weeks of life
Diagnosis is made by test feed or USS
Treatment: Ramstedt pyloromyotomy (open or laparoscopic)
Acute appendicitis
Uncommon under 3 years
When occurs may present atypically
Mesenteric adenitis
Central abdominal pain and URTI
Conservative management
Intussusception
Telescoping bowel
Proximal to or at the level of, ileocaecal valve
6-9 months age
Colicky pain, diarrhoea and vomiting, sausage shaped mass, red jelly stool.
Treatment: reduction with air insufflation
Malrotation
High caecum at the midline
Feature in exomphalos, congenital diaphragmatic hernia, intrinsic duodenal atresia
May be complicated by development of volvulus, infant with volvulus may have bile stained vomiting
Diagnosis is made by upper GI contrast study and USS
Treatment is by laparotomy, if volvulus is present (or at high risk of occurring then a ladds procedure is performed
Hirschsprung’s disease
Absence of ganglion cells from myenteric and submucosal plexuses
Occurs in 1/5000 births
Full thickness rectal biopsy for diagnosis
Delayed passage of meconium and abdominal distension
Treatment is with rectal washouts initially, thereafter an anorectal pull through procedure
Oesophageal atresia
Associated with tracheo-oesophageal fistula and polyhydramnios
May present with choking and cyanotic spells following aspiration
VACTERL associations
Meconium ileus
Usually delayed passage of meconium and abdominal distension
Majority have cystic fibrosis
X-Rays may not show a fluid level as the meconium is viscid (depends upon feeding), PR contrast studies may dislodge meconium plugs and be therapeutic
Infants who do not respond to PR contrast and NG N-acetyl cysteine will require surgery to remove the plugs
Biliary atresia
Jaundice > 14 days
Increased conjugated bilirubin
Urgent Kasai procedure
Necrotising enterocolitis
Prematurity is the main risk factor
Early features include abdominal distension and passage of bloody stools
X-Rays may show pneumatosis intestinalis and evidence of free air
Increased risk when empirical antibiotics are given to infants beyond 5 days
Treatment is with total gut rest and TPN, babies with perforations will require laparotomy

186
Q

A 1 day old child is born by emergency cesarean section for foetal distress. On examination he has decreased air entry on the left side of his chest and a displaced apex beat. Abdominal examination demonstrates a scaphoid abdomen but is otherwise unremarkable.

A.	Duodenal atresia
B.	Hypertrophy of the pyloric sphincter
C.	Budd Chiari Syndrome
D.	Intussceception
E.	Oesophageal atresia
F.	Congenital diaphragmatic hernia
G.	Cystic fibrosis
H.	Intestinal malrotation
I.	Gastroenteritis
A

Congenital diaphragmatic hernia

Displaced apex beat and decreased air entry are suggestive of diaphragmatic hernia. The abdomen may well be scaphoid in some cases. The underlying lung may be hypoplastic and this correlates directly with prognosis.

Pyloric stenosis
M>F
5-10% Family history in parents
Projectile non bile stained vomiting at 4-6 weeks of life
Diagnosis is made by test feed or USS
Treatment: Ramstedt pyloromyotomy (open or laparoscopic)
Acute appendicitis
Uncommon under 3 years
When occurs may present atypically
Mesenteric adenitis
Central abdominal pain and URTI
Conservative management
Intussusception
Telescoping bowel
Proximal to or at the level of, ileocaecal valve
6-9 months age
Colicky pain, diarrhoea and vomiting, sausage shaped mass, red jelly stool.
Treatment: reduction with air insufflation
Malrotation
High caecum at the midline
Feature in exomphalos, congenital diaphragmatic hernia, intrinsic duodenal atresia
May be complicated by development of volvulus, infant with volvulus may have bile stained vomiting
Diagnosis is made by upper GI contrast study and USS
Treatment is by laparotomy, if volvulus is present (or at high risk of occurring then a ladds procedure is performed
Hirschsprung’s disease
Absence of ganglion cells from myenteric and submucosal plexuses
Occurs in 1/5000 births
Full thickness rectal biopsy for diagnosis
Delayed passage of meconium and abdominal distension
Treatment is with rectal washouts initially, thereafter an anorectal pull through procedure
Oesophageal atresia
Associated with tracheo-oesophageal fistula and polyhydramnios
May present with choking and cyanotic spells following aspiration
VACTERL associations
Meconium ileus
Usually delayed passage of meconium and abdominal distension
Majority have cystic fibrosis
X-Rays may not show a fluid level as the meconium is viscid (depends upon feeding), PR contrast studies may dislodge meconium plugs and be therapeutic
Infants who do not respond to PR contrast and NG N-acetyl cysteine will require surgery to remove the plugs
Biliary atresia
Jaundice > 14 days
Increased conjugated bilirubin
Urgent Kasai procedure
Necrotising enterocolitis
Prematurity is the main risk factor
Early features include abdominal distension and passage of bloody stools
X-Rays may show pneumatosis intestinalis and evidence of free air
Increased risk when empirical antibiotics are given to infants beyond 5 days
Treatment is with total gut rest and TPN, babies with perforations will require laparotomy

187
Q

A male infant is born at 28 weeks gestation by emergency cesarean section. He is taken to theatre for a colostomy due to an imperforate anus. He initially seems to be progressing well. However, he begins to develop decerebrate posturing and is becoming increasingly obtunded.

A.	Intraventricular haemorrhage
B.	Chronic sub dural bleed
C.	Acute sub dural bleed
D.	Extra dural haemorrhage
E.	Sub arachnoid haemorrhage
A

Intraventricular haemorrhage

Acute neurological deterioration in premature neonates is usually due to intraventricular haemorrhage. Diagnosis is made by cranial ultrasound. Development of hydrocephalus may necessitate surgery.

Extradural haematoma Bleeding into the space between the dura mater and the skull. Often results from acceleration-deceleration trauma or a blow to the side of the head. The majority of extradural haematomas occur in the temporal region where skull fractures cause a rupture of the middle meningeal artery.

Features
Raised intracranial pressure
Some patients may exhibit a lucid interval
Subdural haematoma Bleeding into the outermost meningeal layer. Most commonly occur around the frontal and parietal lobes. May be either acute or chronic.

Risk factors include old age and alcoholism.

Slower onset of symptoms than a extradural haematoma.
Intracerebral haematoma Usually hyperdense lesions on CT scanning. Arise in areas of traumatic contusion with fuse to become a haematoma. Areas of clot and fresh blood may co-exist on the same CT scan (Swirl sign). Large haematomas and those associated with mass effect should be evacuated.
Subarachnoid haemorrhage Usually occurs spontaneously in the context of a ruptured cerebral aneurysm but may be seen in association with other injuries when a patient has sustained a traumatic brain injury
Intraventricular haemorrhage Haemorrhage that occurs into the ventricular system of the brain. It is relatively rare in adult surgical practice and when it does occur, it is typically associated with severe head injuries. In premature neonates it may occur spontaneously. The blood may clot and occlude CSF flow, hydrocephalus may result.
In neonatal practice the vast majority of IVH occur in the first 72 hours after birth, the aetiology is not well understood and it is suggested to occur as a result of birth trauma combined with cellular hypoxia, together with the delicate neonatal CNS.

188
Q

A 50 year old lady is admitted having fallen down some stairs sustaining multiple rib fractures 36 hours previously. On examination she is confused and agitated and has clinical evidence of lateralising signs. She deteriorates further and then dies with no response to resuscitation.

A.	Intraventricular haemorrhage
B.	Chronic sub dural bleed
C.	Acute sub dural bleed
D.	Extra dural haemorrhage
E.	Sub arachnoid haemorrhage
A

Acute sub dural bleed

The time frame of deterioration of an acute sub dural bleed would fit with this scenario. They are highly lethal and not uncommon injuries. As the bleed enlarges, lateralising signs may be seen and eventually coning and death will occur.

Extradural haematoma Bleeding into the space between the dura mater and the skull. Often results from acceleration-deceleration trauma or a blow to the side of the head. The majority of extradural haematomas occur in the temporal region where skull fractures cause a rupture of the middle meningeal artery.

Features
Raised intracranial pressure
Some patients may exhibit a lucid interval
Subdural haematoma Bleeding into the outermost meningeal layer. Most commonly occur around the frontal and parietal lobes. May be either acute or chronic.

Risk factors include old age and alcoholism.

Slower onset of symptoms than a extradural haematoma.
Intracerebral haematoma Usually hyperdense lesions on CT scanning. Arise in areas of traumatic contusion with fuse to become a haematoma. Areas of clot and fresh blood may co-exist on the same CT scan (Swirl sign). Large haematomas and those associated with mass effect should be evacuated.
Subarachnoid haemorrhage Usually occurs spontaneously in the context of a ruptured cerebral aneurysm but may be seen in association with other injuries when a patient has sustained a traumatic brain injury
Intraventricular haemorrhage Haemorrhage that occurs into the ventricular system of the brain. It is relatively rare in adult surgical practice and when it does occur, it is typically associated with severe head injuries. In premature neonates it may occur spontaneously. The blood may clot and occlude CSF flow, hydrocephalus may result.
In neonatal practice the vast majority of IVH occur in the first 72 hours after birth, the aetiology is not well understood and it is suggested to occur as a result of birth trauma combined with cellular hypoxia, together with the delicate neonatal CNS.

189
Q

A 50 year old alcoholic man attends the emergency department. His main reason for presenting is that he has no home to go to. On examination he has no evidence of involvement in recent trauma, a skull x-ray fails to show any evidence of skull fracture. He is admitted and twelve hours following admission he develops sudden onset headache, becomes comatose and then dies.

A.	Intraventricular haemorrhage
B.	Chronic sub dural bleed
C.	Acute sub dural bleed
D.	Extra dural haemorrhage
E.	Sub arachnoid haemorrhage
A

Sub arachnoid haemorrhage

The absence of trauma here makes an acute sub dural and extra dural bleed unlikely. Chronic sub dural bleeds would usually cause a more gradual deterioration than is seen here. The absence of any skull fracture also makes an underlying intra cranial bleed less likely. Sudden onset headaches, together with sudden deterioration in neurological function are typical of a sub arachnoid haemorrhage.

Extradural haematoma Bleeding into the space between the dura mater and the skull. Often results from acceleration-deceleration trauma or a blow to the side of the head. The majority of extradural haematomas occur in the temporal region where skull fractures cause a rupture of the middle meningeal artery.

Features
Raised intracranial pressure
Some patients may exhibit a lucid interval
Subdural haematoma Bleeding into the outermost meningeal layer. Most commonly occur around the frontal and parietal lobes. May be either acute or chronic.

Risk factors include old age and alcoholism.

Slower onset of symptoms than a extradural haematoma.
Intracerebral haematoma Usually hyperdense lesions on CT scanning. Arise in areas of traumatic contusion with fuse to become a haematoma. Areas of clot and fresh blood may co-exist on the same CT scan (Swirl sign). Large haematomas and those associated with mass effect should be evacuated.
Subarachnoid haemorrhage Usually occurs spontaneously in the context of a ruptured cerebral aneurysm but may be seen in association with other injuries when a patient has sustained a traumatic brain injury
Intraventricular haemorrhage Haemorrhage that occurs into the ventricular system of the brain. It is relatively rare in adult surgical practice and when it does occur, it is typically associated with severe head injuries. In premature neonates it may occur spontaneously. The blood may clot and occlude CSF flow, hydrocephalus may result.
In neonatal practice the vast majority of IVH occur in the first 72 hours after birth, the aetiology is not well understood and it is suggested to occur as a result of birth trauma combined with cellular hypoxia, together with the delicate neonatal CNS.

190
Q

A 32 year old Indian lady presents with a diffuse swelling of the left breast. She has a 4 month old child. Clinically, she has jaundice and there is erythema of the left breast.

A.	Ductal carcinoma in situ
B.	Lobular carcinoma in situ
C.	Invasive ductal carcinoma
D.	Invasive lobular carcinoma
E.	Inflammatory carcinoma
F.	Phyllodes tumour
G.	Paget's disease of the nipple
H.	Fibroadenoma
I.	Mucinous breast carcinoma
A

Inflammatory carcinoma

Inflammatory breast cancers have an aggressive nature. Dissemination occurs early and is more resistant to adjuvent treatments than other types of breast cancer. Often occurs in pregnancy or lactation.

Breast cancer

Commoner in the older age group
Invasive ductal carcinomas are the most common type. Some may arise as a result of ductal carcinoma in situ (DCIS). There are associated carcinomas of special type e.g. Tubular that may carry better prognosis.
The pathological assessment involves assessment of the tumour and lymph nodes, sentinel lymph node biopsy is often used to minimise the morbidity of an axillary dissection.
Treatment, typically this is either wide local excision or mastectomy. There are many sub types of both of these that fall outside of the MRCS. Some key rules to bear in mind.
Whatever operation is contemplated the final cosmetic outcome does have a bearing. A woman with small breasts and a large tumour will tend to fare better with mastectomy, even if clear pathological and clinical margins can be obtained. Conversely a women with larger breasts may be able to undergo breast conserving surgery even with a relatively large primary lesion (NB tumours >4cm used to attract recommendation for mastectomy). For screen detected and impalpable tumour image guidance will be necessary.
Reconstruction is always an option following any resectional procedure. However, its exact type must be tailored to age and co-morbidities of the patient. The main operations in common use include latissimus dorsi myocutaneous flap and sub pectoral implants. Women wishing to avoid a prosthesis may be offered TRAM or DIEP flaps.

Surgical options
Mastectomy vs Wide local excision

Mastectomy	Wide Local Excision
Multifocal tumour	Solitary lesion
Central tumour	Peripheral tumour
Large lesion in small breast	Small lesion in large breast
DCIS >4cm	DCIS <4cm
Patient Choice	Patient choice

Central lesions may be managed using breast conserving surgery where an acceptable cosmetic result may be obtained, this is rarely the case in small breasts

A compelling indication for mastectomy, a larger tumour that would be unsuitable for breast conserving surgery

Image sourced from Wikipedia

Whatever surgical option is chosen the aim should be to have a local recurrence rate of 5% or less at 5 years [1].

Nottingham Prognostic Index
The Nottingham Prognostic Index can be used to give an indication of survival. In this system the tumour size is weighted less heavily than other major prognostic parameters.

Calculation of NPI
Tumour Size x 0.2 + Lymph node score(From table below)+Grade score(From table below).

Score Lymph nodes involved Grade
1 0 1
2 1-3 2
3 >3 3

Prognosis

Score	Percentage 5 year survival
2.0 to 2.4	93%
2.5 to 3.4	85%
3.5 to 5.4	70%
>5.4	50%
191
Q

A 72 year old female presents with a painless breast lump. Clinically she has a 4cm diameter irregular breast mass, with no other palpable masses.

A.	Ductal carcinoma in situ
B.	Lobular carcinoma in situ
C.	Invasive ductal carcinoma
D.	Invasive lobular carcinoma
E.	Inflammatory carcinoma
F.	Phyllodes tumour
G.	Paget's disease of the nipple
H.	Fibroadenoma
I.	Mucinous breast carcinoma
A

Invasive ductal carcinoma

A post menopausal woman is more likely to have a ductal carcinoma and they tend to occur at a single focus within the breast.

Breast cancer

Commoner in the older age group
Invasive ductal carcinomas are the most common type. Some may arise as a result of ductal carcinoma in situ (DCIS). There are associated carcinomas of special type e.g. Tubular that may carry better prognosis.
The pathological assessment involves assessment of the tumour and lymph nodes, sentinel lymph node biopsy is often used to minimise the morbidity of an axillary dissection.
Treatment, typically this is either wide local excision or mastectomy. There are many sub types of both of these that fall outside of the MRCS. Some key rules to bear in mind.
Whatever operation is contemplated the final cosmetic outcome does have a bearing. A woman with small breasts and a large tumour will tend to fare better with mastectomy, even if clear pathological and clinical margins can be obtained. Conversely a women with larger breasts may be able to undergo breast conserving surgery even with a relatively large primary lesion (NB tumours >4cm used to attract recommendation for mastectomy). For screen detected and impalpable tumour image guidance will be necessary.
Reconstruction is always an option following any resectional procedure. However, its exact type must be tailored to age and co-morbidities of the patient. The main operations in common use include latissimus dorsi myocutaneous flap and sub pectoral implants. Women wishing to avoid a prosthesis may be offered TRAM or DIEP flaps.

Surgical options
Mastectomy vs Wide local excision

Mastectomy	Wide Local Excision
Multifocal tumour	Solitary lesion
Central tumour	Peripheral tumour
Large lesion in small breast	Small lesion in large breast
DCIS >4cm	DCIS <4cm
Patient Choice	Patient choice

Central lesions may be managed using breast conserving surgery where an acceptable cosmetic result may be obtained, this is rarely the case in small breasts

A compelling indication for mastectomy, a larger tumour that would be unsuitable for breast conserving surgery

Image sourced from Wikipedia

Whatever surgical option is chosen the aim should be to have a local recurrence rate of 5% or less at 5 years [1].

Nottingham Prognostic Index
The Nottingham Prognostic Index can be used to give an indication of survival. In this system the tumour size is weighted less heavily than other major prognostic parameters.

Calculation of NPI
Tumour Size x 0.2 + Lymph node score(From table below)+Grade score(From table below).

Score Lymph nodes involved Grade
1 0 1
2 1-3 2
3 >3 3

Prognosis

Score	Percentage 5 year survival
2.0 to 2.4	93%
2.5 to 3.4	85%
3.5 to 5.4	70%
>5.4	50%
192
Q

A 72 year old woman presents with 2 breast lumps. She has a history of breast cancer in the opposite breast 5 years ago.

A.	Ductal carcinoma in situ
B.	Lobular carcinoma in situ
C.	Invasive ductal carcinoma
D.	Invasive lobular carcinoma
E.	Inflammatory carcinoma
F.	Phyllodes tumour
G.	Paget's disease of the nipple
H.	Fibroadenoma
I.	Mucinous breast carcinoma
A

Invasive lobular carcinoma

This is likely to be an invasive lobular carcinoma, mainly due to the multifocal lesions and the history of previous breast cancer in the opposite breast.

Breast cancer

Commoner in the older age group
Invasive ductal carcinomas are the most common type. Some may arise as a result of ductal carcinoma in situ (DCIS). There are associated carcinomas of special type e.g. Tubular that may carry better prognosis.
The pathological assessment involves assessment of the tumour and lymph nodes, sentinel lymph node biopsy is often used to minimise the morbidity of an axillary dissection.
Treatment, typically this is either wide local excision or mastectomy. There are many sub types of both of these that fall outside of the MRCS. Some key rules to bear in mind.
Whatever operation is contemplated the final cosmetic outcome does have a bearing. A woman with small breasts and a large tumour will tend to fare better with mastectomy, even if clear pathological and clinical margins can be obtained. Conversely a women with larger breasts may be able to undergo breast conserving surgery even with a relatively large primary lesion (NB tumours >4cm used to attract recommendation for mastectomy). For screen detected and impalpable tumour image guidance will be necessary.
Reconstruction is always an option following any resectional procedure. However, its exact type must be tailored to age and co-morbidities of the patient. The main operations in common use include latissimus dorsi myocutaneous flap and sub pectoral implants. Women wishing to avoid a prosthesis may be offered TRAM or DIEP flaps.

Surgical options
Mastectomy vs Wide local excision

Mastectomy	Wide Local Excision
Multifocal tumour	Solitary lesion
Central tumour	Peripheral tumour
Large lesion in small breast	Small lesion in large breast
DCIS >4cm	DCIS <4cm
Patient Choice	Patient choice

Central lesions may be managed using breast conserving surgery where an acceptable cosmetic result may be obtained, this is rarely the case in small breasts

A compelling indication for mastectomy, a larger tumour that would be unsuitable for breast conserving surgery

Image sourced from Wikipedia

Whatever surgical option is chosen the aim should be to have a local recurrence rate of 5% or less at 5 years [1].

Nottingham Prognostic Index
The Nottingham Prognostic Index can be used to give an indication of survival. In this system the tumour size is weighted less heavily than other major prognostic parameters.

Calculation of NPI
Tumour Size x 0.2 + Lymph node score(From table below)+Grade score(From table below).

Score Lymph nodes involved Grade
1 0 1
2 1-3 2
3 >3 3

Prognosis

Score	Percentage 5 year survival
2.0 to 2.4	93%
2.5 to 3.4	85%
3.5 to 5.4	70%
>5.4	50%
193
Q

A 63 year old lady presents with an obstructing cancer of the sigmoid colon. She is not peritonitic and her imaging demonstrates a solitary liver metastasis.

A. Ileocolic bypass
B. Loop ileostomy
C. High anterior resection
D. Insertion of self expanding metallic stent
E. Left hemicolectomy and on table colonic lavage and primary anastomosis
F. Extended right hemicolectomy and ileocolic anastomosis
G. Low anterior resection
H. Loop colostomy of the transverse colon
I. Loop colostomy of the sigmoid colon
J. Right hemicolectomy

A

Insertion of self expanding metallic stent

Ideally, the distant disease should be managed first and then the primary lesion addressed. A self expanding stent is likely to achieve this and avoids a stoma.

Large bowel obstruction

Colonic obstruction remains a common surgical problem. It is most commonly due to malignancy (60%) and diverticular disease (20%). Volvulus affecting the colon accounts for 5% of cases. Acute colonic pseudo-obstruction remains a potential differential diagnosis in all cases. Intussusception affecting the colon (most often due to tumours in the adult population) remains a rare but recognised cause.
The typical patient will present with gradual onset of progressive abdominal distension, colicky abdominal pain and either obstipation or absolute constipation.
On examination abdominal distension is present, the presence of caecal tenderness (assuming no overt evidence of peritonitis) is a useful sign to elicit. A digital rectal examination and rigid sigmoidoscopy should be performed.
A plain abdominal x-ray is the usual first line test and; the caecal diameter and ileocaecal valve competency should be assessed on this film.

Imaging modalities
Debate long surrounds the use of CT versus gastrograffin enemas. The latter investigation has always been the traditional method of determining whether a structural lesion is indeed present. However, in the UK the use of this technique has declined and in most units a CT scan will be offered as the first line investigation by the majority of radiologists (and is advocated by the ACPGBI). In most cases this will provide sufficient detail to allow operative planning, and since malignancy accounts for most presentations may also stage the disease. In the event that the radiologist cannot provide a clear statement of lesion site, the surgeon should have no hesitation in requesting a contrast enema.

Surgical options
The decision as to when to operate or not is determined firstly by the patients physiological status. Unstable patients require resuscitation prior to surgery and admission to a critical care unit for invasive monitoring and potential inotropic support may be needed. In patients who are otherwise stable the decision then rests on the radiological and clinical findings. As a general rule the old adage that the sun should not rise and set on unrelieved large bowel obstruction still holds true. A caecal diameter of 12cm or more in the presence of complete obstruction with a competent ileocaecal valve and caecal tenderness is a sign of impending perforation and a relative indication for prompt surgery.

Right sided and transverse lesions
Right sided lesions producing large bowel obstruction should generally be treated by right hemicolectomy or its extended variant if the lesion lies in the distal transverse colon or splenic flexure. In these cases an ileocolic anastomosis may be easily constructed and even in the emergency setting has a low risk of anastomotic leak.

Left sided lesions
The options here lie between sub total colectomy and anastomosis, left hemicolectomy with on table lavage and primary anastomosis, left hemicolectomy and end colostomy formation and finally colonic stent insertion.
The usefulness of colonic stents was the subject of a Cochrane review in 2011. The authors concluded that on the basis of the data that they reviewed, there was no benefit from the use of colonic stents over conventional surgical resection with a tendency to better outcomes seen in the surgical group (1). A more recently conducted meta analysis met with the same conclusion (2). However, the recently concluded CREST trial has suggested that self expanding metallic stents can improve outcomes and avoids a stoma.

Rectosigmoid lesions
Lesions below the peritoneal reflection that are causing obstruction should generally be treated with a loop colostomy. Primary resection of unstaged rectal cancer would most likely carry a high CRM positivity rate and cannot be condoned. Where the lesion occupies the distal sigmoid colon the usual practice would be to perform a high anterior resection. The decision surrounding restoration of intestinal continuity would lie with the operating surgeon.

194
Q

A 65 year old man presents with absolute constipation and abdominal pain. On examination he has marked abdominal distension. A digital rectal examination reveals an empty rectum. A rectal contrast study shows an obstructing lesion of the proximal rectum.

A. Ileocolic bypass
B. Loop ileostomy
C. High anterior resection
D. Insertion of self expanding metallic stent
E. Left hemicolectomy and on table colonic lavage and primary anastomosis
F. Extended right hemicolectomy and ileocolic anastomosis
G. Low anterior resection
H. Loop colostomy of the transverse colon
I. Loop colostomy of the sigmoid colon
J. Right hemicolectomy

A

Loop colostomy of the sigmoid colon

Rectal cancers should not be primarily resected prior to definitive staging and a tumour of this nature is likely to have circumferential margin involvement. Whilst a sigmoid and transverse loop colostomy would both provide an equal relief of obstruction the former procedure has the added benefit of making a subsequent resection safer, since a transverse colostomy would have to be taken down and closed during the course of subsequent surgery.

Large bowel obstruction

Colonic obstruction remains a common surgical problem. It is most commonly due to malignancy (60%) and diverticular disease (20%). Volvulus affecting the colon accounts for 5% of cases. Acute colonic pseudo-obstruction remains a potential differential diagnosis in all cases. Intussusception affecting the colon (most often due to tumours in the adult population) remains a rare but recognised cause.
The typical patient will present with gradual onset of progressive abdominal distension, colicky abdominal pain and either obstipation or absolute constipation.
On examination abdominal distension is present, the presence of caecal tenderness (assuming no overt evidence of peritonitis) is a useful sign to elicit. A digital rectal examination and rigid sigmoidoscopy should be performed.
A plain abdominal x-ray is the usual first line test and; the caecal diameter and ileocaecal valve competency should be assessed on this film.

Imaging modalities
Debate long surrounds the use of CT versus gastrograffin enemas. The latter investigation has always been the traditional method of determining whether a structural lesion is indeed present. However, in the UK the use of this technique has declined and in most units a CT scan will be offered as the first line investigation by the majority of radiologists (and is advocated by the ACPGBI). In most cases this will provide sufficient detail to allow operative planning, and since malignancy accounts for most presentations may also stage the disease. In the event that the radiologist cannot provide a clear statement of lesion site, the surgeon should have no hesitation in requesting a contrast enema.

Surgical options
The decision as to when to operate or not is determined firstly by the patients physiological status. Unstable patients require resuscitation prior to surgery and admission to a critical care unit for invasive monitoring and potential inotropic support may be needed. In patients who are otherwise stable the decision then rests on the radiological and clinical findings. As a general rule the old adage that the sun should not rise and set on unrelieved large bowel obstruction still holds true. A caecal diameter of 12cm or more in the presence of complete obstruction with a competent ileocaecal valve and caecal tenderness is a sign of impending perforation and a relative indication for prompt surgery.

Right sided and transverse lesions
Right sided lesions producing large bowel obstruction should generally be treated by right hemicolectomy or its extended variant if the lesion lies in the distal transverse colon or splenic flexure. In these cases an ileocolic anastomosis may be easily constructed and even in the emergency setting has a low risk of anastomotic leak.

Left sided lesions
The options here lie between sub total colectomy and anastomosis, left hemicolectomy with on table lavage and primary anastomosis, left hemicolectomy and end colostomy formation and finally colonic stent insertion.
The usefulness of colonic stents was the subject of a Cochrane review in 2011. The authors concluded that on the basis of the data that they reviewed, there was no benefit from the use of colonic stents over conventional surgical resection with a tendency to better outcomes seen in the surgical group (1). A more recently conducted meta analysis met with the same conclusion (2). However, the recently concluded CREST trial has suggested that self expanding metallic stents can improve outcomes and avoids a stoma.

Rectosigmoid lesions
Lesions below the peritoneal reflection that are causing obstruction should generally be treated with a loop colostomy. Primary resection of unstaged rectal cancer would most likely carry a high CRM positivity rate and cannot be condoned. Where the lesion occupies the distal sigmoid colon the usual practice would be to perform a high anterior resection. The decision surrounding restoration of intestinal continuity would lie with the operating surgeon.

195
Q

A 70 year old lady presents with a two day history of constipation and vomiting. On examination she has right iliac fossa tenderness and little abdominal distension. A CT scan is performed and is suggestive of an obstructing carcinoma of the colonic hepatic flexure (stage T3).

A. Ileocolic bypass
B. Loop ileostomy
C. High anterior resection
D. Insertion of self expanding metallic stent
E. Left hemicolectomy and on table colonic lavage and primary anastomosis
F. Extended right hemicolectomy and ileocolic anastomosis
G. Low anterior resection
H. Loop colostomy of the transverse colon
I. Loop colostomy of the sigmoid colon
J. Right hemicolectomy

A

Right hemicolectomy

This lesion should be amenable to standard right hemicolectomy. Extending the resection to take the middle colic vessels and distal transverse colon is unlikely to provide additional oncological benefit.

Large bowel obstruction

Colonic obstruction remains a common surgical problem. It is most commonly due to malignancy (60%) and diverticular disease (20%). Volvulus affecting the colon accounts for 5% of cases. Acute colonic pseudo-obstruction remains a potential differential diagnosis in all cases. Intussusception affecting the colon (most often due to tumours in the adult population) remains a rare but recognised cause.
The typical patient will present with gradual onset of progressive abdominal distension, colicky abdominal pain and either obstipation or absolute constipation.
On examination abdominal distension is present, the presence of caecal tenderness (assuming no overt evidence of peritonitis) is a useful sign to elicit. A digital rectal examination and rigid sigmoidoscopy should be performed.
A plain abdominal x-ray is the usual first line test and; the caecal diameter and ileocaecal valve competency should be assessed on this film.

Imaging modalities
Debate long surrounds the use of CT versus gastrograffin enemas. The latter investigation has always been the traditional method of determining whether a structural lesion is indeed present. However, in the UK the use of this technique has declined and in most units a CT scan will be offered as the first line investigation by the majority of radiologists (and is advocated by the ACPGBI). In most cases this will provide sufficient detail to allow operative planning, and since malignancy accounts for most presentations may also stage the disease. In the event that the radiologist cannot provide a clear statement of lesion site, the surgeon should have no hesitation in requesting a contrast enema.

Surgical options
The decision as to when to operate or not is determined firstly by the patients physiological status. Unstable patients require resuscitation prior to surgery and admission to a critical care unit for invasive monitoring and potential inotropic support may be needed. In patients who are otherwise stable the decision then rests on the radiological and clinical findings. As a general rule the old adage that the sun should not rise and set on unrelieved large bowel obstruction still holds true. A caecal diameter of 12cm or more in the presence of complete obstruction with a competent ileocaecal valve and caecal tenderness is a sign of impending perforation and a relative indication for prompt surgery.

Right sided and transverse lesions
Right sided lesions producing large bowel obstruction should generally be treated by right hemicolectomy or its extended variant if the lesion lies in the distal transverse colon or splenic flexure. In these cases an ileocolic anastomosis may be easily constructed and even in the emergency setting has a low risk of anastomotic leak.

Left sided lesions
The options here lie between sub total colectomy and anastomosis, left hemicolectomy with on table lavage and primary anastomosis, left hemicolectomy and end colostomy formation and finally colonic stent insertion.
The usefulness of colonic stents was the subject of a Cochrane review in 2011. The authors concluded that on the basis of the data that they reviewed, there was no benefit from the use of colonic stents over conventional surgical resection with a tendency to better outcomes seen in the surgical group (1). A more recently conducted meta analysis met with the same conclusion (2). However, the recently concluded CREST trial has suggested that self expanding metallic stents can improve outcomes and avoids a stoma.

Rectosigmoid lesions
Lesions below the peritoneal reflection that are causing obstruction should generally be treated with a loop colostomy. Primary resection of unstaged rectal cancer would most likely carry a high CRM positivity rate and cannot be condoned. Where the lesion occupies the distal sigmoid colon the usual practice would be to perform a high anterior resection. The decision surrounding restoration of intestinal continuity would lie with the operating surgeon.

196
Q

A 72 year old female is found to have a malignant lesion in her left arm. She had a mastectomy of the left breast 10 years ago and has chronic lymph oedema of the left arm. What is the most likely cause of the malignancy?

	Lymphangiosarcoma
	Lymphoma
	Myeloma
	Angiomyolipoma
	Giant cell tumour
A

Lymphangiosarcoma is a rare condition arising as a result of chronic oedema. It is an aggressive malignancy.

Lymphoedema

Due to impaired lymphatic drainage in the presence of normal capillary function.
Lymphoedema causes the accumulation of protein rich fluid, subdermal fibrosis and dermal thickening.
Characteristically fluid is confined to the epifascial space (skin and subcutaneous tissues); muscle compartments are free of oedema. It involves the foot, unlike other forms of oedema. There may be a ‘buffalo hump’ on the dorsum of the foot and the skin cannot be pinched due to subcutaneous fibrosis.

Causes of lymphoedema

Primary
Congenital < 1 year: sporadic, Milroy’s disease
Onset 1-35 years: sporadic, Meige’s disease
> 35 years: Tarda
Secondary
Bacterial/fungal/parasitic infection (filariasis)
Lymphatic malignancy
Radiotherapy to lymph nodes
Surgical resection of lymph nodes
DVT
Thrombophlebitis

Indications for surgery
Marked disability or deformity from limb swelling
Lymphoedema caused by proximal lymphatic obstruction with patent distal lymphatics suitable for a lymphatic drainage procedure
Lymphocutaneous fistulae and megalymphatics

Procedures
Homans operation Reduction procedure with preservation of overlying skin (which must be in good condition). Skin flaps are raised and the underlying tissue excised. Limb circumference typically reduced by a third.
Charles operation All skin and subcutaneous tissue around the calf are excised down to the deep fascia. Split skin grafts are placed over the site. May be performed if overlying skin is not in good condition. Larger reduction in size than with Homans procedure.
Lymphovenous anastamosis Identifiable lymphatics are anastomosed to sub dermal venules. Usually indicated in 2% of patients with proximal lymphatic obstruction and normal distal lymphatics.

197
Q

A 45 year old lady undergoes a renal transplant from a living related donor. She is well for several months but on review in the outpatient department is noted to have persistent hypertension and a slight deterioration in renal function.

A.	Ureteric anastomotic leak
B.	Renal vein thrombosis
C.	Acute rejection
D.	Chronic allograft nephropathy
E.	Renal artery thrombosis
F.	Renal artery stenosis
G.	Lymphocele
H.	Hyperacute rejection
A

Renal artery stenosis

Renal artery stenosis typically occurs over several months and will usually result in the development of hypertension. Most cases can be assessed using duplex scanning and managed with angioplasty.

Complications of renal transplantation

A number of complications may occur following renal transplantation. A critical aspect of post operative care is evaluation of graft function. Post operatively, urine output is the most readily available, and easily measured, indicator of graft function. If an individual was relatively anuric pre-transplant and has a good urine output following surgery then this is more useful than it would be in someone who had a higher volume diuresis prior to transplantation. Recipients can be divided into three main groups following renal transplantation, with regard to their graft function:
Immediate graft function; brisk diuresis and falling serum creatinine
Slow graft function; modest urine output and slowly falling creatinine levels
Delayed graft function; defined as need for dialysis post transplant

Decreased urine output following surgery can be the result of hypovolvaemia or a blocked catheter (commonest causes). Other important causes include rejection, or a vascular complication.

Vascular complications
These may involve the donor vessels, those of the recipient or both. Renal artery thrombosis usually occurs early post transplant, but is uncommon with an incidence of less than 1%. It typically results in graft loss. It usually occurs as a result of a technical problem such a vessel torsion or sub intimal flaps. The usual presenting feature is a sudden cessation of urine output. When suspected, the occlusion is usually well demonstrated with duplex scanning. Ideally immediate surgical re-exploration should occur. Sadly, the graft has usually been lost by this stage and will require graft nephrectomy. Renal vein thrombosis is not as common as arterial graft thrombosis and the usual presenting features include discomfort at the graft site and swelling of the graft associated with loss of urine output. Again, duplex scanning is indicated. Unfortunately, this complication is also associated with a high incidence of graft loss.
Over a longer time frame (typically months) some individuals will develop renal artery stenosis. These individuals will typically develop hypertension and over time graft function will decline as hypertensive nephropathy occurs. It is usually demonstrated by duplex scanning and is usually amenable to endovascular intervention.

Urological complications
Urinary tract complications manifesting as leakage or obstruction are common complications following renal transplantation and occur in up to 10% of patients. The main underlying cause is the relatively poor blood supply to the transplanted ureter. Patients typically present relatively early in the first 5 weeks following transplantation with pain and swelling at the graft site. Imaging with USS is often the initial test. Therapeutic options include surgical re-implantation of the ureter for large leaks and stent insertion and nephrostomy placement for smaller leaks.

Lymphocele
These do not generally occur until 2 weeks or longer after surgery. They are, however, relatively common and may be seen in up to 18% of patients. Symptoms usually occur as a result of mass effect with compression of adjacent structures. These include the vessels supplying both the graft, with deterioration in graft function, the ureter, with alteration in urine output and the recipients lower limb vessels, with development of leg swelling. Creation of a laparoscopic or open peritoneal window is a favored treatment.

Rejection
Four types of graft rejection are recognised; hyperacute, accelerated acute, acute and chronic.

Type of rejection Key features
Hyperacute Occurs within minutes of clamp release
Due to pre formed antibodies
Immediate loss of graft occurs
Accelerated acute Occurs in first few days following surgery
Involved both cellular and antibody mediated injury
Pre-sensitisation of the donor is a common cause
Acute Traditionally the most common type of rejection
Seen days to weeks after surgery
Predominantly a cell mediated process mediated by lymphocytes
Organ biopsy demonstrates cellular infiltrates and graft cell apoptosis
Chronic Increasingly common problem
Typically; graft atrophy and atherosclerosis are seen. Fibrosis often occurs as a late event

198
Q

A 43 year old lady undergoes a live related renal transplant. At the conclusion of the operation she has a good urine output and the graft appeared well perfused. On the ward she suddenly becomes anuric.

A.	Ureteric anastomotic leak
B.	Renal vein thrombosis
C.	Acute rejection
D.	Chronic allograft nephropathy
E.	Renal artery thrombosis
F.	Renal artery stenosis
G.	Lymphocele
H.	Hyperacute rejection
A

Renal artery thrombosis

Sudden loss of urine output is most commonly due to a blocked catheter. However, if this is excluded (and is not included in the options) the most worrisome cause is arterial thrombosis. This will often be a delayed diagnosis and the rate of graft loss is high.

Complications of renal transplantation

A number of complications may occur following renal transplantation. A critical aspect of post operative care is evaluation of graft function. Post operatively, urine output is the most readily available, and easily measured, indicator of graft function. If an individual was relatively anuric pre-transplant and has a good urine output following surgery then this is more useful than it would be in someone who had a higher volume diuresis prior to transplantation. Recipients can be divided into three main groups following renal transplantation, with regard to their graft function:
Immediate graft function; brisk diuresis and falling serum creatinine
Slow graft function; modest urine output and slowly falling creatinine levels
Delayed graft function; defined as need for dialysis post transplant

Decreased urine output following surgery can be the result of hypovolvaemia or a blocked catheter (commonest causes). Other important causes include rejection, or a vascular complication.

Vascular complications
These may involve the donor vessels, those of the recipient or both. Renal artery thrombosis usually occurs early post transplant, but is uncommon with an incidence of less than 1%. It typically results in graft loss. It usually occurs as a result of a technical problem such a vessel torsion or sub intimal flaps. The usual presenting feature is a sudden cessation of urine output. When suspected, the occlusion is usually well demonstrated with duplex scanning. Ideally immediate surgical re-exploration should occur. Sadly, the graft has usually been lost by this stage and will require graft nephrectomy. Renal vein thrombosis is not as common as arterial graft thrombosis and the usual presenting features include discomfort at the graft site and swelling of the graft associated with loss of urine output. Again, duplex scanning is indicated. Unfortunately, this complication is also associated with a high incidence of graft loss.
Over a longer time frame (typically months) some individuals will develop renal artery stenosis. These individuals will typically develop hypertension and over time graft function will decline as hypertensive nephropathy occurs. It is usually demonstrated by duplex scanning and is usually amenable to endovascular intervention.

Urological complications
Urinary tract complications manifesting as leakage or obstruction are common complications following renal transplantation and occur in up to 10% of patients. The main underlying cause is the relatively poor blood supply to the transplanted ureter. Patients typically present relatively early in the first 5 weeks following transplantation with pain and swelling at the graft site. Imaging with USS is often the initial test. Therapeutic options include surgical re-implantation of the ureter for large leaks and stent insertion and nephrostomy placement for smaller leaks.

Lymphocele
These do not generally occur until 2 weeks or longer after surgery. They are, however, relatively common and may be seen in up to 18% of patients. Symptoms usually occur as a result of mass effect with compression of adjacent structures. These include the vessels supplying both the graft, with deterioration in graft function, the ureter, with alteration in urine output and the recipients lower limb vessels, with development of leg swelling. Creation of a laparoscopic or open peritoneal window is a favored treatment.

Rejection
Four types of graft rejection are recognised; hyperacute, accelerated acute, acute and chronic.

Type of rejection Key features
Hyperacute Occurs within minutes of clamp release
Due to pre formed antibodies
Immediate loss of graft occurs
Accelerated acute Occurs in first few days following surgery
Involved both cellular and antibody mediated injury
Pre-sensitisation of the donor is a common cause
Acute Traditionally the most common type of rejection
Seen days to weeks after surgery
Predominantly a cell mediated process mediated by lymphocytes
Organ biopsy demonstrates cellular infiltrates and graft cell apoptosis
Chronic Increasingly common problem
Typically; graft atrophy and atherosclerosis are seen. Fibrosis often occurs as a late event

199
Q

A 39 year old lady undergoes a live related renal transplant. She progresses well. Two weeks following the transplant she is noted to have swelling overlying the transplant site and swelling of the ipsilateral limb.Urine output is acceptable and creatinine unchanged.

A.	Ureteric anastomotic leak
B.	Renal vein thrombosis
C.	Acute rejection
D.	Chronic allograft nephropathy
E.	Renal artery thrombosis
F.	Renal artery stenosis
G.	Lymphocele
H.	Hyperacute rejection
A

Lymphocele

Swelling over the graft site is often due to a lymphocele and this is further suggested by the normal renal function. They cause symptoms through mass effect and limb swelling may occur. Treatment is often surgical.

Complications of renal transplantation

A number of complications may occur following renal transplantation. A critical aspect of post operative care is evaluation of graft function. Post operatively, urine output is the most readily available, and easily measured, indicator of graft function. If an individual was relatively anuric pre-transplant and has a good urine output following surgery then this is more useful than it would be in someone who had a higher volume diuresis prior to transplantation. Recipients can be divided into three main groups following renal transplantation, with regard to their graft function:
Immediate graft function; brisk diuresis and falling serum creatinine
Slow graft function; modest urine output and slowly falling creatinine levels
Delayed graft function; defined as need for dialysis post transplant

Decreased urine output following surgery can be the result of hypovolvaemia or a blocked catheter (commonest causes). Other important causes include rejection, or a vascular complication.

Vascular complications
These may involve the donor vessels, those of the recipient or both. Renal artery thrombosis usually occurs early post transplant, but is uncommon with an incidence of less than 1%. It typically results in graft loss. It usually occurs as a result of a technical problem such a vessel torsion or sub intimal flaps. The usual presenting feature is a sudden cessation of urine output. When suspected, the occlusion is usually well demonstrated with duplex scanning. Ideally immediate surgical re-exploration should occur. Sadly, the graft has usually been lost by this stage and will require graft nephrectomy. Renal vein thrombosis is not as common as arterial graft thrombosis and the usual presenting features include discomfort at the graft site and swelling of the graft associated with loss of urine output. Again, duplex scanning is indicated. Unfortunately, this complication is also associated with a high incidence of graft loss.
Over a longer time frame (typically months) some individuals will develop renal artery stenosis. These individuals will typically develop hypertension and over time graft function will decline as hypertensive nephropathy occurs. It is usually demonstrated by duplex scanning and is usually amenable to endovascular intervention.

Urological complications
Urinary tract complications manifesting as leakage or obstruction are common complications following renal transplantation and occur in up to 10% of patients. The main underlying cause is the relatively poor blood supply to the transplanted ureter. Patients typically present relatively early in the first 5 weeks following transplantation with pain and swelling at the graft site. Imaging with USS is often the initial test. Therapeutic options include surgical re-implantation of the ureter for large leaks and stent insertion and nephrostomy placement for smaller leaks.

Lymphocele
These do not generally occur until 2 weeks or longer after surgery. They are, however, relatively common and may be seen in up to 18% of patients. Symptoms usually occur as a result of mass effect with compression of adjacent structures. These include the vessels supplying both the graft, with deterioration in graft function, the ureter, with alteration in urine output and the recipients lower limb vessels, with development of leg swelling. Creation of a laparoscopic or open peritoneal window is a favored treatment.

Rejection
Four types of graft rejection are recognised; hyperacute, accelerated acute, acute and chronic.

Type of rejection Key features
Hyperacute Occurs within minutes of clamp release
Due to pre formed antibodies
Immediate loss of graft occurs
Accelerated acute Occurs in first few days following surgery
Involved both cellular and antibody mediated injury
Pre-sensitisation of the donor is a common cause
Acute Traditionally the most common type of rejection
Seen days to weeks after surgery
Predominantly a cell mediated process mediated by lymphocytes
Organ biopsy demonstrates cellular infiltrates and graft cell apoptosis
Chronic Increasingly common problem
Typically; graft atrophy and atherosclerosis are seen. Fibrosis often occurs as a late event

200
Q

A 58 year old lady has a two year history of worsening dysphagia. In addition to this she has a nocturnal cough. Over this time she has lost a total of 8kg in weight. Which of the processes below is the most likely explanation for this?

Gastro-oesophageal reflux disease
Barretts oesophagus
Carcinoma
Mallory Weiss tear
A

Such marked weight loss should arouse suspicion of cancer. She is most likely to have a longstanding stricture associated with oesophagitis and Barretts oesophagus. This may progress to carcinoma which will tend to occur in association with worsening dysphagia and weight loss.

Oesophageal cancer

Incidence is increasing
In most cases in the Western world this increase is accounted for by a rise in the number of cases of adenocarcinoma. In the UK adenocarcinomas account for 65% of cases.
Barretts oesophagus is a major risk factor for most cases of oesophageal adenocarcinoma.
In other regions of the world squamous cancer is more common and is linked to smoking, alcohol intake, diets rich in nitrosamines and achalasia.
Surveillance of Barretts is important, as it imparts a 30 fold increase in cancer risk and if invasive malignancy is diagnosed early then survival may approach 85% at 5 years.

Diagnosis
Upper GI endoscopy is the first line test
Contrast swallow may be of benefit in classifying benign motility disorders but has no place in the assessment of tumours
Staging is initially undertaken with CT scanning of the chest, abdomen and pelvis. If overt metastatic disease is identified using this modality then further complex imaging is unnecessary
If CT does not show metastatic disease, then local stage may be more accurately assessed by use of endoscopic ultrasound.
Staging laparoscopy is performed to detect occult peritoneal disease. PET CT is performed in those with negative laparoscopy. Thoracoscopy is not routinely performed.

Treatment
Operable disease is best managed by surgical resection. The most standard procedure is an Ivor- Lewis type oesophagectomy. This procedure involves the mobilisation of the stomach and division of the oesophageal hiatus. The abdomen is closed and a right sided thoracotomy performed. The stomach is brought into the chest and the oesophagus mobilised further. An intrathoracic oesophagogastric anastomosis is constructed. Alternative surgical strategies include a transhiatal resection (for distal lesions), a left thoraco-abdominal resection (difficult access due to thoracic aorta) and a total oesophagectomy (McKeown) with a cervical oesophagogastric anastomosis.
The biggest surgical challenge is that of anastomotic leak, with an intrathoracic anastomosis this will result in mediastinitis. With high mortality. The McKeown technique has an intrinsically lower systemic insult in the event of anastomotic leakage.

In addition to surgical resection many patients will be treated with adjuvant chemotherapy.

201
Q

A 56 year old man is investigated with an abdominal CT scan for a change of bowel habit towards constipation. It shows no colonic lesions. However, a right sided adrenal lesion is noted and measures 2.5cm in diameter. What is the most appropriate course of action?

	Arrange an MRI of the adrenal gland
	Arrange an adrenal USS
	Arrange an image guided core biopsy
	Undertake an adrenalectomy
	Arrange a hormonal assay
A

Arrange a hormonal assay

The vast majority of small adrenal lesions are incidental, benign and non functioning adenomas. Apart from minimal workup, no further investigation is needed. Of note, if there are concerns about malignancy, the only surgical option is adrenalectomy.

Adrenal lesions- Incidental

Incidentaloma of the adrenal glands have become increasingly common as CT scanning of the abdomen is widely undertaken. Prevalences range from 1.5-9% in autopsy studies. Overall, 75% will be non functioning adenomas. However, a thorough diagnostic work up is required to exclude a more significant lesion.

Investigation
Morning and midnight plasma cortisol measurements
Dexamethasone suppression test
24 hour urinary cortisol excretion
24 hour urinary excretion of catecholamines
Serum potassium, aldosterone and renin levels

Management
The risk of malignancy is related to the size of the lesion and 25% of all masses greater than 4cm will be malignant. Such lesions should usually be excised. Where a lesion is a suspected metastatic deposit a biopsy may be considered.

202
Q

A 63 year old man is admitted with obstructive jaundice that has developed over the past 3 weeks. He was previously well and on examination has a smooth mass in his right upper quadrant.

A.	Carcinoma of the head of the pancreas
B.	Bile duct stricture
C.	Mirizzi syndrome
D.	Bile duct stones
E.	Chronic cholecystitis
F.	Peri hilar lymphadenopathy
G.	Fitz - Hugh Curtis syndrome
A

Carcinoma of the head of the pancreas

Carcinoma of the pancreas (Courvoisiers law!). The development of jaundice in association with a smooth right upper quadrant mass is typical of distal biliary obstruction secondary to pancreatic malignancy. A bile duct stricture would not present in this way, all the other choices are related to gallstones and Fitz Hugh Curtis syndrome is a complication of pelvic inflammatory disease.

Courvoisiers Law:
Obstructive jaundice in the presence of a palpable gallbladder is unlikely to be due to stones.
This is due to the fibrotic effect that stones have on the gallbladder. Like all these laws there are numerous exceptions and many cases will not present in the typical manner.

Bile duct injury
Inadvertent bile duct injury during laparoscopic surgery should be referred to a specialist hepatobiliary surgeon. Outcomes are far worse when repair is undertaken by a non specialist surgeon in a district hospital.

Gallstones Typically history of biliary colic or episodes of chlolecystitis. Obstructive type history and test results. Usually small calibre gallstones which can pass through the cystic duct. In Mirizzi syndrome the stone may compress the bile duct directly- one of the rare times that cholecystitis may present with jaundice
Cholangitis Usually obstructive and will have Charcots triad of symptoms (pain, fever, jaundice) Ascending infection of the bile ducts usually by E. coli and by definition occurring in a pool of stagnant bile.
Pancreatic cancer Typically painless jaundice with palpable gallbladder (Courvoisier’s Law) Direct occlusion of distal bile duct or pancreatic duct by tumour. Sometimes nodal disease at the portal hepatis may be the culprit in which case the bile duct may be of normal calibre.
TPN associated jaundice Usually follows long term use and is usually painless with non obstructive features Often due to hepatic dysfunction and fatty liver which may occur with long term TPN usage.
Bile duct injury Depending upon the type of injury may be of sudden or gradual onset and is usually of obstructive type Often due to a difficult cholecystectomy when anatomy in Calots triangle is not appreciated. In the worst scenario the bile duct is excised and jaundice offers rapidly post operatively. More insidious is that of bile duct stenosis which may be caused by clips or diathermy injury.
Cholangiocarcinoma Gradual onset obstructive pattern Direct occlusion by disease and also extrinsic compression by nodal disease at the porta hepatis.
Septic surgical patient Usually hepatic features Combination of impaired biliary excretion and drugs such as ciprofloxacin which may cause cholestasis.
Metastatic disease Mixed hepatic and post hepatic Combination of liver synthetic failure (late) and extrinsic compression by nodal disease and anatomical compression of intra hepatic structures (earlier)

Diagnosis
An ultrasound of the liver and biliary tree is the most commonly used first line test. This will establish bile duct calibre, often ascertain the presence of gallstones, may visualise pancreatic masses and other lesions. The most important clinical question is essentially the extent of biliary dilatation and its distribution.

Where pancreatic neoplasia is suspected, the next test should be a pancreatic protocol CT scan. With liver tumours and cholangiocarcinoma an MRI/ MRCP is often the preferred option. PET scans may be used to stage a number of malignancies but do not routinely form part of first line testing.

Where MRCP fails to give adequate information an ERCP may be necessary. In many cases this may form part of patient management. It is however, invasive and certainly not without risk and highly operator dependent.

Management
Clearly this will depend to an extent upon the underlying cause but relief of jaundice is important, even if surgery forms part of the planned treatment. Patients with unrelieved jaundice have a much higher incidence of septic complications, bleeding and death.

Screen for and address any clotting irregularities

In patients with malignancy a stent will need to be inserted. These come in two main types; metal and plastic. Plastic stents are cheap and easy to replace and should be used if any surgical intervention (e.g. Whipples) is planned. However, they are prone to displacement and blockage. Metal stents are much more expensive and may compromise a surgical resection. However, they are far less prone to displacement and to a lesser extent blockage than their plastic counterparts.

If malignancy is in bile duct/ pancreatic head and stenting has been attempted and has failed, then an alternative strategy is to drain the biliary system percutaneously via a transhepatic route. It may also be possible to insert a stent in this way. One of the main problems with temporary PTC’s is their propensity to displacement, which may result in a bile leak.

In patients who have a bile duct injury surgery will be required to repair the defect. If the bile duct has been inadvertently excised then a hepatico-jejunostomy will need to be created (difficult!)

If gallstones are the culprit, then these may be removed by ERCP and a cholecystectomy performed. Where there is doubt about the efficacy of the ERCP an operative cholangiogram should be performed and bile duct exploration undertaken where stones remain. When the bile duct has been formally opened the options are between closure over a T tube, a choledochoduodenostomy or choledochojejunostomy.

Patients with cholangitis should receive high dose broad spectrum antibiotics via the intravenous route. Biliary decompression should follow soon afterwards, instrumenting the bile duct of these patients will often provoke a septic episode (but should be done anyway).

203
Q

A 41 year old lady is admitted with colicky right upper quadrant pain. On clinical examination she has a mild pyrexia and is clinically jaundiced. An ultrasound scan is reported as showing gallstones and the patient is taken to theatre for an open cholecystectomy. At operation, Calots triangle is almost completely impossible to delineate.

A.	Carcinoma of the head of the pancreas
B.	Bile duct stricture
C.	Mirizzi syndrome
D.	Bile duct stones
E.	Chronic cholecystitis
F.	Peri hilar lymphadenopathy
G.	Fitz - Hugh Curtis syndrome
A

Mirizzi syndrome

In Mirizzi syndrome the gallstone becomes impacted in Hartmans pouch. Episodes of recurrent inflammation occur and this causes compression of the bile duct. In severe cases this then progresses to fistulation. Surgery is extremely difficult as Calots triangle is often completely obliterated and the risks of causing injury to the CBD are high.

Courvoisiers Law:
Obstructive jaundice in the presence of a palpable gallbladder is unlikely to be due to stones.
This is due to the fibrotic effect that stones have on the gallbladder. Like all these laws there are numerous exceptions and many cases will not present in the typical manner.

Bile duct injury
Inadvertent bile duct injury during laparoscopic surgery should be referred to a specialist hepatobiliary surgeon. Outcomes are far worse when repair is undertaken by a non specialist surgeon in a district hospital.

Gallstones Typically history of biliary colic or episodes of chlolecystitis. Obstructive type history and test results. Usually small calibre gallstones which can pass through the cystic duct. In Mirizzi syndrome the stone may compress the bile duct directly- one of the rare times that cholecystitis may present with jaundice
Cholangitis Usually obstructive and will have Charcots triad of symptoms (pain, fever, jaundice) Ascending infection of the bile ducts usually by E. coli and by definition occurring in a pool of stagnant bile.
Pancreatic cancer Typically painless jaundice with palpable gallbladder (Courvoisier’s Law) Direct occlusion of distal bile duct or pancreatic duct by tumour. Sometimes nodal disease at the portal hepatis may be the culprit in which case the bile duct may be of normal calibre.
TPN associated jaundice Usually follows long term use and is usually painless with non obstructive features Often due to hepatic dysfunction and fatty liver which may occur with long term TPN usage.
Bile duct injury Depending upon the type of injury may be of sudden or gradual onset and is usually of obstructive type Often due to a difficult cholecystectomy when anatomy in Calots triangle is not appreciated. In the worst scenario the bile duct is excised and jaundice offers rapidly post operatively. More insidious is that of bile duct stenosis which may be caused by clips or diathermy injury.
Cholangiocarcinoma Gradual onset obstructive pattern Direct occlusion by disease and also extrinsic compression by nodal disease at the porta hepatis.
Septic surgical patient Usually hepatic features Combination of impaired biliary excretion and drugs such as ciprofloxacin which may cause cholestasis.
Metastatic disease Mixed hepatic and post hepatic Combination of liver synthetic failure (late) and extrinsic compression by nodal disease and anatomical compression of intra hepatic structures (earlier)

Diagnosis
An ultrasound of the liver and biliary tree is the most commonly used first line test. This will establish bile duct calibre, often ascertain the presence of gallstones, may visualise pancreatic masses and other lesions. The most important clinical question is essentially the extent of biliary dilatation and its distribution.

Where pancreatic neoplasia is suspected, the next test should be a pancreatic protocol CT scan. With liver tumours and cholangiocarcinoma an MRI/ MRCP is often the preferred option. PET scans may be used to stage a number of malignancies but do not routinely form part of first line testing.

Where MRCP fails to give adequate information an ERCP may be necessary. In many cases this may form part of patient management. It is however, invasive and certainly not without risk and highly operator dependent.

Management
Clearly this will depend to an extent upon the underlying cause but relief of jaundice is important, even if surgery forms part of the planned treatment. Patients with unrelieved jaundice have a much higher incidence of septic complications, bleeding and death.

Screen for and address any clotting irregularities

In patients with malignancy a stent will need to be inserted. These come in two main types; metal and plastic. Plastic stents are cheap and easy to replace and should be used if any surgical intervention (e.g. Whipples) is planned. However, they are prone to displacement and blockage. Metal stents are much more expensive and may compromise a surgical resection. However, they are far less prone to displacement and to a lesser extent blockage than their plastic counterparts.

If malignancy is in bile duct/ pancreatic head and stenting has been attempted and has failed, then an alternative strategy is to drain the biliary system percutaneously via a transhepatic route. It may also be possible to insert a stent in this way. One of the main problems with temporary PTC’s is their propensity to displacement, which may result in a bile leak.

In patients who have a bile duct injury surgery will be required to repair the defect. If the bile duct has been inadvertently excised then a hepatico-jejunostomy will need to be created (difficult!)

If gallstones are the culprit, then these may be removed by ERCP and a cholecystectomy performed. Where there is doubt about the efficacy of the ERCP an operative cholangiogram should be performed and bile duct exploration undertaken where stones remain. When the bile duct has been formally opened the options are between closure over a T tube, a choledochoduodenostomy or choledochojejunostomy.

Patients with cholangitis should receive high dose broad spectrum antibiotics via the intravenous route. Biliary decompression should follow soon afterwards, instrumenting the bile duct of these patients will often provoke a septic episode (but should be done anyway).

204
Q

A 72 year old man undergoes a distal gastrectomy for carcinoma of the stomach. He presents with jaundice approximately 8 months post operatively. Ultrasound of the liver and bile ducts shows no focal liver lesion and normal calibre common bile duct with intra hepatic duct dilatation.

A.	Carcinoma of the head of the pancreas
B.	Bile duct stricture
C.	Mirizzi syndrome
D.	Bile duct stones
E.	Chronic cholecystitis
F.	Peri hilar lymphadenopathy
G.	Fitz - Hugh Curtis syndrome
A

Peri hilar lymphadenopathy

Unfortunately metastatic disease is the most likely event. Peri hilar lymphadenopathy would be a common culprit.

Courvoisiers Law:
Obstructive jaundice in the presence of a palpable gallbladder is unlikely to be due to stones.
This is due to the fibrotic effect that stones have on the gallbladder. Like all these laws there are numerous exceptions and many cases will not present in the typical manner.

Bile duct injury
Inadvertent bile duct injury during laparoscopic surgery should be referred to a specialist hepatobiliary surgeon. Outcomes are far worse when repair is undertaken by a non specialist surgeon in a district hospital.

Gallstones Typically history of biliary colic or episodes of chlolecystitis. Obstructive type history and test results. Usually small calibre gallstones which can pass through the cystic duct. In Mirizzi syndrome the stone may compress the bile duct directly- one of the rare times that cholecystitis may present with jaundice
Cholangitis Usually obstructive and will have Charcots triad of symptoms (pain, fever, jaundice) Ascending infection of the bile ducts usually by E. coli and by definition occurring in a pool of stagnant bile.
Pancreatic cancer Typically painless jaundice with palpable gallbladder (Courvoisier’s Law) Direct occlusion of distal bile duct or pancreatic duct by tumour. Sometimes nodal disease at the portal hepatis may be the culprit in which case the bile duct may be of normal calibre.
TPN associated jaundice Usually follows long term use and is usually painless with non obstructive features Often due to hepatic dysfunction and fatty liver which may occur with long term TPN usage.
Bile duct injury Depending upon the type of injury may be of sudden or gradual onset and is usually of obstructive type Often due to a difficult cholecystectomy when anatomy in Calots triangle is not appreciated. In the worst scenario the bile duct is excised and jaundice offers rapidly post operatively. More insidious is that of bile duct stenosis which may be caused by clips or diathermy injury.
Cholangiocarcinoma Gradual onset obstructive pattern Direct occlusion by disease and also extrinsic compression by nodal disease at the porta hepatis.
Septic surgical patient Usually hepatic features Combination of impaired biliary excretion and drugs such as ciprofloxacin which may cause cholestasis.
Metastatic disease Mixed hepatic and post hepatic Combination of liver synthetic failure (late) and extrinsic compression by nodal disease and anatomical compression of intra hepatic structures (earlier)

Diagnosis
An ultrasound of the liver and biliary tree is the most commonly used first line test. This will establish bile duct calibre, often ascertain the presence of gallstones, may visualise pancreatic masses and other lesions. The most important clinical question is essentially the extent of biliary dilatation and its distribution.

Where pancreatic neoplasia is suspected, the next test should be a pancreatic protocol CT scan. With liver tumours and cholangiocarcinoma an MRI/ MRCP is often the preferred option. PET scans may be used to stage a number of malignancies but do not routinely form part of first line testing.

Where MRCP fails to give adequate information an ERCP may be necessary. In many cases this may form part of patient management. It is however, invasive and certainly not without risk and highly operator dependent.

Management
Clearly this will depend to an extent upon the underlying cause but relief of jaundice is important, even if surgery forms part of the planned treatment. Patients with unrelieved jaundice have a much higher incidence of septic complications, bleeding and death.

Screen for and address any clotting irregularities

In patients with malignancy a stent will need to be inserted. These come in two main types; metal and plastic. Plastic stents are cheap and easy to replace and should be used if any surgical intervention (e.g. Whipples) is planned. However, they are prone to displacement and blockage. Metal stents are much more expensive and may compromise a surgical resection. However, they are far less prone to displacement and to a lesser extent blockage than their plastic counterparts.

If malignancy is in bile duct/ pancreatic head and stenting has been attempted and has failed, then an alternative strategy is to drain the biliary system percutaneously via a transhepatic route. It may also be possible to insert a stent in this way. One of the main problems with temporary PTC’s is their propensity to displacement, which may result in a bile leak.

In patients who have a bile duct injury surgery will be required to repair the defect. If the bile duct has been inadvertently excised then a hepatico-jejunostomy will need to be created (difficult!)

If gallstones are the culprit, then these may be removed by ERCP and a cholecystectomy performed. Where there is doubt about the efficacy of the ERCP an operative cholangiogram should be performed and bile duct exploration undertaken where stones remain. When the bile duct has been formally opened the options are between closure over a T tube, a choledochoduodenostomy or choledochojejunostomy.

Patients with cholangitis should receive high dose broad spectrum antibiotics via the intravenous route. Biliary decompression should follow soon afterwards, instrumenting the bile duct of these patients will often provoke a septic episode (but should be done anyway).

205
Q

A 70 year old lady presents with a number of skin lesions that she describes as unsightly. On examination she has a number of raised lesions with a greasy surface located over her trunk. Apart from having a greasy surface the lesions also seem to have scattered keratin plugs located within them.

A.	Basal cell carcinoma
B.	Dermatofibroma
C.	Pilar cyst
D.	Epidermoid cyst
E.	Spitz naevus
F.	Seborrhoeic keratosis
G.	Atypical naevus
H.	Capillary cavernous haemangioma
A

Seborrhoeic keratosis

Seborrhoeic keratosis may have a number of appearances. However, the scaly, thick, greasy surface with scattered keratin plugs makes this the most likely diagnosis.

Benign skin diseases

Seborrhoeic keratosis
Most commonly arise in patients over the age of 50 years, often idiopathic
Equal sex incidence and prevalence
Usually multiple lesions over face and trunk
Flat, raised, filiform and pedunculated subtypes are recognised
Variable colours and surface may have greasy scale overlying it
Treatment options consist of leaving alone or simple shave excision

Melanocytic naevi
Congenital melanocytic naevi
Typically appear at, or soon after, birth
Usually greater than 1cm diameter
Increased risk of malignant transformation (increased risk greatest for large lesions)
Junctional melanocytic naevi
Circular macules
May have heterogeneous colour even within same lesion
Most naevi of the palms, soles and mucous membranes are of this type
Compound naevi
Domed pigmented nodules up to 1cm in diameter
Arise from junctional naevi, usually have uniform colour and are smooth
Spitz naevus
Usually develop over a few months in children
May be pink or red in colour, most common on face and legs
May grow up to 1cm and growth can be rapid, this usually results in excision
Atypical naevus syndrome
Atypical melanocytic naevi that may be autosomally dominantly inherited
Some individuals are at increased risk of melanoma (usually have mutations of CDKN2A gene)
Many people with atypical naevus syndrome AND a parent sibling with melanoma will develop melanoma

Epidermoid cysts
Common and affect face and trunk
They have a central punctum, they may contain small quantities of sebum
The cyst lining is either normal epidermis (epidermoid cyst) or outer root sheath of hair follicle (pilar cyst)

Dermatofibroma
Solitary dermal nodules
Usually affect extremities of young adults
Lesions feel larger than they appear visually
Histologically they consist of proliferating fibroblasts merging with sparsely cellular dermal tissues

Painful skin lesions
Eccrine spiradenoma
Neuroma
Glomus tumour
Leimyoma
Angiolipoma
Neurofibroma (rarely painful) and dermatofibroma (rarely painful)
206
Q

A 28 year old female presents with a small nodule located on the back of her neck. It is excised for cosmetic reasons and the histology report states that the lesion consists of a sebum filled lesion surrounded by the outer root sheath of a hair follicle.

A.	Basal cell carcinoma
B.	Dermatofibroma
C.	Pilar cyst
D.	Epidermoid cyst
E.	Spitz naevus
F.	Seborrhoeic keratosis
G.	Atypical naevus
H.	Capillary cavernous haemangioma
A

Pilar cyst

Pilar cysts may contain foul smelling cheesy material and are surrounded by the outer part of a hair follicle. Because of their histological appearances they are more correctly termed pilar cysts than sebaceous cysts.

Benign skin diseases

Seborrhoeic keratosis
Most commonly arise in patients over the age of 50 years, often idiopathic
Equal sex incidence and prevalence
Usually multiple lesions over face and trunk
Flat, raised, filiform and pedunculated subtypes are recognised
Variable colours and surface may have greasy scale overlying it
Treatment options consist of leaving alone or simple shave excision

Melanocytic naevi
Congenital melanocytic naevi
Typically appear at, or soon after, birth
Usually greater than 1cm diameter
Increased risk of malignant transformation (increased risk greatest for large lesions)
Junctional melanocytic naevi
Circular macules
May have heterogeneous colour even within same lesion
Most naevi of the palms, soles and mucous membranes are of this type
Compound naevi
Domed pigmented nodules up to 1cm in diameter
Arise from junctional naevi, usually have uniform colour and are smooth
Spitz naevus
Usually develop over a few months in children
May be pink or red in colour, most common on face and legs
May grow up to 1cm and growth can be rapid, this usually results in excision
Atypical naevus syndrome
Atypical melanocytic naevi that may be autosomally dominantly inherited
Some individuals are at increased risk of melanoma (usually have mutations of CDKN2A gene)
Many people with atypical naevus syndrome AND a parent sibling with melanoma will develop melanoma

Epidermoid cysts
Common and affect face and trunk
They have a central punctum, they may contain small quantities of sebum
The cyst lining is either normal epidermis (epidermoid cyst) or outer root sheath of hair follicle (pilar cyst)

Dermatofibroma
Solitary dermal nodules
Usually affect extremities of young adults
Lesions feel larger than they appear visually
Histologically they consist of proliferating fibroblasts merging with sparsely cellular dermal tissues

Painful skin lesions
Eccrine spiradenoma
Neuroma
Glomus tumour
Leimyoma
Angiolipoma
Neurofibroma (rarely painful) and dermatofibroma (rarely painful)
207
Q

A 21 year old lady presents with a nodule on the posterior aspect of her right calf. It has been present at the site for the past 6 months and occurred at the site of a previous insect bite. Although the nodule appears small, on palpation it appears to be nearly twice the size it appears on examination. The overlying skin is faintly pigmented.

A.	Basal cell carcinoma
B.	Dermatofibroma
C.	Pilar cyst
D.	Epidermoid cyst
E.	Spitz naevus
F.	Seborrhoeic keratosis
G.	Atypical naevus
H.	Capillary cavernous haemangioma
A

Dermatofibroma

Dermatofibromas may be pigmented and are often larger than they appear. They frequently occur at sites of previous trauma.

Benign skin diseases

Seborrhoeic keratosis
Most commonly arise in patients over the age of 50 years, often idiopathic
Equal sex incidence and prevalence
Usually multiple lesions over face and trunk
Flat, raised, filiform and pedunculated subtypes are recognised
Variable colours and surface may have greasy scale overlying it
Treatment options consist of leaving alone or simple shave excision

Melanocytic naevi
Congenital melanocytic naevi
Typically appear at, or soon after, birth
Usually greater than 1cm diameter
Increased risk of malignant transformation (increased risk greatest for large lesions)
Junctional melanocytic naevi
Circular macules
May have heterogeneous colour even within same lesion
Most naevi of the palms, soles and mucous membranes are of this type
Compound naevi
Domed pigmented nodules up to 1cm in diameter
Arise from junctional naevi, usually have uniform colour and are smooth
Spitz naevus
Usually develop over a few months in children
May be pink or red in colour, most common on face and legs
May grow up to 1cm and growth can be rapid, this usually results in excision
Atypical naevus syndrome
Atypical melanocytic naevi that may be autosomally dominantly inherited
Some individuals are at increased risk of melanoma (usually have mutations of CDKN2A gene)
Many people with atypical naevus syndrome AND a parent sibling with melanoma will develop melanoma

Epidermoid cysts
Common and affect face and trunk
They have a central punctum, they may contain small quantities of sebum
The cyst lining is either normal epidermis (epidermoid cyst) or outer root sheath of hair follicle (pilar cyst)

Dermatofibroma
Solitary dermal nodules
Usually affect extremities of young adults
Lesions feel larger than they appear visually
Histologically they consist of proliferating fibroblasts merging with sparsely cellular dermal tissues

Painful skin lesions
Eccrine spiradenoma
Neuroma
Glomus tumour
Leimyoma
Angiolipoma
Neurofibroma (rarely painful) and dermatofibroma (rarely painful)
208
Q

A 20 year old male notices a mild painful swelling of his right scrotum. He also complains of abdominal pain. Clinically the patient is found to have a swollen right testicle. Apart from a supraclavicular node, there is no obvious lymphadenopathy.

A.	Antibiotics
B.	Aspiration
C.	Testicular exploration after 6 hours
D.	Testicular exploration within 6 hours
E.	Orchidectomy via inguinal approach
F.	Orchidectomy via scrotal approach
G.	No treatment needed
A

Orchidectomy via inguinal approach

The patient is likely to have a teratoma which has metastasized to the supraclavicular nodes. There is suspicion of spread to the para-aortic nodes due to the abdominal pain. He will need orchidectomy and combination chemotherapy. There is no role for orchidectomy via scrotal approach in malignancy.

Inguinal hernia If inguinoscrotal swelling; cannot “get above it” on examination
Cough impulse may be present
May be reducible
Testicular tumours Often discrete testicular nodule (may have associated hydrocele)
Symptoms of metastatic disease may be present
USS scrotum and serum AFP and β HCG required
Acute epididymo-orchitis Often history of dysuria and urethral discharge
Swelling may be tender and eased by elevating testis
Most cases due to Chlamydia
Infections with other gram negative organisms may be associated with underlying structural abnormality
Epididymal cysts Single or multiple cysts
May contain clear or opalescent fluid (spermatoceles)
Usually occur over 40 years of age
Painless
Lie above and behind testis
It is usually possible to “get above the lump” on examination
Hydrocele Non painful, soft fluctuant swelling
Often possible to “get above it” on examination
Usually contain clear fluid
Will often transilluminate
May be presenting feature of testicular cancer in young men
Testicular torsion Severe, sudden onset testicular pain
Risk factors include abnormal testicular lie
Typically affects adolescents and young males
On examination testis is tender and pain not eased by elevation
Urgent surgery is indicated, the contra lateral testis should also be fixed
Varicocele Varicosities of the pampiniform plexus
Typically occur on left (because testicular vein drains into renal vein)
May be presenting feature of renal cell carcinoma
Affected testis may be smaller and bilateral varicoceles may affect fertility

Management
Testicular malignancy is always treated with orchidectomy via an inguinal approach. This allows high ligation of the testicular vessels and avoids exposure of another lymphatic field to the tumour.
Torsion is commonest in young teenagers and the history in older children can be difficult to elicit. Intermittent torsion is a recognised problem. The treatment is prompt surgical exploration and testicular fixation. This can be achieved using sutures or by placement of the testis in a Dartos pouch.
Varicoceles are usually managed conservatively. If there are concerns about testicular function of infertility then surgery or radiological management can be considered.
Epididymal cysts can be excised using a scrotal approach
Hydroceles are managed differently in children where the underlying pathology is a patent processus vaginalis and therefore an inguinal approach is used in children so that the processus can be ligated. In adults a scrotal approach is preferred and the hydrocele sac excised or plicated.

209
Q

A 40 year old male presents with a non painful, bilateral scrotal swellings over 3 years. The testis is felt separately and the swelling transilluminates.

 A.	Antibiotics
B.	Aspiration
C.	Testicular exploration after 6 hours
D.	Testicular exploration within 6 hours
E.	Orchidectomy via inguinal approach
F.	Orchidectomy via scrotal approach
G.	No treatment needed
A

No treatment needed

This is an epididymal cyst, the testis is palpated therefore this differentiates it from a hydrocele.

Inguinal hernia If inguinoscrotal swelling; cannot “get above it” on examination
Cough impulse may be present
May be reducible
Testicular tumours Often discrete testicular nodule (may have associated hydrocele)
Symptoms of metastatic disease may be present
USS scrotum and serum AFP and β HCG required
Acute epididymo-orchitis Often history of dysuria and urethral discharge
Swelling may be tender and eased by elevating testis
Most cases due to Chlamydia
Infections with other gram negative organisms may be associated with underlying structural abnormality
Epididymal cysts Single or multiple cysts
May contain clear or opalescent fluid (spermatoceles)
Usually occur over 40 years of age
Painless
Lie above and behind testis
It is usually possible to “get above the lump” on examination
Hydrocele Non painful, soft fluctuant swelling
Often possible to “get above it” on examination
Usually contain clear fluid
Will often transilluminate
May be presenting feature of testicular cancer in young men
Testicular torsion Severe, sudden onset testicular pain
Risk factors include abnormal testicular lie
Typically affects adolescents and young males
On examination testis is tender and pain not eased by elevation
Urgent surgery is indicated, the contra lateral testis should also be fixed
Varicocele Varicosities of the pampiniform plexus
Typically occur on left (because testicular vein drains into renal vein)
May be presenting feature of renal cell carcinoma
Affected testis may be smaller and bilateral varicoceles may affect fertility

Management
Testicular malignancy is always treated with orchidectomy via an inguinal approach. This allows high ligation of the testicular vessels and avoids exposure of another lymphatic field to the tumour.
Torsion is commonest in young teenagers and the history in older children can be difficult to elicit. Intermittent torsion is a recognised problem. The treatment is prompt surgical exploration and testicular fixation. This can be achieved using sutures or by placement of the testis in a Dartos pouch.
Varicoceles are usually managed conservatively. If there are concerns about testicular function of infertility then surgery or radiological management can be considered.
Epididymal cysts can be excised using a scrotal approach
Hydroceles are managed differently in children where the underlying pathology is a patent processus vaginalis and therefore an inguinal approach is used in children so that the processus can be ligated. In adults a scrotal approach is preferred and the hydrocele sac excised or plicated.

210
Q

A 32 year old male presents with a swollen, painful right scrotum after being kicked in the groin area. There is a painful swelling of the right scrotum and the underlying testis cannot be easily palpated.

A.	Antibiotics
B.	Aspiration
C.	Testicular exploration after 6 hours
D.	Testicular exploration within 6 hours
E.	Orchidectomy via inguinal approach
F.	Orchidectomy via scrotal approach
G.	No treatment needed
A

Testicular exploration within 6 hours

Acute haematocele: tense, tender and non transilluminating mass. The testis will need surgical exploration to evacuate the blood and repair any damage.

Inguinal hernia If inguinoscrotal swelling; cannot “get above it” on examination
Cough impulse may be present
May be reducible
Testicular tumours Often discrete testicular nodule (may have associated hydrocele)
Symptoms of metastatic disease may be present
USS scrotum and serum AFP and β HCG required
Acute epididymo-orchitis Often history of dysuria and urethral discharge
Swelling may be tender and eased by elevating testis
Most cases due to Chlamydia
Infections with other gram negative organisms may be associated with underlying structural abnormality
Epididymal cysts Single or multiple cysts
May contain clear or opalescent fluid (spermatoceles)
Usually occur over 40 years of age
Painless
Lie above and behind testis
It is usually possible to “get above the lump” on examination
Hydrocele Non painful, soft fluctuant swelling
Often possible to “get above it” on examination
Usually contain clear fluid
Will often transilluminate
May be presenting feature of testicular cancer in young men
Testicular torsion Severe, sudden onset testicular pain
Risk factors include abnormal testicular lie
Typically affects adolescents and young males
On examination testis is tender and pain not eased by elevation
Urgent surgery is indicated, the contra lateral testis should also be fixed
Varicocele Varicosities of the pampiniform plexus
Typically occur on left (because testicular vein drains into renal vein)
May be presenting feature of renal cell carcinoma
Affected testis may be smaller and bilateral varicoceles may affect fertility

Management
Testicular malignancy is always treated with orchidectomy via an inguinal approach. This allows high ligation of the testicular vessels and avoids exposure of another lymphatic field to the tumour.
Torsion is commonest in young teenagers and the history in older children can be difficult to elicit. Intermittent torsion is a recognised problem. The treatment is prompt surgical exploration and testicular fixation. This can be achieved using sutures or by placement of the testis in a Dartos pouch.
Varicoceles are usually managed conservatively. If there are concerns about testicular function of infertility then surgery or radiological management can be considered.
Epididymal cysts can be excised using a scrotal approach
Hydroceles are managed differently in children where the underlying pathology is a patent processus vaginalis and therefore an inguinal approach is used in children so that the processus can be ligated. In adults a scrotal approach is preferred and the hydrocele sac excised or plicated.

211
Q

An 8 year old boy is examined by his doctor as part of a routine clinical examination. The doctor notices a smooth swelling in the right iliac fossa. It is mobile and he is otherwise well. What is the most likely underlying cause?

	Meckels diverticulum
	Spigelian hernia
	Mesenteric cyst
	Appendix mass
	Liposarcoma
A

Mesenteric cysts are often smooth. Imaging with ultrasound and CT is usually sufficient. Although rare, they most often occur in young children (up to 30% present before the age of 15). Many are asymptomatic and discovered incidentally. Acute presentations are recognised and may occur following cyst torsion, infarction or rupture. Most cysts will be surgically resected.
Spigelian hernias are very rare in children, liposarcomas are not smooth swellings. An appendix mass will usually produce systemic illness.

Appendicitis	
Pain radiating to right iliac fossa
Anorexia (very common)
Short history
Diarrhoea and profuse vomiting rare

Crohn’s disease
Often long history
Signs of malnutrition
Change in bowel habit, especially diarrhoea

Mesenteric adenitis
Mainly affects children
Causes include Adenoviruses, Epstein Barr Virus, beta-haemolytic Streptococcus, Staphylococcus spp., Escherichia coli, Streptococcus viridans and Yersinia spp.
Patients have a higher temperature than those with appendicitis
If laparotomy is performed, enlarged mesenteric lymph nodes will be present

Diverticulitis
Both left and right sided disease may present with right iliac fossa pain
Clinical history may be similar, although some change in bowel habit is usual
When suspected, a CT scan may help in refining the diagnosis

Meckel’s diverticulitis
A Meckel’s diverticulum is a congenital abnormality that is present in about 2% of the population
Typically 2 feet proximal to the ileocaecal valve
May be lined by ectopic gastric mucosal tissue and produce bleeding

Perforated peptic ulcer
This usually produces upper quadrant pain but pain may be lower
Perforations typically have a sharp sudden onset of pain in the history

Incarcerated right inguinal or femoral hernia
Usually only right iliac fossa pain if right sided or bowel obstruction.
Bowel perforation secondary to caecal or colon carcinoma
Seldom localised to right iliac fossa, although complete large bowel obstruction with caecal distension may cause pain prior to perforation.

Gynaecological causes
Pelvic inflammatory disease/salpingitis/pelvic abscess/Ectopic pregnancy/Ovarian torsion/Threatened or complete abortion/Mittelschmerz

Urological causes
Ureteric colic/UTI/Testicular torsion
Other causes
TB/Typhoid/Herpes Zoster/AAA/Situs inversus

212
Q

A 21 year old women is admitted with a 48 hour history of worsening right iliac fossa pain. She has been nauseated and vomited twice. On examination, she is markedly tender in the right iliac fossa with localised guarding. Vaginal examination is unremarkable. Urine dipstick (including beta HCG) is negative. Blood tests show a WCC of 13.5 and CRP 70. What is the most appropriate course of action?

	Open appendicectomy
	Laparotomy
	Abdominal ultrasound
	Laparoscopic appendicectomy
	Abdominal CT scan
A

Laparoscopic appendicectomy

The most likely diagnosis is appendicitis. The negative vaginal examination (and therefore by definition the absence of cervical excitation) makes pelvic inflammatory disease unlikely. Given the raised inflammatory markers, the correct course of action is to proceed with surgery. In females, there are considerable advantages of undertaking this laparoscopically as it allows evaluation of the pelvic viscera. Imaging with USS is unlikely to alter management as it has a false negative rate and given the context of the clinical findings, surgery is likely to occur in any case. Whilst a CT scan would allow for an accurate pre-operative diagnosis, it carries a significant radiation dose, and again, is unlikely to alter management.

Appendicitis	
Pain radiating to right iliac fossa
Anorexia (very common)
Short history
Diarrhoea and profuse vomiting rare

Crohn’s disease
Often long history
Signs of malnutrition
Change in bowel habit, especially diarrhoea

Mesenteric adenitis
Mainly affects children
Causes include Adenoviruses, Epstein Barr Virus, beta-haemolytic Streptococcus, Staphylococcus spp., Escherichia coli, Streptococcus viridans and Yersinia spp.
Patients have a higher temperature than those with appendicitis
If laparotomy is performed, enlarged mesenteric lymph nodes will be present

Diverticulitis
Both left and right sided disease may present with right iliac fossa pain
Clinical history may be similar, although some change in bowel habit is usual
When suspected, a CT scan may help in refining the diagnosis

Meckel’s diverticulitis
A Meckel’s diverticulum is a congenital abnormality that is present in about 2% of the population
Typically 2 feet proximal to the ileocaecal valve
May be lined by ectopic gastric mucosal tissue and produce bleeding

Perforated peptic ulcer
This usually produces upper quadrant pain but pain may be lower
Perforations typically have a sharp sudden onset of pain in the history

Incarcerated right inguinal or femoral hernia
Usually only right iliac fossa pain if right sided or bowel obstruction.
Bowel perforation secondary to caecal or colon carcinoma
Seldom localised to right iliac fossa, although complete large bowel obstruction with caecal distension may cause pain prior to perforation.

Gynaecological causes
Pelvic inflammatory disease/salpingitis/pelvic abscess/Ectopic pregnancy/Ovarian torsion/Threatened or complete abortion/Mittelschmerz

Urological causes
Ureteric colic/UTI/Testicular torsion
Other causes
TB/Typhoid/Herpes Zoster/AAA/Situs inversus

213
Q

A 1-day-old baby girl is noted to become profoundly cyanotic whilst feeding and crying. A diagnosis of congenital heart disease is suspected. What is the most likely cause?

	Transposition of the great arteries
	Coarctation of the aorta
	Patent ductus arteriosus
	Hypoplastic left heart
	Ventricular septal defect
A

Congenital heart disease
Cyanotic: TGA most common at birth, Fallot’s most common overall
Acyanotic: VSD most common cause

It is important to be aware of common congenital cardiac abnormalities. The main differentiating factor is whether the patient is cyanotic or acyanotic. In the neonate, TGA is the most common condition presenting with profound cyanosis.

The other options are causes of acyanotic congenital heart disease

Congenital heart disease

Acyanotic - most common causes
Ventricular septal defects (VSD) - most common, accounts for 30%
Atrial septal defect (ASD)
Patent ductus arteriosus (PDA)
Coarctation of the aorta
Aortic valve stenosis

VSDs are more common than ASDs. However, in adult patients ASDs are the more common new diagnosis as they generally present later.

Cyanotic - most common causes
Tetralogy of Fallot
Transposition of the great arteries (TGA)
Tricuspid atresia
Pulmonary valve stenosis
214
Q

A 44 year old lady presents with a mass in the upper outer quadrant of her right breast. Imaging, histology and clinical examination confirm a 1.5cm malignant mass lesion with no clinical evidence of axillary nodal disease.

A. Simple mastectomy alone
B. Radical mastectomy alone
C. Simple mastectomy and sentinel lymph node biopsy
D. Wide local excision and sentinel lymph node biopsy
E. Simple mastectomy and axillary node clearance
F. Radical mastectomy and axillary node clearance
G. Wide local excision and axillary node clearance
H. Wide local excision alone

A

Wide local excision and sentinel lymph node biopsy

A small peripheral lesion such as this would usually be suitable for breast conserving surgery. Since imaging and clinical examination is not suspicious for axillary disease, a sentinel lymph node biopsy should be performed.

Breast cancer management

Surgery is performed in most patients suffering from breast cancer.
Chemotherapy may be used to downstage tumours and allow breast conserving surgery. Hormonal therapy may also be used for the same purposes.
Radiotherapy is given to all patients who have undergone breast conserving surgery.
Patients who have undergone mastectomy may be offered a reconstructive procedure either in conjunction with their primary resection or as a staged procedure at a later date.

Surgical options
Mastectomy vs Wide local excision

Mastectomy	Wide Local Excision
Multifocal tumour	Solitary lesion
Central tumour	Peripheral tumour
Large lesion in small breast	Small lesion in large breast
DCIS >4cm	DCIS <4cm
Patient Choice	Patient choice

Central lesions may be managed using breast conserving surgery, where an acceptable cosmetic result may be obtained, this is rarely the case in small breasts

Axillary disease
As a minimum, all patients with invasive breast cancer should have their axilla staged. In those who do not have overt evidence of axillary nodal involvement this can be undertaken using sentinel lymph node biopsy.
Patients with a positive sentinel lymph node biopsy or who have imaging and cytological or histological evidence of axillary nodal metastasis should undergo axillary node clearance.
Axillary node clearance is associated with the development of lymphoedema, increased risk of cellulitis and frozen shoulder.

215
Q

A 44 year old lady presents with a mass lesion in the upper outer quadrant of the left breast. On clinical examination she has a 2cm mass lesion which on core biopsy is demonstrated to have invasive ductal carcinoma. An FNA of a bulky axillary lymph node contains malignant cells.

A. Simple mastectomy alone
B. Radical mastectomy alone
C. Simple mastectomy and sentinel lymph node biopsy
D. Wide local excision and sentinel lymph node biopsy
E. Simple mastectomy and axillary node clearance
F. Radical mastectomy and axillary node clearance
G. Wide local excision and axillary node clearance
H. Wide local excision alone

A

Wide local excision and axillary node clearance

Although the primary lesion is small enough for breast conserving surgery, the presence of overt axillary lymph node metastasis will attract a recommendation for axillary node clearance.

Breast cancer management

Surgery is performed in most patients suffering from breast cancer.
Chemotherapy may be used to downstage tumours and allow breast conserving surgery. Hormonal therapy may also be used for the same purposes.
Radiotherapy is given to all patients who have undergone breast conserving surgery.
Patients who have undergone mastectomy may be offered a reconstructive procedure either in conjunction with their primary resection or as a staged procedure at a later date.

Surgical options
Mastectomy vs Wide local excision

Mastectomy	Wide Local Excision
Multifocal tumour	Solitary lesion
Central tumour	Peripheral tumour
Large lesion in small breast	Small lesion in large breast
DCIS >4cm	DCIS <4cm
Patient Choice	Patient choice

Central lesions may be managed using breast conserving surgery, where an acceptable cosmetic result may be obtained, this is rarely the case in small breasts

Axillary disease
As a minimum, all patients with invasive breast cancer should have their axilla staged. In those who do not have overt evidence of axillary nodal involvement this can be undertaken using sentinel lymph node biopsy.
Patients with a positive sentinel lymph node biopsy or who have imaging and cytological or histological evidence of axillary nodal metastasis should undergo axillary node clearance.
Axillary node clearance is associated with the development of lymphoedema, increased risk of cellulitis and frozen shoulder.

216
Q

A 39 year old lady presents with a mass lesion in her right breast. Clinical examination, biopsy and imaging confirm a 2.5 cm lesion in the upper inner quadrant of her right breast and a 1.5 cm lesion at the central aspect of the same breast. Her axilla shows lymphadenopathy and a fine needle aspirate from the node shows malignant cells.

A. Simple mastectomy alone
B. Radical mastectomy alone
C. Simple mastectomy and sentinel lymph node biopsy
D. Wide local excision and sentinel lymph node biopsy
E. Simple mastectomy and axillary node clearance
F. Radical mastectomy and axillary node clearance
G. Wide local excision and axillary node clearance
H. Wide local excision alone

A

Simple mastectomy and axillary node clearance

A combination of established axillary disease and multifocal invasive lesions attracts an indication for mastectomy and axillary clearance. A radical mastectomy is less frequently indicated in modern surgical practice, disease that is locally advanced is often best downstaged using medical therapy, rather than embarking on the operations for breast cancer that were first popularised over 100 years ago.

Breast cancer management

Surgery is performed in most patients suffering from breast cancer.
Chemotherapy may be used to downstage tumours and allow breast conserving surgery. Hormonal therapy may also be used for the same purposes.
Radiotherapy is given to all patients who have undergone breast conserving surgery.
Patients who have undergone mastectomy may be offered a reconstructive procedure either in conjunction with their primary resection or as a staged procedure at a later date.

Surgical options
Mastectomy vs Wide local excision

Mastectomy	Wide Local Excision
Multifocal tumour	Solitary lesion
Central tumour	Peripheral tumour
Large lesion in small breast	Small lesion in large breast
DCIS >4cm	DCIS <4cm
Patient Choice	Patient choice

Central lesions may be managed using breast conserving surgery, where an acceptable cosmetic result may be obtained, this is rarely the case in small breasts

Axillary disease
As a minimum, all patients with invasive breast cancer should have their axilla staged. In those who do not have overt evidence of axillary nodal involvement this can be undertaken using sentinel lymph node biopsy.
Patients with a positive sentinel lymph node biopsy or who have imaging and cytological or histological evidence of axillary nodal metastasis should undergo axillary node clearance.
Axillary node clearance is associated with the development of lymphoedema, increased risk of cellulitis and frozen shoulder.

217
Q

A 58 year old man with long standing Barretts oesophagus is found to have a nodule on endoscopic surveillence. Biopsies and endoscopic USS suggest this is at most a 1cm foci of T1 disease in the distal oesophagus 4 cm proximal to the oesophagogastric junction.

A.	Endo lumenal brachytherapy
B.	Chemo-radiotherapy
C.	Radiotherapy alone
D.	Insertion of expanding metallic stent
E.	Ivor-Lewis oesophagectomy
F.	Total oesophagectomy
G.	Segmental resection of mid oesophagus
H.	Endoscopic mucosal resection
A

Endoscopic mucosal resection

EMR is an reasonable option for small areas of malignancy occurring on a background of Barretts change. Segmental resections of the oesophagus are not practised and the only resectional strategy in this scenario would be an Ivor- Lewis type resection. The morbidity such a strategy in T1 disease is probably not justified.

Oesophageal cancer - treatment

Treatments for SCC’s and adenocarcinomas of the oesophagus differ. This is primarily due to the positive outcomes that are observed when localised SCC’s (particularly of the proximal oesophagus are treated with radical chemoradiotherapy (obviating the need for surgery).
Only those patients whose staging investigations are negative for metastatic disease should be considered for surgery.

Surgical options
Endoscopic mucosal resection Treatment for early localised adenocarcinoma of the distal oesophagus. Survival mirrors that of surgical resection for Tis and T1 disease
Transhiatal oeosphagectomy Most commonly used for junctional (type II) (1) tumours where limited thoracic oesophageal resection is required. Less morbidity than two field oesophagectomy
Ivor Lewis oesophagectomy Two stage approach for middle and distal tumours. Very commonly performed, intrathoracic anastomosis will result in mediastinitis in event of anastomotic leak. Lower incidence of recurrent laryngeal nerve injury
McKeown oesophagectomy Three field approach, may be useful for proximal tumours. Anastomotic leakage is less serious. Higher incidence of recurrent laryngeal nerve injury

Neoadjuvent and adjuvent treatment
Neoadjuvent radiotherapy alone prior to resection confers little benefit and is not routinely performed (2)
Preoperative chemotherapy is associated with a survival advantage (OE02 trial)
Peri operative (pre and post operative) chemotherapy confers a survival advantage in junctional tumours
Post operative chemotherapy is not generally recommended following oesophageal resections outside clinical trials

Palliation strategies
Combination chemotherapy improves quality of life and survival in non operable disease (3)
Trastuzumab may improve survival in patients with HER 2 positive tumours
Oesophageal intubation with self expanding metal stents is the treatment of choice in patients with occluding tumours >2cm from the cricopharyngeus
Covered metal stents are useful in cases of malignant fistulas
Laser therapy and argon plasma coagulation may be useful as therapies for tumour overgrowth and bleeding
Photodynamic therapy and ethanol injections confer little benefit and should not be routinely used

218
Q

An 82 year old man presents with dysphagia and on investigation is found to have a stenosing tumour of the mid oesophagus with a single mestastasis in the right lobe of the liver (segment VI).

A.	Endo lumenal brachytherapy
B.	Chemo-radiotherapy
C.	Radiotherapy alone
D.	Insertion of expanding metallic stent
E.	Ivor-Lewis oesophagectomy
F.	Total oesophagectomy
G.	Segmental resection of mid oesophagus
H.	Endoscopic mucosal resection
A

Insertion of expanding metallic stent

Distant disease in patients with oesophageal cancer is a contra indication to a resectional strategy and downstaging with chemotherapy is not routinely undertaken in this age group as the results are poor. An expanding stent will provide rapid and durable palliation.

Oesophageal cancer - treatment

Treatments for SCC’s and adenocarcinomas of the oesophagus differ. This is primarily due to the positive outcomes that are observed when localised SCC’s (particularly of the proximal oesophagus are treated with radical chemoradiotherapy (obviating the need for surgery).
Only those patients whose staging investigations are negative for metastatic disease should be considered for surgery.

Surgical options
Endoscopic mucosal resection Treatment for early localised adenocarcinoma of the distal oesophagus. Survival mirrors that of surgical resection for Tis and T1 disease
Transhiatal oeosphagectomy Most commonly used for junctional (type II) (1) tumours where limited thoracic oesophageal resection is required. Less morbidity than two field oesophagectomy
Ivor Lewis oesophagectomy Two stage approach for middle and distal tumours. Very commonly performed, intrathoracic anastomosis will result in mediastinitis in event of anastomotic leak. Lower incidence of recurrent laryngeal nerve injury
McKeown oesophagectomy Three field approach, may be useful for proximal tumours. Anastomotic leakage is less serious. Higher incidence of recurrent laryngeal nerve injury

Neoadjuvent and adjuvent treatment
Neoadjuvent radiotherapy alone prior to resection confers little benefit and is not routinely performed (2)
Preoperative chemotherapy is associated with a survival advantage (OE02 trial)
Peri operative (pre and post operative) chemotherapy confers a survival advantage in junctional tumours
Post operative chemotherapy is not generally recommended following oesophageal resections outside clinical trials

Palliation strategies
Combination chemotherapy improves quality of life and survival in non operable disease (3)
Trastuzumab may improve survival in patients with HER 2 positive tumours
Oesophageal intubation with self expanding metal stents is the treatment of choice in patients with occluding tumours >2cm from the cricopharyngeus
Covered metal stents are useful in cases of malignant fistulas
Laser therapy and argon plasma coagulation may be useful as therapies for tumour overgrowth and bleeding
Photodynamic therapy and ethanol injections confer little benefit and should not be routinely used

219
Q

A 56 year old man presents with odynophagia and on investigation is found to have a squamous cell carcinoma of the upper third of the oesophagus. Staging investigations are negative for metastatic disease.

A.	Endo lumenal brachytherapy
B.	Chemo-radiotherapy
C.	Radiotherapy alone
D.	Insertion of expanding metallic stent
E.	Ivor-Lewis oesophagectomy
F.	Total oesophagectomy
G.	Segmental resection of mid oesophagus
H.	Endoscopic mucosal resection
A

Chemo-radiotherapy

SCC of the oesophagus is treated with chemo-radiotherapy in the first instance.

Oesophageal cancer - treatment

Treatments for SCC’s and adenocarcinomas of the oesophagus differ. This is primarily due to the positive outcomes that are observed when localised SCC’s (particularly of the proximal oesophagus are treated with radical chemoradiotherapy (obviating the need for surgery).
Only those patients whose staging investigations are negative for metastatic disease should be considered for surgery.

Surgical options
Endoscopic mucosal resection Treatment for early localised adenocarcinoma of the distal oesophagus. Survival mirrors that of surgical resection for Tis and T1 disease
Transhiatal oeosphagectomy Most commonly used for junctional (type II) (1) tumours where limited thoracic oesophageal resection is required. Less morbidity than two field oesophagectomy
Ivor Lewis oesophagectomy Two stage approach for middle and distal tumours. Very commonly performed, intrathoracic anastomosis will result in mediastinitis in event of anastomotic leak. Lower incidence of recurrent laryngeal nerve injury
McKeown oesophagectomy Three field approach, may be useful for proximal tumours. Anastomotic leakage is less serious. Higher incidence of recurrent laryngeal nerve injury

Neoadjuvent and adjuvent treatment
Neoadjuvent radiotherapy alone prior to resection confers little benefit and is not routinely performed (2)
Preoperative chemotherapy is associated with a survival advantage (OE02 trial)
Peri operative (pre and post operative) chemotherapy confers a survival advantage in junctional tumours
Post operative chemotherapy is not generally recommended following oesophageal resections outside clinical trials

Palliation strategies
Combination chemotherapy improves quality of life and survival in non operable disease (3)
Trastuzumab may improve survival in patients with HER 2 positive tumours
Oesophageal intubation with self expanding metal stents is the treatment of choice in patients with occluding tumours >2cm from the cricopharyngeus
Covered metal stents are useful in cases of malignant fistulas
Laser therapy and argon plasma coagulation may be useful as therapies for tumour overgrowth and bleeding
Photodynamic therapy and ethanol injections confer little benefit and should not be routinely used

220
Q

Which investigation is best for initial assessment of recurrence of follicular carcinoma of the thyroid?

	Free T4
	Thyroid stimulating hormone
	Scintigraphy
	Serum thyroglobulin
	USS thyroid gland
A

Elevated thyroglobulin levels raises suspicion of recurrence.

Thyroid malignancy

Papillary carcinoma
Commonest sub-type
Accurately diagnosed on fine needle aspiration cytology
Histologically, they may demonstrate psammoma bodies (areas of calcification) and so called ‘orphan Annie’ nuclei
They typically metastasise via the lymphatics and thus laterally located apparently ectopic thyroid tissue is usually a metastasis from a well differentiated papillary carcinoma

Follicular carcinoma
Are less common than papillary lesions
Like papillary tumours, they may present as a discrete nodule. Although they appear to be well encapsulated macroscopically there is invasion on microscopic evaluation
Lymph node metastases are uncommon and these tumours tend to spread haematogenously. This translates into a higher mortality rate
Follicular lesions cannot be accurately diagnosed on fine needle aspiration cytology and thus all follicular FNA’s (THY 3f) will require at least a hemi thyroidectomy

Anaplastic carcinoma
Less common and tend to occur in elderly females
Disease is usually advanced at presentation and often only palliative decompression and radiotherapy can be offered.

Medullary carcinoma
These are tumours of the parafollicular cells ( C Cells) and are of neural crest origin.
The serum calcitonin may be elevated which is of use when monitoring for recurrence.
They may be familial and occur as part of the MEN -2A disease spectrum.
Spread may be either lymphatic or haematogenous and as these tumours are not derived primarily from thyroid cells they are not responsive to radioiodine.

Lymphoma
These respond well to radiotherapy
Radical surgery is unnecessary once the disease has been diagnosed on biopsy material. Such biopsy material is not generated by an FNA and thus a core biopsy has to be obtained (with care!).

221
Q

A 41 year old lady with colicky right upper quadrant pain is identified as having gallstones on an abdominal ultrasound scan. What is the most appropriate course of action?

	Laparoscopic cholecystectomy
	Open cholecystectomy
	Liver function tests
	MRCP
	ERCP
A

Liver function testing is part of the core diagnostic work up of biliary colic and surgical planning cannot proceed until this (and the diameter of the CBD on USS) are known.

Gallstones

Up to 24% of women and 12% of men may have gallstones. Of these up to 30% may develop local infection and cholecystitis. In patients subjected to surgery 12% will have stones contained within the common bile duct. The majority of gallstones are of a mixed composition (50%) with pure cholesterol stones accounting for 20% of cases.
The aetiology of CBD stones differs in the world, in the West most CBD stones are the result of migration. In the East a far higher proportion arise in the CBD de novo.
The classical symptoms are of colicky right upper quadrant pain that occurs post prandially. The symptoms are usually worst following a fatty meal when cholecystokinin levels are highest and gallbladder contraction is maximal.

Investigation
In almost all suspected cases the standard diagnostic work up consists of abdominal ultrasound and liver function tests. Of patients who have stones within the bile duct, 60% will have at least one abnormal result on LFT’s. Ultrasound is an important test, but is operator dependent and therefore may occasionally need to be repeated if a negative result is at odds with the clinical picture. Where stones are suspected in the bile duct, the options lie between magnetic resonance cholangiography and intraoperative imaging. The choice between these two options is determined by the skills and experience of the surgeon. The advantages of intra operative imaging are less useful in making therapeutic decisions if the operator is unhappy about proceeding the bile duct exploration, and in such circumstances pre operative MRCP is probably a better option.

Specific gallstone and gallbladder related disease
Disease Features Management
Biliary colic Colicky abdominal pain, worse post prandially, worse after fatty foods If imaging shows gallstones and history compatible then laparoscopic cholecystectomy
Acute cholecystitis
Right upper quadrant pain
Fever
Murphys sign on examination
Occasionally mildly deranged LFT’s (especially if Mirizzi syndrome)
Imaging (USS) and cholecystectomy (ideally within 48 hours of presentation) (2)
Gallbladder abscess
Usually prodromal illness and right upper quadrant pain
Swinging pyrexia
Patient may be systemically unwell
Generalised peritonism not present
Imaging with USS +/- CT Scanning
Ideally surgery, sub total cholecystectomy may be needed if Calots triangle is hostile
In unfit patients percutaneous drainage may be considered
Cholangitis
Patient severely septic and unwell
Jaundice
Right upper quadrant pain
Fluid resuscitation
Broad spectrum intravenous antibiotics
Correct any coagulopathy
Early ERCP
Gallstone ileus
Patients may have a history of previous cholecystitis and known gallstones
Small bowel obstruction (may be intermittent)
Laparotomy and removal of gallstone from small bowel, the enterotomy must be made proximal to the site of obstruction and not at the site of obstruction. The fistula between the gallbladder and duodenum should not be interfered with.
Acalculous cholecystitis
Patients with inter current illness (e.g. diabetes, organ failure)
Patient of systemically unwell
Gallbladder inflammation in absence of stones
High fever
If patient fit then cholecystectomy, if unfit then percutaneous cholecystostomy

Treatment
Patients with asymptomatic gallstones rarely develop symptoms related to them (less than 2% per year) and may therefore be managed expectantly. In almost all cases of symptomatic gallstones the treatment of choice is cholecystectomy performed via the laparoscopic route. In the very frail patient there is sometimes a role for selective use of ultrasound guided cholecystostomy.

During the course of the procedure some surgeons will routinely perform either intra operative cholangiography or laparoscopic USS to either confirm anatomy or to exclude CBD stones. The latter may be more easily achieved by use of laparoscopic ultrasound. If stones are found then the options lie between early ERCP in the day or so following surgery or immediate surgical exploration of the bile duct. When performed via the trans cystic route this adds little in the way of morbidity and certainly results in faster recovery. Where transcystic exploration fails the alternative strategy is that of formal choledochotomy. The exploration of a small duct is challenging and ducts of less than 8mm should not be explored. Small stones that measure less than 5mm may be safely left and most will pass spontaneously.

Risks of ERCP(1)
Bleeding 0.9% (rises to 1.5% if sphincterotomy performed)
Duodenal perforation 0.4%
Cholangitis 1.1%
Pancreatitis 1.5%
222
Q

You embark on a laparoscopic appendicectomy and find an appendix mass. There is no free fluid and the patient has no evidence of peritonitis. Which is the best option?

Convert to a midline laparotomy and perform a limited right hemicolectomy and end ileostomy
Convert to midline laparotomy and perform and appendicectomy after taking down the adhesions
Place a drain laparoscopically and administer parenteral antibiotics
Send the patient for CT guided drainage
Wrap omentum around the area and avoid drainage
A

Place a drain laparoscopically and administer parenteral antibiotics
Attempt conservative management for appendix mass without peritonitis.

Dissection of appendix masses can be associated with a considerable degree of morbidity, the gains of formally dissecting them over simple drainage and antibiotics are minimal.
This was initially described as the Ochsner-Sherren regime and was based on the teachings of Albert Ochsner of Chicago and James Sherren of the London hospital.

Appendicitis

History
Peri umbilical abdominal pain (visceral stretching of appendix lumen and appendix is mid gut structure) radiating to the right iliac fossa due to localised parietal peritoneal inflammation.
Vomit once or twice but marked and persistent vomiting is unusual.
Diarrhoea is rare. However, pelvic appendicitis may cause localised rectal irritation and some loose stools. A pelvic abscess may also cause diarrhoea.
Mild pyrexia is common - temperature is usually 37.5 -38oC. Higher temperatures are more typical of conditions like mesenteric adenitis.
Anorexia is very common. It is very unusual for patients with appendicitis to be hungry.

Examination
Generalised peritonitis if perforation has occurred or localised peritonism.
Retrocaecal appendicitis may have relatively few signs.
Digital rectal examination may reveal boggy sensation if pelvic abscess is present, or even tenderness with a pelvic appendix.

Diagnosis
Typically raised inflammatory markers coupled with compatible history and examination findings should be enough to justify appendicectomy.
Urine analysis may show mild leucocytosis but no nitrites.
Ultrasound is useful in females where pelvic organ pathology is suspected. Although it is not always possible to visualise the appendix on ultrasound, the presence of free fluid (always pathological in males) should raise suspicion.

Ultrasound examination may show evidence of luminal obstruction and thickening of the appendiceal wall as shown below

Image sourced from Wikipedia

Treatment
Appendicectomy which can be performed via either an open or laparoscopic approach.
Administration of metronidazole reduces wound infection rates.
Patients with perforated appendicitis require copious abdominal lavage.
Patients without peritonitis who have an appendix mass should receive broad spectrum antibiotics and consideration given to performing an interval appendicectomy.
Be wary in the older patients who may have either an underlying caecal malignancy or perforated sigmoid diverticular disease.

223
Q

A 52 year old obese lady reports a painless grape sized mass in her groin area. She has no medical conditions apart from some varicose veins. There is a cough impulse and the mass disappears on lying down.

A.	Femoral hernia
B.	Lymphadenitis
C.	Inguinal hernia
D.	Psoas abscess
E.	Saphenous varix
F.	Femoral artery aneurysm
G.	Metastatic lymphadenopathy
H.	Lymphoma
I.	False femoral artery aneurysm
A

Saphenous varix

The history of varicose veins should indicate a more likely diagnosis of a varix. The varix can enlarge during coughing/sneezing. A blue discolouration may be noted.

Groin masses are common and include:
Herniae
Lipomas
Lymph nodes
Undescended testis
Femoral aneurysm
Saphena varix (more a swelling than a mass!)

In the history, features relating to systemic illness and tempo of onset will often give a clue as to the most likely underlying diagnosis.

Groin lumps- some key questions
Is there a cough impulse
Is it pulsatile AND is it expansile (to distinguish between false and true aneurysm)
Are both testes intra scrotal
Any lesions in the legs such as malignancy or infections (?lymph nodes)
Examine the ano rectum as anal cancer may metastasise to the groin
Is the lump soft, small and very superficial (?lipoma)

Scrotal lumps - some key questions
Is the lump entirely intra scrotal
Does it transilluminate (?hydrocele)
Is there a cough impulse (?hernia)

In most cases a diagnosis can be made clinically. Where it is not clear an ultrasound scan is often the most convenient next investigation.

224
Q

A 32 year old male is noted to have a tender mass in the right groin area. There are also red streaks on the thigh, extending from a small abrasion.

A.	Femoral hernia
B.	Lymphadenitis
C.	Inguinal hernia
D.	Psoas abscess
E.	Saphenous varix
F.	Femoral artery aneurysm
G.	Metastatic lymphadenopathy
H.	Lymphoma
I.	False femoral artery aneurysm
A

Lymphadenitis

The red streaks are along the line of the lymphatics, indicating infection of the lymphatic vessels. Lymphadenitis is infection of the local lymph nodes.

Groin masses are common and include:
Herniae
Lipomas
Lymph nodes
Undescended testis
Femoral aneurysm
Saphena varix (more a swelling than a mass!)

In the history, features relating to systemic illness and tempo of onset will often give a clue as to the most likely underlying diagnosis.

Groin lumps- some key questions
Is there a cough impulse
Is it pulsatile AND is it expansile (to distinguish between false and true aneurysm)
Are both testes intra scrotal
Any lesions in the legs such as malignancy or infections (?lymph nodes)
Examine the ano rectum as anal cancer may metastasise to the groin
Is the lump soft, small and very superficial (?lipoma)

Scrotal lumps - some key questions
Is the lump entirely intra scrotal
Does it transilluminate (?hydrocele)
Is there a cough impulse (?hernia)

In most cases a diagnosis can be made clinically. Where it is not clear an ultrasound scan is often the most convenient next investigation.

225
Q

A 23 year old male suffering from hepatitis C presents with right groin pain and swelling. On examination there is a large abscess in the groin. Adjacent to this is a pulsatile swelling. There is no cough impulse.

A.	Femoral hernia
B.	Lymphadenitis
C.	Inguinal hernia
D.	Psoas abscess
E.	Saphenous varix
F.	Femoral artery aneurysm
G.	Metastatic lymphadenopathy
H.	Lymphoma
I.	False femoral artery aneurysm
A

False femoral artery aneurysm

False aneurysms may occur following arterial trauma in IVDU. They may have associated blood borne virus infections and should undergo duplex scanning prior to surgery. False aneurysms do not contain all layers of the arterial wall.

Groin masses are common and include:
Herniae
Lipomas
Lymph nodes
Undescended testis
Femoral aneurysm
Saphena varix (more a swelling than a mass!)

In the history, features relating to systemic illness and tempo of onset will often give a clue as to the most likely underlying diagnosis.

Groin lumps- some key questions
Is there a cough impulse
Is it pulsatile AND is it expansile (to distinguish between false and true aneurysm)
Are both testes intra scrotal
Any lesions in the legs such as malignancy or infections (?lymph nodes)
Examine the ano rectum as anal cancer may metastasise to the groin
Is the lump soft, small and very superficial (?lipoma)

Scrotal lumps - some key questions
Is the lump entirely intra scrotal
Does it transilluminate (?hydrocele)
Is there a cough impulse (?hernia)

In most cases a diagnosis can be made clinically. Where it is not clear an ultrasound scan is often the most convenient next investigation.

226
Q

A 73 year old lady develops a cold, pulseless hand 3 days following a myocardial infarction.

A.	Vasculitis
B.	Steal syndrome
C.	Thrombosis
D.	Foreign body embolus
E.	Clot embolus
F.	Vasospasm
G.	Direct arterial injury
A

Clot embolus

The development of mural or atrial appendage thrombi may occur following a myocardial infarct and co-existing atrial fibrillation may contribute to the formation. They tend to present with classical features of an embolic event.

Embolus Sudden onset
Depending upon level of occlusion; limb may show typical features of pain, loss of pulses and pallor. Sensory perceptive changes may also be present
Thrombosis Usually known disease and prodromal symptoms e.g. claudication
Disruption to flow may be incomplete
If background disease process present then collaterals may be present and picture less dramatic
Vasospasm May be due to Raynauds and affect extremities
Symptoms are often temperature related
Discolouration of the hands may occur (pale, dark, red)
Symptoms improve during pregnancy (hyperdynamic circulation)
Steal syndromes Occur secondary to arteriovenous fistula, or partial arterial occlusions (e.g. cervical rib)
Pain and diminished pulses distal to fistula are seen

Vasculitis
Vessel diameter and vasculitis classification
Aorta and branches	
Takayasu's arteritis
Buergers disease
Giant cell arteritis
Large and medium sized arteries	
Buergers disease
Giant cell arteritis
Polyarteritis nodosa
Medium sized muscular arteries	
Polyarteritis nodosa
Wegeners granulomatosis
Small muscular arteries	
Wegeners granulomatosis
Rheumatoid vasculitis

Specific conditions
Takyasu’s arteritis
Inflammatory, obliterative arteritis affecting aorta and branches
Females> Males
Symptoms may include upper limb claudication
Clinical findings include diminished or absent pulses
ESR often affected during the acute phase
Buergers disease
Segmental thrombotic occlusions of the small and medium sized lower limb vessels
Commonest in young male smokers
Proximal pulses usually present, but pedal pulses are lost
An acuter hypercellular occlusive thrombus is often present
Tortuous corkscrew shaped collateral vessels may be seen on angiography
Giant cell arteritis
Systemic granulomatous arteritis that usually affects large and medium sized vessels
Females > Males
Temporal arteritis is commonest type
Granulomatous lesions may be seen on biopsy (although up to 50% are normal)
Polyarteritis nodosa
Systemic necrotising vasculitis affecting small and medium sized muscular arteries
Most common in populations with high prevalence of hepatitis B
Renal disease is seen in 70% cases
Angiography may show saccular or fusiform aneurysms and arterial stenoses
Wegeners granulomatosis
Predominantly affects small and medium sized arteries
Systemic necrotising granulomatous vasculitis
Cutaneous vascular lesions may be seen (ulceration, nodules and purpura)
Sinus imaging may show mucosal thickening and air fluid levels

227
Q

A 6 year old child has suffered a displaced supracondylar humeral fracture. On examination, they have a cold and insensate hand with absent pulses.

A.	Vasculitis
B.	Steal syndrome
C.	Thrombosis
D.	Foreign body embolus
E.	Clot embolus
F.	Vasospasm
G.	Direct arterial injury
A

Direct arterial injury

Both vasospasm and arterial injury may complicate supracondylar fractures and are seen in 1% of all cases. Vasospasm is usually transient and more likely when the injury is minor and reduced early. Severely displaced injuries and those with more advanced signs are usually associated with direct arterial injury.

Embolus Sudden onset
Depending upon level of occlusion; limb may show typical features of pain, loss of pulses and pallor. Sensory perceptive changes may also be present
Thrombosis Usually known disease and prodromal symptoms e.g. claudication
Disruption to flow may be incomplete
If background disease process present then collaterals may be present and picture less dramatic
Vasospasm May be due to Raynauds and affect extremities
Symptoms are often temperature related
Discolouration of the hands may occur (pale, dark, red)
Symptoms improve during pregnancy (hyperdynamic circulation)
Steal syndromes Occur secondary to arteriovenous fistula, or partial arterial occlusions (e.g. cervical rib)
Pain and diminished pulses distal to fistula are seen

Vasculitis
Vessel diameter and vasculitis classification
Aorta and branches	
Takayasu's arteritis
Buergers disease
Giant cell arteritis
Large and medium sized arteries	
Buergers disease
Giant cell arteritis
Polyarteritis nodosa
Medium sized muscular arteries	
Polyarteritis nodosa
Wegeners granulomatosis
Small muscular arteries	
Wegeners granulomatosis
Rheumatoid vasculitis

Specific conditions
Takyasu’s arteritis
Inflammatory, obliterative arteritis affecting aorta and branches
Females> Males
Symptoms may include upper limb claudication
Clinical findings include diminished or absent pulses
ESR often affected during the acute phase
Buergers disease
Segmental thrombotic occlusions of the small and medium sized lower limb vessels
Commonest in young male smokers
Proximal pulses usually present, but pedal pulses are lost
An acuter hypercellular occlusive thrombus is often present
Tortuous corkscrew shaped collateral vessels may be seen on angiography
Giant cell arteritis
Systemic granulomatous arteritis that usually affects large and medium sized vessels
Females > Males
Temporal arteritis is commonest type
Granulomatous lesions may be seen on biopsy (although up to 50% are normal)
Polyarteritis nodosa
Systemic necrotising vasculitis affecting small and medium sized muscular arteries
Most common in populations with high prevalence of hepatitis B
Renal disease is seen in 70% cases
Angiography may show saccular or fusiform aneurysms and arterial stenoses
Wegeners granulomatosis
Predominantly affects small and medium sized arteries
Systemic necrotising granulomatous vasculitis
Cutaneous vascular lesions may be seen (ulceration, nodules and purpura)
Sinus imaging may show mucosal thickening and air fluid levels

228
Q

A 26 year old man who smokes heavily develops aching, crampy pains in his legs. On examination distal limb pulses are diminished.

A.	Vasculitis
B.	Steal syndrome
C.	Thrombosis
D.	Foreign body embolus
E.	Clot embolus
F.	Vasospasm
G.	Direct arterial injury
A

Vasculitis

This is likely to represent Buergers disease. It is commonest in young males who smoke heavily.

Embolus Sudden onset
Depending upon level of occlusion; limb may show typical features of pain, loss of pulses and pallor. Sensory perceptive changes may also be present
Thrombosis Usually known disease and prodromal symptoms e.g. claudication
Disruption to flow may be incomplete
If background disease process present then collaterals may be present and picture less dramatic
Vasospasm May be due to Raynauds and affect extremities
Symptoms are often temperature related
Discolouration of the hands may occur (pale, dark, red)
Symptoms improve during pregnancy (hyperdynamic circulation)
Steal syndromes Occur secondary to arteriovenous fistula, or partial arterial occlusions (e.g. cervical rib)
Pain and diminished pulses distal to fistula are seen

Vasculitis
Vessel diameter and vasculitis classification
Aorta and branches	
Takayasu's arteritis
Buergers disease
Giant cell arteritis
Large and medium sized arteries	
Buergers disease
Giant cell arteritis
Polyarteritis nodosa
Medium sized muscular arteries	
Polyarteritis nodosa
Wegeners granulomatosis
Small muscular arteries	
Wegeners granulomatosis
Rheumatoid vasculitis

Specific conditions
Takyasu’s arteritis
Inflammatory, obliterative arteritis affecting aorta and branches
Females> Males
Symptoms may include upper limb claudication
Clinical findings include diminished or absent pulses
ESR often affected during the acute phase
Buergers disease
Segmental thrombotic occlusions of the small and medium sized lower limb vessels
Commonest in young male smokers
Proximal pulses usually present, but pedal pulses are lost
An acuter hypercellular occlusive thrombus is often present
Tortuous corkscrew shaped collateral vessels may be seen on angiography
Giant cell arteritis
Systemic granulomatous arteritis that usually affects large and medium sized vessels
Females > Males
Temporal arteritis is commonest type
Granulomatous lesions may be seen on biopsy (although up to 50% are normal)
Polyarteritis nodosa
Systemic necrotising vasculitis affecting small and medium sized muscular arteries
Most common in populations with high prevalence of hepatitis B
Renal disease is seen in 70% cases
Angiography may show saccular or fusiform aneurysms and arterial stenoses
Wegeners granulomatosis
Predominantly affects small and medium sized arteries
Systemic necrotising granulomatous vasculitis
Cutaneous vascular lesions may be seen (ulceration, nodules and purpura)
Sinus imaging may show mucosal thickening and air fluid levels

229
Q

A 28 year old man presents with hypertension and haematuria. Haematological investigations show polycythaemia but otherwise no abnormality. CT scanning shows a left renal mass.

A.	Retroperitoneal liposarcoma
B.	Transitional cell carcinoma
C.	Retroperitoneal fibrosis
D.	Renal squamous cell carcinoma
E.	Renal adenocarcinoma
F.	Nephroblastoma
A

Renal adenocarcinoma

Renal adenocarcinoma is the most common variant and is associated with polycythaemia.

Causes of haematuria

Trauma
Injury to renal tract
Renal trauma commonly due to blunt injury (others penetrating injuries)
Ureter trauma rare: iatrogenic
Bladder trauma: due to RTA or pelvic fractures
Infection
Remember TB
Malignancy
Renal cell carcinoma (remember paraneoplastic syndromes): painful or painless
Urothelial malignancies: 90% are transitional cell carcinoma, can occur anywhere along the urinary tract. Painless haematuria.
Squamous cell carcinoma and adenocarcinoma: rare bladder tumours
Prostate cancer
Penile cancers: SCC
Renal disease
Glomerulonephritis
Stones
Microscopic haematuria common
Structural abnormalities
Benign prostatic hyperplasia (BPH) causes haematuria due to hypervascularity of the prostate gland
Cystic renal lesions e.g. polycystic kidney disease
Vascular malformations
Renal vein thrombosis due to renal cell carcinoma
Coagulopathy
Causes bleeding of underlying lesions
Drugs
Cause tubular necrosis or interstitial nephritis: aminoglycosides, chemotherapy
Interstitial nephritis: penicillin, sulphonamides, and NSAIDs
Anticoagulants
Benign
Exercise
Gynaecological
Endometriosis: flank pain, dysuria, and haematuria that is cyclical
Iatrogenic
Catheterisation
Radiotherapy; cystitis, severe haemorrhage, bladder necrosis
Pseudohaematuria For example following consumption of beetroot

230
Q

A 68 year of man presents with recurrent episodes of left sided ureteric colic and haematuria. Investigations show some dilatation of the renal pelvis but the outline is irregular.

A.	Retroperitoneal liposarcoma
B.	Transitional cell carcinoma
C.	Retroperitoneal fibrosis
D.	Renal squamous cell carcinoma
E.	Renal adenocarcinoma
F.	Nephroblastoma
A

Transitional cell carcinoma

These arise from urothelium and necessitate a nephroureterectomy.

Causes of haematuria

Trauma
Injury to renal tract
Renal trauma commonly due to blunt injury (others penetrating injuries)
Ureter trauma rare: iatrogenic
Bladder trauma: due to RTA or pelvic fractures
Infection
Remember TB
Malignancy
Renal cell carcinoma (remember paraneoplastic syndromes): painful or painless
Urothelial malignancies: 90% are transitional cell carcinoma, can occur anywhere along the urinary tract. Painless haematuria.
Squamous cell carcinoma and adenocarcinoma: rare bladder tumours
Prostate cancer
Penile cancers: SCC
Renal disease
Glomerulonephritis
Stones
Microscopic haematuria common
Structural abnormalities
Benign prostatic hyperplasia (BPH) causes haematuria due to hypervascularity of the prostate gland
Cystic renal lesions e.g. polycystic kidney disease
Vascular malformations
Renal vein thrombosis due to renal cell carcinoma
Coagulopathy
Causes bleeding of underlying lesions
Drugs
Cause tubular necrosis or interstitial nephritis: aminoglycosides, chemotherapy
Interstitial nephritis: penicillin, sulphonamides, and NSAIDs
Anticoagulants
Benign
Exercise
Gynaecological
Endometriosis: flank pain, dysuria, and haematuria that is cyclical
Iatrogenic
Catheterisation
Radiotherapy; cystitis, severe haemorrhage, bladder necrosis
Pseudohaematuria For example following consumption of beetroot

231
Q

A 4 year old boy presents with haematuria and on examination is found to have a right sided renal mass.

A.	Retroperitoneal liposarcoma
B.	Transitional cell carcinoma
C.	Retroperitoneal fibrosis
D.	Renal squamous cell carcinoma
E.	Renal adenocarcinoma
F.	Nephroblastoma
A

Nephroblastoma

Wilms tumours (nephroblastoma) usually present in the first 4 years of life and may cause lung metastases.

Causes of haematuria

Trauma
Injury to renal tract
Renal trauma commonly due to blunt injury (others penetrating injuries)
Ureter trauma rare: iatrogenic
Bladder trauma: due to RTA or pelvic fractures
Infection
Remember TB
Malignancy
Renal cell carcinoma (remember paraneoplastic syndromes): painful or painless
Urothelial malignancies: 90% are transitional cell carcinoma, can occur anywhere along the urinary tract. Painless haematuria.
Squamous cell carcinoma and adenocarcinoma: rare bladder tumours
Prostate cancer
Penile cancers: SCC
Renal disease
Glomerulonephritis
Stones
Microscopic haematuria common
Structural abnormalities
Benign prostatic hyperplasia (BPH) causes haematuria due to hypervascularity of the prostate gland
Cystic renal lesions e.g. polycystic kidney disease
Vascular malformations
Renal vein thrombosis due to renal cell carcinoma
Coagulopathy
Causes bleeding of underlying lesions
Drugs
Cause tubular necrosis or interstitial nephritis: aminoglycosides, chemotherapy
Interstitial nephritis: penicillin, sulphonamides, and NSAIDs
Anticoagulants
Benign
Exercise
Gynaecological
Endometriosis: flank pain, dysuria, and haematuria that is cyclical
Iatrogenic
Catheterisation
Radiotherapy; cystitis, severe haemorrhage, bladder necrosis
Pseudohaematuria For example following consumption of beetroot

232
Q

Which of the following statements relating to omphalocele is false?

	The herniated organs lie outside the peritoneal sac
	Cardiac abnormalities co-exist in 25%
	The caecum is usually right sided
	The defect occurs through the umbilicus
	Mortality may be as high as 15%
A

They are contained within the peritoneal sac and therefore do not have the fluid losses seen in gastroschisis. True malrotation is unusual and minor variants may not result in a requirement for surgery.

Gastroschisis: Isolated abnormality, bowel lies outside abdominal wall through defect located to right of umbilicus.
Exomphalos: Liver and gut remain covered with membranous sac connected to umbilical cord. It is associated with other developmental defects.

Pyloric stenosis
M>F
5-10% Family history in parents
Projectile non bile stained vomiting at 4-6 weeks of life
Diagnosis is made by test feed or USS
Treatment: Ramstedt pyloromyotomy (open or laparoscopic)
Acute appendicitis
Uncommon under 3 years
When occurs may present atypically
Mesenteric adenitis
Central abdominal pain and URTI
Conservative management
Intussusception
Telescoping bowel
Proximal to or at the level of, ileocaecal valve
6-9 months age
Colicky pain, diarrhoea and vomiting, sausage shaped mass, red jelly stool.
Treatment: reduction with air insufflation
Malrotation
High caecum at the midline
Feature in exomphalos, congenital diaphragmatic hernia, intrinsic duodenal atresia
May be complicated by development of volvulus, infant with volvulus may have bile stained vomiting
Diagnosis is made by upper GI contrast study and USS
Treatment is by laparotomy, if volvulus is present (or at high risk of occurring then a ladds procedure is performed
Hirschsprung’s disease
Absence of ganglion cells from myenteric and submucosal plexuses
Occurs in 1/5000 births
Full thickness rectal biopsy for diagnosis
Delayed passage of meconium and abdominal distension
Treatment is with rectal washouts initially, thereafter an anorectal pull through procedure
Oesophageal atresia
Associated with tracheo-oesophageal fistula and polyhydramnios
May present with choking and cyanotic spells following aspiration
VACTERL associations
Meconium ileus
Usually delayed passage of meconium and abdominal distension
Majority have cystic fibrosis
X-Rays may not show a fluid level as the meconium is viscid (depends upon feeding), PR contrast studies may dislodge meconium plugs and be therapeutic
Infants who do not respond to PR contrast and NG N-acetyl cysteine will require surgery to remove the plugs
Biliary atresia
Jaundice > 14 days
Increased conjugated bilirubin
Urgent Kasai procedure
Necrotising enterocolitis
Prematurity is the main risk factor
Early features include abdominal distension and passage of bloody stools
X-Rays may show pneumatosis intestinalis and evidence of free air
Increased risk when empirical antibiotics are given to infants beyond 5 days
Treatment is with total gut rest and TPN, babies with perforations will require laparotomy

233
Q

A 42 year old singer is admitted for a thyroidectomy. Post operatively she is only able to make a gargling noise. Her voice sounds breathy.

A.	Vagus nerve injury
B.	Thyroid nerve injury
C.	Superior laryngeal nerve injury
D.	Unilateral inferior laryngeal nerve injury
E.	Bilateral inferior laryngeal nerves injuries
F.	Stroke
G.	Lacunar infarcts
H.	None of the above
A

Unilateral inferior laryngeal nerve injury

This patient has diplophonia which causes a gargling sound. This is associated with dysphagia. This can also be caused by a vagus nerve lesion, but the recurrent laryngeal nerve is more at risk of damage.

Voice production

There are 2 main nerves involved:

Superior laryngeal nerve (SLN)
Innervates the cricothyroid muscle

Since the cricothyroid muscle is involved in adjusting the tension of the vocal fold for high notes during singing, SLN paresis and paralysis result in:

a. Abnormalities in pitch
b. Inability to sing with smooth change to each higher note (glissando or pitch glide)

Recurrent laryngeal nerve (RLN)/Inferior laryngeal nerve
Innervates intrinsic larynx muscles

a. Opening vocal folds (as in breathing, coughing)
b. Closing vocal folds for vocal fold vibration during voice use
c. Closing vocal folds during swallowing

234
Q

A 42 year old singer is admitted for a thyroidectomy. Post operatively she is unable to sing high pitched notes.

A.	Vagus nerve injury
B.	Thyroid nerve injury
C.	Superior laryngeal nerve injury
D.	Unilateral inferior laryngeal nerve injury
E.	Bilateral inferior laryngeal nerves injuries
F.	Stroke
G.	Lacunar infarcts
H.	None of the above
A

Superior laryngeal nerve injury

SLN lesions cause difficulty in voice pitch.

Voice production

There are 2 main nerves involved:

Superior laryngeal nerve (SLN)
Innervates the cricothyroid muscle

Since the cricothyroid muscle is involved in adjusting the tension of the vocal fold for high notes during singing, SLN paresis and paralysis result in:

a. Abnormalities in pitch
b. Inability to sing with smooth change to each higher note (glissando or pitch glide)

Recurrent laryngeal nerve (RLN)/Inferior laryngeal nerve
Innervates intrinsic larynx muscles

a. Opening vocal folds (as in breathing, coughing)
b. Closing vocal folds for vocal fold vibration during voice use
c. Closing vocal folds during swallowing

235
Q

A 42 year old singer is admitted for a thyroidectomy. Post operatively the patient develops stridor and is unable to speak.

 A.	Vagus nerve injury
B.	Thyroid nerve injury
C.	Superior laryngeal nerve injury
D.	Unilateral inferior laryngeal nerve injury
E.	Bilateral inferior laryngeal nerves injuries
F.	Stroke
G.	Lacunar infarcts
H.	None of the above
A

Bilateral inferior laryngeal nerves injuries

This patient has aphonia due to bilateral damage to the recurrent laryngeal nerve.

Voice production

There are 2 main nerves involved:

Superior laryngeal nerve (SLN)
Innervates the cricothyroid muscle

Since the cricothyroid muscle is involved in adjusting the tension of the vocal fold for high notes during singing, SLN paresis and paralysis result in:

a. Abnormalities in pitch
b. Inability to sing with smooth change to each higher note (glissando or pitch glide)

Recurrent laryngeal nerve (RLN)/Inferior laryngeal nerve
Innervates intrinsic larynx muscles

a. Opening vocal folds (as in breathing, coughing)
b. Closing vocal folds for vocal fold vibration during voice use
c. Closing vocal folds during swallowing

236
Q

A 56 year old man presents with symptoms of nasal pain, anosmia and rhinorrhea. He has been well until recently and has worked as a wood carver for many years.

A.	Ethmoid sinus cancer
B.	Maxillary sinus cancer
C.	Ethmoid adenoma
D.	Maxillary adenoma
E.	Ethmoidal fracture
F.	Nasal polyps
G.	Sphenoid osteoma
H.	Ethmoidal sinusitis
I.	Maxillary sinusitis
A

Ethmoid sinus cancer

Nasopharyngeal cancer is strongly associated with wood work. Most cases require an occupational exposure of greater than 10 years and are adenocarcinomas on histology.
Most cases are ethmoidal in origin (Hadfield E. Ann R Coll Surg Engl. 1970 June; 46(6): 301319)

Diseases of nose and sinuses
Benign Tumours
Simple papillomas may be an incidental finding or present with obstructive symptoms. Excision under general anaesthesia is sufficient management.
Transitional cell papillomas may be more extensive and produce obstructive symptoms. Erosion of local structures is a recognised complication. These lesions may rarely undergo malignant transformation and therefore careful and complete excision is required, some cases may require partial or total maxillectomy.
Pleomorphic adenomas of the maxillary sinuses are reported but are extremely rare, their symptoms typically include nasal obstruction and pain if the sinus is obstructed. Treatment is by complete surgical excision, the diagnosis is not infrequently made post operatively.
Benign osteomas may develop in the paranasal sinuses, the frontal sinus is the most frequent location of such lesions. Symptoms include; pain, rhinorrhoea and anosmia. Most osteomas may be observed if asymptomatic, sphenoid osteomas should be resected soon after diagnosis as enlargement may compromise visual fields. Many sinus osteomas can now be resected endoscopically, complete surgical resection is required.
Nasal polyps are benign lesions of the ethmoid sinus mucosa. Many patients may also have asthma, cystic fibrosis and a sensitivity to aspirin. Symptoms include watery rhinorrhoea, infection and anosmia. The polyps are usually a semi transparent grey mass. They are rare in childhood. Treatment is either with systemic steroids or surgical resection. The latter should be combined with antral washout. Low dose, nasal, steroid drops may reduce the risk of recurrence.

Malignant disease
Malignancies encountered in the nose and paranasal sinuses include; adenoid cystic carcinoma, squamous cell carcinoma and adenocarcinoma.
Adenocarcinoma of the paranasal sinuses and nasopharynx is strongly linked to exposure to hard wood dust (after >10 years exposure).
Adenoid cystic carcinoma usually originate in the smaller salivary glands.
The majority of cancers (50%) arise from the lateral nasal wall, a smaller number (33%) arise from the maxillary antrum, ethmoid and sphenoid cancers comprise only 7%.
Signs of malignancy on clinical examination include loose teeth, cranial nerve palsies and lymphadenopathy.
Nasopharyngeal cancers are most common in individuals presenting from China and Asia and are linked to viral infection with Epstein Barr Virus. Radiotherapy and chemotherapy are the most commonly used modalities.

Maxillary sinusitis
Common symptoms include post nasal discharge, pain, headache and toothache.
Imaging may show a fluid level in the antrum.
Common organisms include Haemophilus influenzae or Streptococcus pneumoniae.
Treatment with antral lavage may facilitate diagnosis and relieve symptoms. Antimicrobial therapy has to be continued for long periods. Antrostomy may be needed.

Frontoethmoidal sinusitis
Usually presents with frontal headache, nasal obstruction and altered sense of smell.
Inflammation may progress to involve periorbital tissues. Ocular symptoms may occur and secondary CNS involvement brought about by infection entering via emissary veins.
CT scanning is the imaging modality of choice. Early cases may be managed with antibiotics. More severe cases usually require surgical drainage.

237
Q

A 32 year old female presents with recurrent episodes of rhinorrhoea, the discharge is watery. She has a medical history of asthma and intolerance of aspirin. On examination she has multiple soft, semi- transparent polyps within her nasal cavity.

A.	Ethmoid sinus cancer
B.	Maxillary sinus cancer
C.	Ethmoid adenoma
D.	Maxillary adenoma
E.	Ethmoidal fracture
F.	Nasal polyps
G.	Sphenoid osteoma
H.	Ethmoidal sinusitis
I.	Maxillary sinusitis
A

Nasal polyps

The combination of nasal polyps and atopy is well described. Some cases will respond favourably to systemic steroids and avoid surgery.

Diseases of nose and sinuses
Benign Tumours
Simple papillomas may be an incidental finding or present with obstructive symptoms. Excision under general anaesthesia is sufficient management.
Transitional cell papillomas may be more extensive and produce obstructive symptoms. Erosion of local structures is a recognised complication. These lesions may rarely undergo malignant transformation and therefore careful and complete excision is required, some cases may require partial or total maxillectomy.
Pleomorphic adenomas of the maxillary sinuses are reported but are extremely rare, their symptoms typically include nasal obstruction and pain if the sinus is obstructed. Treatment is by complete surgical excision, the diagnosis is not infrequently made post operatively.
Benign osteomas may develop in the paranasal sinuses, the frontal sinus is the most frequent location of such lesions. Symptoms include; pain, rhinorrhoea and anosmia. Most osteomas may be observed if asymptomatic, sphenoid osteomas should be resected soon after diagnosis as enlargement may compromise visual fields. Many sinus osteomas can now be resected endoscopically, complete surgical resection is required.
Nasal polyps are benign lesions of the ethmoid sinus mucosa. Many patients may also have asthma, cystic fibrosis and a sensitivity to aspirin. Symptoms include watery rhinorrhoea, infection and anosmia. The polyps are usually a semi transparent grey mass. They are rare in childhood. Treatment is either with systemic steroids or surgical resection. The latter should be combined with antral washout. Low dose, nasal, steroid drops may reduce the risk of recurrence.

Malignant disease
Malignancies encountered in the nose and paranasal sinuses include; adenoid cystic carcinoma, squamous cell carcinoma and adenocarcinoma.
Adenocarcinoma of the paranasal sinuses and nasopharynx is strongly linked to exposure to hard wood dust (after >10 years exposure).
Adenoid cystic carcinoma usually originate in the smaller salivary glands.
The majority of cancers (50%) arise from the lateral nasal wall, a smaller number (33%) arise from the maxillary antrum, ethmoid and sphenoid cancers comprise only 7%.
Signs of malignancy on clinical examination include loose teeth, cranial nerve palsies and lymphadenopathy.
Nasopharyngeal cancers are most common in individuals presenting from China and Asia and are linked to viral infection with Epstein Barr Virus. Radiotherapy and chemotherapy are the most commonly used modalities.

Maxillary sinusitis
Common symptoms include post nasal discharge, pain, headache and toothache.
Imaging may show a fluid level in the antrum.
Common organisms include Haemophilus influenzae or Streptococcus pneumoniae.
Treatment with antral lavage may facilitate diagnosis and relieve symptoms. Antimicrobial therapy has to be continued for long periods. Antrostomy may be needed.

Frontoethmoidal sinusitis
Usually presents with frontal headache, nasal obstruction and altered sense of smell.
Inflammation may progress to involve periorbital tissues. Ocular symptoms may occur and secondary CNS involvement brought about by infection entering via emissary veins.
CT scanning is the imaging modality of choice. Early cases may be managed with antibiotics. More severe cases usually require surgical drainage.

238
Q

A child is brought to casualty complaining of a headache and a sensation of pressure between the eyes. On examination she is febrile with a smooth swelling overlying the superomedial aspect of the right eye. The eye is uncomfortable and there is a purulent discharge from the inner canthus.

A.	Ethmoid sinus cancer
B.	Maxillary sinus cancer
C.	Ethmoid adenoma
D.	Maxillary adenoma
E.	Ethmoidal fracture
F.	Nasal polyps
G.	Sphenoid osteoma
H.	Ethmoidal sinusitis
I.	Maxillary sinusitis
A

Ethmoidal sinusitis

Ethmoidal sinusitis may spread to the periorbital tissues resulting in periorbital cellulitis. The superomedial distribution makes a maxillary sinusitis less likely.

Diseases of nose and sinuses
Benign Tumours
Simple papillomas may be an incidental finding or present with obstructive symptoms. Excision under general anaesthesia is sufficient management.
Transitional cell papillomas may be more extensive and produce obstructive symptoms. Erosion of local structures is a recognised complication. These lesions may rarely undergo malignant transformation and therefore careful and complete excision is required, some cases may require partial or total maxillectomy.
Pleomorphic adenomas of the maxillary sinuses are reported but are extremely rare, their symptoms typically include nasal obstruction and pain if the sinus is obstructed. Treatment is by complete surgical excision, the diagnosis is not infrequently made post operatively.
Benign osteomas may develop in the paranasal sinuses, the frontal sinus is the most frequent location of such lesions. Symptoms include; pain, rhinorrhoea and anosmia. Most osteomas may be observed if asymptomatic, sphenoid osteomas should be resected soon after diagnosis as enlargement may compromise visual fields. Many sinus osteomas can now be resected endoscopically, complete surgical resection is required.
Nasal polyps are benign lesions of the ethmoid sinus mucosa. Many patients may also have asthma, cystic fibrosis and a sensitivity to aspirin. Symptoms include watery rhinorrhoea, infection and anosmia. The polyps are usually a semi transparent grey mass. They are rare in childhood. Treatment is either with systemic steroids or surgical resection. The latter should be combined with antral washout. Low dose, nasal, steroid drops may reduce the risk of recurrence.

Malignant disease
Malignancies encountered in the nose and paranasal sinuses include; adenoid cystic carcinoma, squamous cell carcinoma and adenocarcinoma.
Adenocarcinoma of the paranasal sinuses and nasopharynx is strongly linked to exposure to hard wood dust (after >10 years exposure).
Adenoid cystic carcinoma usually originate in the smaller salivary glands.
The majority of cancers (50%) arise from the lateral nasal wall, a smaller number (33%) arise from the maxillary antrum, ethmoid and sphenoid cancers comprise only 7%.
Signs of malignancy on clinical examination include loose teeth, cranial nerve palsies and lymphadenopathy.
Nasopharyngeal cancers are most common in individuals presenting from China and Asia and are linked to viral infection with Epstein Barr Virus. Radiotherapy and chemotherapy are the most commonly used modalities.

Maxillary sinusitis
Common symptoms include post nasal discharge, pain, headache and toothache.
Imaging may show a fluid level in the antrum.
Common organisms include Haemophilus influenzae or Streptococcus pneumoniae.
Treatment with antral lavage may facilitate diagnosis and relieve symptoms. Antimicrobial therapy has to be continued for long periods. Antrostomy may be needed.

Frontoethmoidal sinusitis
Usually presents with frontal headache, nasal obstruction and altered sense of smell.
Inflammation may progress to involve periorbital tissues. Ocular symptoms may occur and secondary CNS involvement brought about by infection entering via emissary veins.
CT scanning is the imaging modality of choice. Early cases may be managed with antibiotics. More severe cases usually require surgical drainage.

239
Q

A 33 year old lady develops a thunderclap headache and collapses. A CT scan shows that she has developed a subarachnoid haemorrhage. She currently has no evidence of raised intracranial pressure. Which of the following drugs should be administered?

	None
	Atenotol
	Labetolol
	Nimodipine
	Mannitol
A

Nimodipine is a calcium channel blocker. It reduces cerebral vasospasm and improves outcomes. It is administered to most cases of sub arachnoid haemorrhage.

Sub arachnoid haemorrhage
Spontaneous intracranial haemorrhage
Most commonly sub arachnoid haemorrhage. It is due to intra cranial aneurysm in 85% cases. Approximately 10% of cases will have normal angiography and the cause will remain unclear. Patients with inherited connective tissue disorders are at higher risk although most cases are sporadic.
>95% cases will have headache (often thunderclap)
>15% will have coma

Investigation
CT scan for all (although as CSF blood clears the sensitivity declines)
Lumbar puncture if CT normal (very unlikely if normal)
CT angiogram to look for aneurysms.

Management
Supportive treatment, optimising BP (not too high if untreated aneurysm) and ventilation if needed.
Nimodipine reduces cerebral vasospasm and reduces poor outcomes.
Untreated patients most likely to rebleed in first 2 weeks.
Patients developing hydrocephalus will need a V-P shunt (external ventricular drain acutely).
Electrolytes require careful monitoring and hyponatraemia is common.

Treatment of aneurysm
>80% aneuryms arise from the anterior circulation
Craniotomy and clipping of aneurysm is standard treatment, alternatively suitable lesions may be coiled using an endovascular approach. Where both options are suitable data suggests that outcomes are better with coiling than surgery.

240
Q

A 52 year old man with long standing Barretts oesophagus is diagnosed with high grade dysplasia on recent endoscopy. The lesions are multifocal and mainly distally sited.

A.	Oesophagectomy
B.	Endoscopic sub mucosal dissection
C.	Photodynamic therapy
D.	Insertion of oesophageal stent
E.	Chemotherapy
F.	Radiotherapy
A

Oesophagectomy

Some may argue for local therapy. However, in young patients who are otherwise fit, multifocal disease such as this should probably be resected.

Treatment of oesophageal cancer
In general resections are not offered to those patients with distant metastasis, and usually not to those with N2 disease.
Local nodal involvement is not in itself a contra indication to resection.
Surgical resection is the mainstay of treatment.
Neoadjuvent chemotherapy is given in most cases prior to surgery.
In situ disease may be managed by endoscopic mucosal resection, although this is still debated.
In patients with lower third lesions an Ivor - Lewis type procedure is most commonly performed. Very distal tumours may be suitable to a transhiatal procedure. Which is an attractive option as the penetration of two visceral cavities required for an Ivor- Lewis type procedure increases the morbidity considerably.
More proximal lesions will require a total oesphagectomy (Mckeown type) with anastomosis to the cervical oesophagus.
Patients with unresectable disease may derive benefit from local ablative procedures, palliative chemotherapy or stent insertion.

Operative details of Ivor- Lewis procedure
Combined laparotomy and right thoracotomy

Indication
Lower and middle third oesophageal tumours

Preparation
Staging with a combination of CT chest abdomen and pelvis- if no metastatic disease detected then patients will undergo a staging laparoscopy to detect peritoneal disease.
If both these modalities are negative then patients will finally undergo a PET CT scan to detect occult metastatic disease. Only in those whom no evidence of advanced disease is detected will proceed to resection.
Patients receive a GA, double lumen endotracheal tube to allow for lung deflation, CVP and arterial monitoring.

Procedure
A rooftop incision is made to access the stomach and duodenum.

Laparotomy To mobilize the stomach
The greater omentum is incised away from its attachment to the right gastroepiploic vessels along the greater curvature of the stomach.
Then the short gastric vessels are ligated and detached from the greater curvature from the spleen.
The lesser omentum is incised, preserving the right gastric artery.
The retroperitoneal attachments of the duodenum in its second and third portions are incised, allowing the pylorus to reach the oesophageal hiatus. Some surgeons perform a pyloroplasty at this point to facilitate gastric emptying.
The left gastric vessels are then ligated, avoiding any injury to the common hepatic or splenic arteries. Care must be taken to avoid inadvertently devascularising the liver owing to variations in anatomy.

Right Thoracotomy Oesophageal resection and oesophagogastric anastomosis
Through 5th intercostal space
Dissection performed 10cm above the tumour
This may involve transection of the azygos vein.
The oesophagus is then removed with the stomach creating a gastric tube.
An anastomosis is created.

The chest is closed with underwater seal drainage and tube drains to the abdominal cavity.

Post operatively
Patients will typically recover in ITU initially.
A nasogastric tube will have been inserted intraoperatively and must remain in place during the early phases of recovery.
Post operatively these patients are at relatively high risk of developing complications:

  • Atelectasis- due to the effects of thoracotomy and lung collapse
  • Anastomotic leakage. The risk is relatively high owing to the presence of a relatively devascularised stomach. Often the only blood supply is from the gastroepiploic artery as all others will have been divided. If a leak does occur then many will attempt to manage conservatively with prolonged nasogastric tube drainage and TPN. The reality is that up to 50% of patients developing an anastomotic leak will not survive to discharge.
  • Delayed gastric emptying (may be avoided by performing a pyloroplasty).
241
Q

A 82 year old man presents with dysphagia. He is investigated and found to have an adenocarcinoma of the distal oesophagus. His staging investigations have revealed a solitary metastatic lesion in the right lobe of his liver.

A.	Oesophagectomy
B.	Endoscopic sub mucosal dissection
C.	Photodynamic therapy
D.	Insertion of oesophageal stent
E.	Chemotherapy
F.	Radiotherapy
A

Insertion of oesophageal stent

Although he may be palliated with chemotherapy a stent will produce the quickest clinical response. Metastatic disease is usually a contra indication to oesophageal resection

Treatment of oesophageal cancer
In general resections are not offered to those patients with distant metastasis, and usually not to those with N2 disease.
Local nodal involvement is not in itself a contra indication to resection.
Surgical resection is the mainstay of treatment.
Neoadjuvent chemotherapy is given in most cases prior to surgery.
In situ disease may be managed by endoscopic mucosal resection, although this is still debated.
In patients with lower third lesions an Ivor - Lewis type procedure is most commonly performed. Very distal tumours may be suitable to a transhiatal procedure. Which is an attractive option as the penetration of two visceral cavities required for an Ivor- Lewis type procedure increases the morbidity considerably.
More proximal lesions will require a total oesphagectomy (Mckeown type) with anastomosis to the cervical oesophagus.
Patients with unresectable disease may derive benefit from local ablative procedures, palliative chemotherapy or stent insertion.

Operative details of Ivor- Lewis procedure
Combined laparotomy and right thoracotomy

Indication
Lower and middle third oesophageal tumours

Preparation
Staging with a combination of CT chest abdomen and pelvis- if no metastatic disease detected then patients will undergo a staging laparoscopy to detect peritoneal disease.
If both these modalities are negative then patients will finally undergo a PET CT scan to detect occult metastatic disease. Only in those whom no evidence of advanced disease is detected will proceed to resection.
Patients receive a GA, double lumen endotracheal tube to allow for lung deflation, CVP and arterial monitoring.

Procedure
A rooftop incision is made to access the stomach and duodenum.

Laparotomy To mobilize the stomach
The greater omentum is incised away from its attachment to the right gastroepiploic vessels along the greater curvature of the stomach.
Then the short gastric vessels are ligated and detached from the greater curvature from the spleen.
The lesser omentum is incised, preserving the right gastric artery.
The retroperitoneal attachments of the duodenum in its second and third portions are incised, allowing the pylorus to reach the oesophageal hiatus. Some surgeons perform a pyloroplasty at this point to facilitate gastric emptying.
The left gastric vessels are then ligated, avoiding any injury to the common hepatic or splenic arteries. Care must be taken to avoid inadvertently devascularising the liver owing to variations in anatomy.

Right Thoracotomy Oesophageal resection and oesophagogastric anastomosis
Through 5th intercostal space
Dissection performed 10cm above the tumour
This may involve transection of the azygos vein.
The oesophagus is then removed with the stomach creating a gastric tube.
An anastomosis is created.

The chest is closed with underwater seal drainage and tube drains to the abdominal cavity.

Post operatively
Patients will typically recover in ITU initially.
A nasogastric tube will have been inserted intraoperatively and must remain in place during the early phases of recovery.
Post operatively these patients are at relatively high risk of developing complications:

  • Atelectasis- due to the effects of thoracotomy and lung collapse
  • Anastomotic leakage. The risk is relatively high owing to the presence of a relatively devascularised stomach. Often the only blood supply is from the gastroepiploic artery as all others will have been divided. If a leak does occur then many will attempt to manage conservatively with prolonged nasogastric tube drainage and TPN. The reality is that up to 50% of patients developing an anastomotic leak will not survive to discharge.
  • Delayed gastric emptying (may be avoided by performing a pyloroplasty).
242
Q

An 83 year old lady with long standing Barretts oesophagus is diagnosed with a 1cm focus of high grade dysplasia 3cm from the gastrooesophageal junction.

A.	Oesophagectomy
B.	Endoscopic sub mucosal dissection
C.	Photodynamic therapy
D.	Insertion of oesophageal stent
E.	Chemotherapy
F.	Radiotherapy
A

Endoscopic sub mucosal dissection

As she is elderly and the disease localised EMR is an appropriate first line step.
The technique involves raising the mucosa containing the lesion and then using an endoscopic snare to remove it. This technique is therefore minimally invasive. However, it is only suitable for early superficial lesions. Deeper invasion would carry a high risk of recurrence

Treatment of oesophageal cancer
In general resections are not offered to those patients with distant metastasis, and usually not to those with N2 disease.
Local nodal involvement is not in itself a contra indication to resection.
Surgical resection is the mainstay of treatment.
Neoadjuvent chemotherapy is given in most cases prior to surgery.
In situ disease may be managed by endoscopic mucosal resection, although this is still debated.
In patients with lower third lesions an Ivor - Lewis type procedure is most commonly performed. Very distal tumours may be suitable to a transhiatal procedure. Which is an attractive option as the penetration of two visceral cavities required for an Ivor- Lewis type procedure increases the morbidity considerably.
More proximal lesions will require a total oesphagectomy (Mckeown type) with anastomosis to the cervical oesophagus.
Patients with unresectable disease may derive benefit from local ablative procedures, palliative chemotherapy or stent insertion.

Operative details of Ivor- Lewis procedure
Combined laparotomy and right thoracotomy

Indication
Lower and middle third oesophageal tumours

Preparation
Staging with a combination of CT chest abdomen and pelvis- if no metastatic disease detected then patients will undergo a staging laparoscopy to detect peritoneal disease.
If both these modalities are negative then patients will finally undergo a PET CT scan to detect occult metastatic disease. Only in those whom no evidence of advanced disease is detected will proceed to resection.
Patients receive a GA, double lumen endotracheal tube to allow for lung deflation, CVP and arterial monitoring.

Procedure
A rooftop incision is made to access the stomach and duodenum.

Laparotomy To mobilize the stomach
The greater omentum is incised away from its attachment to the right gastroepiploic vessels along the greater curvature of the stomach.
Then the short gastric vessels are ligated and detached from the greater curvature from the spleen.
The lesser omentum is incised, preserving the right gastric artery.
The retroperitoneal attachments of the duodenum in its second and third portions are incised, allowing the pylorus to reach the oesophageal hiatus. Some surgeons perform a pyloroplasty at this point to facilitate gastric emptying.
The left gastric vessels are then ligated, avoiding any injury to the common hepatic or splenic arteries. Care must be taken to avoid inadvertently devascularising the liver owing to variations in anatomy.

Right Thoracotomy Oesophageal resection and oesophagogastric anastomosis
Through 5th intercostal space
Dissection performed 10cm above the tumour
This may involve transection of the azygos vein.
The oesophagus is then removed with the stomach creating a gastric tube.
An anastomosis is created.

The chest is closed with underwater seal drainage and tube drains to the abdominal cavity.

Post operatively
Patients will typically recover in ITU initially.
A nasogastric tube will have been inserted intraoperatively and must remain in place during the early phases of recovery.
Post operatively these patients are at relatively high risk of developing complications:

  • Atelectasis- due to the effects of thoracotomy and lung collapse
  • Anastomotic leakage. The risk is relatively high owing to the presence of a relatively devascularised stomach. Often the only blood supply is from the gastroepiploic artery as all others will have been divided. If a leak does occur then many will attempt to manage conservatively with prolonged nasogastric tube drainage and TPN. The reality is that up to 50% of patients developing an anastomotic leak will not survive to discharge.
  • Delayed gastric emptying (may be avoided by performing a pyloroplasty).
243
Q

A 3 day old baby presents with recurrent episodes of choking and cyanotic episodes. There is a history of polyhydramnios.

A.	Meconium ileus
B.	Biliary atresia
C.	Oesophageal atresia
D.	Pyloric stenosis
E.	Intussusception
F.	Malrotation
G.	Hirschsprung disease
H.	Mesenteric adenitis.
A

Oesophageal atresia

Diagnosis is confirmed when an nasogastric tube fails to reach the stomach.

Pyloric stenosis
M>F
5-10% Family history in parents
Projectile non bile stained vomiting at 4-6 weeks of life
Diagnosis is made by test feed or USS
Treatment: Ramstedt pyloromyotomy (open or laparoscopic)
Acute appendicitis
Uncommon under 3 years
When occurs may present atypically
Mesenteric adenitis
Central abdominal pain and URTI
Conservative management
Intussusception
Telescoping bowel
Proximal to or at the level of, ileocaecal valve
6-9 months age
Colicky pain, diarrhoea and vomiting, sausage shaped mass, red jelly stool.
Treatment: reduction with air insufflation
Malrotation
High caecum at the midline
Feature in exomphalos, congenital diaphragmatic hernia, intrinsic duodenal atresia
May be complicated by development of volvulus, infant with volvulus may have bile stained vomiting
Diagnosis is made by upper GI contrast study and USS
Treatment is by laparotomy, if volvulus is present (or at high risk of occurring then a ladds procedure is performed
Hirschsprung’s disease
Absence of ganglion cells from myenteric and submucosal plexuses
Occurs in 1/5000 births
Full thickness rectal biopsy for diagnosis
Delayed passage of meconium and abdominal distension
Treatment is with rectal washouts initially, thereafter an anorectal pull through procedure
Oesophageal atresia
Associated with tracheo-oesophageal fistula and polyhydramnios
May present with choking and cyanotic spells following aspiration
VACTERL associations
Meconium ileus
Usually delayed passage of meconium and abdominal distension
Majority have cystic fibrosis
X-Rays may not show a fluid level as the meconium is viscid (depends upon feeding), PR contrast studies may dislodge meconium plugs and be therapeutic
Infants who do not respond to PR contrast and NG N-acetyl cysteine will require surgery to remove the plugs
Biliary atresia
Jaundice > 14 days
Increased conjugated bilirubin
Urgent Kasai procedure
Necrotising enterocolitis
Prematurity is the main risk factor
Early features include abdominal distension and passage of bloody stools
X-Rays may show pneumatosis intestinalis and evidence of free air
Increased risk when empirical antibiotics are given to infants beyond 5 days
Treatment is with total gut rest and TPN, babies with perforations will require laparotomy

244
Q

A 3 day old neonate is developing increasing problems with feeding. On examination she has a pan systolic murmur and her forearms have not developed properly.

A.	Meconium ileus
B.	Biliary atresia
C.	Oesophageal atresia
D.	Pyloric stenosis
E.	Intussusception
F.	Malrotation
G.	Hirschsprung disease
H.	Mesenteric adenitis.
A

Oesophageal atresia

This child has VACTERL, which is a combination of Vertebral, Ano-rectal, Cardiac, Tracheo-oesophageal, Renal and Radial limb anomalies. Half of babies with oesophageal atresia will have VACTERL.

Pyloric stenosis
M>F
5-10% Family history in parents
Projectile non bile stained vomiting at 4-6 weeks of life
Diagnosis is made by test feed or USS
Treatment: Ramstedt pyloromyotomy (open or laparoscopic)
Acute appendicitis
Uncommon under 3 years
When occurs may present atypically
Mesenteric adenitis
Central abdominal pain and URTI
Conservative management
Intussusception
Telescoping bowel
Proximal to or at the level of, ileocaecal valve
6-9 months age
Colicky pain, diarrhoea and vomiting, sausage shaped mass, red jelly stool.
Treatment: reduction with air insufflation
Malrotation
High caecum at the midline
Feature in exomphalos, congenital diaphragmatic hernia, intrinsic duodenal atresia
May be complicated by development of volvulus, infant with volvulus may have bile stained vomiting
Diagnosis is made by upper GI contrast study and USS
Treatment is by laparotomy, if volvulus is present (or at high risk of occurring then a ladds procedure is performed
Hirschsprung’s disease
Absence of ganglion cells from myenteric and submucosal plexuses
Occurs in 1/5000 births
Full thickness rectal biopsy for diagnosis
Delayed passage of meconium and abdominal distension
Treatment is with rectal washouts initially, thereafter an anorectal pull through procedure
Oesophageal atresia
Associated with tracheo-oesophageal fistula and polyhydramnios
May present with choking and cyanotic spells following aspiration
VACTERL associations
Meconium ileus
Usually delayed passage of meconium and abdominal distension
Majority have cystic fibrosis
X-Rays may not show a fluid level as the meconium is viscid (depends upon feeding), PR contrast studies may dislodge meconium plugs and be therapeutic
Infants who do not respond to PR contrast and NG N-acetyl cysteine will require surgery to remove the plugs
Biliary atresia
Jaundice > 14 days
Increased conjugated bilirubin
Urgent Kasai procedure
Necrotising enterocolitis
Prematurity is the main risk factor
Early features include abdominal distension and passage of bloody stools
X-Rays may show pneumatosis intestinalis and evidence of free air
Increased risk when empirical antibiotics are given to infants beyond 5 days
Treatment is with total gut rest and TPN, babies with perforations will require laparotomy

245
Q

A 2 year old child has central abdominal pain. He has had a recent upper respiratory tract infection.

A.	Meconium ileus
B.	Biliary atresia
C.	Oesophageal atresia
D.	Pyloric stenosis
E.	Intussusception
F.	Malrotation
G.	Hirschsprung disease
H.	Mesenteric adenitis.
A

Mesenteric adenitis.

Mesenteric adenitis may complicate upper respiratory tract infection and clinical exclusion of appendicitis can be difficult.

Pyloric stenosis
M>F
5-10% Family history in parents
Projectile non bile stained vomiting at 4-6 weeks of life
Diagnosis is made by test feed or USS
Treatment: Ramstedt pyloromyotomy (open or laparoscopic)
Acute appendicitis
Uncommon under 3 years
When occurs may present atypically
Mesenteric adenitis
Central abdominal pain and URTI
Conservative management
Intussusception
Telescoping bowel
Proximal to or at the level of, ileocaecal valve
6-9 months age
Colicky pain, diarrhoea and vomiting, sausage shaped mass, red jelly stool.
Treatment: reduction with air insufflation
Malrotation
High caecum at the midline
Feature in exomphalos, congenital diaphragmatic hernia, intrinsic duodenal atresia
May be complicated by development of volvulus, infant with volvulus may have bile stained vomiting
Diagnosis is made by upper GI contrast study and USS
Treatment is by laparotomy, if volvulus is present (or at high risk of occurring then a ladds procedure is performed
Hirschsprung’s disease
Absence of ganglion cells from myenteric and submucosal plexuses
Occurs in 1/5000 births
Full thickness rectal biopsy for diagnosis
Delayed passage of meconium and abdominal distension
Treatment is with rectal washouts initially, thereafter an anorectal pull through procedure
Oesophageal atresia
Associated with tracheo-oesophageal fistula and polyhydramnios
May present with choking and cyanotic spells following aspiration
VACTERL associations
Meconium ileus
Usually delayed passage of meconium and abdominal distension
Majority have cystic fibrosis
X-Rays may not show a fluid level as the meconium is viscid (depends upon feeding), PR contrast studies may dislodge meconium plugs and be therapeutic
Infants who do not respond to PR contrast and NG N-acetyl cysteine will require surgery to remove the plugs
Biliary atresia
Jaundice > 14 days
Increased conjugated bilirubin
Urgent Kasai procedure
Necrotising enterocolitis
Prematurity is the main risk factor
Early features include abdominal distension and passage of bloody stools
X-Rays may show pneumatosis intestinalis and evidence of free air
Increased risk when empirical antibiotics are given to infants beyond 5 days
Treatment is with total gut rest and TPN, babies with perforations will require laparotomy

246
Q

A 63 year old man is admitted with rest pain and foot ulceration. An angiogram shows a 3 cm area of occlusion of the distal superficial femoral artery with 3 vessel run off. His ankle - brachial pressure index is 0.4.

A.	Primary amputation
B.	Angioplasty
C.	Arterial bypass surgery using vein
D.	Arterial bypass surgery using PTFE
E.	Conservative management with medical therapy and exercise
F.	Watch and wait
G.	Duplex scanning
A

Angioplasty

Short segment disease and good run off with tissue loss is a compelling indication for angioplasty. He should receive aspirin and a statin if not already taking them.

Peripheral vascular disease

Indications for surgery to revascularise the lower limb
Intermittent claudication
Critical ischaemia
Ulceration
Gangrene

Intermittent claudication that is not disabling may provide a relative indication, whilst the other complaints are often absolute indications depending upon the frailty of the patient.

Assessment
Clinical examination
Ankle brachial pressure index measurement
Duplex arterial ultrasound
Angiography (standard, CT or MRI): usually performed only if intervention being considered.

Angioplasty
In order for angioplasty to be undertaken successfully the artery has to be accessible. The lesion relatively short and reasonable distal vessel runoff. Longer lesions may be amenable to sub-intimal angioplasty.

Surgery
Surgery will be undertaken where attempts at angioplasty have either failed or are unsuitable. Bypass essentially involves bypassing the affected arterial segment by utilising a graft to run from above the disease to below the disease. As with angioplasty good runoff improves the outcome.

Some key concepts with bypass surgery

Superficial femoral artery occlusion to the above knee popliteal
In the ideal scenario, vein (either in situ or reversed LSV) would the used as a conduit. However, prosthetic material has reasonable 5 year patency rates and some would advocate using this in preference to vein so that vein can be used for other procedures in the future. In general terms either technique is usually associated with an excellent outcome (if run off satisfactory).

Procedure
Artery dissected out, IV heparin 3,000 units given and then the vessels are cross clamped
Longitudinal arteriotomy
Graft cut to size and tunneled to arteriotomy sites
Anastomosis to femoral artery usually with 5/0 ‘double ended’ Prolene suture
Distal anastomosis usually using 6/0 ‘double ended’ Prolene

Distal disease
Femoro-distal bypass surgery takes longer to perform, is more technically challenging and has higher failure rates.
In elderly diabetic patients with poor runoff a primary amputation may well be a safer and more effective option. There is no point in embarking on this type of surgery in patients who are wheelchair bound.
In femorodistal bypasses vein gives superior outcomes to PTFE.

Rules
Vein mapping 1st to see whether there is suitable vein (the preferred conduit). Sub intimal hyperplasia occurs early when PTFE is used for the distal anastomosis and will lead to early graft occlusion and failure.
Essential operative procedure as for above knee fem-pop.
If there is insufficient vein for the entire conduit then vein can be attached to the end of the PTFE graft and then used for the distal anastomosis. This type of ‘vein boot’ is technically referred to as a Miller Cuff and is associated with better patency rates than PTFE alone.
Remember the more distal the arterial anastomosis the lower the success rate.

247
Q

A 72 year old man present in the vascular clinic with calf pain present on walking 100 yards. He is an ex-smoker and lives alone. On examination he has reasonable leg pulses. His right dorsalis pedis pulse gives a monophasic doppler signal with an ankle brachial pressure index measurement of 0.7. All other pressures are acceptable. There is no evidence of ulceration or gangrene.

A.	Primary amputation
B.	Angioplasty
C.	Arterial bypass surgery using vein
D.	Arterial bypass surgery using PTFE
E.	Conservative management with medical therapy and exercise
F.	Watch and wait
G.	Duplex scanning
A

Conservative management with medical therapy and exercise

Structured exercise programmes combined with medical therapy will improve many patients. Should his symptoms worsen or fail to improve then imaging with duplex scanning would be required.

Peripheral vascular disease

Indications for surgery to revascularise the lower limb
Intermittent claudication
Critical ischaemia
Ulceration
Gangrene

Intermittent claudication that is not disabling may provide a relative indication, whilst the other complaints are often absolute indications depending upon the frailty of the patient.

Assessment
Clinical examination
Ankle brachial pressure index measurement
Duplex arterial ultrasound
Angiography (standard, CT or MRI): usually performed only if intervention being considered.

Angioplasty
In order for angioplasty to be undertaken successfully the artery has to be accessible. The lesion relatively short and reasonable distal vessel runoff. Longer lesions may be amenable to sub-intimal angioplasty.

Surgery
Surgery will be undertaken where attempts at angioplasty have either failed or are unsuitable. Bypass essentially involves bypassing the affected arterial segment by utilising a graft to run from above the disease to below the disease. As with angioplasty good runoff improves the outcome.

Some key concepts with bypass surgery

Superficial femoral artery occlusion to the above knee popliteal
In the ideal scenario, vein (either in situ or reversed LSV) would the used as a conduit. However, prosthetic material has reasonable 5 year patency rates and some would advocate using this in preference to vein so that vein can be used for other procedures in the future. In general terms either technique is usually associated with an excellent outcome (if run off satisfactory).

Procedure
Artery dissected out, IV heparin 3,000 units given and then the vessels are cross clamped
Longitudinal arteriotomy
Graft cut to size and tunneled to arteriotomy sites
Anastomosis to femoral artery usually with 5/0 ‘double ended’ Prolene suture
Distal anastomosis usually using 6/0 ‘double ended’ Prolene

Distal disease
Femoro-distal bypass surgery takes longer to perform, is more technically challenging and has higher failure rates.
In elderly diabetic patients with poor runoff a primary amputation may well be a safer and more effective option. There is no point in embarking on this type of surgery in patients who are wheelchair bound.
In femorodistal bypasses vein gives superior outcomes to PTFE.

Rules
Vein mapping 1st to see whether there is suitable vein (the preferred conduit). Sub intimal hyperplasia occurs early when PTFE is used for the distal anastomosis and will lead to early graft occlusion and failure.
Essential operative procedure as for above knee fem-pop.
If there is insufficient vein for the entire conduit then vein can be attached to the end of the PTFE graft and then used for the distal anastomosis. This type of ‘vein boot’ is technically referred to as a Miller Cuff and is associated with better patency rates than PTFE alone.
Remember the more distal the arterial anastomosis the lower the success rate.

248
Q

An 83 year old lady is admitted from a nursing home with infected lower leg ulcers. She underwent an attempted long superficial femoral artery sub initimal angioplasty 2 weeks previously. This demonstrated poor runoff below the knee.

A.	Primary amputation
B.	Angioplasty
C.	Arterial bypass surgery using vein
D.	Arterial bypass surgery using PTFE
E.	Conservative management with medical therapy and exercise
F.	Watch and wait
G.	Duplex scanning
A

Primary amputation

Poor runoff and sepsis would equate to poor outcome with attempted bypass surgery.

Peripheral vascular disease

Indications for surgery to revascularise the lower limb
Intermittent claudication
Critical ischaemia
Ulceration
Gangrene

Intermittent claudication that is not disabling may provide a relative indication, whilst the other complaints are often absolute indications depending upon the frailty of the patient.

Assessment
Clinical examination
Ankle brachial pressure index measurement
Duplex arterial ultrasound
Angiography (standard, CT or MRI): usually performed only if intervention being considered.

Angioplasty
In order for angioplasty to be undertaken successfully the artery has to be accessible. The lesion relatively short and reasonable distal vessel runoff. Longer lesions may be amenable to sub-intimal angioplasty.

Surgery
Surgery will be undertaken where attempts at angioplasty have either failed or are unsuitable. Bypass essentially involves bypassing the affected arterial segment by utilising a graft to run from above the disease to below the disease. As with angioplasty good runoff improves the outcome.

Some key concepts with bypass surgery

Superficial femoral artery occlusion to the above knee popliteal
In the ideal scenario, vein (either in situ or reversed LSV) would the used as a conduit. However, prosthetic material has reasonable 5 year patency rates and some would advocate using this in preference to vein so that vein can be used for other procedures in the future. In general terms either technique is usually associated with an excellent outcome (if run off satisfactory).

Procedure
Artery dissected out, IV heparin 3,000 units given and then the vessels are cross clamped
Longitudinal arteriotomy
Graft cut to size and tunneled to arteriotomy sites
Anastomosis to femoral artery usually with 5/0 ‘double ended’ Prolene suture
Distal anastomosis usually using 6/0 ‘double ended’ Prolene

Distal disease
Femoro-distal bypass surgery takes longer to perform, is more technically challenging and has higher failure rates.
In elderly diabetic patients with poor runoff a primary amputation may well be a safer and more effective option. There is no point in embarking on this type of surgery in patients who are wheelchair bound.
In femorodistal bypasses vein gives superior outcomes to PTFE.

Rules
Vein mapping 1st to see whether there is suitable vein (the preferred conduit). Sub intimal hyperplasia occurs early when PTFE is used for the distal anastomosis and will lead to early graft occlusion and failure.
Essential operative procedure as for above knee fem-pop.
If there is insufficient vein for the entire conduit then vein can be attached to the end of the PTFE graft and then used for the distal anastomosis. This type of ‘vein boot’ is technically referred to as a Miller Cuff and is associated with better patency rates than PTFE alone.
Remember the more distal the arterial anastomosis the lower the success rate.

249
Q

Which of the following is not a typical feature of acute appendicitis?

	Neutrophilia
	Profuse vomiting
	Anorexia
	Low grade pyrexia
	Small amounts of protein on urine analysis
A

Profuse vomiting and diarrhoea are rare in early appendicitis

Whilst patients may vomit once or twice, profuse vomiting is unusual, and would fit more with gastroenteritis or an ileus. A trace of protein is not an uncommon occurrence in acute appendicitis. A free lying pelvic appendix may result in localised bladder irritation, with inflammation occurring as a secondary phenomena. This latter feature may result in patients being incorrectly diagnosed as having a urinary tract infection. A urine dipstick test is useful in differentiating between the two conditions.

Appendicitis

History
Peri umbilical abdominal pain (visceral stretching of appendix lumen and appendix is mid gut structure) radiating to the right iliac fossa due to localised parietal peritoneal inflammation.
Vomit once or twice but marked and persistent vomiting is unusual.
Diarrhoea is rare. However, pelvic appendicitis may cause localised rectal irritation and some loose stools. A pelvic abscess may also cause diarrhoea.
Mild pyrexia is common - temperature is usually 37.5 -38oC. Higher temperatures are more typical of conditions like mesenteric adenitis.
Anorexia is very common. It is very unusual for patients with appendicitis to be hungry.

Examination
Generalised peritonitis if perforation has occurred or localised peritonism.
Retrocaecal appendicitis may have relatively few signs.
Digital rectal examination may reveal boggy sensation if pelvic abscess is present, or even tenderness with a pelvic appendix.

Diagnosis
Typically raised inflammatory markers coupled with compatible history and examination findings should be enough to justify appendicectomy.
Urine analysis may show mild leucocytosis but no nitrites.
Ultrasound is useful in females where pelvic organ pathology is suspected. Although it is not always possible to visualise the appendix on ultrasound, the presence of free fluid (always pathological in males) should raise suspicion.

Ultrasound examination may show evidence of luminal obstruction and thickening of the appendiceal wall

Treatment
Appendicectomy which can be performed via either an open or laparoscopic approach.
Administration of metronidazole reduces wound infection rates.
Patients with perforated appendicitis require copious abdominal lavage.
Patients without peritonitis who have an appendix mass should receive broad spectrum antibiotics and consideration given to performing an interval appendicectomy.
Be wary in the older patients who may have either an underlying caecal malignancy or perforated sigmoid diverticular disease.

250
Q

An 28 year old man presents with a direct inguinal hernia. A decision is made to perform an open inguinal hernia repair. Which of the following is the best option for abdominal wall reconstruction in this case?

Suture plication of the transversalis fascia using PDS only
Suture plication of the hernial defect with nylon and placement of prolene mesh anterior to external oblique
Suture plication of the hernia defect using nylon and re-enforcing with a sutured repair of the abdominal wall
Sutured repair of the hernial defect with prolene and placement of prolene mesh over the cord structures in the inguinal canal
Sutured repair of the hernial defect using nylon and placement of a prolene mesh posterior to the cord structures
A

Sutured repair of the hernial defect using nylon and placement of a prolene mesh posterior to the cord structures
Laparoscopic repair- bilateral and recurrent cases

During an inguinal hernia repair in males the cord structures will always lie anterior to the mesh. In the conventional open repairs the cord structures are mobilised and the mesh placed behind them, with a slit made to allow passage of the cord structures through the deep inguinal ring. Placement of the mesh over the cord structures results in chronic pain and usually a higher risk of recurrence.

Laparoscopic inguinal hernia repair is the procedure of choice for bilateral inguinal hernias.

Types of surgery include:
Onlay mesh repair (Lichtenstein style)
Inguinal herniorrhaphy
Shouldice repair
Darn repair
Laparoscopic mesh repair

Open mesh repair and laparoscopic repair are the two main procedures in mainstream use. The Shouldice repair is a useful procedure in cases where a mesh repair would be associated with increased risk of infection, e.g. repair of case with strangulated bowel, as it avoids the use of mesh. It is, however, far more technically challenging to perform.

Inguinal hernias occur when the abdominal viscera protrude through the anterior abdominal wall into the inguinal canal. They may be classified as being either direct or indirect. The distinction between these two rests on their relation to Hesselbach’s triangle.

Boundaries of Hesselbach’s Triangle
Medial: Rectus abdominis
Lateral: Inferior epigastric vessels
Inferior: Inguinal ligament

Hernias occurring within the triangle tend to be direct and those outside - indirect.

Diagnosis
Most cases are diagnosed clinically, a reducible swelling may be located at the level of the inguinal canal. Large hernia’s may extend down into the male scrotum, these will not trans-illuminate and it is not possible to “get above” the swelling.
Cases that are unclear on examination, but suspected from the history, may be further investigated using ultrasound or by performing a herniogram.

Treatment
Hernias associated with few symptoms may be managed conservatively. Symptomatic hernias or those which are at risk of developing complications are usually treated surgically.
First time hernias may be treated by performing an open inguinal hernia repair; the inguinal canal is opened, the hernia reduced and the defect repaired. A prosthetic mesh may be placed posterior to the cord structures to re-enforce the repair and reduce the risk of recurrence.
Recurrent hernias and those which are bilateral are generally managed with a laparoscopic approach. This may be via an intra or extra peritoneal route. As in open surgery a mesh is deployed. However, it will typically lie posterior to the deep ring.

Inguinal hernia in children
Inguinal hernias in children are almost always of an indirect type and therefore are usually dealt with by herniotomy, rather than herniorraphy. Neonatal hernias especially in those children born prematurely are at highest risk of strangulation and should be repaired urgently. Other hernias may be repaired on an elective basis.

251
Q

A newborn baby boy presents with mild abdominal distension and failure to pass meconium after 24 hours. X- Ray reveals dilated loops of bowel with fluid levels. The anus appears normally located.

A.	Ano-rectal atresia
B.	Pyloric stenosis
C.	Hirschsprungs disease
D.	Duodenal atresia
E.	Meconium ileus
F.	Intussusception
G.	Necrotising enterocolitis
H.	Intestinal volvulus
I.	Tracheo-oesophageal fistula
A

Hirschsprungs disease

Hirschsprung’s disease is an absence of ganglion cells in the neural plexus of the intestinal wall. It is more common in boys than girls. The delayed passage of meconium together with distension of abdomen is the usual clinical presentation. A plain abdominal x ray will demonstrate dilated loops of bowel with fluid levels and a barium enema can be helpful when it demonstrates a cone with dilated ganglionic proximal colon and the distal aganglionic bowel failing to distend.

Pyloric stenosis
M>F
5-10% Family history in parents
Projectile non bile stained vomiting at 4-6 weeks of life
Diagnosis is made by test feed or USS
Treatment: Ramstedt pyloromyotomy (open or laparoscopic)
Acute appendicitis
Uncommon under 3 years
When occurs may present atypically
Mesenteric adenitis
Central abdominal pain and URTI
Conservative management
Intussusception
Telescoping bowel
Proximal to or at the level of, ileocaecal valve
6-9 months age
Colicky pain, diarrhoea and vomiting, sausage shaped mass, red jelly stool.
Treatment: reduction with air insufflation
Malrotation
High caecum at the midline
Feature in exomphalos, congenital diaphragmatic hernia, intrinsic duodenal atresia
May be complicated by development of volvulus, infant with volvulus may have bile stained vomiting
Diagnosis is made by upper GI contrast study and USS
Treatment is by laparotomy, if volvulus is present (or at high risk of occurring then a ladds procedure is performed
Hirschsprung’s disease
Absence of ganglion cells from myenteric and submucosal plexuses
Occurs in 1/5000 births
Full thickness rectal biopsy for diagnosis
Delayed passage of meconium and abdominal distension
Treatment is with rectal washouts initially, thereafter an anorectal pull through procedure
Oesophageal atresia
Associated with tracheo-oesophageal fistula and polyhydramnios
May present with choking and cyanotic spells following aspiration
VACTERL associations
Meconium ileus
Usually delayed passage of meconium and abdominal distension
Majority have cystic fibrosis
X-Rays may not show a fluid level as the meconium is viscid (depends upon feeding), PR contrast studies may dislodge meconium plugs and be therapeutic
Infants who do not respond to PR contrast and NG N-acetyl cysteine will require surgery to remove the plugs
Biliary atresia
Jaundice > 14 days
Increased conjugated bilirubin
Urgent Kasai procedure
Necrotising enterocolitis
Prematurity is the main risk factor
Early features include abdominal distension and passage of bloody stools
X-Rays may show pneumatosis intestinalis and evidence of free air
Increased risk when empirical antibiotics are given to infants beyond 5 days
Treatment is with total gut rest and TPN, babies with perforations will require laparotomy

252
Q

A premature infant (30-week gestation) presents with distended and tense abdomen. She is passing blood and mucus per rectum, and she is also manifesting signs of sepsis.

A.	Ano-rectal atresia
B.	Pyloric stenosis
C.	Hirschsprungs disease
D.	Duodenal atresia
E.	Meconium ileus
F.	Intussusception
G.	Necrotising enterocolitis
H.	Intestinal volvulus
I.	Tracheo-oesophageal fistula
A

Necrotising enterocolitis

Necrotising enterocolitis is more common in premature infants. Mesenteric ischemia causes bacterial invasion of the mucosa leading to sepsis. Terminal ileum, caecum and the distal colon are commonly affected. The abdomen is distended and tense, and the infant passes blood and mucus per rectum. X -Ray of the abdomen shows distended loops of intestine and gas bubbles may be seen in the bowel wall.

Pyloric stenosis
M>F
5-10% Family history in parents
Projectile non bile stained vomiting at 4-6 weeks of life
Diagnosis is made by test feed or USS
Treatment: Ramstedt pyloromyotomy (open or laparoscopic)
Acute appendicitis
Uncommon under 3 years
When occurs may present atypically
Mesenteric adenitis
Central abdominal pain and URTI
Conservative management
Intussusception
Telescoping bowel
Proximal to or at the level of, ileocaecal valve
6-9 months age
Colicky pain, diarrhoea and vomiting, sausage shaped mass, red jelly stool.
Treatment: reduction with air insufflation
Malrotation
High caecum at the midline
Feature in exomphalos, congenital diaphragmatic hernia, intrinsic duodenal atresia
May be complicated by development of volvulus, infant with volvulus may have bile stained vomiting
Diagnosis is made by upper GI contrast study and USS
Treatment is by laparotomy, if volvulus is present (or at high risk of occurring then a ladds procedure is performed
Hirschsprung’s disease
Absence of ganglion cells from myenteric and submucosal plexuses
Occurs in 1/5000 births
Full thickness rectal biopsy for diagnosis
Delayed passage of meconium and abdominal distension
Treatment is with rectal washouts initially, thereafter an anorectal pull through procedure
Oesophageal atresia
Associated with tracheo-oesophageal fistula and polyhydramnios
May present with choking and cyanotic spells following aspiration
VACTERL associations
Meconium ileus
Usually delayed passage of meconium and abdominal distension
Majority have cystic fibrosis
X-Rays may not show a fluid level as the meconium is viscid (depends upon feeding), PR contrast studies may dislodge meconium plugs and be therapeutic
Infants who do not respond to PR contrast and NG N-acetyl cysteine will require surgery to remove the plugs
Biliary atresia
Jaundice > 14 days
Increased conjugated bilirubin
Urgent Kasai procedure
Necrotising enterocolitis
Prematurity is the main risk factor
Early features include abdominal distension and passage of bloody stools
X-Rays may show pneumatosis intestinalis and evidence of free air
Increased risk when empirical antibiotics are given to infants beyond 5 days
Treatment is with total gut rest and TPN, babies with perforations will require laparotomy

253
Q

A newborn baby boy presents with gross abdominal distension. He is diagnosed with cystic fibrosis and his abdominal x ray shows distended coils of small bowel, but no fluid levels.

A.	Ano-rectal atresia
B.	Pyloric stenosis
C.	Hirschsprungs disease
D.	Duodenal atresia
E.	Meconium ileus
F.	Intussusception
G.	Necrotising enterocolitis
H.	Intestinal volvulus
I.	Tracheo-oesophageal fistula
A

Meconium ileus

One in 5,000 newborns will have a distal small bowel obstruction secondary to abnormal bulky and viscid meconium. Ninety percent of these infants will have cystic fibrosis and the abnormal meconium is the result of deficient intestinal secretions. This condition presents during the first days of life with gross abdominal distension and bilious vomiting. x Ray of the abdomen shows distended coils of bowel and typical mottled ground glass appearance. Fluid levels are scarce as the meconium is viscid.

Pyloric stenosis
M>F
5-10% Family history in parents
Projectile non bile stained vomiting at 4-6 weeks of life
Diagnosis is made by test feed or USS
Treatment: Ramstedt pyloromyotomy (open or laparoscopic)
Acute appendicitis
Uncommon under 3 years
When occurs may present atypically
Mesenteric adenitis
Central abdominal pain and URTI
Conservative management
Intussusception
Telescoping bowel
Proximal to or at the level of, ileocaecal valve
6-9 months age
Colicky pain, diarrhoea and vomiting, sausage shaped mass, red jelly stool.
Treatment: reduction with air insufflation
Malrotation
High caecum at the midline
Feature in exomphalos, congenital diaphragmatic hernia, intrinsic duodenal atresia
May be complicated by development of volvulus, infant with volvulus may have bile stained vomiting
Diagnosis is made by upper GI contrast study and USS
Treatment is by laparotomy, if volvulus is present (or at high risk of occurring then a ladds procedure is performed
Hirschsprung’s disease
Absence of ganglion cells from myenteric and submucosal plexuses
Occurs in 1/5000 births
Full thickness rectal biopsy for diagnosis
Delayed passage of meconium and abdominal distension
Treatment is with rectal washouts initially, thereafter an anorectal pull through procedure
Oesophageal atresia
Associated with tracheo-oesophageal fistula and polyhydramnios
May present with choking and cyanotic spells following aspiration
VACTERL associations
Meconium ileus
Usually delayed passage of meconium and abdominal distension
Majority have cystic fibrosis
X-Rays may not show a fluid level as the meconium is viscid (depends upon feeding), PR contrast studies may dislodge meconium plugs and be therapeutic
Infants who do not respond to PR contrast and NG N-acetyl cysteine will require surgery to remove the plugs
Biliary atresia
Jaundice > 14 days
Increased conjugated bilirubin
Urgent Kasai procedure
Necrotising enterocolitis
Prematurity is the main risk factor
Early features include abdominal distension and passage of bloody stools
X-Rays may show pneumatosis intestinalis and evidence of free air
Increased risk when empirical antibiotics are given to infants beyond 5 days
Treatment is with total gut rest and TPN, babies with perforations will require laparotomy

254
Q

A 73 year old lady presents with constipation and no organic disease is identified on investigation. Which of the following types of laxatives works by direct bowel stimulation?

	Magnesium sulphate
	Lactulose
	Potassium sodium tatrate
	Methylcellulose
	Senna
A

Senna contains glycosides. It passes unchanged into the colon where bacteria hydrolyse the glycosidic bond, releasing the anthracene derivatives. These stimulate the myenteric plexus.

Laxatives

Bulk forming laxatives
Bran
Psyllium
Methylcellulose

Osmotic laxatives
Magnesium sulphate
Magnesium citrate
Sodium phosphate
Sodium sulphate
Potassium sodium tatrate
Polyethylene glycol
Stimulant laxatives
Docusates
Bisacodyl
Sodium picosulphate
Senna
Ricinoleic acid
255
Q

A 65 year old male with known nasopharyngeal carcinoma presents with double vision over a few weeks. On examination he is found to have left eye proptosis and it is down and out. He reports pain on attempting to move the eye. There is an absent corneal reflex. What is the most likely diagnosis?

	Posterior communicating artery aneurysm
	Cavernous sinus syndrome
	Optic nerve tumour
	Migraine
	Cerebral metastases
A

Cavernous sinus syndrome is most commonly caused by cavernous sinus tumours. In this case, the nasopharyngeal malignancy has locally invaded the left cavernous sinus. Diagnosis is based on signs of pain, opthalmoplegia, proptosis, trigeminal nerve lesion (opthalmic branch) and Horner’s syndrome.

Cavernous sinus

The cavernous sinuses are paired and are situated on the body of the sphenoid bone. It runs from the superior orbital fissure to the petrous temporal bone.

Relations
Medial Lateral
Pituitary fossa
Sphenoid sinus Temporal lobe

Contents
Lateral wall components (from top to bottom:)
Oculomotor nerve
Trochlear nerve
Ophthalmic nerve
Maxillary nerve
Contents of the sinus (from medial to lateral:)
Internal carotid artery (and sympathetic plexus)
Abducens nerve

Blood supply
Ophthalmic vein, superficial cortical veins, basilar plexus of veins posteriorly.

Drains into the internal jugular vein via: the superior and inferior petrosal sinuses

256
Q

A 42 year old female presents with symptoms of biliary colic and on investigation is identified as having gallstones. Of the procedures listed below, which is most likely to increase the risk of gallstone formation?

	Partial gastrectomy
	Jejunal resection
	Liver lobectomy
	Ileal resection
	Left hemicolectomy
A

Bile salt reabsorption occurs at the ileum. Therefore cholesterol gallstones form as a result of ileal resection.

Gallstones Typically history of biliary colic or episodes of chlolecystitis. Obstructive type history and test results. Usually small calibre gallstones which can pass through the cystic duct. In Mirizzi syndrome the stone may compress the bile duct directly- one of the rare times that cholecystitis may present with jaundice
Cholangitis Usually obstructive and will have Charcot’s triad of symptoms (pain, fever, jaundice) Ascending infection of the bile ducts usually by E. coli and by definition occurring in a pool of stagnant bile.
Pancreatic cancer Typically painless jaundice with palpable gallbladder (Courvoisier’s Law) Direct occlusion of distal bile duct or pancreatic duct by tumour. Sometimes nodal disease at the portal hepatis may be the culprit in which case the bile duct may be of normal calibre.
TPN (total parenteral nutrition) associated jaundice Usually follows long term use and is usually painless with non obstructive features Often due to hepatic dysfunction and fatty liver which may occur with long term TPN usage.
Bile duct injury Depending upon the type of injury may be of sudden or gradual onset and is usually of obstructive type Often due to a difficult laparoscopic cholecystectomy when anatomy in Calots triangle is not appreciated. In the worst scenario the bile duct is excised and jaundice develops rapidly post operatively. More insidious is that of bile duct stenosis which may be caused by clips or diathermy injury.
Cholangiocarcinoma Gradual onset obstructive pattern Direct occlusion by disease and also extrinsic compression by nodal disease at the porta hepatis.
Septic surgical patient Usually hepatic features Combination of impaired biliary excretion and drugs such as ciprofloxacin which may cause cholestasis.
Metastatic disease Mixed hepatic and post hepatic Combination of liver synthetic failure (late) and extrinsic compression by nodal disease and anatomical compression of intra hepatic structures (earlier)

A gallbladder may develop a thickened wall in chronic cholecystitis, microscopically Roikitansky-Aschoff Sinuses may be seen

257
Q

A 22 year old women presents with a newly pigmented lesion on her right shin, it has regular borders and normal appearing dermal appendages. However, she reports a recent increase in size.

A.	Excision biopsy
B.	Excision with 0.5 cm margin
C.	Excision with 2 cm margin
D.	Shave biopsy and cautery
E.	Punch biopsy
F.	Excision and full thickness skin graft
G.	Discharge
A

Excision biopsy

Lesion bearing normal dermal appendages and regular borders are likely to be a benign pigmented naevi. Therefore diagnostic and not radical excision is indicated.

Skin lesions may be referred for surgical assessment, but more commonly will come via a dermatologist for definitive surgical management.

Skin malignancies include basal cell carcinoma, squamous cell carcinoma and malignant melanoma.

Basal Cell Carcinoma
Most common form of skin cancer.
Commonly occur on sun exposed sites apart from the ear.
Sub types include nodular, morphoeic, superficial and pigmented.
Typically slow growing with low metastatic potential.
Standard surgical excision, topical chemotherapy and radiotherapy are all successful.
As a minimum a diagnostic punch biopsy should be taken if treatment other than standard surgical excision is planned.

Squamous Cell Carcinoma
Again related to sun exposure.
May arise in pre - existing solar keratoses.
May metastasize if left.
Immunosupression (e.g. following transplant), increases risk.
Wide local excision is the treatment of choice and where a diagnostic excision biopsy has demonstrated SCC, repeat surgery to gain adequate margins may be required.

Malignant Melanoma
The main diagnostic features (major criteria):
Change in size
Change in shape
Change in colour
Secondary features (minor criteria)
Diameter >6mm
Inflammation
Oozing or bleeding
Altered sensation

Treatment
Suspicious lesions should undergo excision biopsy. The lesion should be removed in completely as incision biopsy can make subsequent histopathological assessment difficult.
Once the diagnosis is confirmed the pathology report should be reviewed to determine whether further re-excision of margins is required (see below):

Margins of excision-Related to Breslow thickness
Lesions 0-1mm thick 1cm
Lesions 1-2mm thick 1- 2cm (Depending upon site and pathological features)
Lesions 2-4mm thick 2-3 cm (Depending upon site and pathological features)
Lesions >4 mm thick 3cm
Marsden J et al. Revised UK guidelines for management of Melanoma. Br J Dermatol 2010 163:238-256.

Further treatments such as sentinel lymph node mapping, isolated limb perfusion and block dissection of regional lymph node groups should be selectively applied.

Kaposi Sarcoma
Tumour of vascular and lymphatic endothelium.
Purple cutaneous nodules.
Associated with immuno supression.
Classical form affects elderly males and is slow growing.
Immunosupression form is much more aggressive and tends to affect those with HIV related disease.

Non malignant skin disease

Dermatitis Herpetiformis
Chronic itchy clusters of blisters.
Linked to underlying gluten enteropathy (coeliac disease).

Dermatofibroma
Benign lesion.
Firm elevated nodules.
Usually history of trauma.
Lesion consists of histiocytes, blood vessels and fibrotic changes.
Pyogenic granuloma
Overgrowth of blood vessels.
Red nodules.
Usually follow trauma.
May mimic amelanotic melanoma.

Acanthosis nigricans
Brown to black, poorly defined, velvety hyperpigmentation of the skin.
Usually found in body folds such as the posterior and lateral folds of the neck, the axilla, groin, umbilicus, forehead, and other areas.
The most common cause of acanthosis nigricans is insulin resistance, which leads to increased circulating insulin levels. Insulin spillover into the skin results in its abnormal increase in growth (hyperplasia of the skin).
In the context of a malignant disease, acanthosis nigricans is a paraneoplastic syndrome and is then commonly referred to as acanthosis nigricans maligna. Involvement of mucous membranes is rare and suggests a coexisting malignant condition.

258
Q

A 58 year old lady presents with changes that are suspicious of lichen sclerosis of the perineum.

A.	Excision biopsy
B.	Excision with 0.5 cm margin
C.	Excision with 2 cm margin
D.	Shave biopsy and cautery
E.	Punch biopsy
F.	Excision and full thickness skin graft
G.	Discharge
A

Punch biopsy

Punch biopsies are a useful option for obtaining a full thickness tissues sample with minimal tissue disruption. In this situation the other differential would be AIN or VIN and punch biopsies would be useful in distinguishing these.

Skin lesions may be referred for surgical assessment, but more commonly will come via a dermatologist for definitive surgical management.

Skin malignancies include basal cell carcinoma, squamous cell carcinoma and malignant melanoma.

Basal Cell Carcinoma
Most common form of skin cancer.
Commonly occur on sun exposed sites apart from the ear.
Sub types include nodular, morphoeic, superficial and pigmented.
Typically slow growing with low metastatic potential.
Standard surgical excision, topical chemotherapy and radiotherapy are all successful.
As a minimum a diagnostic punch biopsy should be taken if treatment other than standard surgical excision is planned.

Squamous Cell Carcinoma
Again related to sun exposure.
May arise in pre - existing solar keratoses.
May metastasize if left.
Immunosupression (e.g. following transplant), increases risk.
Wide local excision is the treatment of choice and where a diagnostic excision biopsy has demonstrated SCC, repeat surgery to gain adequate margins may be required.

Malignant Melanoma
The main diagnostic features (major criteria):
Change in size
Change in shape
Change in colour
Secondary features (minor criteria)
Diameter >6mm
Inflammation
Oozing or bleeding
Altered sensation

Treatment
Suspicious lesions should undergo excision biopsy. The lesion should be removed in completely as incision biopsy can make subsequent histopathological assessment difficult.
Once the diagnosis is confirmed the pathology report should be reviewed to determine whether further re-excision of margins is required (see below):

Margins of excision-Related to Breslow thickness
Lesions 0-1mm thick 1cm
Lesions 1-2mm thick 1- 2cm (Depending upon site and pathological features)
Lesions 2-4mm thick 2-3 cm (Depending upon site and pathological features)
Lesions >4 mm thick 3cm
Marsden J et al. Revised UK guidelines for management of Melanoma. Br J Dermatol 2010 163:238-256.

Further treatments such as sentinel lymph node mapping, isolated limb perfusion and block dissection of regional lymph node groups should be selectively applied.

Kaposi Sarcoma
Tumour of vascular and lymphatic endothelium.
Purple cutaneous nodules.
Associated with immuno supression.
Classical form affects elderly males and is slow growing.
Immunosupression form is much more aggressive and tends to affect those with HIV related disease.

Non malignant skin disease

Dermatitis Herpetiformis
Chronic itchy clusters of blisters.
Linked to underlying gluten enteropathy (coeliac disease).

Dermatofibroma
Benign lesion.
Firm elevated nodules.
Usually history of trauma.
Lesion consists of histiocytes, blood vessels and fibrotic changes.
Pyogenic granuloma
Overgrowth of blood vessels.
Red nodules.
Usually follow trauma.
May mimic amelanotic melanoma.

Acanthosis nigricans
Brown to black, poorly defined, velvety hyperpigmentation of the skin.
Usually found in body folds such as the posterior and lateral folds of the neck, the axilla, groin, umbilicus, forehead, and other areas.
The most common cause of acanthosis nigricans is insulin resistance, which leads to increased circulating insulin levels. Insulin spillover into the skin results in its abnormal increase in growth (hyperplasia of the skin).
In the context of a malignant disease, acanthosis nigricans is a paraneoplastic syndrome and is then commonly referred to as acanthosis nigricans maligna. Involvement of mucous membranes is rare and suggests a coexisting malignant condition.

259
Q

A 73 year old man presents with a 1.5cm ulcerated basal cell carcinoma on his back.

A.	Excision biopsy
B.	Excision with 0.5 cm margin
C.	Excision with 2 cm margin
D.	Shave biopsy and cautery
E.	Punch biopsy
F.	Excision and full thickness skin graft
G.	Discharge
A

Excision with 0.5 cm margin

A small lesion such as this is adequately treated by local excision. The British Association of Dermatology guidelines suggest that excision of conventional BCC (<2cm) with margins of 3-5mm have locoregional control rates of 85%. Morpoeic lesions have higher local recurrence rates.

Skin lesions may be referred for surgical assessment, but more commonly will come via a dermatologist for definitive surgical management.

Skin malignancies include basal cell carcinoma, squamous cell carcinoma and malignant melanoma.

Basal Cell Carcinoma
Most common form of skin cancer.
Commonly occur on sun exposed sites apart from the ear.
Sub types include nodular, morphoeic, superficial and pigmented.
Typically slow growing with low metastatic potential.
Standard surgical excision, topical chemotherapy and radiotherapy are all successful.
As a minimum a diagnostic punch biopsy should be taken if treatment other than standard surgical excision is planned.

Squamous Cell Carcinoma
Again related to sun exposure.
May arise in pre - existing solar keratoses.
May metastasize if left.
Immunosupression (e.g. following transplant), increases risk.
Wide local excision is the treatment of choice and where a diagnostic excision biopsy has demonstrated SCC, repeat surgery to gain adequate margins may be required.

Malignant Melanoma
The main diagnostic features (major criteria):
Change in size
Change in shape
Change in colour
Secondary features (minor criteria)
Diameter >6mm
Inflammation
Oozing or bleeding
Altered sensation

Treatment
Suspicious lesions should undergo excision biopsy. The lesion should be removed in completely as incision biopsy can make subsequent histopathological assessment difficult.
Once the diagnosis is confirmed the pathology report should be reviewed to determine whether further re-excision of margins is required (see below):

Margins of excision-Related to Breslow thickness
Lesions 0-1mm thick 1cm
Lesions 1-2mm thick 1- 2cm (Depending upon site and pathological features)
Lesions 2-4mm thick 2-3 cm (Depending upon site and pathological features)
Lesions >4 mm thick 3cm
Marsden J et al. Revised UK guidelines for management of Melanoma. Br J Dermatol 2010 163:238-256.

Further treatments such as sentinel lymph node mapping, isolated limb perfusion and block dissection of regional lymph node groups should be selectively applied.

Kaposi Sarcoma
Tumour of vascular and lymphatic endothelium.
Purple cutaneous nodules.
Associated with immuno supression.
Classical form affects elderly males and is slow growing.
Immunosupression form is much more aggressive and tends to affect those with HIV related disease.

Non malignant skin disease

Dermatitis Herpetiformis
Chronic itchy clusters of blisters.
Linked to underlying gluten enteropathy (coeliac disease).

Dermatofibroma
Benign lesion.
Firm elevated nodules.
Usually history of trauma.
Lesion consists of histiocytes, blood vessels and fibrotic changes.
Pyogenic granuloma
Overgrowth of blood vessels.
Red nodules.
Usually follow trauma.
May mimic amelanotic melanoma.

Acanthosis nigricans
Brown to black, poorly defined, velvety hyperpigmentation of the skin.
Usually found in body folds such as the posterior and lateral folds of the neck, the axilla, groin, umbilicus, forehead, and other areas.
The most common cause of acanthosis nigricans is insulin resistance, which leads to increased circulating insulin levels. Insulin spillover into the skin results in its abnormal increase in growth (hyperplasia of the skin).
In the context of a malignant disease, acanthosis nigricans is a paraneoplastic syndrome and is then commonly referred to as acanthosis nigricans maligna. Involvement of mucous membranes is rare and suggests a coexisting malignant condition.

260
Q

A 32 year old female hits her head on the steering wheel during a collision with another car. She has periorbital swelling and a flattened appearance of the face.

A.	Subdural haematoma
B.	Extradural haematoma
C.	Subarachnoid haemorrhage
D.	Basal skull fracture
E.	Intracerebral haematoma
F.	Le fort 1 fracture of maxilla
G.	Le fort fracture 3 affecting maxilla
H.	Mandibular fracture
A

Le fort fracture 3 affecting maxilla

The flattened appearance of the face is a classical description of the dish/pan face associated with Le fort fracture 2 or 3 of the maxilla.

Head injury

Patients who suffer head injuries should be managed according to ATLS principles and extra cranial injuries should be managed alongside cranial trauma. Inadequate cardiac output will compromise CNS perfusion irrespective of the nature of the cranial injury.

Types of traumatic brain injury
Extradural haematoma Bleeding into the space between the dura mater and the skull. Often results from acceleration-deceleration trauma or a blow to the side of the head. The majority of extradural haematomas occur in the temporal region where skull fractures cause a rupture of the middle meningeal artery.

Features
Raised intracranial pressure
Some patients may exhibit a lucid interval
Subdural haematoma Bleeding into the outermost meningeal layer. Most commonly occur around the frontal and parietal lobes. May be either acute or chronic.

Risk factors include old age and alcoholism.

Slower onset of symptoms than a extradural haematoma.
Subarachnoid haemorrhage Usually occurs spontaneously in the context of a ruptured cerebral aneurysm, but may be seen in association with other injuries when a patient has sustained a traumatic brain injury.

Pathophysiology
Primary brain injury may be focal (contusion/ haematoma) or diffuse (diffuse axonal injury)
Diffuse axonal injury occurs as a result of mechanical shearing following deceleration, causing disruption and tearing of axons
Intra-cranial haematomas can be extradural, subdural or intracerebral, while contusions may occur adjacent to (coup) or contralateral (contre-coup) to the side of impact
Secondary brain injury occurs when cerebral oedema, ischaemia, infection, tonsillar or tentorial herniation exacerbates the original injury. The normal cerebral auto regulatory processes are disrupted following trauma rendering the brain more susceptible to blood flow changes and hypoxia
The Cushings reflex (hypertension and bradycardia) often occurs late and is usually a pre terminal event

Management
Where there is life threatening rising ICP such as in extra dural haematoma and whilst theatre is prepared or transfer arranged use of IV mannitol/ frusemide may be required.
Diffuse cerebral oedema may require decompressive craniotomy
Exploratory Burr Holes have little management in modern practice except where scanning may be unavailable and to thus facilitate creation of formal craniotomy flap
Depressed skull fractures that are open require formal surgical reduction and debridement, closed injuries may be managed non operatively if there is minimal displacement.
ICP monitoring is appropriate in those who have GCS 3-8 and normal CT scan.
ICP monitoring is mandatory in those who have GCS 3-8 and abnormal CT scan.
Hyponatraemia is most likely to be due to syndrome of inappropriate ADH secretion.
Minimum of cerebral perfusion pressure of 70mmHg in adults.
Minimum cerebral perfusion pressure of between 40 and 70 mmHg in children.

Interpretation of pupillary findings in head injuries
Pupil size Light response Interpretation

Unilaterally dilated Sluggish or fixed 3rd nerve compression secondary to tentorial herniation

Bilaterally dilated Sluggish or fixed
Poor CNS perfusion
Bilateral 3rd nerve palsy

Unilaterally dilated or equal Cross reactive (Marcus - Gunn) Optic nerve injury

Bilaterally constricted May be difficult to assess
Opiates
Pontine lesions
Metabolic encephalopathy

Unilaterally constricted Preserved Sympathetic pathway disruption

261
Q

A 29 year bouncer is hit on the side of the head with a bat. He now presents to A&E with odd behaviour and complaining of a headache. Whilst waiting for a CT scan he becomes drowsy and unresponsive.

 A.	Subdural haematoma
B.	Extradural haematoma
C.	Subarachnoid haemorrhage
D.	Basal skull fracture
E.	Intracerebral haematoma
F.	Le fort 1 fracture of maxilla
G.	Le fort fracture 3 affecting maxilla
H.	Mandibular fracture
A

Extradural haematoma

The middle meningeal artery is prone to damage when the temporal side of the head is hit.
Note that there may NOT be any initial LOC or lucid interval.

Head injury

Patients who suffer head injuries should be managed according to ATLS principles and extra cranial injuries should be managed alongside cranial trauma. Inadequate cardiac output will compromise CNS perfusion irrespective of the nature of the cranial injury.

Types of traumatic brain injury
Extradural haematoma Bleeding into the space between the dura mater and the skull. Often results from acceleration-deceleration trauma or a blow to the side of the head. The majority of extradural haematomas occur in the temporal region where skull fractures cause a rupture of the middle meningeal artery.

Features
Raised intracranial pressure
Some patients may exhibit a lucid interval
Subdural haematoma Bleeding into the outermost meningeal layer. Most commonly occur around the frontal and parietal lobes. May be either acute or chronic.

Risk factors include old age and alcoholism.

Slower onset of symptoms than a extradural haematoma.
Subarachnoid haemorrhage Usually occurs spontaneously in the context of a ruptured cerebral aneurysm, but may be seen in association with other injuries when a patient has sustained a traumatic brain injury.

Pathophysiology
Primary brain injury may be focal (contusion/ haematoma) or diffuse (diffuse axonal injury)
Diffuse axonal injury occurs as a result of mechanical shearing following deceleration, causing disruption and tearing of axons
Intra-cranial haematomas can be extradural, subdural or intracerebral, while contusions may occur adjacent to (coup) or contralateral (contre-coup) to the side of impact
Secondary brain injury occurs when cerebral oedema, ischaemia, infection, tonsillar or tentorial herniation exacerbates the original injury. The normal cerebral auto regulatory processes are disrupted following trauma rendering the brain more susceptible to blood flow changes and hypoxia
The Cushings reflex (hypertension and bradycardia) often occurs late and is usually a pre terminal event

Management
Where there is life threatening rising ICP such as in extra dural haematoma and whilst theatre is prepared or transfer arranged use of IV mannitol/ frusemide may be required.
Diffuse cerebral oedema may require decompressive craniotomy
Exploratory Burr Holes have little management in modern practice except where scanning may be unavailable and to thus facilitate creation of formal craniotomy flap
Depressed skull fractures that are open require formal surgical reduction and debridement, closed injuries may be managed non operatively if there is minimal displacement.
ICP monitoring is appropriate in those who have GCS 3-8 and normal CT scan.
ICP monitoring is mandatory in those who have GCS 3-8 and abnormal CT scan.
Hyponatraemia is most likely to be due to syndrome of inappropriate ADH secretion.
Minimum of cerebral perfusion pressure of 70mmHg in adults.
Minimum cerebral perfusion pressure of between 40 and 70 mmHg in children.

Interpretation of pupillary findings in head injuries
Pupil size Light response Interpretation

Unilaterally dilated Sluggish or fixed 3rd nerve compression secondary to tentorial herniation

Bilaterally dilated Sluggish or fixed
Poor CNS perfusion
Bilateral 3rd nerve palsy

Unilaterally dilated or equal Cross reactive (Marcus - Gunn) Optic nerve injury

Bilaterally constricted May be difficult to assess
Opiates
Pontine lesions
Metabolic encephalopathy

Unilaterally constricted Preserved Sympathetic pathway disruption

262
Q

A 40 year old alcoholic presents with worsening confusion over 2 weeks. He has weakness of the left side of the body.

 A.	Subdural haematoma
B.	Extradural haematoma
C.	Subarachnoid haemorrhage
D.	Basal skull fracture
E.	Intracerebral haematoma
F.	Le fort 1 fracture of maxilla
G.	Le fort fracture 3 affecting maxilla
H.	Mandibular fracture
A

Subdural haematoma

Subdural haematomas can have a history over weeks/months. It is common in alcoholics due to cerebral atrophy causing increased stretching of veins.

Head injury

Patients who suffer head injuries should be managed according to ATLS principles and extra cranial injuries should be managed alongside cranial trauma. Inadequate cardiac output will compromise CNS perfusion irrespective of the nature of the cranial injury.

Types of traumatic brain injury
Extradural haematoma Bleeding into the space between the dura mater and the skull. Often results from acceleration-deceleration trauma or a blow to the side of the head. The majority of extradural haematomas occur in the temporal region where skull fractures cause a rupture of the middle meningeal artery.

Features
Raised intracranial pressure
Some patients may exhibit a lucid interval
Subdural haematoma Bleeding into the outermost meningeal layer. Most commonly occur around the frontal and parietal lobes. May be either acute or chronic.

Risk factors include old age and alcoholism.

Slower onset of symptoms than a extradural haematoma.
Subarachnoid haemorrhage Usually occurs spontaneously in the context of a ruptured cerebral aneurysm, but may be seen in association with other injuries when a patient has sustained a traumatic brain injury.

Pathophysiology
Primary brain injury may be focal (contusion/ haematoma) or diffuse (diffuse axonal injury)
Diffuse axonal injury occurs as a result of mechanical shearing following deceleration, causing disruption and tearing of axons
Intra-cranial haematomas can be extradural, subdural or intracerebral, while contusions may occur adjacent to (coup) or contralateral (contre-coup) to the side of impact
Secondary brain injury occurs when cerebral oedema, ischaemia, infection, tonsillar or tentorial herniation exacerbates the original injury. The normal cerebral auto regulatory processes are disrupted following trauma rendering the brain more susceptible to blood flow changes and hypoxia
The Cushings reflex (hypertension and bradycardia) often occurs late and is usually a pre terminal event

Management
Where there is life threatening rising ICP such as in extra dural haematoma and whilst theatre is prepared or transfer arranged use of IV mannitol/ frusemide may be required.
Diffuse cerebral oedema may require decompressive craniotomy
Exploratory Burr Holes have little management in modern practice except where scanning may be unavailable and to thus facilitate creation of formal craniotomy flap
Depressed skull fractures that are open require formal surgical reduction and debridement, closed injuries may be managed non operatively if there is minimal displacement.
ICP monitoring is appropriate in those who have GCS 3-8 and normal CT scan.
ICP monitoring is mandatory in those who have GCS 3-8 and abnormal CT scan.
Hyponatraemia is most likely to be due to syndrome of inappropriate ADH secretion.
Minimum of cerebral perfusion pressure of 70mmHg in adults.
Minimum cerebral perfusion pressure of between 40 and 70 mmHg in children.

Interpretation of pupillary findings in head injuries
Pupil size Light response Interpretation

Unilaterally dilated Sluggish or fixed 3rd nerve compression secondary to tentorial herniation

Bilaterally dilated Sluggish or fixed
Poor CNS perfusion
Bilateral 3rd nerve palsy

Unilaterally dilated or equal Cross reactive (Marcus - Gunn) Optic nerve injury

Bilaterally constricted May be difficult to assess
Opiates
Pontine lesions
Metabolic encephalopathy

Unilaterally constricted Preserved Sympathetic pathway disruption

263
Q

Which of the following is not true of hyper acute solid organ transplant rejection?

It may occur during the surgical procedure itself.
May occur as a result of blood group A, B or O incompatibility.
May be due to pre existing anti HLA antibodies.
On biopsy will typically show neo intimal hyperplasia of donor arterioles.
Complement system activation is one of the key mediators.
A

On biopsy will typically show neo intimal hyperplasia of donor arterioles. - FALSE

These changes are more often seen in the chronic setting. Thrombosis is more commonly seen in the hyperacute phase.

Organ Transplant

A number of different organ and tissue transplants are now available. In many cases an allograft is performed, where an organ is transplanted from one individual to another. Allografts will elicit an immune response and this is one of the main reasons for organ rejection.

Graft rejection occurs because allografts have allelic differences at genes that code immunohistocompatability complex genes. The main antigens that give rise to rejection are:
ABO blood group
Human leucocyte antigens (HLA)
Minor histocompatability antigens

ABO Matching
ABO incompatibility will result in early organ rejection (hyperacute) because of pre existing antibodies to other groups. Group O donors can give organs to any type of ABO recipient whereas group AB donor can only donate to AB recipient.

HLA System
The four most important HLA alleles are:

HLA A
HLA B
HLA C
HLA DR

An ideal organ match would be one in which all 8 alleles are matched (remember 2 from each parent, four each = 8 alleles). Modern immunosuppressive regimes help to manage the potential rejection due to HLA mismatching. However, the greater the number of mismatches the worse the long term outcome will be. T lymphocytes will recognise antigens bound to HLA molecules and will then become activated. Clonal expansion then occurs with a response directed against that antigen.

Types of organ rejection
Hyperacute. This occurs immediately through presence of pre formed antibodies (such as ABO incompatibility).
Acute. Occurs during the first 6 months and is usually T cell mediated. Usually tissue infiltrates and vascular lesions.
Chronic. Occurs after the first 6 months. Vascular changes predominate.

Hyperacute
Renal transplants at greatest risk and liver transplants at least risk. Although ABO incompatibility and HLA Class I incompatible transplants will all fare worse in long term.

Acute
All organs may undergo acute rejection. Mononuclear cell infiltrates predominate. All types of transplanted organ are susceptible and it may occur in up to 50% cases.

Chronic
Again all transplants with HLA mismatch may suffer this fate. Previous acute rejections and other immunosensitising events all increase the risk. Vascular changes are most prominent with myointimal proliferation leading to organ ischaemia. Organ specific changes are also seen such as loss of acinar cells in pancreas transplants and rapidly progressive coronary artery disease in cardiac transplants.

Surgical overview-Renal transplantation
A brief overview of the steps involved in renal transplantation is given.
Patients with end stage renal failure who are dialysis dependent or likely to become so in the immediate future are considered for transplant. Exclusion criteria include; active malignancy, old age (due to limited organ availability). Patients are medically optimised.
Donor kidneys, these may be taken from live related donors and close family, members may have less HLA mismatch than members of the general population. Laparoscopic donor nephrectomy further minimises the operative morbidity for the donor. Other organs are typically taken from brain dead or dying patients who have a cardiac arrest and in whom resuscitation is futile. The key event is to minimise the warm ischaemic time in the donor phase.

The kidney once removed is usually prepared on the bench in theatre by the transplant surgeon immediately prior to implantation and factors such as accessory renal arteries and vessel length are assessed and managed.

For first time recipients the operation is performed under general anaesthesia. A Rutherford-Morrison incision is made on the preferred side. This provides excellent extraperitoneal access to the iliac vessels. The external iliac artery and vein are dissected out and following systemic heparinisation are cross clamped. The vein and artery are anastamosed to the iliacs and the clamps removed. The ureter is then implanted into the bladder and a stent is usually placed to maintain patency. The wounds are then closed and the patient recovered from surgery.

In the immediate phase a common problem encountered in cadaveric kidneys is acute tubular necrosis and this tends to resolve.

Graft survival times from cadaveric donors are typically of the order of 9 years and monozygotic twin transplant (live donor) may survive as long as 25 years.

264
Q

A 66 year old man is referred via the aneurysm screening programme with an abdominal aortic aneurysm measuring 4.4 cm. Apart from well controlled type 2 DM he is otherwise well

 A.	Immediate laparotomy
B.	Immediate CT
C.	AAA repair during next 48 hours
D.	USS in 6 months
E.	CT scan during next 4 weeks
F.	Endovascular aortic aneurysm repair
G.	Discharge
H.	Palliate
I.	None of the above
A

USS in 6 months

At this point continue with ultrasound surveillance

Abdominal aorta aneurysm

Abdominal aortic aneurysms are a common problem in vascular surgery.
They may occur as either true or false aneurysm. With the former all 3 layers of the arterial wall are involved, in the latter only a single layer of fibrous tissue forms the aneurysm wall.
True abdominal aortic aneurysms have an approximate incidence of 0.06 per 1000 people. They are commonest in elderly men and for this reason the UK is now introducing the aneurysm screening program with the aim of performing an abdominal aortic ultrasound measurement in all men aged 65 years.

Causes
Several different groups of patients suffer from aneurysmal disease.
The commonest group is those who suffer from standard arterial disease, i.e. Those who are hypertensive and have been or are smokers.
Other patients such as those suffering from connective tissue diseases such as Marfan’s may also develop aneurysms. In patients with abdominal aortic aneurysms the extracellular matrix becomes disrupted with a change in the balance of collagen and elastic fibres.

Management
Most abdominal aortic aneurysms are an incidental finding.
Symptoms most often relate to rupture or impending rupture.
20% rupture anteriorly into the peritoneal cavity. Very poor prognosis.
80% rupture posteriorly into the retroperitoneal space
The risk of rupture is related to aneurysm size, only 2% of aneurysms measuring less than 4cm in diameter will rupture over a 5 year period. This contrasts with 75% of aneurysms measuring over 7cm in diameter.
This is well explained by Laplaces’ law which relates size to transmural pressure.
For this reason most vascular surgeons will subject patients with an aneurysm size of 5cm or greater to CT scanning of the chest, abdomen and pelvis with the aim of delineating anatomy and planning treatment. Depending upon co-morbidities, surgery is generally offered once the aneurysm is between 5.5cm and 6cm.

A CT reconstruction showing an infrarenal abdominal aortic aneurysm. The walls of the sac are calcified which may facilitate identification on plain x-rays

Image sourced from Wikipedia

Indications for surgery
Symptomatic aneurysms (80% annual mortality if untreated)
Increasing size above 5.5cm if asymptomatic
Rupture (100% mortality without surgery)

Surgical procedures
Abdominal aortic aneurysm repair

Procedure:

GA
Invasive monitoring (A-line, CVP, catheter)
Incision: Midline or transverse
Bowel and distal duodenum mobilised to access aorta.
Aneurysm neck and base dissected out and prepared for cross clamp
Systemic heparinisation
Cross clamp (proximal first)
Longitudinal aortotomy
Atherectomy
Deal with back bleeding from lumbar vessels and inferior mesenteric artery
Insert graft either tube or bifurcated depending upon anatomy
Suture using Prolene (3/0 for proximal , distal anastomosis suture varies according to site)
Clamps off: End tidal CO2 will rise owing to effects of reperfusion, at this point major risk of myocardial events.
Haemostasis
Closure of aneurysm sac to minimise risk of aorto-enteric fistula
Closure: Loop 1 PDS or Prolene to abdominal wall
Skin- surgeons preference

Post operatively:

ITU (Almost all)
Greatest risk of complications following emergency repair
Complications: Embolic- gut and foot infarcts
Cardiac - owing to premorbid states, re-perfusion injury and effects of cross clamp
Wound problems
Later risks related to graft- infection and aorto-enteric fistula

Special groups

Supra renal AAA
These patients will require a supra renal clamp and this carries a far higher risk of complications and risk of renal failure.

Ruptured AAA
Pre-operatively the management depends upon haemodynamic instability. In patients with symptoms of rupture (typical pain, haemodynamic compromise and risk factors) then ideally prompt laparotomy. In those with vague symptoms and haemodynamic stability the ideal test is CT scan to determine whether rupture has occurred or not. Most common rupture site is retroperitoneal 80%. These patients will tend to develop retroperitoneal haematoma. This can be disrupted if Bp is allowed to rise too high so aim for Bp 100mmHg.
Operative details are similar to elective repair although surgery should be swift, blind rushing often makes the situation worse. Plunging vascular clamps blindly into a pool of blood at the aneurysm neck carries the risk of injury the vena cava that these patients do not withstand. Occasionally a supracoeliac clamp is needed to effect temporary control, although leaving this applied for more than 20 minutes tends to carry a dismal outcome.

EVAR
Increasingly patients are now being offered endovascular aortic aneurysm repair. This is undertaken by surgeons and radiologists working jointly. The morphology of the aneurysm is important and not all are suitable. Here is a typical list of those features favoring a suitable aneurysm:
Long neck
Straight iliac vessels
Healthy groin vessels

Clearly few AAA patients possess the above and compromise has to be made. The use of fenestrated grafts can allow supra renal AAA to be treated.

Procedure:

GA
Radiology or theatre
Bilateral groin incisions
Common femoral artery dissected out
Heparinisation
Arteriotomy and insertion of guide wire
Dilation of arteriotomy
Insertion of EVAR Device
Once in satisfactory position it is released
Arteriotomy closed once check angiogram shows good position and no endoleak

Complications:
Endoleaks depending upon site are either Type I or 2. These may necessitate re-intervention and all EVAR patients require follow up . Details are not needed for MRCS.

265
Q

A 72 year old man has a CT scan for abdominal discomfort and the surgeon suspects AAA. This shows a 6.6cm aneurysm with a 3.5cm neck and it continues to involve the right common iliac. The left iliac is occluded. He is hypertensive and has Type 2 DM which is well controlled.

A.	Immediate laparotomy
B.	Immediate CT
C.	AAA repair during next 48 hours
D.	USS in 6 months
E.	CT scan during next 4 weeks
F.	Endovascular aortic aneurysm repair
G.	Discharge
H.	Palliate
I.	None of the above
A

AAA repair during next 48 hours

Assuming he is fit enough. This would be a typical ‘open ‘ case as the marked iliac disease would make EVAR difficult

Abdominal aorta aneurysm

Abdominal aortic aneurysms are a common problem in vascular surgery.
They may occur as either true or false aneurysm. With the former all 3 layers of the arterial wall are involved, in the latter only a single layer of fibrous tissue forms the aneurysm wall.
True abdominal aortic aneurysms have an approximate incidence of 0.06 per 1000 people. They are commonest in elderly men and for this reason the UK is now introducing the aneurysm screening program with the aim of performing an abdominal aortic ultrasound measurement in all men aged 65 years.

Causes
Several different groups of patients suffer from aneurysmal disease.
The commonest group is those who suffer from standard arterial disease, i.e. Those who are hypertensive and have been or are smokers.
Other patients such as those suffering from connective tissue diseases such as Marfan’s may also develop aneurysms. In patients with abdominal aortic aneurysms the extracellular matrix becomes disrupted with a change in the balance of collagen and elastic fibres.

Management
Most abdominal aortic aneurysms are an incidental finding.
Symptoms most often relate to rupture or impending rupture.
20% rupture anteriorly into the peritoneal cavity. Very poor prognosis.
80% rupture posteriorly into the retroperitoneal space
The risk of rupture is related to aneurysm size, only 2% of aneurysms measuring less than 4cm in diameter will rupture over a 5 year period. This contrasts with 75% of aneurysms measuring over 7cm in diameter.
This is well explained by Laplaces’ law which relates size to transmural pressure.
For this reason most vascular surgeons will subject patients with an aneurysm size of 5cm or greater to CT scanning of the chest, abdomen and pelvis with the aim of delineating anatomy and planning treatment. Depending upon co-morbidities, surgery is generally offered once the aneurysm is between 5.5cm and 6cm.

A CT reconstruction showing an infrarenal abdominal aortic aneurysm. The walls of the sac are calcified which may facilitate identification on plain x-rays

Image sourced from Wikipedia

Indications for surgery
Symptomatic aneurysms (80% annual mortality if untreated)
Increasing size above 5.5cm if asymptomatic
Rupture (100% mortality without surgery)

Surgical procedures
Abdominal aortic aneurysm repair

Procedure:

GA
Invasive monitoring (A-line, CVP, catheter)
Incision: Midline or transverse
Bowel and distal duodenum mobilised to access aorta.
Aneurysm neck and base dissected out and prepared for cross clamp
Systemic heparinisation
Cross clamp (proximal first)
Longitudinal aortotomy
Atherectomy
Deal with back bleeding from lumbar vessels and inferior mesenteric artery
Insert graft either tube or bifurcated depending upon anatomy
Suture using Prolene (3/0 for proximal , distal anastomosis suture varies according to site)
Clamps off: End tidal CO2 will rise owing to effects of reperfusion, at this point major risk of myocardial events.
Haemostasis
Closure of aneurysm sac to minimise risk of aorto-enteric fistula
Closure: Loop 1 PDS or Prolene to abdominal wall
Skin- surgeons preference

Post operatively:

ITU (Almost all)
Greatest risk of complications following emergency repair
Complications: Embolic- gut and foot infarcts
Cardiac - owing to premorbid states, re-perfusion injury and effects of cross clamp
Wound problems
Later risks related to graft- infection and aorto-enteric fistula

Special groups

Supra renal AAA
These patients will require a supra renal clamp and this carries a far higher risk of complications and risk of renal failure.

Ruptured AAA
Pre-operatively the management depends upon haemodynamic instability. In patients with symptoms of rupture (typical pain, haemodynamic compromise and risk factors) then ideally prompt laparotomy. In those with vague symptoms and haemodynamic stability the ideal test is CT scan to determine whether rupture has occurred or not. Most common rupture site is retroperitoneal 80%. These patients will tend to develop retroperitoneal haematoma. This can be disrupted if Bp is allowed to rise too high so aim for Bp 100mmHg.
Operative details are similar to elective repair although surgery should be swift, blind rushing often makes the situation worse. Plunging vascular clamps blindly into a pool of blood at the aneurysm neck carries the risk of injury the vena cava that these patients do not withstand. Occasionally a supracoeliac clamp is needed to effect temporary control, although leaving this applied for more than 20 minutes tends to carry a dismal outcome.

EVAR
Increasingly patients are now being offered endovascular aortic aneurysm repair. This is undertaken by surgeons and radiologists working jointly. The morphology of the aneurysm is important and not all are suitable. Here is a typical list of those features favoring a suitable aneurysm:
Long neck
Straight iliac vessels
Healthy groin vessels

Clearly few AAA patients possess the above and compromise has to be made. The use of fenestrated grafts can allow supra renal AAA to be treated.

Procedure:

GA
Radiology or theatre
Bilateral groin incisions
Common femoral artery dissected out
Heparinisation
Arteriotomy and insertion of guide wire
Dilation of arteriotomy
Insertion of EVAR Device
Once in satisfactory position it is released
Arteriotomy closed once check angiogram shows good position and no endoleak

Complications:
Endoleaks depending upon site are either Type I or 2. These may necessitate re-intervention and all EVAR patients require follow up . Details are not needed for MRCS.

266
Q

An 89 year old man presents with hypotension and collapse and is found by the staff in the toilet of his care home. He is moribund and unable to give a clear history. He had suffered a cardiac arrest in the ambulance but has since been resuscitated and now has a Bp of 95 systolic. He has an obviously palpable AAA.

A.	Immediate laparotomy
B.	Immediate CT
C.	AAA repair during next 48 hours
D.	USS in 6 months
E.	CT scan during next 4 weeks
F.	Endovascular aortic aneurysm repair
G.	Discharge
H.	Palliate
I.	None of the above
A

Palliate

He will not survive aortic surgery and whilst some may disagree, I would argue that taking this case to theatre would be futile

Abdominal aorta aneurysm

Abdominal aortic aneurysms are a common problem in vascular surgery.
They may occur as either true or false aneurysm. With the former all 3 layers of the arterial wall are involved, in the latter only a single layer of fibrous tissue forms the aneurysm wall.
True abdominal aortic aneurysms have an approximate incidence of 0.06 per 1000 people. They are commonest in elderly men and for this reason the UK is now introducing the aneurysm screening program with the aim of performing an abdominal aortic ultrasound measurement in all men aged 65 years.

Causes
Several different groups of patients suffer from aneurysmal disease.
The commonest group is those who suffer from standard arterial disease, i.e. Those who are hypertensive and have been or are smokers.
Other patients such as those suffering from connective tissue diseases such as Marfan’s may also develop aneurysms. In patients with abdominal aortic aneurysms the extracellular matrix becomes disrupted with a change in the balance of collagen and elastic fibres.

Management
Most abdominal aortic aneurysms are an incidental finding.
Symptoms most often relate to rupture or impending rupture.
20% rupture anteriorly into the peritoneal cavity. Very poor prognosis.
80% rupture posteriorly into the retroperitoneal space
The risk of rupture is related to aneurysm size, only 2% of aneurysms measuring less than 4cm in diameter will rupture over a 5 year period. This contrasts with 75% of aneurysms measuring over 7cm in diameter.
This is well explained by Laplaces’ law which relates size to transmural pressure.
For this reason most vascular surgeons will subject patients with an aneurysm size of 5cm or greater to CT scanning of the chest, abdomen and pelvis with the aim of delineating anatomy and planning treatment. Depending upon co-morbidities, surgery is generally offered once the aneurysm is between 5.5cm and 6cm.

A CT reconstruction showing an infrarenal abdominal aortic aneurysm. The walls of the sac are calcified which may facilitate identification on plain x-rays

Image sourced from Wikipedia

Indications for surgery
Symptomatic aneurysms (80% annual mortality if untreated)
Increasing size above 5.5cm if asymptomatic
Rupture (100% mortality without surgery)

Surgical procedures
Abdominal aortic aneurysm repair

Procedure:

GA
Invasive monitoring (A-line, CVP, catheter)
Incision: Midline or transverse
Bowel and distal duodenum mobilised to access aorta.
Aneurysm neck and base dissected out and prepared for cross clamp
Systemic heparinisation
Cross clamp (proximal first)
Longitudinal aortotomy
Atherectomy
Deal with back bleeding from lumbar vessels and inferior mesenteric artery
Insert graft either tube or bifurcated depending upon anatomy
Suture using Prolene (3/0 for proximal , distal anastomosis suture varies according to site)
Clamps off: End tidal CO2 will rise owing to effects of reperfusion, at this point major risk of myocardial events.
Haemostasis
Closure of aneurysm sac to minimise risk of aorto-enteric fistula
Closure: Loop 1 PDS or Prolene to abdominal wall
Skin- surgeons preference

Post operatively:

ITU (Almost all)
Greatest risk of complications following emergency repair
Complications: Embolic- gut and foot infarcts
Cardiac - owing to premorbid states, re-perfusion injury and effects of cross clamp
Wound problems
Later risks related to graft- infection and aorto-enteric fistula

Special groups

Supra renal AAA
These patients will require a supra renal clamp and this carries a far higher risk of complications and risk of renal failure.

Ruptured AAA
Pre-operatively the management depends upon haemodynamic instability. In patients with symptoms of rupture (typical pain, haemodynamic compromise and risk factors) then ideally prompt laparotomy. In those with vague symptoms and haemodynamic stability the ideal test is CT scan to determine whether rupture has occurred or not. Most common rupture site is retroperitoneal 80%. These patients will tend to develop retroperitoneal haematoma. This can be disrupted if Bp is allowed to rise too high so aim for Bp 100mmHg.
Operative details are similar to elective repair although surgery should be swift, blind rushing often makes the situation worse. Plunging vascular clamps blindly into a pool of blood at the aneurysm neck carries the risk of injury the vena cava that these patients do not withstand. Occasionally a supracoeliac clamp is needed to effect temporary control, although leaving this applied for more than 20 minutes tends to carry a dismal outcome.

EVAR
Increasingly patients are now being offered endovascular aortic aneurysm repair. This is undertaken by surgeons and radiologists working jointly. The morphology of the aneurysm is important and not all are suitable. Here is a typical list of those features favoring a suitable aneurysm:
Long neck
Straight iliac vessels
Healthy groin vessels

Clearly few AAA patients possess the above and compromise has to be made. The use of fenestrated grafts can allow supra renal AAA to be treated.

Procedure:

GA
Radiology or theatre
Bilateral groin incisions
Common femoral artery dissected out
Heparinisation
Arteriotomy and insertion of guide wire
Dilation of arteriotomy
Insertion of EVAR Device
Once in satisfactory position it is released
Arteriotomy closed once check angiogram shows good position and no endoleak

Complications:
Endoleaks depending upon site are either Type I or 2. These may necessitate re-intervention and all EVAR patients require follow up . Details are not needed for MRCS.

267
Q

Which of the following statements relating to biliary atresia is untrue?

It most commonly presents as prolonged conjugated jaundice in the neonatal period.
Evidence of portal hypertension at diagnosis is seldom present in the UK.
It may be confused with Alagille syndrome.
The Kasai procedure is best performed in the first 8 weeks of life.
Survival following a successful Kasai procedure is approximately 45% at 5 years.
A

Survival following a successful Kasai procedure is approximately 45% at 5 years. - FALSE

Alagille syndrome autosomal dominant disorder characterised by presence of paucity of bile ducts and cardiac defects. Only the embryonic form of biliary atresia is associated with cardiac and other embryological defects.

Biliary atresia usually presents with obstructed jaundice. A Kasai procedure is best performed in the first 8 weeks of life. If a Kasai procedure is successful most patients will not require liver transplantation. 45% of patients post Kasai procedure will require transplantation. However, overall survival following a successful Kasai procedure is 80%.

Biliary atresia

1 in 17000 affected
Biliary tree lumen is obliterated by an inflammatory cholangiopathy causing progressive liver damage

Clinical features
Infant well in 1st few weeks of life
No family history of liver disease
Jaundice in infants > 14 days in term infants (>21 days in pre term infants)
Pale stool, yellow urine (colourless in babies)
Associated with cardiac malformations, polysplenia, situs inversus

Investigation
Conjugated bilirubin (prolonged physiological jaundice or breast milk jaundice will cause a rise in unconjugated bilirubin, whereas those with obstructive liver disease will have a rise in conjugated bilirubin)
Ultrasound of the liver (excludes extrahepatic causes, in biliary atresia infant may have tiny or invisible gallbladder)
Hepato-iminodiacetic acid radionuclide scan (good uptake but no excretion usually seen)

Management
Early recognition is important to prevent liver transplantation.
Nutritional support.
Roux-en-Y portojejunostomy (Kasai procedure).
If Kasai procedure fails or late recognition, a liver transplant becomes the only option.

268
Q

A 1 day old infant is born with severe respiratory compromise. On examination he has a scaphoid abdomen and an absent apex beat.

A.	Umbilical hernia
B.	Para umbilical hernia
C.	Morgagni hernia
D.	Littres hernia
E.	Bochdalek hernia
F.	Richters hernia
G.	Obturator hernia
A

Bochdalek hernia

The large hernia may displace the heart although true dextrocardia is not present. The associated pulmonary hypoplasia will compromise lung development.

Hernias occur when a viscus or part of it protrudes from within its normal anatomical cavity. Specific hernias are covered under their designated titles the remainder are addressed here.

Spigelian hernia
Interparietal hernia occurring at the level of the arcuate line
Rare
May lie beneath internal oblique muscle. Usually between internal and external oblique
Equal sex distribution
Position is lateral to rectus abdominis
Both open and laparoscopic repair are possible, the former in cases of strangulation

Lumbar hernia
The lumbar triangle (through which these may occur) is bounded by:
Crest of ilium (inferiorly)
External oblique (laterally)
Latissimus dorsi (medially)
Primary lumbar herniae are rare, most are incisional hernias following renal surgery
- Direct anatomical repair with or without mesh re-enforcement is the procedure of choice

Obturator hernia
Herniation through the obturator canal
Commoner in females
Usually lies behind pectineus muscle
Elective diagnosis is unusual most will present acutely with obstruction
When presenting acutely most cases with require laparotomy or laparoscopy (and small bowel resection if indicated)

Richters hernia
Condition in which part of the wall of the small bowel (usually the anti mesenteric border) is strangulated within a hernia (of any type)
They do not present with typical features of intestinal obstruction as lumenal patency is preserved
Where vomiting is prominent it usually occurs as a result of paralytic ileus from peritonitis (as these hernias may perforate)

Incisional hernia
Occur through sites of surgical access into the abdominal cavity
Most common following surgical wound infection
To minimise following midline laparotomy Jenkins Rule should be followed and this necessitates a suture length 4x length of incision with bites taken at 1cm intervals, 1 cm from the wound edge
Repair may be performed either at open surgery or laparoscopically and a wide variety of techniques are described

Bochdalek hernia
Typically congenital diaphragmatic hernia
85% cases are located in the left hemi diaphragm
Associated with lung hypoplasia on the affected side
More common in males
Associated with other birth defects
May contain stomach
May be treated by direct anatomical apposition or placement of mesh. In infants that have severe respiratory compromise mechanical ventilation may be needed and mortality rate is high

Morgagni Hernia
Rare type of diaphragmatic hernia (approx 2% cases)
Herniation through foramen of Morgagni
Usually located on the right and tend to be less symptomatic
More advanced cases may contain transverse colon
As defects are small pulmonary hypoplasia is less common
Direct anatomical repair is performed

Umbilical hernia
Hernia through weak umbilicus
Usually presents in childhood
Often symptomatic
Equal sex incidence
95% will resolve by the age of 2 years
Surgery performed after the third birthday
Paraumbilical hernia
Usually a condition of adulthood
Defect is in the linea alba
More common in females
Multiparity and obesity are risk factors
Traditionally repaired using Mayos technique - overlapping repair, mesh may be used though not if small bowel resection is required owing to acute strangulation

Littres hernia
Hernia containing Meckels diverticulum
Resection of the diverticulum is usually required and this will preclude a mesh repair

269
Q

A 2 month old infant is troubled by recurrent colicky abdominal pain and intermittent intestinal obstruction. On imaging the transverse colon is herniated into the thoracic cavity, through a mid line defect.

A.	Umbilical hernia
B.	Para umbilical hernia
C.	Morgagni hernia
D.	Littres hernia
E.	Bochdalek hernia
F.	Richters hernia
G.	Obturator hernia
A

Morgagni hernia

Morgagni hernia may contain the transverse colon. Unless there is substantial herniation, pulmonary hypoplasia is uncommon. As a result, major respiratory compromise is often absent.

Hernias occur when a viscus or part of it protrudes from within its normal anatomical cavity. Specific hernias are covered under their designated titles the remainder are addressed here.

Spigelian hernia
Interparietal hernia occurring at the level of the arcuate line
Rare
May lie beneath internal oblique muscle. Usually between internal and external oblique
Equal sex distribution
Position is lateral to rectus abdominis
Both open and laparoscopic repair are possible, the former in cases of strangulation

Lumbar hernia
The lumbar triangle (through which these may occur) is bounded by:
Crest of ilium (inferiorly)
External oblique (laterally)
Latissimus dorsi (medially)
Primary lumbar herniae are rare, most are incisional hernias following renal surgery
- Direct anatomical repair with or without mesh re-enforcement is the procedure of choice

Obturator hernia
Herniation through the obturator canal
Commoner in females
Usually lies behind pectineus muscle
Elective diagnosis is unusual most will present acutely with obstruction
When presenting acutely most cases with require laparotomy or laparoscopy (and small bowel resection if indicated)

Richters hernia
Condition in which part of the wall of the small bowel (usually the anti mesenteric border) is strangulated within a hernia (of any type)
They do not present with typical features of intestinal obstruction as lumenal patency is preserved
Where vomiting is prominent it usually occurs as a result of paralytic ileus from peritonitis (as these hernias may perforate)

Incisional hernia
Occur through sites of surgical access into the abdominal cavity
Most common following surgical wound infection
To minimise following midline laparotomy Jenkins Rule should be followed and this necessitates a suture length 4x length of incision with bites taken at 1cm intervals, 1 cm from the wound edge
Repair may be performed either at open surgery or laparoscopically and a wide variety of techniques are described

Bochdalek hernia
Typically congenital diaphragmatic hernia
85% cases are located in the left hemi diaphragm
Associated with lung hypoplasia on the affected side
More common in males
Associated with other birth defects
May contain stomach
May be treated by direct anatomical apposition or placement of mesh. In infants that have severe respiratory compromise mechanical ventilation may be needed and mortality rate is high

Morgagni Hernia
Rare type of diaphragmatic hernia (approx 2% cases)
Herniation through foramen of Morgagni
Usually located on the right and tend to be less symptomatic
More advanced cases may contain transverse colon
As defects are small pulmonary hypoplasia is less common
Direct anatomical repair is performed

Umbilical hernia
Hernia through weak umbilicus
Usually presents in childhood
Often symptomatic
Equal sex incidence
95% will resolve by the age of 2 years
Surgery performed after the third birthday
Paraumbilical hernia
Usually a condition of adulthood
Defect is in the linea alba
More common in females
Multiparity and obesity are risk factors
Traditionally repaired using Mayos technique - overlapping repair, mesh may be used though not if small bowel resection is required owing to acute strangulation

Littres hernia
Hernia containing Meckels diverticulum
Resection of the diverticulum is usually required and this will preclude a mesh repair

270
Q

A 78 year old lady is admitted with small bowel obstruction, on examination she has a distended abdomen and the leg is held semi flexed. She has some groin pain radiating to the ipsilateral knee.

 A.	Umbilical hernia
B.	Para umbilical hernia
C.	Morgagni hernia
D.	Littres hernia
E.	Bochdalek hernia
F.	Richters hernia
G.	Obturator hernia
A

Obturator hernia

The groin swelling in obturator hernia is subtle and hard to elicit clinically. There may be pain in the region of sensory distribution of the obturator nerve. The defect is usually repaired from within the abdomen.

Hernias occur when a viscus or part of it protrudes from within its normal anatomical cavity. Specific hernias are covered under their designated titles the remainder are addressed here.

Spigelian hernia
Interparietal hernia occurring at the level of the arcuate line
Rare
May lie beneath internal oblique muscle. Usually between internal and external oblique
Equal sex distribution
Position is lateral to rectus abdominis
Both open and laparoscopic repair are possible, the former in cases of strangulation

Lumbar hernia
The lumbar triangle (through which these may occur) is bounded by:
Crest of ilium (inferiorly)
External oblique (laterally)
Latissimus dorsi (medially)
Primary lumbar herniae are rare, most are incisional hernias following renal surgery
- Direct anatomical repair with or without mesh re-enforcement is the procedure of choice

Obturator hernia
Herniation through the obturator canal
Commoner in females
Usually lies behind pectineus muscle
Elective diagnosis is unusual most will present acutely with obstruction
When presenting acutely most cases with require laparotomy or laparoscopy (and small bowel resection if indicated)

Richters hernia
Condition in which part of the wall of the small bowel (usually the anti mesenteric border) is strangulated within a hernia (of any type)
They do not present with typical features of intestinal obstruction as lumenal patency is preserved
Where vomiting is prominent it usually occurs as a result of paralytic ileus from peritonitis (as these hernias may perforate)

Incisional hernia
Occur through sites of surgical access into the abdominal cavity
Most common following surgical wound infection
To minimise following midline laparotomy Jenkins Rule should be followed and this necessitates a suture length 4x length of incision with bites taken at 1cm intervals, 1 cm from the wound edge
Repair may be performed either at open surgery or laparoscopically and a wide variety of techniques are described

Bochdalek hernia
Typically congenital diaphragmatic hernia
85% cases are located in the left hemi diaphragm
Associated with lung hypoplasia on the affected side
More common in males
Associated with other birth defects
May contain stomach
May be treated by direct anatomical apposition or placement of mesh. In infants that have severe respiratory compromise mechanical ventilation may be needed and mortality rate is high

Morgagni Hernia
Rare type of diaphragmatic hernia (approx 2% cases)
Herniation through foramen of Morgagni
Usually located on the right and tend to be less symptomatic
More advanced cases may contain transverse colon
As defects are small pulmonary hypoplasia is less common
Direct anatomical repair is performed

Umbilical hernia
Hernia through weak umbilicus
Usually presents in childhood
Often symptomatic
Equal sex incidence
95% will resolve by the age of 2 years
Surgery performed after the third birthday
Paraumbilical hernia
Usually a condition of adulthood
Defect is in the linea alba
More common in females
Multiparity and obesity are risk factors
Traditionally repaired using Mayos technique - overlapping repair, mesh may be used though not if small bowel resection is required owing to acute strangulation

Littres hernia
Hernia containing Meckels diverticulum
Resection of the diverticulum is usually required and this will preclude a mesh repair